Deck 1 Flashcards

1
Q

A 25 year old man is shot in the abdomen and is transferred to the operating theatre following arrival in the emergency department, as he is unstable and a FAST scan is positive. At operation there is an extensive laceration to the right lobe of the liver and involvement of the IVC. There is massive haemorrhage. What is the most appropriate approach to blood component therapy?

  • Use Factor VIII concentrates early
  • Avoid use of “o” negative blood
  • Transfuse packed cells, FFP and platelets in fixed ratios of 1:1:1
  • Transfuse packed cells and FFP in a fixed ratio of 4:1
  • Perform goal directed transfusion based on the Hb, PT and TEG studies
A
  • Transfuse packed cells, FFP and platelets in fixed ratios of 1:1:1

There is strong evidence to support the use of haemostatic transfusion in the setting of major haemorrhage due to trauma. This advocates the use of 1:1:1 ratios.
Uncontrolled haemorrhage accounts for up to 39% of all trauma related death. In the UK approximately 2% of all trauma patients will need massive transfusion.Massive transfusion is defined as the replacement of a patient’s total blood volume in less than 24 hours, or as the acute administration of more than half the patient’s estimated blood volume per hour. In haemorrhaging patients following trauma there is evidence to support the initial administration of tranexamic acid (CRASH study). During acute bleeding the practice of haemostatic resuscitation has been shown to reduce mortality rates. The principle of haemostatic resuscitation is that blood components are transfused in fixed ratios. For example; packed red cells, FFP and platelets are administered in a ratio of 1:1:1.

The typical therapeutic end points include:
Hb: 8-10 g/dl
Platelets > 100
PT and APTT (INR)< 1.5
Fibrinogen > 1.0 g/l
Ca2+ > 1 mmol/l
pH: 7.35-7.45
BE: +/- 2
ToC > 36 °C
How well did you know this?
1
Not at all
2
3
4
5
Perfectly
2
Q

A 76 year old woman with a body weight of 50 kg is undergoing an excision of a lipoma from her forehead. It is the first time the senior house officer has performed the procedure. He administers 30ml of 2% lignocaine to the area. The procedure is complicated by bleeding and the patient experiences discomfort, a further 10ml of the same anaesthetic formulation is then administered. Over the following 5 minutes the patient complains of tinnitus and becomes drowsy. Which of the drugs listed below should be administered?

	Temazepam
	Lorazepam
	Naloxone
	Intralipid 20%
	Sodium bicarbonate 20%
A

Intralipid 20%

Intralipid is indicated for the treatment of local anaesthetic toxicity. In this case the safe dose of local anaesthetic has been exceeded and is thus this lady’s symptoms are likely to represent toxicity.
Local anaesthetic toxicity

Toxicity results from either accidental intravascular injection (rapid onset of symptoms-usually correct dose), or from excessive dosage (slower onset). Local anaesthetic agents not only exert a membrane stabilising effect on peripheral nerves but will also act on excitable membranes within the CNS and Heart. The sensory neurones in the CNS are suppressed before the motor ones. As a result the early symptoms will typically be those of circumoral paraesthesia and tinnitus, followed by falling GCS and eventually coma.

Management of toxicity
Stop injecting the anaesthetic agent
High flow 100% oxygen via face mask
Cardiovascular monitoring
Administer lipid emulsion (Intralipid 20%) at 1.5ml/Kg over 1 minute as a bolus
Consider lipid emulsion infusion, at 0.25ml/ Kg/ minute
If toxicity due to prilocaine then administer methylene blue

Safe doses
10ml of lignocaine 1% contains 100mg of drug, this would constitute 70% of the maximum safe dose in a 50 kg patient. Up to 7mg / kg can be administered if adrenaline is added to the solution.

How well did you know this?
1
Not at all
2
3
4
5
Perfectly
3
Q

A 27 year old man is involved in a road traffic accident. He is seen in the emergency department with chest pain. Clinical examination is essentially unremarkable and he is discharged. He subsequently is found dead at home. What is the most likely underlying injury?

	Tracheobronchial tree injury
	Traumatic aortic disruption
	Cardiac laceration
	Diaphragmatic rupture
	Rupture of the oesophagus
A

Traumatic aortic disruption

Aortic injuries that do not die at the scene may have a contained haematoma. Clinical signs are subtle and the diagnosis may not be apparent on clinical examination. Without prompt treatment the haematoma usually bursts and the patient dies.

Thoracic aorta rupture

Mechanism of injury: Decelerating force i.e. RTA, fall from a great height
Most people die at scene
Survivors may have an incomplete laceration at the ligamentum arteriosum of the aorta.

Clinical features
Contained haematoma: persistent hypotension
Detected mainly by history, CXR changes

CXR changes
Widened mediastinum
Trachea/Oesophagus to right
Depression of left main stem bronchus
Widened paratracheal stripe/paraspinal interfaces
Space between aorta and pulmonary artery obliterated
Rib fracture/left haemothorax

Diagnosis
Angiography, usually CT aortogram.

Treatment
Repair or replacement. Ideally they should undergo endovascular repair.

How well did you know this?
1
Not at all
2
3
4
5
Perfectly
4
Q

A 68 year old man presents to the plastics team with severe burns to his hands. He is not distressed by the burns. He has bilateral charcot joints. On examination; there is loss of pain and temperature sensation of the upper limbs.

A.	Osteomyelitis
B.	Potts disease of the spine
C.	Scheuermanns disease
D.	Transverse myelitis
E.	Tabes dorsalis
F.	Subacute degeneration of the cord
G.	Brown-Sequard syndrome
H.	Syringomyelia
I.	Epidural haematoma
A

The correct answer is Syringomyelia

This patient has syringomyelia which selectively affects the spinotholamic tracts. Syringomyelia is a disorder in which a cystic cavity forms within the spinal cord. The commonest variant is the Arnold- Chiari malformation in which the cavity connects with a congenital malformation affecting the cerebellum. Acquired forms of the condition may occur as a result of previous meningitis, surgery or tumours. Many neurological manifestations have been reported, although the classical variety spares the dorsal columns and medial lemniscus and affecting only the spinothalamic tract with loss of pain and temperature sensation. The bilateral distribution of this patients symptoms would therefore favor syringomyelia over SCID or Brown Sequard syndrome. Osteomyelitis would tend to present with back pain and fever in addition to any neurological signs. Epidural haematoma large enough to produce neurological impairment will usually have motor symptoms in addition to any selective sensory loss, and the history is usually shorter.

How well did you know this?
1
Not at all
2
3
4
5
Perfectly
5
Q

A 24 year old man presents with localised spinal pain over 2 months which is worsened on movement. He is known to be an IVDU. He has no history suggestive of tuberculosis. The pain is now excruciating at rest and not improving with analgesia. He has a temperature of 39 oC.

A.	Osteomyelitis
B.	Potts disease of the spine
C.	Scheuermanns disease
D.	Transverse myelitis
E.	Tabes dorsalis
F.	Subacute degeneration of the cord
G.	Brown-Sequard syndrome
H.	Syringomyelia
I.	Epidural haematoma
A

In an IVDU with back pain and pyrexia have a high suspicion for osteomylelitis. The most likely organism is staph aureus and the cervical spine is the most common region affected. TB tends to affect the thoracic spine and in other causes of osteomyelitis the lumbar spine is affected.

How well did you know this?
1
Not at all
2
3
4
5
Perfectly
6
Q

A 22 year man is shot in the back, in the lumbar region. He has increased tone and hyper-reflexia of his right leg. He cannot feel his left leg.

A.	Osteomyelitis
B.	Potts disease of the spine
C.	Scheuermanns disease
D.	Transverse myelitis
E.	Tabes dorsalis
F.	Subacute degeneration of the cord
G.	Brown-Sequard syndrome
H.	Syringomyelia
I.	Epidural haematoma
A

Brown -Sequard syndrome is caused by hemisection of the spinal cord. It may result from stab injuries or lateral vertebral fractures. It results in ipsilateral paralysis (pyramidal tract) , and also loss of proprioception and fine discrimination (dorsal columns). Pain and temperature sensation are lost on the contra-lateral side. This is because the fibres of the spinothalamic tract have decussated below the level of the cord transection.

How well did you know this?
1
Not at all
2
3
4
5
Perfectly
7
Q

A patient is brought to the emergency department following a motor vehicle accident. He is unconscious and has a deep scalp laceration. His heart rate is 120/min, blood pressure is 80/40 mmHg, and respiratory rate is 35/min. Despite rapid administration of 2 litres of Hartmans solution, the patient’s vital signs do not change significantly. The injury likely to explain this patient’s hypotension is:

	Epidural haematoma
	Sub dural haematoma
	Intra parenchymal brain haemorrhage
	Base of skull fracture
	None of the above
A

In the patient described, hypotension and tachycardia should not be uncritically attributed to the head injury, since these findings in the setting of blunt trauma are suggestive of serious thoracic, abdominal, or pelvic hemorrhage. When cardiovascular collapse occurs as a result of rising intracranial pressure, it is generally accompanied by hypertension, bradycardia, and respiratory depression.

Head injury

Patients who suffer head injuries should be managed according to ATLS principles and extra cranial injuries should be managed alongside cranial trauma. Inadequate cardiac output will compromise CNS perfusion irrespective of the nature of the cranial injury.

Types of traumatic brain injury
Extradural haematoma Bleeding into the space between the dura mater and the skull. Often results from acceleration-deceleration trauma or a blow to the side of the head. The majority of extradural haematomas occur in the temporal region where skull fractures cause a rupture of the middle meningeal artery. Raised intracranial pressure
Some patients may exhibit a lucid interval

Subdural haematoma Bleeding into the outermost meningeal layer. Most commonly occur around the frontal and parietal lobes. May be either acute or chronic.
Risk factors include old age and alcoholism.
Slower onset of symptoms than a extradural haematoma.

Subarachnoid haemorrhage Usually occurs spontaneously in the context of a ruptured cerebral aneurysm, but may be seen in association with other injuries when a patient has sustained a traumatic brain injury.

Pathophysiology
Primary brain injury may be focal (contusion/ haematoma) or diffuse (diffuse axonal injury)
Diffuse axonal injury occurs as a result of mechanical shearing following deceleration, causing disruption and tearing of axons
Intra-cranial haematomas can be extradural, subdural or intracerebral, while contusions may occur adjacent to (coup) or contralateral (contre-coup) to the side of impact
Secondary brain injury occurs when cerebral oedema, ischaemia, infection, tonsillar or tentorial herniation exacerbates the original injury. The normal cerebral auto regulatory processes are disrupted following trauma rendering the brain more susceptible to blood flow changes and hypoxia
The Cushings reflex (hypertension and bradycardia) often occurs late and is usually a pre terminal event

Management
Where there is life threatening rising ICP such as in extra dural haematoma and whilst theatre is prepared or transfer arranged use of IV mannitol/ frusemide may be required.
Diffuse cerebral oedema may require decompressive craniotomy
Exploratory Burr Holes have little management in modern practice except where scanning may be unavailable and to thus facilitate creation of formal craniotomy flap
Depressed skull fractures that are open require formal surgical reduction and debridement, closed injuries may be managed non operatively if there is minimal displacement.
ICP monitoring is appropriate in those who have GCS 3-8 and normal CT scan.
ICP monitoring is mandatory in those who have GCS 3-8 and abnormal CT scan.
Hyponatraemia is most likely to be due to syndrome of inappropriate ADH secretion.
Minimum of cerebral perfusion pressure of 70mmHg in adults.
Minimum cerebral perfusion pressure of between 40 and 70 mmHg in children.

How well did you know this?
1
Not at all
2
3
4
5
Perfectly
8
Q

A 19 year old sportswoman presents with knee pain which is worse on walking down the stairs and when sitting still. On examination there is wasting of the quadriceps and pseudolocking of the knee.

A.	Chondromalacia patellae
B.	Dislocated patella
C.	Undisplaced fracture patella
D.	Displaced patella fracture
E.	Avulsion fracture of the tibial tubercle
F.	Quadriceps tendon rupture
G.	Osgood Schlatters disease
A

Chondromalacia patellae

A teenage girl with knee pain on walking down the stairs is characteristic for chondromalacia patellae (anterior knee pain). Most cases are managed with physiotherapy.

Teenage girls, following an injury to knee e.g. Dislocation patella
Typical history of pain on going downstairs or at rest
Tenderness, quadriceps wasting

How well did you know this?
1
Not at all
2
3
4
5
Perfectly
9
Q

A tall 18 year old male athlete is admitted to the emergency room after being hit in the knee by a hockey stick. On examination his knee is tense and swollen. X-ray shows no fractures.

A.	Chondromalacia patellae
B.	Dislocated patella
C.	Undisplaced fracture patella
D.	Displaced patella fracture
E.	Avulsion fracture of the tibial tubercle
F.	Quadriceps tendon rupture
G.	Osgood Schlatters disease
A

Dislocated patella

A patella dislocation is a common cause of haemarthrosis and many will spontaneously reduce when the leg is straightened. In the chronic setting physiotherapy is used to strengthen the quadriceps muscles.

Most commonly occurs as a traumatic primary event, either through direct trauma or through severe contraction of quadriceps with knee stretched in valgus and external rotation
Genu valgum, tibial torsion and high riding patella are risk factors
Skyline x-ray views of patella are required, although displaced patella may be clinically obvious
An osteochondral fracture is present in 5%
The condition has a 20% recurrence rate

How well did you know this?
1
Not at all
2
3
4
5
Perfectly
10
Q

An athletic 15 year old boy presents with knee pain of 3 weeks duration. It is worst during activity and settles with rest. On examination there is tenderness overlying the tibial tuberosity and an associated swelling at this site.

A.	Chondromalacia patellae
B.	Dislocated patella
C.	Undisplaced fracture patella
D.	Displaced patella fracture
E.	Avulsion fracture of the tibial tubercle
F.	Quadriceps tendon rupture
G.	Osgood Schlatters disease
A

Osgood Schlatters disease

Athletic boys and girls may develop this condition in their teenage years. It is caused by multiple micro fractures at the point of insertion of the tendon into the tibial tuberosity. Most cases settle with physiotherapy and rest.

How well did you know this?
1
Not at all
2
3
4
5
Perfectly
11
Q
A 42 year old man is admitted to surgery with acute appendicitis. He is known to have hypertension, psoriatic arthropathy and polymyalgia rheumatica. His medical therapy includes:
Paracetamol 1g qds
Codeine phosphate 30mg qds
Bendrofluazide 2.5 mg od
Ramipril 10mg od
Methotrexate 7.5mg once a week
Prednisolone 5mg od
You are called by the core surgical trainee to assess this man as he has become delirious and hypotensive 2 hours after surgery. His blood results reveal:
Na+	132 mmol/l
K+	5.2 mmol/l
Urea	10 mmol/l
Creatinine	111 µmol/l
Glucose	3.5
CRP	158

Hb 10.2 g/dl
Platelets 156 * 109/l
WBC 14 * 109/l

What is the most likely diagnosis?

	Septic shock secondary to appendicitis
	Neutropenic sepsis
	Phaeochromocytoma
	Perforated bowel
	Addisonian crisis
A

Features of an addisonian crisis:
Hyponatraemia
Hyperkalaemia
Hypoglycaemia

This man is on steroids for polymyalgia rheumatica. Surgery can precipitate acute adrenal deficiency. The diagnosis is further confirmed by the blood results of hyponatraemia, hyperkalaemia and hypoglycaemia. This patient urgently needs hydrocortisone.
Addisonian crisis

Causes
Sepsis or surgery causing an acute exacerbation of chronic insufficiency (Addison’s, Hypopituitarism)
Adrenal haemorrhage eg Waterhouse-Friderichsen syndrome (fulminant meningococcemia)
Steroid withdrawal

Management
Hydrocortisone 100 mg im or iv
1 litre normal saline infused over 30-60 mins or with dextrose if hypoglycaemic
Continue hydrocortisone 6 hourly until the patient is stable. No fludrocortisone is required because high cortisol exerts weak mineralocorticoid action
Oral replacement may begin after 24 hours and be reduced to maintenance over 3-4 days

How well did you know this?
1
Not at all
2
3
4
5
Perfectly
12
Q

A 20 year old woman trips over a step, injuring her ankle. Examination reveals tenderness over the lateral malleolus and an x-ray demonstrates an undisplaced fracture distal to the syndesmosis.

A. Surgical fixation
B. Below knee amputation
C. Application of below knee plaster
D. Application of ankle boot
E. Application of external fixation device
F. Application of compression dressing and physiotherapy
G. Immediate reduction and application of backslab

A

Application of ankle boot

This is a Weber A fracture. It is a stable ankle injury and can therefore be managed conservatively. Whilst this patient could also be treated in a below knee plaster, most clinicians would nowadays treat this injury in an ankle boot. Patients should be advised to mobilise in the ankle boot, as pain allows, and can wean themselves out of the boot as the symptoms improve.

Danis-Weber
Commonly used. Based on the level of the fibula fracture in relation to the syndesmosis. The more proximal, the greater the risk of syndesmotic injury and therefore fracture instability.
A - fracture below the level of the syndesmosis
B - fracture at the level of the syndesmosis / level of the tibial plafond
C - fracture above the level of the syndesmosis. This includes Maisonneuve fractures (proximal fibula fracture), which can be associated with ankle instability. Beware the high fibula fracture - it may be an ankle fracture!

The Weber classification is based purely on the the lateral side. All injuries can include a medial or posterior bony or ligamentous injury which also dictates fracture stability (bimalleolar and trimalleolar fractures are more unstable).
Defining stability of an ankle fracture underpins the treatment decision.
Weber A - Unimalleolar Weber A Weber fractures by definition are stable and therefore can be mobilised fully weight bearing in an ankle boot.

Weber C - Fractures tend to include syndesmotic disruption and are usually bimalleolar (either bony or ligamentous). They are therefore unstable and usually require operative fixation. In addition to the fracture fixation, the syndesmosis usually requires reconstruction/augmentation with screws to restore the joint integrity and function.

Weber B - B fractures vary greatly. They can be part of a trimalleolar injury and therefore extremely unstable, requiring fixation. Alternatively, a uni-malleolar Weber B fracture can be a stable injury, and therefore mobilised immediately in an ankle boot. Defining the stability can be challenging, and often involves stress radiographs, or a trial of mobilisation and repeat radiographs. Defining stability is the subject of much ongoing research. However, treating undisplaced ankle fractures in a below knee plaster, non-weight bearing for six weeks is still widely practised, and a safe approach.

When operative fixation is appropriate, it is usually via open reduction and internal fixation using plates and screws. It must be carried out when soft tissue swelling has settled in order to minimise the risk of wound problems. This can often take a week to settle.

Lauge-Hansen
Comprises two parts: first part is the foot position, and the second part is the force applied. Useful for understanding the forces involved and therefore predict the ligamentous or bony injury. Results in four injury patterns:
Supination - Adduction (SA) - 10-20%
Supination - External rotation (SER) - 40-75%
Pronation - Abduction (PA) - 5-20%
Pronation - External rotation (PER) - 5-20%

How well did you know this?
1
Not at all
2
3
4
5
Perfectly
13
Q

A 30 year old man injures his ankle playing football. On examination he has tenderness over both medial and lateral malleoli. X-ray demonstrates a bimalleolar fracture with a displaced distal fibula fracture, at the level of the syndesmosis and fracture of the medial malleolus with talar shift. The ankle has been provisionally reduced and splinted in the emergency department.

A. Surgical fixation
B. Below knee amputation
C. Application of below knee plaster
D. Application of ankle boot
E. Application of external fixation device
F. Application of compression dressing and physiotherapy
G. Immediate reduction and application of backslab

A

Surgical fixation

This is an unstable fracture pattern with a Weber B fracture of the distal fibula and a fracture of the medial malleolus. Talar shift indicates loss of ankle mortice congruity. This injury should therefore be treated with surgical fixation.

The Weber classification is based purely on the the lateral side. All injuries can include a medial or posterior bony or ligamentous injury which also dictates fracture stability (bimalleolar and trimalleolar fractures are more unstable).
Defining stability of an ankle fracture underpins the treatment decision.
Weber A - Unimalleolar Weber A Weber fractures by definition are stable and therefore can be mobilised fully weight bearing in an ankle boot.

Weber C - Fractures tend to include syndesmotic disruption and are usually bimalleolar (either bony or ligamentous). They are therefore unstable and usually require operative fixation. In addition to the fracture fixation, the syndesmosis usually requires reconstruction/augmentation with screws to restore the joint integrity and function.

Weber B - B fractures vary greatly. They can be part of a trimalleolar injury and therefore extremely unstable, requiring fixation. Alternatively, a uni-malleolar Weber B fracture can be a stable injury, and therefore mobilised immediately in an ankle boot. Defining the stability can be challenging, and often involves stress radiographs, or a trial of mobilisation and repeat radiographs. Defining stability is the subject of much ongoing research. However, treating undisplaced ankle fractures in a below knee plaster, non-weight bearing for six weeks is still widely practised, and a safe approach.

When operative fixation is appropriate, it is usually via open reduction and internal fixation using plates and screws. It must be carried out when soft tissue swelling has settled in order to minimise the risk of wound problems. This can often take a week to settle.

Lauge-Hansen
Comprises two parts: first part is the foot position, and the second part is the force applied. Useful for understanding the forces involved and therefore predict the ligamentous or bony injury. Results in four injury patterns:
Supination - Adduction (SA) - 10-20%
Supination - External rotation (SER) - 40-75%
Pronation - Abduction (PA) - 5-20%
Pronation - External rotation (PER) - 5-20%

How well did you know this?
1
Not at all
2
3
4
5
Perfectly
14
Q

A 50 year old female slips on wet floor injuring her ankle. On examination, she has tenderness over the lateral and medial malleolus. X-rays demonstrate an undisplaced fracture of the distal fiibula at the level of the syndesmosis and a congruent ankle mortice.

A. Surgical fixation
B. Below knee amputation
C. Application of below knee plaster
D. Application of ankle boot
E. Application of external fixation device
F. Application of compression dressing and physiotherapy
G. Immediate reduction and application of backslab

A

Application of below knee plaster

This is a Weber B fracture and therefore potentially unstable. Medial malleolar tenderness indicates deltoid ligament injury. As the fracture is currently undisplaced and the ankle mortice is congruent, the injury can be initially managed conservatively in a below knee plaster but the patient should be monitored in the outpatient clinic for fracture displacement in the first few weeks.

The Weber classification is based purely on the the lateral side. All injuries can include a medial or posterior bony or ligamentous injury which also dictates fracture stability (bimalleolar and trimalleolar fractures are more unstable).
Defining stability of an ankle fracture underpins the treatment decision.
Weber A - Unimalleolar Weber A Weber fractures by definition are stable and therefore can be mobilised fully weight bearing in an ankle boot.

Weber C - Fractures tend to include syndesmotic disruption and are usually bimalleolar (either bony or ligamentous). They are therefore unstable and usually require operative fixation. In addition to the fracture fixation, the syndesmosis usually requires reconstruction/augmentation with screws to restore the joint integrity and function.

Weber B - B fractures vary greatly. They can be part of a trimalleolar injury and therefore extremely unstable, requiring fixation. Alternatively, a uni-malleolar Weber B fracture can be a stable injury, and therefore mobilised immediately in an ankle boot. Defining the stability can be challenging, and often involves stress radiographs, or a trial of mobilisation and repeat radiographs. Defining stability is the subject of much ongoing research. However, treating undisplaced ankle fractures in a below knee plaster, non-weight bearing for six weeks is still widely practised, and a safe approach.

When operative fixation is appropriate, it is usually via open reduction and internal fixation using plates and screws. It must be carried out when soft tissue swelling has settled in order to minimise the risk of wound problems. This can often take a week to settle.
Lauge-Hansen
Comprises two parts: first part is the foot position, and the second part is the force applied. Useful for understanding the forces involved and therefore predict the ligamentous or bony injury. Results in four injury patterns:
Supination - Adduction (SA) - 10-20%
Supination - External rotation (SER) - 40-75%
Pronation - Abduction (PA) - 5-20%
Pronation - External rotation (PER) - 5-20%

How well did you know this?
1
Not at all
2
3
4
5
Perfectly
15
Q

A 30 year old woman presents with pain and swelling of the left shoulder. There is a large radiolucent lesion in the head of the humerus extending to the subchondral plate.

A.	Osteosarcoma
B.	Osteomalacia
C.	Osteoporosis
D.	Metastatic carcinoma
E.	Osteoblastoma
F.	Giant cell tumour
G.	Ewing's sarcoma
A

Giant cell tumours on x-ray have a ‘soap bubble’ appearance. They present as pain or pathological fractures. They commonly metastasize to the lungs.

How well did you know this?
1
Not at all
2
3
4
5
Perfectly
16
Q

A 72 year old woman has a lumbar vertebral crush fracture. She has hypocalcaemia and a low urinary calcium.

A.	Osteosarcoma
B.	Osteomalacia
C.	Osteoporosis
D.	Metastatic carcinoma
E.	Osteoblastoma
F.	Giant cell tumour
G.	Ewing's sarcoma
A

Osteomalacia

Hypocalcemia and low urinary calcium are biochemical features of osteomalacia. Unfortunately surgeons do need to look at some blood results!

How well did you know this?
1
Not at all
2
3
4
5
Perfectly
17
Q

A 16 year old boy presents with severe groin pain after kicking a football. Imaging confirms a pelvic fracture. A previous pelvic x-ray performed 2 weeks ago shows a lytic lesion with ‘onion type’ periosteal reaction.

A.	Osteosarcoma
B.	Osteomalacia
C.	Osteoporosis
D.	Metastatic carcinoma
E.	Osteoblastoma
F.	Giant cell tumour
G.	Ewing's sarcoma
A

Ewing’s sarcoma

A Ewings sarcoma is most common in males between 10-20 years. It can occur in girls. A lytic lesion with a lamellated or onion type periosteal reaction is a classical finding on x-rays. Most patients present with metastatic disease with a 5 year prognosis between 5-10%.

How well did you know this?
1
Not at all
2
3
4
5
Perfectly
18
Q

A 23 year old rugby player falls directly onto his shoulder. There is pain and swelling of the shoulder joint. The clavicle is prominent and there appears to be a step deformity.

A.	Glenohumeral dislocation
B.	Acromioclavicular dislocation
C.	Sternoclavicular dislocation
D.	Biceps tendon tear
E.	Supraspinatus tear
F.	Fracture of the surgical neck of the humerus
G.	Infra spinatus tear
A

Acromioclavicular dislocation

Acromioclavicular joint (ACJ) dislocation normally occurs secondary to direct injury to the superior aspect of the acromion. Loss of shoulder contour and prominent clavicle are key features. Note; rotator cuff tears rarely occur in the second decade.

Shoulder fractures and dislocations
Fractures
Proximal humerus
Background
Third most common fragility fracture in the elderly.
Results from low energy fall in predominantly elderly females, or from high energy trauma in young males.
Can be associated with nerve injury (commonly axillary), and fracture-dislocation of the humeral head. Detailed neurological assessment is essential for all upper limb injuries.

Anatomy
Osteology
Consists of articular head, greater tuberosity, lesser tuberosity, metaphysis and diaphysis. Between the articular head and the tuberosities is the anatomical neck (previous physis). Between the tuberosities and the metaphysis is the surgical neck.
The supraspinatus, infraspinatus and teres minor muscles attach to the greater tuberosity. The subscapularis muscle attaches to the lesser tuberosity.

Vascular Supply
Humeral head is supplied by the anterior and posterior humeral circumflex arteries. Anatomical neck fractures are at greatest risk of osteonecrosis.

Imaging
Imaging aims to both delineate the fracture pattern, and confirm/exlude the presence of an associated dislocation.
Radiographs - True anteroposterior (AP), axillary lateral and/or scapula Y view.
CT - indicated to better define intra-articular involvement and to aid pre-operative planning. MRI is not useful for fracture imaging.

Classification
Description of the fracture is often more useful than classification. Particular attention should be paid to humeral alignment, fracture displacement, and greater tuberosity position (rotator cuff will pull the GT supero-posterioly, which can cause impingement problems with malunion).
- Neer Classification: Most commonly used. Describes fracture as 2,3,or 4 part depending upon the number main fragments. Also comments on the degree of displacement. Fragments:
-greater tuberosity
-lesser tuberosity
- articular surface
- shaft
Displacement: >1cm or angulation >45 degrees.

Treatment
The vast majority of proximal humeral fractures are minimally displaced, and therefore can be managed conservatively. This involves immobilisation in a polysling, and progressive mobilisation. Pendular exercise can commence at 14 days, and active abduction from 4-6 weeks.

How well did you know this?
1
Not at all
2
3
4
5
Perfectly
19
Q

A 22 year old man falls over and presents to casualty. A shoulder x-ray is performed, the radiologist comments that a Hill-Sachs lesion is present.

A.	Glenohumeral dislocation
B.	Acromioclavicular dislocation
C.	Sternoclavicular dislocation
D.	Biceps tendon tear
E.	Supraspinatus tear
F.	Fracture of the surgical neck of the humerus
G.	Infra spinatus tear
A

Glenohumeral dislocation

A Hill-Sachs lesion occurs when the cartilage surface of the humerus is in contact with the rim of the glenoid. About 50% of anterior glenohumeral dislocations are associated with this lesion.

Shoulder fractures and dislocations
Fractures
Proximal humerus
Background
Third most common fragility fracture in the elderly.
Results from low energy fall in predominantly elderly females, or from high energy trauma in young males.
Can be associated with nerve injury (commonly axillary), and fracture-dislocation of the humeral head. Detailed neurological assessment is essential for all upper limb injuries.

Anatomy
Osteology
Consists of articular head, greater tuberosity, lesser tuberosity, metaphysis and diaphysis. Between the articular head and the tuberosities is the anatomical neck (previous physis). Between the tuberosities and the metaphysis is the surgical neck.
The supraspinatus, infraspinatus and teres minor muscles attach to the greater tuberosity. The subscapularis muscle attaches to the lesser tuberosity.

Vascular Supply
Humeral head is supplied by the anterior and posterior humeral circumflex arteries. Anatomical neck fractures are at greatest risk of osteonecrosis.

Imaging
Imaging aims to both delineate the fracture pattern, and confirm/exlude the presence of an associated dislocation.
Radiographs - True anteroposterior (AP), axillary lateral and/or scapula Y view.
CT - indicated to better define intra-articular involvement and to aid pre-operative planning. MRI is not useful for fracture imaging.

Classification
Description of the fracture is often more useful than classification. Particular attention should be paid to humeral alignment, fracture displacement, and greater tuberosity position (rotator cuff will pull the GT supero-posterioly, which can cause impingement problems with malunion).
- Neer Classification: Most commonly used. Describes fracture as 2,3,or 4 part depending upon the number main fragments. Also comments on the degree of displacement. Fragments:
-greater tuberosity
-lesser tuberosity
- articular surface
- shaft
Displacement: >1cm or angulation >45 degrees.

Treatment
The vast majority of proximal humeral fractures are minimally displaced, and therefore can be managed conservatively. This involves immobilisation in a polysling, and progressive mobilisation. Pendular exercise can commence at 14 days, and active abduction from 4-6 weeks.

How well did you know this?
1
Not at all
2
3
4
5
Perfectly
20
Q

An 82 year old female presents to A&E after tripping on a step. She complains of shoulder pain. On examination there is pain to 90o on abduction.

A.	Glenohumeral dislocation
B.	Acromioclavicular dislocation
C.	Sternoclavicular dislocation
D.	Biceps tendon tear
E.	Supraspinatus tear
F.	Fracture of the surgical neck of the humerus
G.	Infra spinatus tear
A

Supraspinatus tear

A supraspinatus tear is the most common of rotator cuff tears. It occurs as a result of degeneration and is rare in younger adults.

Shoulder fractures and dislocations
Fractures
Proximal humerus
Background
Third most common fragility fracture in the elderly.
Results from low energy fall in predominantly elderly females, or from high energy trauma in young males.
Can be associated with nerve injury (commonly axillary), and fracture-dislocation of the humeral head. Detailed neurological assessment is essential for all upper limb injuries.

Anatomy
Osteology
Consists of articular head, greater tuberosity, lesser tuberosity, metaphysis and diaphysis. Between the articular head and the tuberosities is the anatomical neck (previous physis). Between the tuberosities and the metaphysis is the surgical neck.
The supraspinatus, infraspinatus and teres minor muscles attach to the greater tuberosity. The subscapularis muscle attaches to the lesser tuberosity.

Vascular Supply
Humeral head is supplied by the anterior and posterior humeral circumflex arteries. Anatomical neck fractures are at greatest risk of osteonecrosis.

Imaging
Imaging aims to both delineate the fracture pattern, and confirm/exlude the presence of an associated dislocation.
Radiographs - True anteroposterior (AP), axillary lateral and/or scapula Y view.
CT - indicated to better define intra-articular involvement and to aid pre-operative planning. MRI is not useful for fracture imaging.

Classification
Description of the fracture is often more useful than classification. Particular attention should be paid to humeral alignment, fracture displacement, and greater tuberosity position (rotator cuff will pull the GT supero-posterioly, which can cause impingement problems with malunion).
- Neer Classification: Most commonly used. Describes fracture as 2,3,or 4 part depending upon the number main fragments. Also comments on the degree of displacement. Fragments:
-greater tuberosity
-lesser tuberosity
- articular surface
- shaft
Displacement: >1cm or angulation >45 degrees.

Treatment
The vast majority of proximal humeral fractures are minimally displaced, and therefore can be managed conservatively. This involves immobilisation in a polysling, and progressive mobilisation. Pendular exercise can commence at 14 days, and active abduction from 4-6 weeks.

Pathological fractures
Genetic conditions, such as osteogenesis imperfecta, may cause pathological fractures.

Osteogenesis imperfecta
Defective osteoid formation due to congenital inability to produce adequate intercellular substances like osteoid, collagen and dentine.
Failure of maturation of collagen in all the connective tissues.
Radiology may show translucent bones, multiple fractures, particularly of the long bones, wormian bones (irregular patches of ossification) and a trefoil pelvis.

Subtypes
Type I The collagen is normal quality but insufficient quantity.
Type II- Poor collagen quantity and quality.
Type III- Collagen poorly formed. Normal quantity.
Type IV- Sufficient collagen quantity but poor quality.

Osteopetrosis
Bones become harder and more dense.
Autosomal recessive condition.
It is commonest in young adults.
Radiology reveals a lack of differentiation between the cortex and the medulla described as marble bone.
How well did you know this?
1
Not at all
2
3
4
5
Perfectly
21
Q

A 15 year-old boy presents to the out-patient clinic with tiredness, recurrent throat and chest infections, and gradual loss of vision. Multiple x-rays show brittle bones with no differentiation between the cortex and the medulla.

A.	Rickets
B.	Craniocleidodysostosis
C.	Achondroplasia
D.	Scurvy
E.	Pagets disease
F.	Multiple myeloma
G.	Osteogenesis imperfecta
H.	Osteomalacia
I.	Osteopetrosis
J.	None of the above
A

Osteopetrosis

Osteopetrosis is an autosomal recessive condition. It is commonest in young adults. They may present with symptoms of anaemia or thrombocytopaenia due to decreased marrow space. Radiology reveals a lack of differentiation between the cortex and the medulla described as marble bone. These bones are very dense and brittle.

Pathological fractures
Genetic conditions, such as osteogenesis imperfecta, may cause pathological fractures.

Osteogenesis imperfecta
Defective osteoid formation due to congenital inability to produce adequate intercellular substances like osteoid, collagen and dentine.
Failure of maturation of collagen in all the connective tissues.
Radiology may show translucent bones, multiple fractures, particularly of the long bones, wormian bones (irregular patches of ossification) and a trefoil pelvis.

Subtypes
Type I The collagen is normal quality but insufficient quantity.
Type II- Poor collagen quantity and quality.
Type III- Collagen poorly formed. Normal quantity.
Type IV- Sufficient collagen quantity but poor quality.

Osteopetrosis
Bones become harder and more dense.
Autosomal recessive condition.
It is commonest in young adults.
Radiology reveals a lack of differentiation between the cortex and the medulla described as marble bone.
How well did you know this?
1
Not at all
2
3
4
5
Perfectly
22
Q

A 12 year-old boy who is small for his age presents to the clinic with poor muscular development and hyper-mobile fingers. His x-rays show multiple fractures of the long bones and irregular patches of ossification.

A.	Rickets
B.	Craniocleidodysostosis
C.	Achondroplasia
D.	Scurvy
E.	Pagets disease
F.	Multiple myeloma
G.	Osteogenesis imperfecta
H.	Osteomalacia
I.	Osteopetrosis
J.	None of the above
A

Osteogenesis imperfecta

Osteogenesis imperfecta is caused by defective osteoid formation due to congenital inability to produce adequate intercellular substances like osteoid, collagen and dentine. There is a failure of maturation of collagen in all the connective tissues.Radiology may show translucent bones, multiple fractures, particularly of the long bones, wormian bones (irregular patches of ossification) and a trefoil pelvis.

How well did you know this?
1
Not at all
2
3
4
5
Perfectly
23
Q

A 1 year-old is brought to the Emergency Department with a history of failure to thrive. On examination, the child is small for age and has a large head. X-ray shows a cupped appearance of the epiphysis of the wrist.

A.	Rickets
B.	Craniocleidodysostosis
C.	Achondroplasia
D.	Scurvy
E.	Pagets disease
F.	Multiple myeloma
G.	Osteogenesis imperfecta
H.	Osteomalacia
I.	Osteopetrosis
J.	None of the above
A

Rickets

Rickets is the childhood form of osteomalacia. It is due to the failure of the osteoid to ossify due to vitamin D deficiency. Symptoms start about the age of one. The child is small for age and there is a history of failure to thrive. Bony deformities include bowing of the femur and tibia, a large head, deformity of the chest wall with thickening of the costochondral junction (rickettary rosary), and a transverse sulcus in the chest caused by the pull of the diaphragm (Harrison’s sulcus). X- Rays show widening and cupping of the epiphysis of the long bones, most readily apparent in the wrist.

Pathological fractures
Genetic conditions, such as osteogenesis imperfecta, may cause pathological fractures.

Osteogenesis imperfecta
Defective osteoid formation due to congenital inability to produce adequate intercellular substances like osteoid, collagen and dentine.
Failure of maturation of collagen in all the connective tissues.
Radiology may show translucent bones, multiple fractures, particularly of the long bones, wormian bones (irregular patches of ossification) and a trefoil pelvis.

Subtypes
Type I The collagen is normal quality but insufficient quantity.
Type II- Poor collagen quantity and quality.
Type III- Collagen poorly formed. Normal quantity.
Type IV- Sufficient collagen quantity but poor quality.

Osteopetrosis
Bones become harder and more dense.
Autosomal recessive condition.
It is commonest in young adults.
Radiology reveals a lack of differentiation between the cortex and the medulla described as marble bone.
How well did you know this?
1
Not at all
2
3
4
5
Perfectly
24
Q

A 24 year old motorist is involved in a road traffic accident in which he collides with the wall of a tunnel in a head on car crash, speed 85mph. He is wearing a seatbelt and the airbags have deployed. When rescuers arrive he is lucid and conscious and then dies suddenly.

A.	Tension pneumothorax
B.	Haemopericardium
C.	Haemothorax
D.	Aortic transection
E.	Ruptured spleen
F.	Duodeno-jejunal flexure disruption
G.	Aorto iliac disruption
H.	Ileo-colic junction disruption
A

Aortic transection

Aortic transections typically occur distal to the ligamentum arteriosum. A temporary haematoma may prevent the immediate death that usually occurs. This is a deceleration injury. A widened mediastinum may be seen on x-ray.

How well did you know this?
1
Not at all
2
3
4
5
Perfectly
25
Q

A 30 year old women is involved in a road traffic accident she is a passenger in a car involved in a head on collision with another vehicle. Her car is travelling at 60mph. She has been haemodynamically stable throughout with only minimal tachycardia. On examination she has marked abdominal tenderness and a large amount of intra abdominal fluid on CT scan

A.	Tension pneumothorax
B.	Haemopericardium
C.	Haemothorax
D.	Aortic transection
E.	Ruptured spleen
F.	Duodeno-jejunal flexure disruption
G.	Aorto iliac disruption
H.	Ileo-colic junction disruption
A

Duodeno-jejunal flexure disruption

This is another site of sudden deceleration injury. Given the large amount of free fluid, if it were blood, then a greater degree of haemodynamic instability would be expected.

How well did you know this?
1
Not at all
2
3
4
5
Perfectly
26
Q

A 17 year old boy is involved in a motorcycle accident in which he is thrown from his motorcycle. On admission he has distended neck veins and a weak pulse. The trachea is central.

A.	Tension pneumothorax
B.	Haemopericardium
C.	Haemothorax
D.	Aortic transection
E.	Ruptured spleen
F.	Duodeno-jejunal flexure disruption
G.	Aorto iliac disruption
H.	Ileo-colic junction disruption
A

Haemopericardium

This is most likely a cardiac tamponade produced by haemopericardium. As little as 100ml of blood may result in tamponade as the pericardial sac is not distensible. Diagnosis is suggested by muffled heart sounds, paradoxical pulse and jugular vein distension.

How well did you know this?
1
Not at all
2
3
4
5
Perfectly
27
Q

A 6 year old boy pulls over a kettle and suffers superficial partial thickness burns to his legs. Which of the following will not occur?

	Preservation of hair follicles
	Formation of vesicles or bullae
	Damage to sweat glands
	Healing by re-epithelialisation
	Pain at the burn site
A

Damage to sweat glands

Partial thickness burns are divided into superficial and deep burns, however, this is often not possible on initial assessment and it may be a week or more before the distinction is clear cut. Dermal appendages are, by definition, intact. Superficial partial thickness burns will typically heal by re-epithelialisation, deeper burns will heal with scarring.

Burns may be thermal, chemical or electrical. In the former category are burns which occur as a result of heat. Chemical burns occur when the skin is exposed to an extremely caustic or alkaline substance. Electrical burns occur following exposure to electrical current. The immediate management includes removal of the burning source which usually includes irrigation of the burned area. A detailed assessment then needs to be made of the extent of the burns and a number of charts are available for recording this information. The degree of injury relates to the temperature and duration of exposure. Most domestic burns are mainly scalds in young children.

Following the burn, there is a local response with progressive tissue loss and release of inflammatory cytokines. Systemically, there are cardiovascular effects resulting from fluid loss and sequestration of fluid into the third space. There is a marked catabolic response. Immunosupression is common with large burns and bacterial translocation from the gut lumen is a recognised event. Sepsis is a common cause of death following major burns.

Management
The initial aim is to stop the burning process and resuscitate the patient. Intravenous fluids will be required for children with burns greater than 10% of total body surface area. Adults with burns greater than 15% of total body surface area will also require IV fluids. The fluids are calculated using the Parkland formula which is; volume of fluid= total body surface area of the burn % x weight (Kg) x4. Half of the fluid is administered in the first 8 hours. A urinary catheter should be inserted. Analgesia should be given. Complex burns, burns involving the hand perineum and face and burns >10% in adults and >5% in children should be transferred to a burns unit.

Circumferential burns affecting a limb or severe torso burns impeding respiration may require escharotomy to divide the burnt tissue.

Conservative management is appropriate for superficial burns and mixed superficial burns that will heal in 2 weeks. More complex burns may require excision and skin grafting. Excision and primary closure is not generally practised as there is a high risk of infection.

There is no evidence to support the use of anti microbial prophylaxis or topical antibiotics in burn patients.

Escharotomies
Indicated in circumferential full thickness burns to the torso or limbs.
Careful division of the encasing band of burn tissue will potentially improve ventilation (if the burn involves the torso), or relieve compartment syndrome and oedema (where a limb is involved)

How well did you know this?
1
Not at all
2
3
4
5
Perfectly
28
Q

Based on the current guidelines, which option regarding management of head injuries is false?

Opiates should be avoided
Consider intubation if the GCS is <8 or = 8
Immediate CT head if there is > 1 episode of vomiting
Half hourly GCS assessment until GCS is 15
Contact neurosurgeons if suspected penetrating injury

A

Opiates should be avoided

Head injury management- NICE Guidelines

Summary of guidelines
All patients should be assessed within 15 minutes on arrival to A&E
Document all 3 components of the GCS
If GCS <8 or = to 8, consider stabilising the airway
Treat pain with low dose IV opiates (if safe)
Full spine immobilisation until assessment if:

  • GCS < 15
  • neck pain/tenderness
  • paraesthesia extremities
  • focal neurological deficit
  • suspected c-spine injury

If a c-spine injury is suspected a 3 view c-spine x-ray is indicated. CT c-spine is preferred if:

  • Intubated
  • GCS <13
  • Normal x-ray but continued concerns regarding c-spine injury
  • Any focal neurology
  • A CT head scan is being performed
  • Initial plain films are abnormal
Immediate CT head (within 1 hour) if:
GCS < 13 on admission
GCS < 15 2 hours after admission
Suspected open or depressed skull fracture
Suspected skull base fracture (panda eyes, Battle's sign, CSF from nose/ear, bleeding ear)
Focal neurology
Vomiting > 1 episode
Post traumatic seizure
Coagulopathy
Contact neurosurgeon if:
Persistent GCS < 8 or = 8
Unexplained confusion > 4h
Reduced GCS after admission
Progressive neurological signs
Incomplete recovery post seizure
Penetrating injury
Cerebrospinal fluid leak

Observations
1/2 hourly GCS until 15

How well did you know this?
1
Not at all
2
3
4
5
Perfectly
29
Q

A 73 year old lady develops a cold, pulseless hand 3 days following a myocardial infarction.

A.	Vasculitis
B.	Steal syndrome
C.	Thrombosis
D.	Foreign body embolus
E.	Clot embolus
F.	Vasospasm
G.	Direct arterial injury
A

Clot embolus

The development of mural or atrial appendage thrombi may occur following a myocardial infarct and co-existing atrial fibrillation may contribute to the formation. They tend to present with classical features of an embolic event.

How well did you know this?
1
Not at all
2
3
4
5
Perfectly
30
Q

A 6 year old child has suffered a displaced supracondylar humeral fracture. On examination, they have a cold and insensate hand with absent pulses.

A.	Vasculitis
B.	Steal syndrome
C.	Thrombosis
D.	Foreign body embolus
E.	Clot embolus
F.	Vasospasm
G.	Direct arterial injury
A

Direct arterial injury

Both vasospasm and arterial injury may complicate supracondylar fractures and are seen in 1% of all cases. Vasospasm is usually transient and more likely when the injury is minor and reduced early. Severely displaced injuries and those with more advanced signs are usually associated with direct arterial injury.

How well did you know this?
1
Not at all
2
3
4
5
Perfectly
31
Q

A 26 year old man who smokes heavily develops aching, crampy pains in his legs. On examination distal limb pulses are diminished.

A.	Vasculitis
B.	Steal syndrome
C.	Thrombosis
D.	Foreign body embolus
E.	Clot embolus
F.	Vasospasm
G.	Direct arterial injury
A

Vasculitis

This is likely to represent Buergers disease. It is commonest in young males who smoke heavily.

How well did you know this?
1
Not at all
2
3
4
5
Perfectly
32
Q

A 30 year old man is admitted overnight, following a road traffic accident. He has an open tibial fracture with a 20 cm wound and extensive periosteal stripping. He is neurovascularly intact and IV antibiotics and wound dressing have been administered in the emergency department.

A. Immediate skeletal stabilisation and application of negative pressure dressing
B. Combined skeletal and soft tissue reconstruction on a scheduled operating list
C. Thorough wound debridement in the emergency department
D. Immediate vascuIar shunting, followed by temporary skeletal stabilisation and vascular reconstruction
E. Intravenous antibiotics, photography and application of saline soaked gauze with impermeable dressing
F. Application of external fixator and conversion to internal fixation after two weeks
G. Fasciotomy with four compartment decompression
H. Skeletal fixation followed by vascular reconstruction

A

Combined skeletal and soft tissue reconstruction on a scheduled operating list

This patient has a Gustillo-Anderson Grade 3B open fracture. He will require definitive skeletal and soft tissue reconstruction, which should be performed on a combined ortho-plastic scheduled operating list, as per the BOA/BAPRAS guidelines. The surgery does not have to be performed out of scheduled hours unless there is marine/ sewage contamination, vascular compromise or it is a polytrauma.
Whilst it is reasonable to apply an external fixator prior to definitive skeletal and soft tissue reconstruction, this should be converted to internal fixation within 72 hours.

How well did you know this?
1
Not at all
2
3
4
5
Perfectly
33
Q

A 50 year old man is admitted after falling from scaffolding. He has an open fracture of his tibia with a 15 cm wound. He is neurovascularly intact.

A. Immediate skeletal stabilisation and application of negative pressure dressing
B. Combined skeletal and soft tissue reconstruction on a scheduled operating list
C. Thorough wound debridement in the emergency department
D. Immediate vascuIar shunting, followed by temporary skeletal stabilisation and vascular reconstruction
E. Intravenous antibiotics, photography and application of saline soaked gauze with impermeable dressing
F. Application of external fixator and conversion to internal fixation after two weeks
G. Fasciotomy with four compartment decompression
H. Skeletal fixation followed by vascular reconstruction

A

Intravenous antibiotics, photography and application of saline soaked gauze with impermeable dressing

The initial management of open fractures should include administration of intravenous antibiotics, photography of wound and application of a sterile soaked gauze and impermeable film. The wound should only be handled to remove gross contamination. The patient is then likely to require definitive skeletal and soft tissue reconstruction.

How well did you know this?
1
Not at all
2
3
4
5
Perfectly
34
Q

A 40 year old woman is admitted after being knocked off her bike. She has an open fracture of her tibia, with a 10 cm wound. No peripheral pulses are palpable. Intravenous antibiotics have been administered in the emergency department and the wound has been dressed.

A. Immediate skeletal stabilisation and application of negative pressure dressing
B. Combined skeletal and soft tissue reconstruction on a scheduled operating list
C. Thorough wound debridement in the emergency department
D. Immediate vascuIar shunting, followed by temporary skeletal stabilisation and vascular reconstruction
E. Intravenous antibiotics, photography and application of saline soaked gauze with impermeable dressing
F. Application of external fixator and conversion to internal fixation after two weeks
G. Fasciotomy with four compartment decompression
H. Skeletal fixation followed by vascular reconstruction

A

Immediate vascuIar shunting, followed by temporary skeletal stabilisation and vascular reconstruction

This patient has a Gustillo-Anderson Grade 3C open fracture with vascular injury. Vascular impairment requires immediate surgery and restoration of circulation, ideally within 3-4 hours. This should follow the sequence of shunting, temporary skeletal stabilisation and then vascular reconstruction as per BOA / BAPRAS guidelines. Revascularisation using vascular shunts should be performed before skeletal fixation.

How well did you know this?
1
Not at all
2
3
4
5
Perfectly
35
Q

A 23 year old man sustains a severe facial fracture and reconstruction is planned. Which of the following investigations will facilitate pre-operative planning?

	Mandibular tomography
	Magnetic resonance scan of face
	Skull X-ray
	Computerised tomography of the head
	Orthopantomogram
A

Computerised tomography of the head

Significant facial fractures may have intracranial communication. CT scanning will allow delineation of injury extent and 3D reconstruction images can be created. An Orthopantomogram (OPT) will provide good images of mandible and surrounding bony structures but will not give intracranial detail. A skull x-ray lacks the detail for modern practice.

Ocular injuries
Superior orbital fissure syndrome
Severe force to the lateral wall of the orbit resulting in compression of neurovascular structures. Results in :
Complete opthalmoplegia and ptosis (Cranial nerves 3, 4, 6 and nerve to levator palpebrae superioris)
Relative afferent pupillary defect
Dilatation of the pupil and loss of accommodation and corneal reflexes
Altered sensation from forehead to vertex (frontal branch of trigeminal nerve)

Orbital blow out fracture
Typically occurs when an object of slightly larger diameter than the orbital rim strikes the incompressible eyeball. The bone fragment is displaced downwards into the antral cavity, remaining attached to the orbital periosteum. Periorbital fat may be herniated through the defect, interfering with the inferior rectus and inferior oblique muscles which are contained within the same fascial sheath. This prevents upward movement and outward rotation of the eye and the patient experiences diplopia on upward gaze. The initial bruising and swelling may make assessment difficult and patients should usually be reviewed 5 days later. Residual defects may require orbital floor reconstruction.

Nasal Fractures
Common injury
Ensure new and not old deformity
Control epistaxis
CSF rhinorrhoea implies that the cribriform plate has been breached and antibiotics will be required.
Usually best to allow bruising and swelling to settle and then review patient clinically. Major persistent deformity requires fracture manipulation, best performed within 10 days of injury.

Retrobulbar haemorrhage
Rare but important ocular emergency. Presents with:
Pain (usually sharp and within the globe)
Proptosis
Pupil reactions are lost
Paralysis (eye movements lost)
Visual acuity is lost (colour vision is lost first)
May be the result of Le Fort type facial fractures.

Management:
Mannitol 1g/Kg as 20% infusion, Osmotic diuretic, Contra-indicated in congestive heart failure and pulmonary oedema
Acetazolamide 500mg IV, (Monitor FBC/U+E) Reduces aqueous pressure by inhibition of carbonic anhydrase (used in glaucoma)
Dexamethasone 8mg orally or intravenously
In a traumatic setting an urgent cantholysis may be needed prior to definitive surgery.

Consider
Papaverine 40mg smooth muscle relaxant
Dextran 40 500mls IV improves perfusion

How well did you know this?
1
Not at all
2
3
4
5
Perfectly
36
Q

A 32 year old man presents with a painful swelling over the volar aspect of his hand after receiving a hard blow to his palm. On examination, he experiences pain on moving the wrist and on longitudinal compression of the thumb.

A.	Pulled elbow
B.	Fracture of the coronoid process
C.	Scaphoid fracture
D.	Fracture of the distal humerus
E.	Bennets fracture
F.	Fracture of the shaft of the radius and ulnar
G.	Galeazzi fracture
H.	Fracture of the olecranon
I.	Fracture of the radial head
A

Scaphoid fracture

Scaphoid fractures usually occur as a result of direct hard blow to the palm or following a fall on the out-stretched hand. The main physical signs are swelling and tenderness in the anatomical snuff box, and pain on wrist movements and on longitudinal compression of the thumb

Upper limb fractures

Colles’ fracture
Fall onto extended outstretched hands
Described as a dinner fork type deformity
Classical Colles’ fractures have the following 3 features:

Features of the injury

  1. Transverse fracture of the radius
  2. 1 inch proximal to the radio-carpal joint
  3. Dorsal displacement and angulation

Smith’s fracture (reverse Colles’ fracture)
Volar angulation of distal radius fragment (Garden spade deformity)
Caused by falling backwards onto the palm of an outstretched hand or falling with wrists flexed

Bennett’s fracture
Intra-articular fracture of the first carpometacarpal joint
Impact on flexed metacarpal, caused by fist fights
X-ray: triangular fragment at ulnar base of metacarpal

Monteggia’s fracture
Dislocation of the proximal radioulnar joint in association with an ulna fracture
Fall on outstretched hand with forced pronation
Needs prompt diagnosis to avoid disability

Galeazzi fracture
Radial shaft fracture with associated dislocation of the distal radioulnar joint
Occur after a fall on the hand with a rotational force superimposed on it.
On examination, there is bruising, swelling and tenderness over the lower end of the forearm.
X Rays reveal the displaced fracture of the radius and a prominent ulnar head due to dislocation of the inferior radio-ulnar joint.

Barton’s fracture
Distal radius fracture (Colles’/Smith’s) with associated radiocarpal dislocation
Fall onto extended and pronated wrist

Scaphoid fractures
Scaphoid fractures are the commonest carpal fractures.
Surface of scaphoid is covered by articular cartilage with small area available for blood vessels (fracture risks blood supply)
Forms floor of anatomical snuffbox
Risk of fracture associated with fall onto outstretched hand (tubercle, waist, or proximal 1/3)
The main physical signs are swelling and tenderness in the anatomical snuff box, and pain on wrist movements and on longitudinal compression of the thumb.
Ulnar deviation AP needed for visualization of scaphoid
Immobilization of scaphoid fractures difficult

Radial head fracture
Fracture of the radial head is common in young adults.
It is usually caused by a fall on the outstretched hand.
On examination, there is marked local tenderness over the head of the radius, impaired movements at the elbow, and a sharp pain at the lateral side of the elbow at the extremes of rotation (pronation and supination).

How well did you know this?
1
Not at all
2
3
4
5
Perfectly
37
Q

A 26 year old man presents to the emergency department with a swelling over his left elbow after a fall on an outstretched hand. On examination, he has tenderness over the proximal part of his forearm, and has severely restricted supination and pronation movements.

A.	Pulled elbow
B.	Fracture of the coronoid process
C.	Scaphoid fracture
D.	Fracture of the distal humerus
E.	Bennets fracture
F.	Fracture of the shaft of the radius and ulnar
G.	Galeazzi fracture
H.	Fracture of the olecranon
I.	Fracture of the radial head
A

Fracture of the radial head

Fracture of the radial head is common in young adults. It is usually caused by a fall on the outstretched hand. On examination, there is marked local tenderness over the head of the radius, impaired movements at the elbow, and a sharp pain at the lateral side of the elbow at the extremes of rotation (pronation and supination).

Upper limb fractures

Colles’ fracture
Fall onto extended outstretched hands
Described as a dinner fork type deformity
Classical Colles’ fractures have the following 3 features:

Features of the injury

  1. Transverse fracture of the radius
  2. 1 inch proximal to the radio-carpal joint
  3. Dorsal displacement and angulation

Smith’s fracture (reverse Colles’ fracture)
Volar angulation of distal radius fragment (Garden spade deformity)
Caused by falling backwards onto the palm of an outstretched hand or falling with wrists flexed

Bennett’s fracture
Intra-articular fracture of the first carpometacarpal joint
Impact on flexed metacarpal, caused by fist fights
X-ray: triangular fragment at ulnar base of metacarpal

Monteggia’s fracture
Dislocation of the proximal radioulnar joint in association with an ulna fracture
Fall on outstretched hand with forced pronation
Needs prompt diagnosis to avoid disability

Galeazzi fracture
Radial shaft fracture with associated dislocation of the distal radioulnar joint
Occur after a fall on the hand with a rotational force superimposed on it.
On examination, there is bruising, swelling and tenderness over the lower end of the forearm.
X Rays reveal the displaced fracture of the radius and a prominent ulnar head due to dislocation of the inferior radio-ulnar joint.

Barton’s fracture
Distal radius fracture (Colles’/Smith’s) with associated radiocarpal dislocation
Fall onto extended and pronated wrist

Scaphoid fractures
Scaphoid fractures are the commonest carpal fractures.
Surface of scaphoid is covered by articular cartilage with small area available for blood vessels (fracture risks blood supply)
Forms floor of anatomical snuffbox
Risk of fracture associated with fall onto outstretched hand (tubercle, waist, or proximal 1/3)
The main physical signs are swelling and tenderness in the anatomical snuff box, and pain on wrist movements and on longitudinal compression of the thumb.
Ulnar deviation AP needed for visualization of scaphoid
Immobilization of scaphoid fractures difficult

Radial head fracture
Fracture of the radial head is common in young adults.
It is usually caused by a fall on the outstretched hand.
On examination, there is marked local tenderness over the head of the radius, impaired movements at the elbow, and a sharp pain at the lateral side of the elbow at the extremes of rotation (pronation and supination).

How well did you know this?
1
Not at all
2
3
4
5
Perfectly
38
Q

A 56 year old lady presents with a painful swelling over the lower end of the forearm following a fall. Imaging reveals a distal radial fracture with disruption of the distal radio-ulnar joint.

A.	Pulled elbow
B.	Fracture of the coronoid process
C.	Scaphoid fracture
D.	Fracture of the distal humerus
E.	Bennets fracture
F.	Fracture of the shaft of the radius and ulnar
G.	Galeazzi fracture
H.	Fracture of the olecranon
I.	Fracture of the radial head
A

Galeazzi fracture

Galeazzi fractures occur after a fall on the hand with a rotational force superimposed on it. On examination, there is bruising, swelling and tenderness over the lower end of the forearm. X- Rays reveal a displaced fracture of the radius and a prominent ulnar head due to dislocation of the inferior radio-ulnar joint.

Upper limb fractures

Colles’ fracture
Fall onto extended outstretched hands
Described as a dinner fork type deformity
Classical Colles’ fractures have the following 3 features:

Features of the injury

  1. Transverse fracture of the radius
  2. 1 inch proximal to the radio-carpal joint
  3. Dorsal displacement and angulation

Smith’s fracture (reverse Colles’ fracture)
Volar angulation of distal radius fragment (Garden spade deformity)
Caused by falling backwards onto the palm of an outstretched hand or falling with wrists flexed

Bennett’s fracture
Intra-articular fracture of the first carpometacarpal joint
Impact on flexed metacarpal, caused by fist fights
X-ray: triangular fragment at ulnar base of metacarpal

Monteggia’s fracture
Dislocation of the proximal radioulnar joint in association with an ulna fracture
Fall on outstretched hand with forced pronation
Needs prompt diagnosis to avoid disability

Galeazzi fracture
Radial shaft fracture with associated dislocation of the distal radioulnar joint
Occur after a fall on the hand with a rotational force superimposed on it.
On examination, there is bruising, swelling and tenderness over the lower end of the forearm.
X Rays reveal the displaced fracture of the radius and a prominent ulnar head due to dislocation of the inferior radio-ulnar joint.

Barton’s fracture
Distal radius fracture (Colles’/Smith’s) with associated radiocarpal dislocation
Fall onto extended and pronated wrist

Scaphoid fractures
Scaphoid fractures are the commonest carpal fractures.
Surface of scaphoid is covered by articular cartilage with small area available for blood vessels (fracture risks blood supply)
Forms floor of anatomical snuffbox
Risk of fracture associated with fall onto outstretched hand (tubercle, waist, or proximal 1/3)
The main physical signs are swelling and tenderness in the anatomical snuff box, and pain on wrist movements and on longitudinal compression of the thumb.
Ulnar deviation AP needed for visualization of scaphoid
Immobilization of scaphoid fractures difficult

Radial head fracture
Fracture of the radial head is common in young adults.
It is usually caused by a fall on the outstretched hand.
On examination, there is marked local tenderness over the head of the radius, impaired movements at the elbow, and a sharp pain at the lateral side of the elbow at the extremes of rotation (pronation and supination).

How well did you know this?
1
Not at all
2
3
4
5
Perfectly
39
Q

A 42 year old skier falls and impacts his hand on his ski pole. On examination he is tender in the anatomical snuffbox and on bimanual palpation. X-rays with scaphoid views show no evidence of fracture.

A. Admission and surgical debridement
B. Application of futura splint and fracture clinic review
C. Application of tubigrip bandage and fracture clinic review
D. Admission for open reduction and fixation
E. Discharge with reassurance
F. Commence oral prednisolone
G. Commence oral diclofenac

A

Application of futura splint and fracture clinic review

A fracture may still be present and should be immobilised until repeat imaging can be performed. If clinical suspicion persists then subsequent imaging should be with MRI scanning or CT if MRI is contra-indicated.

Scaphoid fractures:
80% of all carpal fractures
80% occur in men
80% occur at the waist of the scaphoid

Scaphoid fractures are the commonest carpal fractures.
Surface of scaphoid is covered by articular cartilage with small area available for blood vessels (fracture risks blood supply)
Forms floor of anatomical snuffbox
Risk of fracture associated with fall onto outstretched hand (tubercle, waist, or proximal third)
Ulnar deviation AP needed for visualization of scaphoid
Immobilization of scaphoid fractures difficult

Complications
Non union of scaphoid
Avascular necrosis of the scaphoid
Scapholunate disruption and wrist collapse
Degenerative changes of the adjacent joint

How well did you know this?
1
Not at all
2
3
4
5
Perfectly
40
Q

A 43 year old man falls over landing on his left hand. Although there was anatomical snuffbox tenderness no x-rays either at the time or subsequently have shown evidence of scaphoid fracture. He has been immobilised in a futura splint for two weeks and is now asymptomatic.

A. Admission and surgical debridement
B. Application of futura splint and fracture clinic review
C. Application of tubigrip bandage and fracture clinic review
D. Admission for open reduction and fixation
E. Discharge with reassurance
F. Commence oral prednisolone
G. Commence oral diclofenac

A

Discharge with reassurance

This patient is at extremely low risk of having sustained a scaphoid injury and may be discharged.

Scaphoid fractures:
80% of all carpal fractures
80% occur in men
80% occur at the waist of the scaphoid

Scaphoid fractures are the commonest carpal fractures.
Surface of scaphoid is covered by articular cartilage with small area available for blood vessels (fracture risks blood supply)
Forms floor of anatomical snuffbox
Risk of fracture associated with fall onto outstretched hand (tubercle, waist, or proximal third)
Ulnar deviation AP needed for visualization of scaphoid
Immobilization of scaphoid fractures difficult

Complications
Non union of scaphoid
Avascular necrosis of the scaphoid
Scapholunate disruption and wrist collapse
Degenerative changes of the adjacent joint

How well did you know this?
1
Not at all
2
3
4
5
Perfectly
41
Q

A builder falls from scaffolding and lands on his left hand he suffers a severe laceration to his palm. An x-ray shows evidence of scaphoid fracture that is minimally displaced.

A. Admission and surgical debridement
B. Application of futura splint and fracture clinic review
C. Application of tubigrip bandage and fracture clinic review
D. Admission for open reduction and fixation
E. Discharge with reassurance
F. Commence oral prednisolone
G. Commence oral diclofenac

A

Admission and surgical debridement

This is technically an open fracture and should be debrided prior to attempted fixation (which should occur soon after).

Scaphoid fractures:
80% of all carpal fractures
80% occur in men
80% occur at the waist of the scaphoid

Scaphoid fractures are the commonest carpal fractures.
Surface of scaphoid is covered by articular cartilage with small area available for blood vessels (fracture risks blood supply)
Forms floor of anatomical snuffbox
Risk of fracture associated with fall onto outstretched hand (tubercle, waist, or proximal third)
Ulnar deviation AP needed for visualization of scaphoid
Immobilization of scaphoid fractures difficult

Complications
Non union of scaphoid
Avascular necrosis of the scaphoid
Scapholunate disruption and wrist collapse
Degenerative changes of the adjacent joint

How well did you know this?
1
Not at all
2
3
4
5
Perfectly
42
Q

A 4 year boy presents with an abnormal gait. He has a history of recent viral illness. His WCC is 11 and ESR is 30.

A.	Musculoskeletal pain
B.	Congenital dysplasia of the hip
C.	Slipped upper femoral epiphysis
D.	Transient synovitis
E.	Septic arthritis
F.	Perthes disease
G.	Tibial fracture
A

Transient synovitis

Viral illnesses can be associated with transient synovitis. The WCC should ideally be > 12 and the ESR > 40 to suggest septic arthritis.

How well did you know this?
1
Not at all
2
3
4
5
Perfectly
43
Q

A 6 year old boy presents with groin pain. He is known to be disruptive in class. He reports that he is bullied for being short. On examination he has an antalgic gait and pain on internal rotation of the right hip.

A.	Musculoskeletal pain
B.	Congenital dysplasia of the hip
C.	Slipped upper femoral epiphysis
D.	Transient synovitis
E.	Septic arthritis
F.	Perthes disease
G.	Tibial fracture
A

Perthes disease

This child is short, has hyperactivity (disruptive behaviour) and is within the age range for Perthes disease. Hyperactivity and short stature are associated with Perthes disease.

How well did you know this?
1
Not at all
2
3
4
5
Perfectly
44
Q

An obese 12 year old boy is referred with pain in the left knee and hip. On examination he has an antaglic gait and limitation of internal rotation. His knee has normal range of passive and active movement.

A.	Musculoskeletal pain
B.	Congenital dysplasia of the hip
C.	Slipped upper femoral epiphysis
D.	Transient synovitis
E.	Septic arthritis
F.	Perthes disease
G.	Tibial fracture
A

Slipped upper femoral epiphysis

Slipped upper femoral epiphysis is commonest in obese adolescent males. The x-ray will show displacement of the femoral epiphysis inferolaterally. Treatment is usually with rest and non weight bearing crutches.

How well did you know this?
1
Not at all
2
3
4
5
Perfectly
45
Q

A 63 year old man is admitted with rest pain and foot ulceration. An angiogram shows a 3 cm area of occlusion of the distal superficial femoral artery with 3 vessel run off. His ankle - brachial pressure index is 0.4.

A.	Primary amputation
B.	Angioplasty
C.	Arterial bypass surgery using vein
D.	Arterial bypass surgery using PTFE
E.	Conservative management with medical therapy and exercise
F.	Watch and wait
G.	Duplex scanning
A

Angioplasty

Short segment disease and good run off with tissue loss is a compelling indication for angioplasty. He should receive aspirin and a statin if not already taking them.

Peripheral vascular disease

Indications for surgery to revascularise the lower limb
Intermittent claudication
Critical ischaemia
Ulceration
Gangrene

Intermittent claudication that is not disabling may provide a relative indication, whilst the other complaints are often absolute indications depending upon the frailty of the patient.

Assessment
Clinical examination
Ankle brachial pressure index measurement
Duplex arterial ultrasound
Angiography (standard, CT or MRI): usually performed only if intervention being considered.

Angioplasty
In order for angioplasty to be undertaken successfully the artery has to be accessible. The lesion relatively short and reasonable distal vessel runoff. Longer lesions may be amenable to sub-intimal angioplasty.

Surgery
Surgery will be undertaken where attempts at angioplasty have either failed or are unsuitable. Bypass essentially involves bypassing the affected arterial segment by utilising a graft to run from above the disease to below the disease. As with angioplasty good runoff improves the outcome.

Some key concepts with bypass surgery

Superficial femoral artery occlusion to the above knee popliteal
In the ideal scenario, vein (either in situ or reversed LSV) would the used as a conduit. However, prosthetic material has reasonable 5 year patency rates and some would advocate using this in preference to vein so that vein can be used for other procedures in the future. In general terms either technique is usually associated with an excellent outcome (if run off satisfactory).

Procedure
Artery dissected out, IV heparin 3,000 units given and then the vessels are cross clamped
Longitudinal arteriotomy
Graft cut to size and tunneled to arteriotomy sites
Anastomosis to femoral artery usually with 5/0 ‘double ended’ Prolene suture
Distal anastomosis usually using 6/0 ‘double ended’ Prolene

Distal disease
Femoro-distal bypass surgery takes longer to perform, is more technically challenging and has higher failure rates.
In elderly diabetic patients with poor runoff a primary amputation may well be a safer and more effective option. There is no point in embarking on this type of surgery in patients who are wheelchair bound.
In femorodistal bypasses vein gives superior outcomes to PTFE.

Rules
Vein mapping 1st to see whether there is suitable vein (the preferred conduit). Sub intimal hyperplasia occurs early when PTFE is used for the distal anastomosis and will lead to early graft occlusion and failure.
Essential operative procedure as for above knee fem-pop.
If there is insufficient vein for the entire conduit then vein can be attached to the end of the PTFE graft and then used for the distal anastomosis. This type of ‘vein boot’ is technically referred to as a Miller Cuff and is associated with better patency rates than PTFE alone.
Remember the more distal the arterial anastomosis the lower the success rate.

How well did you know this?
1
Not at all
2
3
4
5
Perfectly
46
Q

A 72 year old man present in the vascular clinic with calf pain present on walking 100 yards. He is an ex-smoker and lives alone. On examination he has reasonable leg pulses. His right dorsalis pedis pulse gives a monophasic doppler signal with an ankle brachial pressure index measurement of 0.7. All other pressures are acceptable. There is no evidence of ulceration or gangrene.

A.	Primary amputation
B.	Angioplasty
C.	Arterial bypass surgery using vein
D.	Arterial bypass surgery using PTFE
E.	Conservative management with medical therapy and exercise
F.	Watch and wait
G.	Duplex scanning
A

Conservative management with medical therapy and exercise

Structured exercise programmes combined with medical therapy will improve many patients. Should his symptoms worsen or fail to improve then imaging with duplex scanning would be required.

Peripheral vascular disease

Indications for surgery to revascularise the lower limb
Intermittent claudication
Critical ischaemia
Ulceration
Gangrene

Intermittent claudication that is not disabling may provide a relative indication, whilst the other complaints are often absolute indications depending upon the frailty of the patient.

Assessment
Clinical examination
Ankle brachial pressure index measurement
Duplex arterial ultrasound
Angiography (standard, CT or MRI): usually performed only if intervention being considered.

Angioplasty
In order for angioplasty to be undertaken successfully the artery has to be accessible. The lesion relatively short and reasonable distal vessel runoff. Longer lesions may be amenable to sub-intimal angioplasty.

Surgery
Surgery will be undertaken where attempts at angioplasty have either failed or are unsuitable. Bypass essentially involves bypassing the affected arterial segment by utilising a graft to run from above the disease to below the disease. As with angioplasty good runoff improves the outcome.

Some key concepts with bypass surgery

Superficial femoral artery occlusion to the above knee popliteal
In the ideal scenario, vein (either in situ or reversed LSV) would the used as a conduit. However, prosthetic material has reasonable 5 year patency rates and some would advocate using this in preference to vein so that vein can be used for other procedures in the future. In general terms either technique is usually associated with an excellent outcome (if run off satisfactory).

Procedure
Artery dissected out, IV heparin 3,000 units given and then the vessels are cross clamped
Longitudinal arteriotomy
Graft cut to size and tunneled to arteriotomy sites
Anastomosis to femoral artery usually with 5/0 ‘double ended’ Prolene suture
Distal anastomosis usually using 6/0 ‘double ended’ Prolene

Distal disease
Femoro-distal bypass surgery takes longer to perform, is more technically challenging and has higher failure rates.
In elderly diabetic patients with poor runoff a primary amputation may well be a safer and more effective option. There is no point in embarking on this type of surgery in patients who are wheelchair bound.
In femorodistal bypasses vein gives superior outcomes to PTFE.

Rules
Vein mapping 1st to see whether there is suitable vein (the preferred conduit). Sub intimal hyperplasia occurs early when PTFE is used for the distal anastomosis and will lead to early graft occlusion and failure.
Essential operative procedure as for above knee fem-pop.
If there is insufficient vein for the entire conduit then vein can be attached to the end of the PTFE graft and then used for the distal anastomosis. This type of ‘vein boot’ is technically referred to as a Miller Cuff and is associated with better patency rates than PTFE alone.
Remember the more distal the arterial anastomosis the lower the success rate.

How well did you know this?
1
Not at all
2
3
4
5
Perfectly
47
Q

An 83 year old lady is admitted from a nursing home with infected lower leg ulcers. She underwent an attempted long superficial femoral artery sub initimal angioplasty 2 weeks previously. This demonstrated poor runoff below the knee.

A.	Primary amputation
B.	Angioplasty
C.	Arterial bypass surgery using vein
D.	Arterial bypass surgery using PTFE
E.	Conservative management with medical therapy and exercise
F.	Watch and wait
G.	Duplex scanning
A

Primary amputation

Poor runoff and sepsis would equate to poor outcome with attempted bypass surgery.

Peripheral vascular disease

Indications for surgery to revascularise the lower limb
Intermittent claudication
Critical ischaemia
Ulceration
Gangrene

Intermittent claudication that is not disabling may provide a relative indication, whilst the other complaints are often absolute indications depending upon the frailty of the patient.

Assessment
Clinical examination
Ankle brachial pressure index measurement
Duplex arterial ultrasound
Angiography (standard, CT or MRI): usually performed only if intervention being considered.

Angioplasty
In order for angioplasty to be undertaken successfully the artery has to be accessible. The lesion relatively short and reasonable distal vessel runoff. Longer lesions may be amenable to sub-intimal angioplasty.

Surgery
Surgery will be undertaken where attempts at angioplasty have either failed or are unsuitable. Bypass essentially involves bypassing the affected arterial segment by utilising a graft to run from above the disease to below the disease. As with angioplasty good runoff improves the outcome.

Some key concepts with bypass surgery

Superficial femoral artery occlusion to the above knee popliteal
In the ideal scenario, vein (either in situ or reversed LSV) would the used as a conduit. However, prosthetic material has reasonable 5 year patency rates and some would advocate using this in preference to vein so that vein can be used for other procedures in the future. In general terms either technique is usually associated with an excellent outcome (if run off satisfactory).

Procedure
Artery dissected out, IV heparin 3,000 units given and then the vessels are cross clamped
Longitudinal arteriotomy
Graft cut to size and tunneled to arteriotomy sites
Anastomosis to femoral artery usually with 5/0 ‘double ended’ Prolene suture
Distal anastomosis usually using 6/0 ‘double ended’ Prolene

Distal disease
Femoro-distal bypass surgery takes longer to perform, is more technically challenging and has higher failure rates.
In elderly diabetic patients with poor runoff a primary amputation may well be a safer and more effective option. There is no point in embarking on this type of surgery in patients who are wheelchair bound.
In femorodistal bypasses vein gives superior outcomes to PTFE.

Rules
Vein mapping 1st to see whether there is suitable vein (the preferred conduit). Sub intimal hyperplasia occurs early when PTFE is used for the distal anastomosis and will lead to early graft occlusion and failure.
Essential operative procedure as for above knee fem-pop.
If there is insufficient vein for the entire conduit then vein can be attached to the end of the PTFE graft and then used for the distal anastomosis. This type of ‘vein boot’ is technically referred to as a Miller Cuff and is associated with better patency rates than PTFE alone.
Remember the more distal the arterial anastomosis the lower the success rate.

How well did you know this?
1
Not at all
2
3
4
5
Perfectly
48
Q

A 28 year old man falls on the back of his hand. On x-ray he has a fractured distal radius demonstrating volar displacement of the fracture.

A.	Smith's
B.	Bennett's
C.	Monteggia's
D.	Colles'
E.	Galeazzi
F.	Pott's
G.	Barton's
A

Smith’s

This is a Smith fracture (reverse Colles’ fracture); unlike a Colles’ this is a high velocity injury and may require surgical correction. Note that Colles’ fractures are usually dorsally displaced.

Eponymous fractures

Colles’ fracture (dinner fork deformity)
Fall onto extended outstretched hand
Classical Colles’ fractures have the following 3 features:

  1. Transverse fracture of the radius
  2. 1 inch proximal to the radio-carpal joint
  3. Dorsal displacement and angulation

Smith’s fracture (reverse Colles’ fracture)
Volar angulation of distal radius fragment (Garden spade deformity)
Caused by falling backwards onto the palm of an outstretched hand or falling with wrists flexed

Bennett’s fracture
Intra-articular fracture of the first carpometacarpal joint
Impact on flexed metacarpal, caused by fist fights
X-ray: triangular fragment at ulnar base of metacarpal

Image sourced from Wikipedia
Monteggia’s fracture
Dislocation of the proximal radioulnar joint in association with an ulna fracture
Fall on outstretched hand with forced pronation
Needs prompt diagnosis to avoid disability

Image sourced from Wikipedia
Galeazzi fracture
Radial shaft fracture with associated dislocation of the distal radioulnar joint
Direct blow

Pott’s fracture
Bimalleolar ankle fracture
Forced foot eversion

Barton’s fracture
Distal radius fracture (Colles’/Smith’s) with associated radiocarpal dislocation
Fall onto extended and pronated wrist
Involvement of the joint is a defining feature

How well did you know this?
1
Not at all
2
3
4
5
Perfectly
49
Q

A 38 year old window cleaner falls from his ladder. He lands on his left arm and notices an obvious injury. An x-ray and clinical examination demonstrate that he has a fracture of the proximal ulna and associated radial dislocation.

A.	Smith's
B.	Bennett's
C.	Monteggia's
D.	Colles'
E.	Galeazzi
F.	Pott's
G.	Barton's
A

Monteggia’s

This constellation of injuries is referred to as a Monteggia’s fracture.

Eponymous fractures

Colles’ fracture (dinner fork deformity)
Fall onto extended outstretched hand
Classical Colles’ fractures have the following 3 features:

  1. Transverse fracture of the radius
  2. 1 inch proximal to the radio-carpal joint
  3. Dorsal displacement and angulation

Smith’s fracture (reverse Colles’ fracture)
Volar angulation of distal radius fragment (Garden spade deformity)
Caused by falling backwards onto the palm of an outstretched hand or falling with wrists flexed

Bennett’s fracture
Intra-articular fracture of the first carpometacarpal joint
Impact on flexed metacarpal, caused by fist fights
X-ray: triangular fragment at ulnar base of metacarpal

Image sourced from Wikipedia
Monteggia’s fracture
Dislocation of the proximal radioulnar joint in association with an ulna fracture
Fall on outstretched hand with forced pronation
Needs prompt diagnosis to avoid disability

Image sourced from Wikipedia
Galeazzi fracture
Radial shaft fracture with associated dislocation of the distal radioulnar joint
Direct blow

Pott’s fracture
Bimalleolar ankle fracture
Forced foot eversion

Barton’s fracture
Distal radius fracture (Colles’/Smith’s) with associated radiocarpal dislocation
Fall onto extended and pronated wrist
Involvement of the joint is a defining feature

How well did you know this?
1
Not at all
2
3
4
5
Perfectly
50
Q

A 32 year old man falls from scaffolding and sustains an injury to his forearm. Clinical examination and x-ray shows that he has sustained a radial fracture with dislocation of the distal radio-ulna joint.

A.	Smith's
B.	Bennett's
C.	Monteggia's
D.	Colles'
E.	Galeazzi
F.	Pott's
G.	Barton's
A

Galeazzi

Isolated fracture of the radius alone can occur but is rare. Always check for associated injury.

Eponymous fractures

Colles’ fracture (dinner fork deformity)
Fall onto extended outstretched hand
Classical Colles’ fractures have the following 3 features:

  1. Transverse fracture of the radius
  2. 1 inch proximal to the radio-carpal joint
  3. Dorsal displacement and angulation

Smith’s fracture (reverse Colles’ fracture)
Volar angulation of distal radius fragment (Garden spade deformity)
Caused by falling backwards onto the palm of an outstretched hand or falling with wrists flexed

Bennett’s fracture
Intra-articular fracture of the first carpometacarpal joint
Impact on flexed metacarpal, caused by fist fights
X-ray: triangular fragment at ulnar base of metacarpal

Image sourced from Wikipedia
Monteggia’s fracture
Dislocation of the proximal radioulnar joint in association with an ulna fracture
Fall on outstretched hand with forced pronation
Needs prompt diagnosis to avoid disability

Image sourced from Wikipedia
Galeazzi fracture
Radial shaft fracture with associated dislocation of the distal radioulnar joint
Direct blow

Pott’s fracture
Bimalleolar ankle fracture
Forced foot eversion

Barton’s fracture
Distal radius fracture (Colles’/Smith’s) with associated radiocarpal dislocation
Fall onto extended and pronated wrist
Involvement of the joint is a defining feature

How well did you know this?
1
Not at all
2
3
4
5
Perfectly
51
Q

A 62 year old woman presents with acute bowel obstruction. She has been vomiting up to 15 times a day and is taking erythromycin. She suddenly complains of dizziness. Her ECG shows torsades de pointes. What is the management of choice?

	IV Atropine
	IV Potassium
	IV Magnesium sulphate
	IV Bicarbonate
	IV Adrenaline
A

Torsades de pointes: Treatment IV magnesium sulphate

This woman is likely to have hypokalaemia and hypomagnasaemia as a result of vomiting. In addition to this, the erythromycin will predispose her to torsades de pointes. The patient needs Magnesium 2g over 10 minutes. Knowledge of the management of this peri arrest diagnosis is hence important in surgical practice.

Torsades de pointes

Torsades de pointes (‘twisting of the points’) is a rare arrhythmia associated with a long QT interval. It may deteriorate into ventricular fibrillation and hence lead to sudden death

Causes of long QT interval
congenital: Jervell-Lange-Nielsen syndrome, Romano-Ward syndrome
antiarrhythmics: amiodarone, sotalol, class 1a antiarrhythmic drugs
tricyclic antidepressants
antipsychotics
chloroquine
terfenadine
erythromycin
electrolyte: hypocalcaemia, hypokalaemia, hypomagnesaemia
myocarditis
hypothermia
subarachnoid haemorrhage

Management
IV magnesium sulphate

How well did you know this?
1
Not at all
2
3
4
5
Perfectly
52
Q

A 27 year old man sustains a single gunshot wound to the left thigh. In the emergency department, he is noted to have a large haematoma of his medial thigh. He complains of parasthesia in his foot. On examination, there are weak pulses palpable distal to the injury and the patient is unable to move his foot. The appropriate initial management of this patient is:

	Conventional angiography
	Immediate exploration and repair
	Fasciotomy of the anterior compartment
	Observation for resolution of spasm
	Local wound exploration
A

Immediate exploration and repair

The five P’s of arterial injury include pain, parasthesias, pallor, pulselessness and paralysis. In the extremities, the tissues most sensitive to anoxia are the peripheral nerves and striated muscle. The early developments of paresthesias and paralysis are signals that there is significant ischemia present, and immediate exploration and repair are warranted. The presence of palpable pulse does not exclude an arterial injury because this presence may represent a transmitted pulsation through a blood clot. When severe ischemia is present, the repair must be completed within 6 to 8 h to prevent irreversible muscle ischemia and loss of limb function. Delay to obtain a conventional angiogram or to observe for change needlessly prolongs the ischemic time. A CT angiogram may be a reasonable alternative. Fasciotomy may be required but should be done in conjunction with and after re-establishment of arterial flow. Local wound exploration is not recommended because brisk hemorrhage may be encountered without the securing of prior vascular control.

How well did you know this?
1
Not at all
2
3
4
5
Perfectly
53
Q

A 54-year-old man presents to the Emergency Department with a 2 day history of a swollen, painful left knee. You aspirate the joint to avoid admission to the orthopaedic wards. Aspirated joint fluid shows calcium pyrophosphate crystals. Which of the following blood tests is most useful in revealing an underlying cause?

	Transferrin saturation
	ACTH
	ANA
	Serum ferritin
	LDH
A

This is a typical presentation of pseudogout. An elevated transferrin saturation may indicate haemochromatosis, a recognised cause of pseudogout.

A high ferritin level is also seen in haemochromatosis but can be raised in a variety of infective and inflammatory processes, including pseudogout, as part of an acute phase response.

Pseudogout

Pseudogout is a form of microcrystal synovitis caused by the deposition of calcium pyrophosphate dihydrate in the synovium

Risk factors
hyperparathyroidism
hypothyroidism
haemochromatosis
acromegaly
low magnesium, low phosphate
Wilson's disease

Features
knee, wrist and shoulders most commonly affected
joint aspiration: weakly-positively birefringent rhomboid shaped crystals
x-ray: chondrocalcinosis

Management
aspiration of joint fluid, to exclude septic arthritis
NSAIDs or intra-articular, intra-muscular or oral steroids as for gout

How well did you know this?
1
Not at all
2
3
4
5
Perfectly
54
Q

A 3 year old child inserts a crayon into their external auditory meatus. Attempts to remove it have not been successful.

A.	Manage conservatively
B.	Immediate emergency theatre
C.	Treat in emergency department
D.	Treat in emergency department under sedation
E.	Operate on next emergency list
A

Operate on next emergency list

They would not tolerate removal in the emergency department. The tympanic membrane should be carefully inspected and again this will be easier under general anaesthesia.

Management of acute cases- Paediatric

Children will often insert objects into orifices such as the nose and external auditory meatus
Assessment includes assessment of airway and haemodynamic status
Where the airway is not immediately threatened decisions can be made as to whether to manage in the emergency department or transfer to theatre
In general children do not tolerate procedures well and it is usually safer to remove objects in theatre and under general anaesthesia with a secure airway
A chest x-ray is required to ensure that no object is present in the chest, not all objects are radioopaque. However, signs such as focal consolidation may indicate small airway obstruction
In the case of small bore missile injuries the decision relating to surgery depends on the size of the missile and its location. Airgun pellets are a common culprit, if there is a long time interval between the incident and presentation and the object has not caused any significant problems then it may be best left alone
Airgun pellets (and glass) lodged in the soft tissues are usually notoriously difficult to localise and extract, no matter how superficial. Removal in theatre is usually the best option. If the object is radioopaque then an image intensifier should be used

How well did you know this?
1
Not at all
2
3
4
5
Perfectly
55
Q

A 2 year old accidentally inhales a peanut. They arrive in the emergency department extremely distressed and cyanotic. Imaging shows it to be lodged in the left main bronchus.

A.	Manage conservatively
B.	Immediate emergency theatre
C.	Treat in emergency department
D.	Treat in emergency department under sedation
E.	Operate on next emergency list
A

Immediate emergency theatre

As they are cyanosed it requires immediate removal and this should be undertaken in a fully staffed theatre. Ideally a rigid bronchoscopy should be performed.

Management of acute cases- Paediatric

Children will often insert objects into orifices such as the nose and external auditory meatus
Assessment includes assessment of airway and haemodynamic status
Where the airway is not immediately threatened decisions can be made as to whether to manage in the emergency department or transfer to theatre
In general children do not tolerate procedures well and it is usually safer to remove objects in theatre and under general anaesthesia with a secure airway
A chest x-ray is required to ensure that no object is present in the chest, not all objects are radioopaque. However, signs such as focal consolidation may indicate small airway obstruction
In the case of small bore missile injuries the decision relating to surgery depends on the size of the missile and its location. Airgun pellets are a common culprit, if there is a long time interval between the incident and presentation and the object has not caused any significant problems then it may be best left alone
Airgun pellets (and glass) lodged in the soft tissues are usually notoriously difficult to localise and extract, no matter how superficial. Removal in theatre is usually the best option. If the object is radioopaque then an image intensifier should be used

How well did you know this?
1
Not at all
2
3
4
5
Perfectly
56
Q

A 10 year old boy is shot in the head with an airgun pellet. He is concerned that he will get into trouble and the injury remains concealed for 10 days. Imaging using CT scanning shows it to be lodged in the frontal lobe.

A.	Manage conservatively
B.	Immediate emergency theatre
C.	Treat in emergency department
D.	Treat in emergency department under sedation
E.	Operate on next emergency list
A

Manage conservatively

The pellet is small and no serious injury has occurred at this stage. This should therefore be managed conservatively.

Management of acute cases- Paediatric

Children will often insert objects into orifices such as the nose and external auditory meatus
Assessment includes assessment of airway and haemodynamic status
Where the airway is not immediately threatened decisions can be made as to whether to manage in the emergency department or transfer to theatre
In general children do not tolerate procedures well and it is usually safer to remove objects in theatre and under general anaesthesia with a secure airway
A chest x-ray is required to ensure that no object is present in the chest, not all objects are radioopaque. However, signs such as focal consolidation may indicate small airway obstruction
In the case of small bore missile injuries the decision relating to surgery depends on the size of the missile and its location. Airgun pellets are a common culprit, if there is a long time interval between the incident and presentation and the object has not caused any significant problems then it may be best left alone
Airgun pellets (and glass) lodged in the soft tissues are usually notoriously difficult to localise and extract, no matter how superficial. Removal in theatre is usually the best option. If the object is radioopaque then an image intensifier should be used

How well did you know this?
1
Not at all
2
3
4
5
Perfectly
57
Q

A 66 year old man is referred via the aneurysm screening programme with an abdominal aortic aneurysm measuring 4.4 cm. Apart from well controlled type 2 DM he is otherwise well

A.	Immediate laparotomy
B.	Immediate CT
C.	AAA repair during next 48 hours
D.	USS in 6 months
E.	CT scan during next 4 weeks
F.	Endovascular aortic aneurysm repair
G.	Discharge
H.	Palliate
I.	None of the above
A

USS in 6 months

At this point continue with ultrasound surveillance

Abdominal aorta aneurysm

Abdominal aortic aneurysms are a common problem in vascular surgery.
They may occur as either true or false aneurysm. With the former all 3 layers of the arterial wall are involved, in the latter only a single layer of fibrous tissue forms the aneurysm wall.
True abdominal aortic aneurysms have an approximate incidence of 0.06 per 1000 people. They are commonest in elderly men and for this reason the UK is now introducing the aneurysm screening program with the aim of performing an abdominal aortic ultrasound measurement in all men aged 65 years.

Causes
Several different groups of patients suffer from aneurysmal disease.
The commonest group is those who suffer from standard arterial disease, i.e. Those who are hypertensive and have been or are smokers.
Other patients such as those suffering from connective tissue diseases such as Marfan’s may also develop aneurysms. In patients with abdominal aortic aneurysms the extracellular matrix becomes disrupted with a change in the balance of collagen and elastic fibres.

Management
Most abdominal aortic aneurysms are an incidental finding.
Symptoms most often relate to rupture or impending rupture.
20% rupture anteriorly into the peritoneal cavity. Very poor prognosis.
80% rupture posteriorly into the retroperitoneal space
The risk of rupture is related to aneurysm size, only 2% of aneurysms measuring less than 4cm in diameter will rupture over a 5 year period. This contrasts with 75% of aneurysms measuring over 7cm in diameter.
This is well explained by Laplaces’ law which relates size to transmural pressure.
For this reason most vascular surgeons will subject patients with an aneurysm size of 5cm or greater to CT scanning of the chest, abdomen and pelvis with the aim of delineating anatomy and planning treatment. Depending upon co-morbidities, surgery is generally offered once the aneurysm is between 5.5cm and 6cm.

A CT reconstruction showing an infrarenal abdominal aortic aneurysm. The walls of the sac are calcified which may facilitate identification on plain x-rays

Image sourced from Wikipedia

Indications for surgery
Symptomatic aneurysms (80% annual mortality if untreated)
Increasing size above 5.5cm if asymptomatic
Rupture (100% mortality without surgery)

Surgical procedures
Abdominal aortic aneurysm repair

Procedure:

GA
Invasive monitoring (A-line, CVP, catheter)
Incision: Midline or transverse
Bowel and distal duodenum mobilised to access aorta.
Aneurysm neck and base dissected out and prepared for cross clamp
Systemic heparinisation
Cross clamp (proximal first)
Longitudinal aortotomy
Atherectomy
Deal with back bleeding from lumbar vessels and inferior mesenteric artery
Insert graft either tube or bifurcated depending upon anatomy
Suture using Prolene (3/0 for proximal , distal anastomosis suture varies according to site)
Clamps off: End tidal CO2 will rise owing to effects of reperfusion, at this point major risk of myocardial events.
Haemostasis
Closure of aneurysm sac to minimise risk of aorto-enteric fistula
Closure: Loop 1 PDS or Prolene to abdominal wall
Skin- surgeons preference

Post operatively:

ITU (Almost all)
Greatest risk of complications following emergency repair
Complications: Embolic- gut and foot infarcts
Cardiac - owing to premorbid states, re-perfusion injury and effects of cross clamp
Wound problems
Later risks related to graft- infection and aorto-enteric fistula

Special groups

Supra renal AAA
These patients will require a supra renal clamp and this carries a far higher risk of complications and risk of renal failure.

Ruptured AAA
Pre-operatively the management depends upon haemodynamic instability. In patients with symptoms of rupture (typical pain, haemodynamic compromise and risk factors) then ideally prompt laparotomy. In those with vague symptoms and haemodynamic stability the ideal test is CT scan to determine whether rupture has occurred or not. Most common rupture site is retroperitoneal 80%. These patients will tend to develop retroperitoneal haematoma. This can be disrupted if Bp is allowed to rise too high so aim for Bp 100mmHg.
Operative details are similar to elective repair although surgery should be swift, blind rushing often makes the situation worse. Plunging vascular clamps blindly into a pool of blood at the aneurysm neck carries the risk of injury the vena cava that these patients do not withstand. Occasionally a supracoeliac clamp is needed to effect temporary control, although leaving this applied for more than 20 minutes tends to carry a dismal outcome.

EVAR
Increasingly patients are now being offered endovascular aortic aneurysm repair. This is undertaken by surgeons and radiologists working jointly. The morphology of the aneurysm is important and not all are suitable. Here is a typical list of those features favoring a suitable aneurysm:
Long neck
Straight iliac vessels
Healthy groin vessels

Clearly few AAA patients possess the above and compromise has to be made. The use of fenestrated grafts can allow supra renal AAA to be treated.

Procedure:

GA
Radiology or theatre
Bilateral groin incisions
Common femoral artery dissected out
Heparinisation
Arteriotomy and insertion of guide wire
Dilation of arteriotomy
Insertion of EVAR Device
Once in satisfactory position it is released
Arteriotomy closed once check angiogram shows good position and no endoleak

Complications:
Endoleaks depending upon site are either Type I or 2. These may necessitate re-intervention and all EVAR patients require follow up . Details are not needed for MRCS.

How well did you know this?
1
Not at all
2
3
4
5
Perfectly
58
Q

A 72 year old man has a CT scan for abdominal discomfort and the surgeon suspects AAA. This shows a 6.6cm aneurysm with a 3.5cm neck and it continues to involve the right common iliac. The left iliac is occluded. He is hypertensive and has Type 2 DM which is well controlled.

A.	Immediate laparotomy
B.	Immediate CT
C.	AAA repair during next 48 hours
D.	USS in 6 months
E.	CT scan during next 4 weeks
F.	Endovascular aortic aneurysm repair
G.	Discharge
H.	Palliate
I.	None of the above
A

AAA repair during next 48 hours

Assuming he is fit enough. This would be a typical ‘open ‘ case as the marked iliac disease would make EVAR difficult

Abdominal aorta aneurysm

Abdominal aortic aneurysms are a common problem in vascular surgery.
They may occur as either true or false aneurysm. With the former all 3 layers of the arterial wall are involved, in the latter only a single layer of fibrous tissue forms the aneurysm wall.
True abdominal aortic aneurysms have an approximate incidence of 0.06 per 1000 people. They are commonest in elderly men and for this reason the UK is now introducing the aneurysm screening program with the aim of performing an abdominal aortic ultrasound measurement in all men aged 65 years.

Causes
Several different groups of patients suffer from aneurysmal disease.
The commonest group is those who suffer from standard arterial disease, i.e. Those who are hypertensive and have been or are smokers.
Other patients such as those suffering from connective tissue diseases such as Marfan’s may also develop aneurysms. In patients with abdominal aortic aneurysms the extracellular matrix becomes disrupted with a change in the balance of collagen and elastic fibres.

Management
Most abdominal aortic aneurysms are an incidental finding.
Symptoms most often relate to rupture or impending rupture.
20% rupture anteriorly into the peritoneal cavity. Very poor prognosis.
80% rupture posteriorly into the retroperitoneal space
The risk of rupture is related to aneurysm size, only 2% of aneurysms measuring less than 4cm in diameter will rupture over a 5 year period. This contrasts with 75% of aneurysms measuring over 7cm in diameter.
This is well explained by Laplaces’ law which relates size to transmural pressure.
For this reason most vascular surgeons will subject patients with an aneurysm size of 5cm or greater to CT scanning of the chest, abdomen and pelvis with the aim of delineating anatomy and planning treatment. Depending upon co-morbidities, surgery is generally offered once the aneurysm is between 5.5cm and 6cm.

A CT reconstruction showing an infrarenal abdominal aortic aneurysm. The walls of the sac are calcified which may facilitate identification on plain x-rays

Image sourced from Wikipedia

Indications for surgery
Symptomatic aneurysms (80% annual mortality if untreated)
Increasing size above 5.5cm if asymptomatic
Rupture (100% mortality without surgery)

Surgical procedures
Abdominal aortic aneurysm repair

Procedure:

GA
Invasive monitoring (A-line, CVP, catheter)
Incision: Midline or transverse
Bowel and distal duodenum mobilised to access aorta.
Aneurysm neck and base dissected out and prepared for cross clamp
Systemic heparinisation
Cross clamp (proximal first)
Longitudinal aortotomy
Atherectomy
Deal with back bleeding from lumbar vessels and inferior mesenteric artery
Insert graft either tube or bifurcated depending upon anatomy
Suture using Prolene (3/0 for proximal , distal anastomosis suture varies according to site)
Clamps off: End tidal CO2 will rise owing to effects of reperfusion, at this point major risk of myocardial events.
Haemostasis
Closure of aneurysm sac to minimise risk of aorto-enteric fistula
Closure: Loop 1 PDS or Prolene to abdominal wall
Skin- surgeons preference

Post operatively:

ITU (Almost all)
Greatest risk of complications following emergency repair
Complications: Embolic- gut and foot infarcts
Cardiac - owing to premorbid states, re-perfusion injury and effects of cross clamp
Wound problems
Later risks related to graft- infection and aorto-enteric fistula

Special groups

Supra renal AAA
These patients will require a supra renal clamp and this carries a far higher risk of complications and risk of renal failure.

Ruptured AAA
Pre-operatively the management depends upon haemodynamic instability. In patients with symptoms of rupture (typical pain, haemodynamic compromise and risk factors) then ideally prompt laparotomy. In those with vague symptoms and haemodynamic stability the ideal test is CT scan to determine whether rupture has occurred or not. Most common rupture site is retroperitoneal 80%. These patients will tend to develop retroperitoneal haematoma. This can be disrupted if Bp is allowed to rise too high so aim for Bp 100mmHg.
Operative details are similar to elective repair although surgery should be swift, blind rushing often makes the situation worse. Plunging vascular clamps blindly into a pool of blood at the aneurysm neck carries the risk of injury the vena cava that these patients do not withstand. Occasionally a supracoeliac clamp is needed to effect temporary control, although leaving this applied for more than 20 minutes tends to carry a dismal outcome.

EVAR
Increasingly patients are now being offered endovascular aortic aneurysm repair. This is undertaken by surgeons and radiologists working jointly. The morphology of the aneurysm is important and not all are suitable. Here is a typical list of those features favoring a suitable aneurysm:
Long neck
Straight iliac vessels
Healthy groin vessels

Clearly few AAA patients possess the above and compromise has to be made. The use of fenestrated grafts can allow supra renal AAA to be treated.

Procedure:

GA
Radiology or theatre
Bilateral groin incisions
Common femoral artery dissected out
Heparinisation
Arteriotomy and insertion of guide wire
Dilation of arteriotomy
Insertion of EVAR Device
Once in satisfactory position it is released
Arteriotomy closed once check angiogram shows good position and no endoleak

Complications:
Endoleaks depending upon site are either Type I or 2. These may necessitate re-intervention and all EVAR patients require follow up . Details are not needed for MRCS.

How well did you know this?
1
Not at all
2
3
4
5
Perfectly
59
Q

An 89 year old man presents with hypotension and collapse and is found by the staff in the toilet of his care home. He is moribund and unable to give a clear history. He had suffered a cardiac arrest in the ambulance but has since been resuscitated and now has a Bp of 95 systolic. He has an obviously palpable AAA.

A.	Immediate laparotomy
B.	Immediate CT
C.	AAA repair during next 48 hours
D.	USS in 6 months
E.	CT scan during next 4 weeks
F.	Endovascular aortic aneurysm repair
G.	Discharge
H.	Palliate
I.	None of the above
A

Palliate

He will not survive aortic surgery and whilst some may disagree, I would argue that taking this case to theatre would be futile

Abdominal aorta aneurysm

Abdominal aortic aneurysms are a common problem in vascular surgery.
They may occur as either true or false aneurysm. With the former all 3 layers of the arterial wall are involved, in the latter only a single layer of fibrous tissue forms the aneurysm wall.
True abdominal aortic aneurysms have an approximate incidence of 0.06 per 1000 people. They are commonest in elderly men and for this reason the UK is now introducing the aneurysm screening program with the aim of performing an abdominal aortic ultrasound measurement in all men aged 65 years.

Causes
Several different groups of patients suffer from aneurysmal disease.
The commonest group is those who suffer from standard arterial disease, i.e. Those who are hypertensive and have been or are smokers.
Other patients such as those suffering from connective tissue diseases such as Marfan’s may also develop aneurysms. In patients with abdominal aortic aneurysms the extracellular matrix becomes disrupted with a change in the balance of collagen and elastic fibres.

Management
Most abdominal aortic aneurysms are an incidental finding.
Symptoms most often relate to rupture or impending rupture.
20% rupture anteriorly into the peritoneal cavity. Very poor prognosis.
80% rupture posteriorly into the retroperitoneal space
The risk of rupture is related to aneurysm size, only 2% of aneurysms measuring less than 4cm in diameter will rupture over a 5 year period. This contrasts with 75% of aneurysms measuring over 7cm in diameter.
This is well explained by Laplaces’ law which relates size to transmural pressure.
For this reason most vascular surgeons will subject patients with an aneurysm size of 5cm or greater to CT scanning of the chest, abdomen and pelvis with the aim of delineating anatomy and planning treatment. Depending upon co-morbidities, surgery is generally offered once the aneurysm is between 5.5cm and 6cm.

A CT reconstruction showing an infrarenal abdominal aortic aneurysm. The walls of the sac are calcified which may facilitate identification on plain x-rays

Image sourced from Wikipedia

Indications for surgery
Symptomatic aneurysms (80% annual mortality if untreated)
Increasing size above 5.5cm if asymptomatic
Rupture (100% mortality without surgery)

Surgical procedures
Abdominal aortic aneurysm repair

Procedure:

GA
Invasive monitoring (A-line, CVP, catheter)
Incision: Midline or transverse
Bowel and distal duodenum mobilised to access aorta.
Aneurysm neck and base dissected out and prepared for cross clamp
Systemic heparinisation
Cross clamp (proximal first)
Longitudinal aortotomy
Atherectomy
Deal with back bleeding from lumbar vessels and inferior mesenteric artery
Insert graft either tube or bifurcated depending upon anatomy
Suture using Prolene (3/0 for proximal , distal anastomosis suture varies according to site)
Clamps off: End tidal CO2 will rise owing to effects of reperfusion, at this point major risk of myocardial events.
Haemostasis
Closure of aneurysm sac to minimise risk of aorto-enteric fistula
Closure: Loop 1 PDS or Prolene to abdominal wall
Skin- surgeons preference

Post operatively:

ITU (Almost all)
Greatest risk of complications following emergency repair
Complications: Embolic- gut and foot infarcts
Cardiac - owing to premorbid states, re-perfusion injury and effects of cross clamp
Wound problems
Later risks related to graft- infection and aorto-enteric fistula

Special groups

Supra renal AAA
These patients will require a supra renal clamp and this carries a far higher risk of complications and risk of renal failure.

Ruptured AAA
Pre-operatively the management depends upon haemodynamic instability. In patients with symptoms of rupture (typical pain, haemodynamic compromise and risk factors) then ideally prompt laparotomy. In those with vague symptoms and haemodynamic stability the ideal test is CT scan to determine whether rupture has occurred or not. Most common rupture site is retroperitoneal 80%. These patients will tend to develop retroperitoneal haematoma. This can be disrupted if Bp is allowed to rise too high so aim for Bp 100mmHg.
Operative details are similar to elective repair although surgery should be swift, blind rushing often makes the situation worse. Plunging vascular clamps blindly into a pool of blood at the aneurysm neck carries the risk of injury the vena cava that these patients do not withstand. Occasionally a supracoeliac clamp is needed to effect temporary control, although leaving this applied for more than 20 minutes tends to carry a dismal outcome.

EVAR
Increasingly patients are now being offered endovascular aortic aneurysm repair. This is undertaken by surgeons and radiologists working jointly. The morphology of the aneurysm is important and not all are suitable. Here is a typical list of those features favoring a suitable aneurysm:
Long neck
Straight iliac vessels
Healthy groin vessels

Clearly few AAA patients possess the above and compromise has to be made. The use of fenestrated grafts can allow supra renal AAA to be treated.

Procedure:

GA
Radiology or theatre
Bilateral groin incisions
Common femoral artery dissected out
Heparinisation
Arteriotomy and insertion of guide wire
Dilation of arteriotomy
Insertion of EVAR Device
Once in satisfactory position it is released
Arteriotomy closed once check angiogram shows good position and no endoleak

Complications:
Endoleaks depending upon site are either Type I or 2. These may necessitate re-intervention and all EVAR patients require follow up . Details are not needed for MRCS.

How well did you know this?
1
Not at all
2
3
4
5
Perfectly
60
Q

A 19 year old soldier has just returned from a prolonged marching exercise and presents with a sudden onset, severe pain, in the forefoot. Clinical examination reveals tenderness along the second metatarsal. Plain x-rays are taken of the area, these demonstrate callus surrounding the shaft of the second metatarsal. What is the most likely diagnosis?

	Stress fracture
	Mortons neuroma
	Osteochondroma
	Acute osteomyelitis
	Freiberg's disease
A

Stress fractures

Repetitive activity and loading of normal bone may result in small hairline fractures. Whilst these may be painful they are seldom displaced. Surrounding soft tissue injury is unusual. They may present late following the injury, in which case callus formation may be identified on radiographs. Such cases may not require formal immobilisation, injuries associated with severe pain and presenting at an earlier stage may benefit from immobilisation tailored to the site of injury.

A short history of pain together with clinical examination and radiological signs affecting the second metatarsal favour a stress fracture. The fact that callus is present suggests that immobilisation is unlikely to be beneficial. Freibergs disease is an anterior metatarsalgia affecting the head of the second metarsal, it typically occurs in the pubertal growth spurt. The initial injury was thought to be due to stress microfractures at the growth plate. The key feature in the history which distinguishes the injury as being stress fracture is the radiology. In Freibergs disease the x-ray changes include; joint space widening, formation of bony spurs, sclerosis and flattening of the metatarsal head.

How well did you know this?
1
Not at all
2
3
4
5
Perfectly
61
Q

A 33 year old female is admitted for varicose vein surgery. She is fit and well. After the procedure she is persistently bleeding. She is known to have menorrhagia. Investigations show a prolonged bleeding time and increased APTT. She has a normal PT and platelet count.

A.	Vitamin K deficiency
B.	von Willebrand's disease
C.	Acquired haemophilia
D.	Haemophilia B
E.	Protein C deficiency
F.	Disseminated intravascular coagulation
G.	Factor V Leiden
H.	Excess heparin
I.	Warfarin overdose
J.	Antiphospholipid syndrome
A

von Willebrand’s disease

Bleeding post operatively, epistaxis and menorrhagia may indicate a diagnosis of vWD. Haemoarthroses are rare. The bleeding time is usually normal in haemophilia (X-linked) and vitamin K deficiency.

How well did you know this?
1
Not at all
2
3
4
5
Perfectly
62
Q
A 70 year old heavy smoker presents with 3 weeks of haematuria and bruising. He is normally fit and well. He is on no medications. His results reveal:
Hb 9.0
WCC 11
Pl 255
PT 16 (normal)
APTT 58 (increased)
Thrombin time 20 (normal).
A.	Vitamin K deficiency
B.	von Willebrand's disease
C.	Acquired haemophilia
D.	Haemophilia B
E.	Protein C deficiency
F.	Disseminated intravascular coagulation
G.	Factor V Leiden
H.	Excess heparin
I.	Warfarin overdose
J.	Antiphospholipid syndrome
A

Acquired haemophilia

This patient has Factor 8 acquired disorder. He is likely to have developed a lung malignancy (smoker) and as a result aquired a haemophilia disorder. The elderly, pregnancy, malignancy and autoimmune conditions are associated with acquired haemophilia. Prolonged APTT is key to the diagnosis. Management involves steroids.

How well did you know this?
1
Not at all
2
3
4
5
Perfectly
63
Q

A 28 year old female is attends the gynaecology unit for a D+C following an incomplete miscarriage. She has previously had recurrent pulmonary embolic events. After the procedure she is persistently bleeding. Her APTT is 52 (increased).

A.	Vitamin K deficiency
B.	von Willebrand's disease
C.	Acquired haemophilia
D.	Haemophilia B
E.	Protein C deficiency
F.	Disseminated intravascular coagulation
G.	Factor V Leiden
H.	Excess heparin
I.	Warfarin overdose
J.	Antiphospholipid syndrome
A

Antiphospholipid syndrome

A combination of thromboembolism and bleeding in a young woman should raise the possibility of antiphospholipid syndrome. Other features may include foetal loss, venous and arterial thrombosis and thrombocytopenia. A Lupus anticoagulant may be present and the APTT is prolonged.

How well did you know this?
1
Not at all
2
3
4
5
Perfectly
64
Q

During short saphenous vein surgery for varicose veins which of the following nerves is particularly at risk?

	Sural nerve
	Popliteal nerve
	Tibial nerve
	Femoral nerve
	Saphenous nerve
A

Sural nerve

Saphenous vein

Long saphenous vein
This vein may be harvested for bypass surgery, or removed as treatment for varicose veins with saphenofemoral junction incompetence.

Originates at the 1st digit where the dorsal vein merges with the dorsal venous arch of the foot
Passes anterior to the medial malleolus and runs up the medial side of the leg
At the knee, it runs over the posterior border of the medial epicondyle of the femur bone
Then passes laterally to lie on the anterior surface of the thigh before entering an opening in the fascia lata called the saphenous opening
It joins with the femoral vein in the region of the femoral triangle at the saphenofemoral junction

Tributaries
Medial marginal
Superficial epigastric
Superficial iliac circumflex
Superficial external pudendal veins

Short saphenous vein
Originates at the 5th digit where the dorsal vein merges with the dorsal venous arch of the foot, which attaches to the great saphenous vein.
It passes around the lateral aspect of the foot (inferior and posterior to the lateral malleolus) and runs along the posterior aspect of the leg (with the sural nerve)
Passes between the heads of the gastrocnemius muscle, and drains into the popliteal vein, approximately at or above the level of the knee joint.

How well did you know this?
1
Not at all
2
3
4
5
Perfectly
65
Q

A 78-year-old woman is discharged following a fractured neck of femur. On review, she is making good progress but consideration is given to secondary prevention of further fractures. Unfortunately the orthogeriatricians are all on annual leave and the consultant has asked you to arrange suitable management. Which is the best option?

	Alendronate
	Alendronate, calcium and vitamin D supplementation
	Strontium
	Arrange a DEXA scan
	Hormone replacement therapy
A

A bisphosphonate, calcium and vitamin D supplementation should be given to all patients aged over 75 years after having a fracture. A DEXA scan is only needed of the patient is aged below 75 years. Hormone replacement therapy has been shown to reduce vertebral and non vertebral fractures, however the risks of cardiovascular disease and breast malignancy make this a less favourable option.

Osteoporosis: secondary prevention

NICE guidelines were updated in 2008 on the secondary prevention of osteoporotic fractures in postmenopausal women.

Key points include
Treatment is indicated following osteoporotic fragility fractures in postmenopausal women who are confirmed to have osteoporosis (a T-score of - 2.5 SD or below).
In women aged 75 years or older, a DEXA scan may not be required ‘if the responsible clinician considers it to be clinically inappropriate or unfeasible’
Vitamin D and calcium supplementation should be offered to all women unless the clinician is confident they have adequate calcium intake and are vitamin D replete
Alendronate is first-line
Around 25% of patients cannot tolerate alendronate, usually due to upper gastrointestinal problems. These patients should be offered risedronate or etidronate (see treatment criteria below)
Strontium ranelate and raloxifene are recommended if patients cannot tolerate bisphosphonates (see treatment criteria below)

Supplementary notes on treatment

Bisphosphonates
Alendronate, risedronate and etidronate are all licensed for the prevention and treatment of post-menopausal and glucocorticoid-induced osteoporosis
All three have been shown to reduce the risk of both vertebral and non-vertebral fractures although alendronate, risedronate may be superior to etidronate in preventing hip fractures
Ibandronate is a once-monthly oral bisphosphonate

Vitamin D and calcium
Poor evidence base to suggest reduced fracture rates in the general population at risk of osteoporotic fractures - may reduce rates in frail, housebound patients

Raloxifene - selective oestrogen receptor modulator (SERM)
Has been shown to prevent bone loss and to reduce the risk of vertebral fractures, but has not yet been shown to reduce the risk of non-vertebral fractures
Has been shown to increase bone density in the spine and proximal femur
May worsen menopausal symptoms
Increased risk of thromboembolic events
May decrease risk of breast cancer

Strontium ranelate
‘Dual action bone agent’ - increases deposition of new bone by osteoblasts and reduces the resorption of bone by osteoclasts
Strong evidence base, may be second-line treatment in near future
Increased risk of thromboembolic events

How well did you know this?
1
Not at all
2
3
4
5
Perfectly
66
Q

Which of the following statements relating to avascular necrosis is false?

When associated with fracture may occur despite the radiological evidence of fracture union.
Pain and stiffness will typically precede radiological evidence of the condition.
Drilling of affected bony fragments may be used to facilitate angiogenesis where arthroplasty is not warranted.
The earliest detectable radiological evidence is a radiolucency of the affected area coupled with subchondral collapse.
It is less likely when prompt anatomical alignment of fracture fragments is achieved.
A

The earliest detectable radiological evidence is a radiolucency of the affected area coupled with subchondral collapse.

Radiolucency and subchondral collapse are late changes. The earliest evidence on plain films is the affected area appearing as being more radio-opaque due to hyperaemia and resorption of the neighboring area. It may be diagnosed earlier using bone scans and MRI.

Avascular necrosis

Cellular death of bone components due to interruption of the blood supply, causing bone destruction
Main joints affected are hip, scaphoid, lunate and the talus.
It is not the same as non union. The fracture has usually united.
Radiological evidence is slow to appear.
Vascular ingrowth into the affected bone may occur. However, many joints will develop secondary osteoarthritis.

Causes
P ancreatitis 
L upus 
A lcohol 
S teroids 
T rauma 
I diopathic, infection 
C aisson disease, collagen vascular disease 
R adiation, rheumatoid arthritis 
A myloid 
G aucher disease 
S ickle cell disease

Presentation
Usually pain. Often despite apparent fracture union.

Investigation
MRI scanning will show changes earlier than plain films.

Treatment
In fractures at high risk sites anticipation is key. Early prompt and accurate reduction is essential.

Non weight bearing may help to facilitate vascular regeneration.

Joint replacement may be necessary, or even the preferred option (e.g. Hip in the elderly).

How well did you know this?
1
Not at all
2
3
4
5
Perfectly
67
Q

A 22 year old rugby player falls onto an outstretched hand and sustains a fracture of the distal radius. The x-ray shows a dorsally angulated comminuted fracture.

A. Discharge home with arm sling and fracture clinic appointment
B. Discharge home with futura splint and fracture clinic appointment
C. Admit for open reduction and fixation
D. Fasciotomy
E. Active observation for progression of neurovascular compromise
F. Reduction of fracture in casualty and application of plaster backslab, followed by discharge home.

A

Admit for open reduction and fixation

Unlike an osteoporotic fracture in an elderly lady this is a high velocity injury and will require surgical fixation.

Fracture management

Bony injury resulting in a fracture may arise from trauma (excessive forces applied to bone), stress related (repetitive low velocity injury) or pathological (abnormal bone which fractures during normal use of following minimal trauma)
Diagnosis involves not just evaluating the fracture ; such as site and type of injury but also other associated injuries and distal neurovascular deficits. This may entail not just clinical examination but radiographs of proximal and distal joints.
When assessing x-rays it is important to assess for changes in length of the bone, the angulation of the distal bone, rotational effects, presence of material such as glass.

Key points in management of fractures
Immobilise the fracture including the proximal and distal joints
Carefully monitor and document neurovascular status, particularly following reduction and immobilisation
Manage infection including tetanus prophylaxis
IV broad spectrum antibiotics for open injuries
As a general principle all open fractures should be thoroughly debrided ( and internal fixation devices avoided or used with extreme caution)
Open fractures constitute an emergency and should be debrided and lavaged within 6 hours of injury

How well did you know this?
1
Not at all
2
3
4
5
Perfectly
68
Q

A 10 year old boy undergoes a delayed open reduction and fixation of a significantly displaced supracondylar fracture. On the ward he complains of significant forearm pain and paraesthesia of the hand. Radial pulse is normal.

A. Discharge home with arm sling and fracture clinic appointment
B. Discharge home with futura splint and fracture clinic appointment
C. Admit for open reduction and fixation
D. Fasciotomy
E. Active observation for progression of neurovascular compromise
F. Reduction of fracture in casualty and application of plaster backslab, followed by discharge home.

A

Fasciotomy

The delay is the significant factor here. These injuries often have neurovascular compromise and inactivity now places him at risk of developing complications. In compartment syndrome the loss of arterial pulsation occurs late.

Fracture management

Bony injury resulting in a fracture may arise from trauma (excessive forces applied to bone), stress related (repetitive low velocity injury) or pathological (abnormal bone which fractures during normal use of following minimal trauma)
Diagnosis involves not just evaluating the fracture ; such as site and type of injury but also other associated injuries and distal neurovascular deficits. This may entail not just clinical examination but radiographs of proximal and distal joints.
When assessing x-rays it is important to assess for changes in length of the bone, the angulation of the distal bone, rotational effects, presence of material such as glass.

Key points in management of fractures
Immobilise the fracture including the proximal and distal joints
Carefully monitor and document neurovascular status, particularly following reduction and immobilisation
Manage infection including tetanus prophylaxis
IV broad spectrum antibiotics for open injuries
As a general principle all open fractures should be thoroughly debrided ( and internal fixation devices avoided or used with extreme caution)
Open fractures constitute an emergency and should be debrided and lavaged within 6 hours of injury

How well did you know this?
1
Not at all
2
3
4
5
Perfectly
69
Q

A 28 year old man falls onto an outstretched hand. On examination there is tenderness of the anatomical snuffbox. However, forearm and hand x-rays are normal.

A. Discharge home with arm sling and fracture clinic appointment
B. Discharge home with futura splint and fracture clinic appointment
C. Admit for open reduction and fixation
D. Fasciotomy
E. Active observation for progression of neurovascular compromise
F. Reduction of fracture in casualty and application of plaster backslab, followed by discharge home.

A

Discharge home with futura splint and fracture clinic appointment

This could well be a scaphoid fracture and should be temporarily immobilised pending further review. A futura splint will immobilise better than an arm sling for this problem.

Fracture management

Bony injury resulting in a fracture may arise from trauma (excessive forces applied to bone), stress related (repetitive low velocity injury) or pathological (abnormal bone which fractures during normal use of following minimal trauma)
Diagnosis involves not just evaluating the fracture ; such as site and type of injury but also other associated injuries and distal neurovascular deficits. This may entail not just clinical examination but radiographs of proximal and distal joints.
When assessing x-rays it is important to assess for changes in length of the bone, the angulation of the distal bone, rotational effects, presence of material such as glass.

Key points in management of fractures
Immobilise the fracture including the proximal and distal joints
Carefully monitor and document neurovascular status, particularly following reduction and immobilisation
Manage infection including tetanus prophylaxis
IV broad spectrum antibiotics for open injuries
As a general principle all open fractures should be thoroughly debrided ( and internal fixation devices avoided or used with extreme caution)
Open fractures constitute an emergency and should be debrided and lavaged within 6 hours of injury

How well did you know this?
1
Not at all
2
3
4
5
Perfectly
70
Q

A 32 year old man is involved in a road traffic accident and sustains a significant laceration to the lateral aspect of the nose which is associated with tissue loss.

A.	Wound excision and primary closure
B.	Simple primary closure
C.	Delayed primary closure
D.	Debridement and healing by secondary intention
E.	Split thickness skin graft
F.	Full thickness skin graft
G.	Free flap
H.	Pedicled flap
I.	Debridement and rotational flap
A

Debridement and rotational flap

Nasal injuries can be challenging to manage and where there is tissue loss, it can be difficult to primarily close them and still obtain a satisfactory aesthetic result. Debridement together with a rotational flap would obtain the best results here.

Primary closure
Clean wound, usually surgically created or following minor trauma
Standard suturing methods will usually suffice
Wound heals by primary intention

Delayed primary closure
Similar methods of actual closure to primary closure
May be used in situations where primary closure is either not achievable or not advisable e.g. infection

Vacuum assisted closure
Uses negative pressure therapy to facilitate wound closure
Sponge is inserted into wound cavity and then negative pressure applied
Advantages include removal of exudate and versatility
Disadvantages include cost and risk of fistulation if used incorrectly on sites such as bowel

Split thickness skin grafts
Superficial dermis removed with Watson knife or dermatome (commonly from thigh)
Remaining epithelium regenerates from dermal appendages
Coverage may be increased by meshing

Full thickness skin grafts
Whole dermal thickness is removed
Sub dermal fat is then removed and graft placed over donor site
Better cosmesis and flexibility at recipient site
Donor site “cost”

Flaps
Viable tissue with a blood supply
May be pedicled or free
Pedicled flaps are more reliable, but limited in range
Free flaps have greater range but carry greater risk of breakdown as they require vascular anastomosis

How well did you know this?
1
Not at all
2
3
4
5
Perfectly
71
Q

A 7 year old boy falls over and sustains a 6cm laceration to his head. On inspection his wound contains some dirt in it.

A.	Wound excision and primary closure
B.	Simple primary closure
C.	Delayed primary closure
D.	Debridement and healing by secondary intention
E.	Split thickness skin graft
F.	Full thickness skin graft
G.	Free flap
H.	Pedicled flap
I.	Debridement and rotational flap
A

Wound excision and primary closure

By debriding the wound, the area can then be primarily closed. Prophylactic antibiotics should be administered.

Primary closure
Clean wound, usually surgically created or following minor trauma
Standard suturing methods will usually suffice
Wound heals by primary intention

Delayed primary closure
Similar methods of actual closure to primary closure
May be used in situations where primary closure is either not achievable or not advisable e.g. infection

Vacuum assisted closure
Uses negative pressure therapy to facilitate wound closure
Sponge is inserted into wound cavity and then negative pressure applied
Advantages include removal of exudate and versatility
Disadvantages include cost and risk of fistulation if used incorrectly on sites such as bowel

Split thickness skin grafts
Superficial dermis removed with Watson knife or dermatome (commonly from thigh)
Remaining epithelium regenerates from dermal appendages
Coverage may be increased by meshing

Full thickness skin grafts
Whole dermal thickness is removed
Sub dermal fat is then removed and graft placed over donor site
Better cosmesis and flexibility at recipient site
Donor site “cost”

Flaps
Viable tissue with a blood supply
May be pedicled or free
Pedicled flaps are more reliable, but limited in range
Free flaps have greater range but carry greater risk of breakdown as they require vascular anastomosis

How well did you know this?
1
Not at all
2
3
4
5
Perfectly
72
Q

A 45 year old man is gardening and damages his foot with a fork. On examination there are cutaneous defects and the surrounding skin looks dusky.

A.	Wound excision and primary closure
B.	Simple primary closure
C.	Delayed primary closure
D.	Debridement and healing by secondary intention
E.	Split thickness skin graft
F.	Full thickness skin graft
G.	Free flap
H.	Pedicled flap
I.	Debridement and rotational flap
A

Debridement and healing by secondary intention

The skin changes described here should be debrided. Closure would not be safe with the skin changes documented and the wound should be left open.

Primary closure
Clean wound, usually surgically created or following minor trauma
Standard suturing methods will usually suffice
Wound heals by primary intention

Delayed primary closure
Similar methods of actual closure to primary closure
May be used in situations where primary closure is either not achievable or not advisable e.g. infection

Vacuum assisted closure
Uses negative pressure therapy to facilitate wound closure
Sponge is inserted into wound cavity and then negative pressure applied
Advantages include removal of exudate and versatility
Disadvantages include cost and risk of fistulation if used incorrectly on sites such as bowel

Split thickness skin grafts
Superficial dermis removed with Watson knife or dermatome (commonly from thigh)
Remaining epithelium regenerates from dermal appendages
Coverage may be increased by meshing

Full thickness skin grafts
Whole dermal thickness is removed
Sub dermal fat is then removed and graft placed over donor site
Better cosmesis and flexibility at recipient site
Donor site “cost”

Flaps
Viable tissue with a blood supply
May be pedicled or free
Pedicled flaps are more reliable, but limited in range
Free flaps have greater range but carry greater risk of breakdown as they require vascular anastomosis

How well did you know this?
1
Not at all
2
3
4
5
Perfectly
73
Q

A 19 year old student is involved in a head on car collision. He complains of severe chest pain. A Chest x-ray performed as part of a trauma series shows widening of the mediastinum. Which is the most likely injury in this scenario?

Rupture of the distal oesophagus
Rupture of the left main bronchus
Rupture of the aorta proximal to the left subclavian artery
Rupture of the aorta distal to the left subclavian artery
Rupture of the inferior vena cava
A

Rupture of the aorta distal to the left subclavian artery

The aorta may be injured in deceleration accidents. In the setting of deceleration injury, chest pain and mediastinal widening the most likely problem is aortic rupture. This will typically occur distal to the left subclavian artery. Rupture of the proximal aorta may occur. However, survival is unlikely. It is important to note that the question uses the term Most likely injury as this is the component that distinguishes an ascending rupture from a descending rupture.

Thoracic aorta rupture

Mechanism of injury: Decelerating force i.e. RTA, fall from a great height
Most people die at scene
Survivors may have an incomplete laceration at the ligamentum arteriosum of the aorta.

Clinical features
Contained haematoma: persistent hypotension
Detected mainly by history, CXR changes

CXR changes
Widened mediastinum
Trachea/Oesophagus to right
Depression of left main stem bronchus
Widened paratracheal stripe/paraspinal interfaces
Space between aorta and pulmonary artery obliterated
Rib fracture/left haemothorax

Diagnosis
Angiography, usually CT aortogram.

How well did you know this?
1
Not at all
2
3
4
5
Perfectly
74
Q

A 22 year old mechanic is involved in a fight. He is hit on the head with a hammer. On examination he had clinical evidence of an open depressed skull fracture and a GCS of 6/15.

A.	Observation
B.	CT head within 1 hour
C.	CT head within 8 hours
D.	Urgent neurosurgical review (even before CT head performed)
E.	3 view c-spine xray
F.	2 view c-spine xray
G.	CT c-spine
A

Urgent neurosurgical review (even before CT head performed)

A patient with GCS <8 or = to 8 needs urgent neurosurgical review. Especially when an open fracture is present.

Head injury management- NICE Guidelines

Summary of guidelines
All patients should be assessed within 15 minutes on arrival to A&E
Document all 3 components of the GCS
If GCS <8 or = to 8, consider stabilising the airway
Treat pain with low dose IV opiates (if safe)
Full spine immobilisation until assessment if:

  • GCS < 15
  • neck pain/tenderness
  • paraesthesia extremities
  • focal neurological deficit
  • suspected c-spine injury

If a c-spine injury is suspected a 3 view c-spine x-ray is indicated. CT c-spine is preferred if:

  • Intubated
  • GCS <13
  • Normal x-ray but continued concerns regarding c-spine injury
  • Any focal neurology
  • A CT head scan is being performed
  • Initial plain films are abnormal
Immediate CT head (within 1 hour) if:
GCS < 13 on admission
GCS < 15 2 hours after admission
Suspected open or depressed skull fracture
Suspected skull base fracture (panda eyes, Battle's sign, CSF from nose/ear, bleeding ear)
Focal neurology
Vomiting > 1 episode
Post traumatic seizure
Coagulopathy
Contact neurosurgeon if:
Persistent GCS < 8 or = 8
Unexplained confusion > 4h
Reduced GCS after admission
Progressive neurological signs
Incomplete recovery post seizure
Penetrating injury
Cerebrospinal fluid leak

Observations
1/2 hourly GCS until 15

How well did you know this?
1
Not at all
2
3
4
5
Perfectly
75
Q

A 67 year old retired lawyer falls down the stairs. His GCS is 15/15 and he has some bruising over the mastoid.

A.	Observation
B.	CT head within 1 hour
C.	CT head within 8 hours
D.	Urgent neurosurgical review (even before CT head performed)
E.	3 view c-spine xray
F.	2 view c-spine xray
G.	CT c-spine
A

CT head within 1 hour

This patient has a basal skull fracture, which is indicated by a positive Battle’s sign. He should have a CT head within 1h.

Head injury management- NICE Guidelines

Summary of guidelines
All patients should be assessed within 15 minutes on arrival to A&E
Document all 3 components of the GCS
If GCS <8 or = to 8, consider stabilising the airway
Treat pain with low dose IV opiates (if safe)
Full spine immobilisation until assessment if:

  • GCS < 15
  • neck pain/tenderness
  • paraesthesia extremities
  • focal neurological deficit
  • suspected c-spine injury

If a c-spine injury is suspected a 3 view c-spine x-ray is indicated. CT c-spine is preferred if:

  • Intubated
  • GCS <13
  • Normal x-ray but continued concerns regarding c-spine injury
  • Any focal neurology
  • A CT head scan is being performed
  • Initial plain films are abnormal
Immediate CT head (within 1 hour) if:
GCS < 13 on admission
GCS < 15 2 hours after admission
Suspected open or depressed skull fracture
Suspected skull base fracture (panda eyes, Battle's sign, CSF from nose/ear, bleeding ear)
Focal neurology
Vomiting > 1 episode
Post traumatic seizure
Coagulopathy
Contact neurosurgeon if:
Persistent GCS < 8 or = 8
Unexplained confusion > 4h
Reduced GCS after admission
Progressive neurological signs
Incomplete recovery post seizure
Penetrating injury
Cerebrospinal fluid leak

Observations
1/2 hourly GCS until 15

How well did you know this?
1
Not at all
2
3
4
5
Perfectly
76
Q

A 52 year old secretary falls down the stairs. She complains of neck pain. She has a GCS of 15/15 and no neurology. She is unable to rotate her c-spine 45 degrees to the left and right.

A.	Observation
B.	CT head within 1 hour
C.	CT head within 8 hours
D.	Urgent neurosurgical review (even before CT head performed)
E.	3 view c-spine xray
F.	2 view c-spine xray
G.	CT c-spine
A

3 view c-spine xray

In the January 2014 NICE guidance relating to the diagnosis of head and spinal injury the evidence for initial imaging of the C spine was reviewed. The current UK practice is that “low risk” patients with pain but no neurology undergo a 3 view C spine x-ray. The more detailed 5 view x-ray was not found to be any better than 3 view films. In patients with high risk injuries (which the patient in the scenario does not have) there is a likelihood that 1 in 6 injuries would be missed on plain films alone and therefore CT scanning of the C spine is recommended in this group.

Head injury management- NICE Guidelines

Summary of guidelines
All patients should be assessed within 15 minutes on arrival to A&E
Document all 3 components of the GCS
If GCS <8 or = to 8, consider stabilising the airway
Treat pain with low dose IV opiates (if safe)
Full spine immobilisation until assessment if:

  • GCS < 15
  • neck pain/tenderness
  • paraesthesia extremities
  • focal neurological deficit
  • suspected c-spine injury

If a c-spine injury is suspected a 3 view c-spine x-ray is indicated. CT c-spine is preferred if:

  • Intubated
  • GCS <13
  • Normal x-ray but continued concerns regarding c-spine injury
  • Any focal neurology
  • A CT head scan is being performed
  • Initial plain films are abnormal
Immediate CT head (within 1 hour) if:
GCS < 13 on admission
GCS < 15 2 hours after admission
Suspected open or depressed skull fracture
Suspected skull base fracture (panda eyes, Battle's sign, CSF from nose/ear, bleeding ear)
Focal neurology
Vomiting > 1 episode
Post traumatic seizure
Coagulopathy
Contact neurosurgeon if:
Persistent GCS < 8 or = 8
Unexplained confusion > 4h
Reduced GCS after admission
Progressive neurological signs
Incomplete recovery post seizure
Penetrating injury
Cerebrospinal fluid leak

Observations
1/2 hourly GCS until 15

How well did you know this?
1
Not at all
2
3
4
5
Perfectly
77
Q

A 21 year old badminton player attends A&E with a painful, swollen right arm. He is right handed. Clinically he has dusky fingers and his upper limb pulses are present. An axillary vein thrombosis is confirmed. What is the best acute treatment to achieve vein patency?

	Intravenous heparin
	Warfarin
	Catheter directed tPA
	Low molecular weight heparin
	Aspirin
A

Catheter directed tPA

Axillary vein thrombosis

1-2% of all deep venous thrombosis
Primary cause is associated with trauma, thoracic outlet obstruction or repeated effort in a dominant arm (young active individuals)
Secondary causes include central line insertion, malignancy, pacemakers

Clinical features
Pain and swelling (non pitting)
Numbness
Discolouration: mottling, dusky
Pulses present
Congested veins
Investigations
FBC: viscosity, platelet function
Clotting
Liver function tests
D-dimer
Duplex scan: investigation of choice
CT scan: thoracic outlet obstruction

Treatment
Local catheter directed TPA
Heparin
Warfarin

How well did you know this?
1
Not at all
2
3
4
5
Perfectly
78
Q

A 22 year old man is having a blood transfusion after losing blood from a peptic ulcer. He is normally fit and well. Four hours after the transfusion; he complains of sudden onset shortness of breath and chest pain. On examination his temperature is 37.2, saturations are 88% on air, blood pressure 100/55 mmHg and HR 110 bpm. He has crepitations bilaterally up to the midzones. He is given IV frusemide, but deteriorates and is admitted to ITU. A pulmonary catheter is inserted and the PCWP is 10 mmHg.

A.	Neutrophilic febrile reaction
B.	Acute haemolytic transfusion reaction
C.	Delayed haemolytic transfusion reaction
D.	Pulmonary oedema
E.	Sickle cell crisis
F.	Transfusion associated lung injury
G.	Graft vs. Host disease
A

Transfusion associated lung injury

The pulmonary catheter reading indicates that this is not a case of fluid overload (the PCWP should be high, normal values PCWP systolic 7mmHg, diastolic 10mmHg). Transfusion associated lung injury is a rare reaction causing neutrophilic mediated allergic pulmonary oedema. Patient’s have antibodies to donor leukocytes. It is important to consider this as a diagnosis when patients don’t respond to treatment for pulmonary oedema. Patients normally respond to supportive therapy including fluids and oxygen.

Blood transfusion reactions

Acute transfusion reactions present as adverse signs or symptoms during or within 24 hours of a blood transfusion. The most frequent reactions are fever, chills, pruritus, or urticaria, which typically resolve promptly without specific treatment or complications. Other signs occurring in temporal relationship with a blood transfusion, such as severe dyspnoea, pyrexia, or loss of consciousness may be the first indication of a more severe potentially fatal reaction.
The causes of adverse reactions are multi-factorial. Immune mediated reactions, some of the most feared, occur as a result of component mismatch, the commonest cause of which is clerical error. More common, non immune mediated, complications may occur as a result of product contamination, this may be bacterial or viral.
Transfusion related lung injury is well recognised and there are two proposed mechanisms which underpin this. One involves the sequestration of primed neutrophils within the recipient pulmonary capillary bed. The other proposed mechanism suggests that HLA mismatches between donor neutrophils and recipient lung tissue is to blame.

How well did you know this?
1
Not at all
2
3
4
5
Perfectly
79
Q

A 32 year male with leukaemia attends the day unit for a blood transfusion. Five days after the transfusion he attends A&E with a temperature of 38.5, erythroderma and desquamation.

A.	Neutrophilic febrile reaction
B.	Acute haemolytic transfusion reaction
C.	Delayed haemolytic transfusion reaction
D.	Pulmonary oedema
E.	Sickle cell crisis
F.	Transfusion associated lung injury
G.	Graft vs. Host disease
A

Graft vs. Host disease

This is associated with transfusion of unirradiated blood in immunosupressed patients. Transfusion associated GVHD can occur 4-30 days after a transfusion and follows a sub acute pathway. Patients may also have diarrhoea and abnormal liver function tests. Management involves steroid therapy.

Blood transfusion reactions

Acute transfusion reactions present as adverse signs or symptoms during or within 24 hours of a blood transfusion. The most frequent reactions are fever, chills, pruritus, or urticaria, which typically resolve promptly without specific treatment or complications. Other signs occurring in temporal relationship with a blood transfusion, such as severe dyspnoea, pyrexia, or loss of consciousness may be the first indication of a more severe potentially fatal reaction.
The causes of adverse reactions are multi-factorial. Immune mediated reactions, some of the most feared, occur as a result of component mismatch, the commonest cause of which is clerical error. More common, non immune mediated, complications may occur as a result of product contamination, this may be bacterial or viral.
Transfusion related lung injury is well recognised and there are two proposed mechanisms which underpin this. One involves the sequestration of primed neutrophils within the recipient pulmonary capillary bed. The other proposed mechanism suggests that HLA mismatches between donor neutrophils and recipient lung tissue is to blame.

How well did you know this?
1
Not at all
2
3
4
5
Perfectly
80
Q

A 22 year old man is having a blood transfusion after losing blood from haemorrhoids. He is normally fit and well. 3h during the transfusion he complains of sudden onset abdominal pain and nausea. His temperature is 39 degrees, Blood pressure 98/42 mmHg, HR 105 bpm and saturations 94% air. His urine appears dark.

A.	Neutrophilic febrile reaction
B.	Acute haemolytic transfusion reaction
C.	Delayed haemolytic transfusion reaction
D.	Pulmonary oedema
E.	Sickle cell crisis
F.	Transfusion associated lung injury
G.	Graft vs. Host disease
A

Acute haemolytic transfusion reaction

Rapid intravascular haemolysis leading to shock, DIC and death can occur with this reaction.

Blood transfusion reactions

Acute transfusion reactions present as adverse signs or symptoms during or within 24 hours of a blood transfusion. The most frequent reactions are fever, chills, pruritus, or urticaria, which typically resolve promptly without specific treatment or complications. Other signs occurring in temporal relationship with a blood transfusion, such as severe dyspnoea, pyrexia, or loss of consciousness may be the first indication of a more severe potentially fatal reaction.
The causes of adverse reactions are multi-factorial. Immune mediated reactions, some of the most feared, occur as a result of component mismatch, the commonest cause of which is clerical error. More common, non immune mediated, complications may occur as a result of product contamination, this may be bacterial or viral.
Transfusion related lung injury is well recognised and there are two proposed mechanisms which underpin this. One involves the sequestration of primed neutrophils within the recipient pulmonary capillary bed. The other proposed mechanism suggests that HLA mismatches between donor neutrophils and recipient lung tissue is to blame.

How well did you know this?
1
Not at all
2
3
4
5
Perfectly
81
Q

A 49-year-old male sustained a severe blunt injury just below the bridge of the nose with industrial machinery. Imaging demonstrates a fracture involving the superior orbital fissure. On examination an ipsilateral pupillary defect is present and loss of the corneal reflexes. In addition to these examination findings, which of the following will not be present?

	Altered cutaneous sensation from the forehead to the vertex
	Ptosis
	Complete opthalmoplegia
	Nystagmus
	Enopthalmos
A

Nystagmus

Orbital apex syndrome
This is an extension of superior orbital fissure syndrome and includes compression of the optic nerve passing through the optic foramen. It is indicated by features of superior orbital fissure syndrome and ipsilateral afferent pupillary defect.
This type of injury will result in the orbital apex syndrome (See above). As such opthalmoplegia will be present and nystagmus cannot occur.

Le Fort 1 The fracture extends from the nasal septum to the lateral pyriform rims, travels horizontally above the teeth apices, crosses below the zygomaticomaxillary junction, and traverses the pterygomaxillary junction to interrupt the pterygoid plates.
Le Fort 2 These fractures have a pyramidal shape and extend from the nasal bridge at or below the nasofrontal suture through the frontal process of the maxilla, inferolaterally through the lacrimal bones and inferior orbital floor and rim through or near the inferior orbital foramen, and inferiorly through the anterior wall of the maxillary sinus; it then travels under the zygoma, across the pterygomaxillary fissure, and through the pterygoid plates.
Le Fort 3 These fractures start at the nasofrontal and frontomaxillary sutures and extend posteriorly along the medial wall of the orbit through the nasolacrimal groove and ethmoid bones. The thicker sphenoid bone posteriorly usually prevents continuation of the fracture into the optic canal. Instead, the fracture continues along the floor of the orbit along the inferior orbital fissure and continues superolaterally through the lateral orbital wall, through the zygomaticofrontal junction and the zygomatic arch. Intranasally, a branch of the fracture extends through the base of the perpendicular plate of the ethmoid, through the vomer, and through the interface of the pterygoid plates to the base of the sphenoid. This type of fracture predisposes the patient to CSF rhinorrhea more commonly than the other types.

Ocular injuries
Superior orbital fissure syndrome
Severe force to the lateral wall of the orbit resulting in compression of neurovascular structures. Results in :
Complete opthalmoplegia and ptosis (Cranial nerves 3, 4, 6 and nerve to levator palpebrae superioris)
Relative afferent pupillary defect
Dilatation of the pupil and loss of accommodation and corneal reflexes
Altered sensation from forehead to vertex (frontal branch of trigeminal nerve)

Orbital blow out fracture
Typically occurs when an object of slightly larger diameter than the orbital rim strikes the incompressible eyeball. The bone fragment is displaced downwards into the antral cavity, remaining attached to the orbital periosteum. Periorbital fat may be herniated through the defect, interfering with the inferior rectus and inferior oblique muscles which are contained within the same fascial sheath. This prevents upward movement and outward rotation of the eye and the patient experiences diplopia on upward gaze. The initial bruising and swelling may make assessment difficult and patients should usually be reviewed 5 days later. Residual defects may require orbital floor reconstruction.

Nasal Fractures
Common injury
Ensure new and not old deformity
Control epistaxis
CSF rhinorrhoea implies that the cribriform plate has been breached and antibiotics will be required.
Usually best to allow bruising and swelling to settle and then review patient clinically. Major persistent deformity requires fracture manipulation, best performed within 10 days of injury.

Retrobulbar haemorrhage
Rare but important ocular emergency. Presents with:
Pain (usually sharp and within the globe)
Proptosis
Pupil reactions are lost
Paralysis (eye movements lost)
Visual acuity is lost (colour vision is lost first)
May be the result of Le Fort type facial fractures.

Management:
Mannitol 1g/Kg as 20% infusion, Osmotic diuretic, Contra-indicated in congestive heart failure and pulmonary oedema
Acetazolamide 500mg IV, (Monitor FBC/U+E) Reduces aqueous pressure by inhibition of carbonic anhydrase (used in glaucoma)
Dexamethasone 8mg orally or intravenously
In a traumatic setting an urgent cantholysis may be needed prior to definitive surgery.

Consider
Papaverine 40mg smooth muscle relaxant
Dextran 40 500mls IV improves perfusion

How well did you know this?
1
Not at all
2
3
4
5
Perfectly
82
Q

A 10 year old child is admitted with severe 30% burns following a house fire. After wound cleaning and dressings he is admitted to critical care. 1 day following skin grafts he becomes tachycardic and hypotensive. He vomits twice and this shows evidence of haematemesis

A.	Deep vein thrombosis
B.	Curlings Ulcer
C.	Contracture
D.	Type I respiratory failure
E.	Type II respiratory failure
F.	Toxic shock syndrome
G.	Compartment syndrome
H.	Rhabdomyolysis
I.	Disseminated intravascular coagulation
A

Curlings Ulcer

Stress ulcers may occur in the duodenum of burns patients and are more common in children.

Depth of burn assessment
Bleeding on needle prick
Sensation
Appearance
Blanching to pressure

Percentage burn estimation
Lund Browder chart: most accurate even in children
Wallace rule of nines
Palmar surface: surface area palm = 0.8% burn

> 15% body surface area burns in adults needs urgent burn fluid resuscitation

Transfer to burn centre if:
Need burn shock resuscitation
Face/hands/genitals affected
Deep partial thickness or full thickness burns
Significant electrical/chemical burns

Management
The initial aim is to stop the burning process and resuscitate the patient. Intravenous fluids will be required for children with burns greater than 10% of total body surface area. Adults with burns greater than 15% of total body surface area will also require IV fluids. The fluids are calculated using the Parkland formula which is; volume of fluid= total body surface area of the burn % x weight (Kg) x4. Half of the fluid is administered in the first 8 hours. A urinary catheter should be inserted. Analgesia should be given. Complex burns, burns involving the hand perineum and face and burns >10% in adults and >5% in children should be transferred to a burns unit.

Circumferential burns affecting a limb or severe torso burns impeding respiration may require escharotomy to divide the burnt tissue.

Conservative management is appropriate for superficial burns and mixed superficial burns that will heal in 2 weeks. More complex burns may require excision and skin grafting. Excision and primary closure is not generally practised as there is a high risk of infection.

There is no evidence to support the use of anti microbial prophylaxis or topical antibiotics in burn patients.

Escharotomies
Indicated in circumferential full thickness burns to the torso or limbs.
Careful division of the encasing band of burn tissue will potentially improve ventilation (if the burn involves the torso), or relieve compartment syndrome and oedema (where a limb is involved)

How well did you know this?
1
Not at all
2
3
4
5
Perfectly
83
Q

A 26 year old electrician suffers a full thickness high voltage burn to his leg. On routine urine analysis he has + blood. His U+E’s show mild hyperkalaemia and a CK of 3000

A.	Deep vein thrombosis
B.	Curlings Ulcer
C.	Contracture
D.	Type I respiratory failure
E.	Type II respiratory failure
F.	Toxic shock syndrome
G.	Compartment syndrome
H.	Rhabdomyolysis
I.	Disseminated intravascular coagulation
A

Rhabdomyolysis

Electrical high voltage burns are associated with rhabdomyolysis. Acute tubular necrosis may occur. Aggressive IV fluids should be given

Depth of burn assessment
Bleeding on needle prick
Sensation
Appearance
Blanching to pressure

Percentage burn estimation
Lund Browder chart: most accurate even in children
Wallace rule of nines
Palmar surface: surface area palm = 0.8% burn

> 15% body surface area burns in adults needs urgent burn fluid resuscitation

Transfer to burn centre if:
Need burn shock resuscitation
Face/hands/genitals affected
Deep partial thickness or full thickness burns
Significant electrical/chemical burns

Management
The initial aim is to stop the burning process and resuscitate the patient. Intravenous fluids will be required for children with burns greater than 10% of total body surface area. Adults with burns greater than 15% of total body surface area will also require IV fluids. The fluids are calculated using the Parkland formula which is; volume of fluid= total body surface area of the burn % x weight (Kg) x4. Half of the fluid is administered in the first 8 hours. A urinary catheter should be inserted. Analgesia should be given. Complex burns, burns involving the hand perineum and face and burns >10% in adults and >5% in children should be transferred to a burns unit.

Circumferential burns affecting a limb or severe torso burns impeding respiration may require escharotomy to divide the burnt tissue.

Conservative management is appropriate for superficial burns and mixed superficial burns that will heal in 2 weeks. More complex burns may require excision and skin grafting. Excision and primary closure is not generally practised as there is a high risk of infection.

There is no evidence to support the use of anti microbial prophylaxis or topical antibiotics in burn patients.

Escharotomies
Indicated in circumferential full thickness burns to the torso or limbs.
Careful division of the encasing band of burn tissue will potentially improve ventilation (if the burn involves the torso), or relieve compartment syndrome and oedema (where a limb is involved)

How well did you know this?
1
Not at all
2
3
4
5
Perfectly
84
Q

A 45 year old man is admitted after his clothing caught fire. He suffers a full thickness circumferential burn to his lower thigh. He complains of increasing pain in lower leg and on examination there is parasthesia and severe pain in the lower leg. Foot pulses are normal

A.	Deep vein thrombosis
B.	Curlings Ulcer
C.	Contracture
D.	Type I respiratory failure
E.	Type II respiratory failure
F.	Toxic shock syndrome
G.	Compartment syndrome
H.	Rhabdomyolysis
I.	Disseminated intravascular coagulation
A

Compartment syndrome

Circumferential burns may constrict the limb and cause a compartment syndrome to develop. Eshcarotomy is required, and compartmental decompression.

Depth of burn assessment
Bleeding on needle prick
Sensation
Appearance
Blanching to pressure

Percentage burn estimation
Lund Browder chart: most accurate even in children
Wallace rule of nines
Palmar surface: surface area palm = 0.8% burn

> 15% body surface area burns in adults needs urgent burn fluid resuscitation

Transfer to burn centre if:
Need burn shock resuscitation
Face/hands/genitals affected
Deep partial thickness or full thickness burns
Significant electrical/chemical burns

Management
The initial aim is to stop the burning process and resuscitate the patient. Intravenous fluids will be required for children with burns greater than 10% of total body surface area. Adults with burns greater than 15% of total body surface area will also require IV fluids. The fluids are calculated using the Parkland formula which is; volume of fluid= total body surface area of the burn % x weight (Kg) x4. Half of the fluid is administered in the first 8 hours. A urinary catheter should be inserted. Analgesia should be given. Complex burns, burns involving the hand perineum and face and burns >10% in adults and >5% in children should be transferred to a burns unit.

Circumferential burns affecting a limb or severe torso burns impeding respiration may require escharotomy to divide the burnt tissue.

Conservative management is appropriate for superficial burns and mixed superficial burns that will heal in 2 weeks. More complex burns may require excision and skin grafting. Excision and primary closure is not generally practised as there is a high risk of infection.

There is no evidence to support the use of anti microbial prophylaxis or topical antibiotics in burn patients.

Escharotomies
Indicated in circumferential full thickness burns to the torso or limbs.
Careful division of the encasing band of burn tissue will potentially improve ventilation (if the burn involves the torso), or relieve compartment syndrome and oedema (where a limb is involved)

How well did you know this?
1
Not at all
2
3
4
5
Perfectly
85
Q

A 52 year old lady develops lower leg swelling following redo varicose vein surgery. There is evidence of swelling of the left leg up to the knee. The overlying skin appears healthy.

A.	Homans operation
B.	Charles operation
C.	Frusemide at high doses
D.	Frusemide at low doses
E.	Multilayer compression bandaging
F.	Lymphovenous anastomosis
A

Multilayer compression bandaging

Unfortunately lymphoedema may complicate redo varicose vein surgery (in 0.5% of cases). As the presentation is mild, she should be managed using compression hosiery. Diuretics do not help in cases of true lymphoedema and a dramatic response suggests an alternative underlying cause.
Lymphoedema

Due to impaired lymphatic drainage in the presence of normal capillary function.
Lymphoedema causes the accumulation of protein rich fluid, subdermal fibrosis and dermal thickening.
Characteristically fluid is confined to the epifascial space (skin and subcutaneous tissues); muscle compartments are free of oedema. It involves the foot, unlike other forms of oedema. There may be a ‘buffalo hump’ on the dorsum of the foot and the skin cannot be pinched due to subcutaneous fibrosis.

Causes of lymphoedema

Primary
Congenital < 1 year: sporadic, Milroy’s disease
Onset 1-35 years: sporadic, Meige’s disease
> 35 years: Tarda
Secondary
Bacterial/fungal/parasitic infection (filariasis)
Lymphatic malignancy
Radiotherapy to lymph nodes
Surgical resection of lymph nodes
DVT
Thrombophlebitis

Indications for surgery
Marked disability or deformity from limb swelling
Lymphoedema caused by proximal lymphatic obstruction with patent distal lymphatics suitable for a lymphatic drainage procedure
Lymphocutaneous fistulae and megalymphatics

Procedures
Homans operation Reduction procedure with preservation of overlying skin (which must be in good condition). Skin flaps are raised and the underlying tissue excised. Limb circumference typically reduced by a third.
Charles operation All skin and subcutaneous tissue around the calf are excised down to the deep fascia. Split skin grafts are placed over the site. May be performed if overlying skin is not in good condition. Larger reduction in size than with Homans procedure.
Lymphovenous anastamosis Identifiable lymphatics are anastomosed to sub dermal venules. Usually indicated in 2% of patients with proximal lymphatic obstruction and normal distal lymphatics.

How well did you know this?
1
Not at all
2
3
4
5
Perfectly
86
Q

A 57 year old lady has suffered from lymphoedema for many years. The left leg is swollen to the mid thigh. Severe limb deformity has developed as a result of process and in spite of compression hosiery. Lymphoscintography shows no patent lymphatics in the proximal leg. The overlying skin is healthy.

A.	Homans operation
B.	Charles operation
C.	Frusemide at high doses
D.	Frusemide at low doses
E.	Multilayer compression bandaging
F.	Lymphovenous anastomosis
A

Homans operation

Surgery is indicated in less than 10% of cases. However, severe deformity is one of the indications for surgery. Lymphovenous anastomosis is indicated where the proximal lymphatics are not patent. When the overlying skin is healthy (and limb deformity a problem), a Homans procedure is a reasonable first line operative option.

Lymphoedema

Due to impaired lymphatic drainage in the presence of normal capillary function.
Lymphoedema causes the accumulation of protein rich fluid, subdermal fibrosis and dermal thickening.
Characteristically fluid is confined to the epifascial space (skin and subcutaneous tissues); muscle compartments are free of oedema. It involves the foot, unlike other forms of oedema. There may be a ‘buffalo hump’ on the dorsum of the foot and the skin cannot be pinched due to subcutaneous fibrosis.

Causes of lymphoedema

Primary
Congenital < 1 year: sporadic, Milroy’s disease
Onset 1-35 years: sporadic, Meige’s disease
> 35 years: Tarda
Secondary
Bacterial/fungal/parasitic infection (filariasis)
Lymphatic malignancy
Radiotherapy to lymph nodes
Surgical resection of lymph nodes
DVT
Thrombophlebitis

Indications for surgery
Marked disability or deformity from limb swelling
Lymphoedema caused by proximal lymphatic obstruction with patent distal lymphatics suitable for a lymphatic drainage procedure
Lymphocutaneous fistulae and megalymphatics

Procedures
Homans operation Reduction procedure with preservation of overlying skin (which must be in good condition). Skin flaps are raised and the underlying tissue excised. Limb circumference typically reduced by a third.
Charles operation All skin and subcutaneous tissue around the calf are excised down to the deep fascia. Split skin grafts are placed over the site. May be performed if overlying skin is not in good condition. Larger reduction in size than with Homans procedure.
Lymphovenous anastamosis Identifiable lymphatics are anastomosed to sub dermal venules. Usually indicated in 2% of patients with proximal lymphatic obstruction and normal distal lymphatics.

How well did you know this?
1
Not at all
2
3
4
5
Perfectly
87
Q

A 38 year old lady is troubled by lymphoedema that occurred following a block dissection of the groin for malignant melanoma many years previously. Despite therapy with compression bandages she has persistent lower leg swelling impairing her activities of daily living. She has no evidence of recurrent malignancy. Lymphoscintography demonstrates occlusion of the groin lymphatics. However, the distal lymphatic system appears healthy.

A.	Homans operation
B.	Charles operation
C.	Frusemide at high doses
D.	Frusemide at low doses
E.	Multilayer compression bandaging
F.	Lymphovenous anastomosis
A

Lymphovenous anastomosis

In young patients with proximal disease and healthy distal lymphatics a lymphovenous anastomosis may be considered. Such cases are rare.

Lymphoedema

Due to impaired lymphatic drainage in the presence of normal capillary function.
Lymphoedema causes the accumulation of protein rich fluid, subdermal fibrosis and dermal thickening.
Characteristically fluid is confined to the epifascial space (skin and subcutaneous tissues); muscle compartments are free of oedema. It involves the foot, unlike other forms of oedema. There may be a ‘buffalo hump’ on the dorsum of the foot and the skin cannot be pinched due to subcutaneous fibrosis.

Causes of lymphoedema

Primary
Congenital < 1 year: sporadic, Milroy’s disease
Onset 1-35 years: sporadic, Meige’s disease
> 35 years: Tarda
Secondary
Bacterial/fungal/parasitic infection (filariasis)
Lymphatic malignancy
Radiotherapy to lymph nodes
Surgical resection of lymph nodes
DVT
Thrombophlebitis

Indications for surgery
Marked disability or deformity from limb swelling
Lymphoedema caused by proximal lymphatic obstruction with patent distal lymphatics suitable for a lymphatic drainage procedure
Lymphocutaneous fistulae and megalymphatics

Procedures
Homans operation Reduction procedure with preservation of overlying skin (which must be in good condition). Skin flaps are raised and the underlying tissue excised. Limb circumference typically reduced by a third.
Charles operation All skin and subcutaneous tissue around the calf are excised down to the deep fascia. Split skin grafts are placed over the site. May be performed if overlying skin is not in good condition. Larger reduction in size than with Homans procedure.
Lymphovenous anastamosis Identifiable lymphatics are anastomosed to sub dermal venules. Usually indicated in 2% of patients with proximal lymphatic obstruction and normal distal lymphatics.

How well did you know this?
1
Not at all
2
3
4
5
Perfectly
88
Q
A 42 year old woman is admitted to surgery with acute cholecystitis. She is known to have hypertension, rheumatoid arthritis and polymyalgia rheumatica. Her medical therapy includes:
Paracetamol 1g qds
Codeine phosphate 30mg qds
Bendrofluazide 2.5 mg od
Ramipril 10mg od
Methotrexate 7.5mg once a week
Prednisolone 5mg od
You are called by the CT1 to assess this lady as she has become delirious and hypotensive 2 hours after surgery. Her blood results reveal:

Na+ 132 mmol/l
K+ 5.3 mmol/l
Urea 7 mmol/l
Creatinine 108 µmol/l

Hb 12.4 g/dl
Platelets 178 * 109/l
WBC 15.4 * 109/l

What management is needed immediately?

	Ceftriaxone IV
	Hydrocortisone 50mg IV
	CT scan abdomen
	Urgent exploratory laparotomy
	Hydrocortisone 100mg IV
A

Hydrocortisone 100mg IV

This patient has acute adrenal insufficiency and urgently needs steroid replacement.

Addisonian crisis

Causes
Sepsis or surgery causing an acute exacerbation of chronic insufficiency (Addison’s, Hypopituitarism)
Adrenal haemorrhage eg Waterhouse-Friderichsen syndrome (fulminant meningococcemia)
Steroid withdrawal

Management
Hydrocortisone 100 mg im or iv
1 litre normal saline infused over 30-60 mins or with dextrose if hypoglycaemic
Continue hydrocortisone 6 hourly until the patient is stable. No fludrocortisone is required because high cortisol exerts weak mineralocorticoid action
Oral replacement may begin after 24 hours and be reduced to maintenance over 3-4 days

How well did you know this?
1
Not at all
2
3
4
5
Perfectly
89
Q

A 28 year old male is involved in a road traffic accident he is thrown from his motorbike onto the pavement and sustains a haemopneumothorax and flail segment of the right chest.

A. Thoracotomy
B. Manage conservatively
C. Intercostal tube drain insertion
D. CT scanning
E. Bronchoscopy
F. Negative pressure intercostal tube drainage
G. Video assisted thoracoscopy and pleurectomy

A

Intercostal tube drain insertion

He requires a chest drain and analgesia. In general all haemopneumothoraces should be managed by intercostal chest drain insertion as they have a risk of becoming a tension pneumothorax until the lung laceration has sealed.

Thoracic trauma

Types of thoracic trauma

Tension pneumothorax
Often laceration to lung parenchyma with flap
Pressure develops in thorax
Most common cause is mechanical ventilation in patient with pleural injury
Symptoms overlap with cardiac tamponade, hyper-resonant percussion note is more likely in tension pnemothorax
Flail chest
Chest wall disconnects from thoracic cage
Multiple rib fractures (at least two fractures per rib in at least two ribs)
Associated with pulmonary contusion
Abnormal chest motion
Avoid over hydration and fluid overload
Pneumothorax
Most common cause is lung laceration with air leakage
Most traumatic pneumothoraces should have a chest drain
Patients with traumatic pneumothorax should never be mechanically ventilated until a chest drain is inserted
Haemothorax
Most commonly due to laceration of lung, intercostal vessel or internal mammary artery
Haemothoraces large enough to appear on CXR are treated with large bore chest drain
Surgical exploration is warranted if >1500ml blood drained immediately
Cardiac tamponade
Beck’s triad: elevated venous pressure, reduced arterial pressure, reduced heart sounds
Pulsus paradoxus
May occur with as little as 100ml blood
Pulmonary contusion
Most common potentially lethal chest injury
Arterial blood gases and pulse oximetry important
Early intubation within an hour if significant hypoxia
Blunt cardiac injury
Usually occurs secondary to chest wall injury
ECG may show features of myocardial infarction
Sequelae: hypotension, arrhythmias, cardiac wall motion abnormalities
Aorta disruption
Deceleration injuries
Contained haematoma
Widened mediastinum
Diaphragm disruption
Most due to motor vehicle accidents and blunt trauma causing large radial tears (laceration injuries result in small tears)
More common on left side
Insert gastric tube, may pass into intrathoracic stomach
Mediastinal traversing wounds
Entrance wound in one hemithorax and exit wound/foreign body in opposite hemithorax
Mediastinal haematoma or pleural cap suggests great vessel injury
Mortality is 20%

How well did you know this?
1
Not at all
2
3
4
5
Perfectly
90
Q

A 19 year old man is stabbed in the chest at a nightclub. He develops a cardiac arrest in casualty following an attempted transfer to the CT scanning room

A. Thoracotomy
B. Manage conservatively
C. Intercostal tube drain insertion
D. CT scanning
E. Bronchoscopy
F. Negative pressure intercostal tube drainage
G. Video assisted thoracoscopy and pleurectomy

A

Thoracotomy

This is one indication for an ‘emergency room’ thoracotomy, there are not many others! Typical injuries include ventricular penetration, great vessel disruption and hilar lung injuries.

Thoracic trauma

Types of thoracic trauma

Tension pneumothorax
Often laceration to lung parenchyma with flap
Pressure develops in thorax
Most common cause is mechanical ventilation in patient with pleural injury
Symptoms overlap with cardiac tamponade, hyper-resonant percussion note is more likely in tension pnemothorax
Flail chest
Chest wall disconnects from thoracic cage
Multiple rib fractures (at least two fractures per rib in at least two ribs)
Associated with pulmonary contusion
Abnormal chest motion
Avoid over hydration and fluid overload
Pneumothorax
Most common cause is lung laceration with air leakage
Most traumatic pneumothoraces should have a chest drain
Patients with traumatic pneumothorax should never be mechanically ventilated until a chest drain is inserted
Haemothorax
Most commonly due to laceration of lung, intercostal vessel or internal mammary artery
Haemothoraces large enough to appear on CXR are treated with large bore chest drain
Surgical exploration is warranted if >1500ml blood drained immediately
Cardiac tamponade
Beck’s triad: elevated venous pressure, reduced arterial pressure, reduced heart sounds
Pulsus paradoxus
May occur with as little as 100ml blood
Pulmonary contusion
Most common potentially lethal chest injury
Arterial blood gases and pulse oximetry important
Early intubation within an hour if significant hypoxia
Blunt cardiac injury
Usually occurs secondary to chest wall injury
ECG may show features of myocardial infarction
Sequelae: hypotension, arrhythmias, cardiac wall motion abnormalities
Aorta disruption
Deceleration injuries
Contained haematoma
Widened mediastinum
Diaphragm disruption
Most due to motor vehicle accidents and blunt trauma causing large radial tears (laceration injuries result in small tears)
More common on left side
Insert gastric tube, may pass into intrathoracic stomach
Mediastinal traversing wounds
Entrance wound in one hemithorax and exit wound/foreign body in opposite hemithorax
Mediastinal haematoma or pleural cap suggests great vessel injury
Mortality is 20%

How well did you know this?
1
Not at all
2
3
4
5
Perfectly
91
Q

A 32 year old male falls over and sustains a small pneumothorax following a simple rib fracture. He has no physiological compromise.

A. Thoracotomy
B. Manage conservatively
C. Intercostal tube drain insertion
D. CT scanning
E. Bronchoscopy
F. Negative pressure intercostal tube drainage
G. Video assisted thoracoscopy and pleurectomy

A

Intercostal tube drain insertion

Unlike spontaneous pneumothoraces most would advocate chest tube drainage in the context of pneumothorax resulting from trauma. This is because of the risk of the lung laceration developing a tension. Once there is no further evidence of air leak the chest drain may be removed and a check x-ray performed to check there is no re-accumulation prior to discharge.

Thoracic trauma

Types of thoracic trauma

Tension pneumothorax
Often laceration to lung parenchyma with flap
Pressure develops in thorax
Most common cause is mechanical ventilation in patient with pleural injury
Symptoms overlap with cardiac tamponade, hyper-resonant percussion note is more likely in tension pnemothorax
Flail chest
Chest wall disconnects from thoracic cage
Multiple rib fractures (at least two fractures per rib in at least two ribs)
Associated with pulmonary contusion
Abnormal chest motion
Avoid over hydration and fluid overload
Pneumothorax
Most common cause is lung laceration with air leakage
Most traumatic pneumothoraces should have a chest drain
Patients with traumatic pneumothorax should never be mechanically ventilated until a chest drain is inserted
Haemothorax
Most commonly due to laceration of lung, intercostal vessel or internal mammary artery
Haemothoraces large enough to appear on CXR are treated with large bore chest drain
Surgical exploration is warranted if >1500ml blood drained immediately
Cardiac tamponade
Beck’s triad: elevated venous pressure, reduced arterial pressure, reduced heart sounds
Pulsus paradoxus
May occur with as little as 100ml blood
Pulmonary contusion
Most common potentially lethal chest injury
Arterial blood gases and pulse oximetry important
Early intubation within an hour if significant hypoxia
Blunt cardiac injury
Usually occurs secondary to chest wall injury
ECG may show features of myocardial infarction
Sequelae: hypotension, arrhythmias, cardiac wall motion abnormalities
Aorta disruption
Deceleration injuries
Contained haematoma
Widened mediastinum
Diaphragm disruption
Most due to motor vehicle accidents and blunt trauma causing large radial tears (laceration injuries result in small tears)
More common on left side
Insert gastric tube, may pass into intrathoracic stomach
Mediastinal traversing wounds
Entrance wound in one hemithorax and exit wound/foreign body in opposite hemithorax
Mediastinal haematoma or pleural cap suggests great vessel injury
Mortality is 20%

How well did you know this?
1
Not at all
2
3
4
5
Perfectly
92
Q

A 38 year old man is playing football when he slips over during a tackle. His knee is painful immediately following the fall. Several hours later he notices that the knee has become swollen. Following a course of non steroidal anti inflammatory drugs and rest the situation improves. However, complains of recurrent pain. On assessment in clinic you notice that it is impossible to fully extend the knee, although the patient is able to do so when asked.

A.	Anterior cruciate ligament rupture
B.	Posterior cruciate ligament rupture
C.	Medial collateral ligament tear
D.	Lateral collateral ligament tear
E.	Torn meniscus
F.	Chondromalacia patellae
G.	Dislocated patella
H.	Fractured patella
I.	Tibial plateau fracture
A

Torn meniscus

Twisting sporting injuries followed by delayed onset of knee swelling and locking are strongly suggestive of a menisceal tear. Arthroscopic menisectomy is the usual treatment.

Knee injury

Types of injury

Ruptured anterior cruciate ligament
Sport injury
Mechanism: high twisting force applied to a bent knee
Typically presents with: loud crack, pain and RAPID joint swelling (haemoarthrosis)
Poor healing
Management: intense physiotherapy or surgery
Ruptured posterior cruciate ligament
Mechanism: hyperextension injuries
Tibia lies back on the femur
Paradoxical anterior draw test
Rupture of medial collateral ligament
Mechanism: leg forced into valgus via force outside the leg
Knee unstable when put into valgus position
Menisceal tear
Rotational sporting injuries
Delayed knee swelling
Joint locking (Patient may develop skills to “unlock” the knee
Recurrent episodes of pain and effusions are common, often following minor trauma
Chondromalacia patellae
Teenage girls, following an injury to knee e.g. Dislocation patella
Typical history of pain on going downstairs or at rest
Tenderness, quadriceps wasting
Dislocation of the patella
Most commonly occurs as a traumatic primary event, either through direct trauma or through severe contraction of quadriceps with knee stretched in valgus and external rotation
Genu valgum, tibial torsion and high riding patella are risk factors
Skyline x-ray views of patella are required, although displaced patella may be clinically obvious
An osteochondral fracture is present in 5%
The condition has a 20% recurrence rate
Fractured patella
2 types:
i. Direct blow to patella causing undisplaced fragments
ii. Avulsion fracture
Tibial plateau fracture
Occur in the elderly (or following significant trauma in young)
Mechanism: knee forced into valgus or varus, but the knee fractures before the ligaments rupture
Varus injury affects medial plateau and if valgus injury, lateral plateau depressed fracture occurs
Classified using the Schatzker system (see below)

How well did you know this?
1
Not at all
2
3
4
5
Perfectly
93
Q

A 34 year old woman is a passenger in a car during an accident. Her knee hits the dashboard. On examination the tibia looks posterior compared to the non injured knee.

A.	Anterior cruciate ligament rupture
B.	Posterior cruciate ligament rupture
C.	Medial collateral ligament tear
D.	Lateral collateral ligament tear
E.	Torn meniscus
F.	Chondromalacia patellae
G.	Dislocated patella
H.	Fractured patella
I.	Tibial plateau fracture
A

Posterior cruciate ligament rupture

In ruptured posterior cruciate ligament the tibia lies back on the femur and can be drawn forward during a paradoxical draw test.

Knee injury

Types of injury

Ruptured anterior cruciate ligament
Sport injury
Mechanism: high twisting force applied to a bent knee
Typically presents with: loud crack, pain and RAPID joint swelling (haemoarthrosis)
Poor healing
Management: intense physiotherapy or surgery
Ruptured posterior cruciate ligament
Mechanism: hyperextension injuries
Tibia lies back on the femur
Paradoxical anterior draw test
Rupture of medial collateral ligament
Mechanism: leg forced into valgus via force outside the leg
Knee unstable when put into valgus position
Menisceal tear
Rotational sporting injuries
Delayed knee swelling
Joint locking (Patient may develop skills to “unlock” the knee
Recurrent episodes of pain and effusions are common, often following minor trauma
Chondromalacia patellae
Teenage girls, following an injury to knee e.g. Dislocation patella
Typical history of pain on going downstairs or at rest
Tenderness, quadriceps wasting
Dislocation of the patella
Most commonly occurs as a traumatic primary event, either through direct trauma or through severe contraction of quadriceps with knee stretched in valgus and external rotation
Genu valgum, tibial torsion and high riding patella are risk factors
Skyline x-ray views of patella are required, although displaced patella may be clinically obvious
An osteochondral fracture is present in 5%
The condition has a 20% recurrence rate
Fractured patella
2 types:
i. Direct blow to patella causing undisplaced fragments
ii. Avulsion fracture
Tibial plateau fracture
Occur in the elderly (or following significant trauma in young)
Mechanism: knee forced into valgus or varus, but the knee fractures before the ligaments rupture
Varus injury affects medial plateau and if valgus injury, lateral plateau depressed fracture occurs
Classified using the Schatzker system (see below)

How well did you know this?
1
Not at all
2
3
4
5
Perfectly
94
Q

A 28 year old professional footballer is admitted to the emergency department. During a tackle he is twisted with his knee flexed. He hears a loud crack and his knee rapidly becomes swollen.

A.	Anterior cruciate ligament rupture
B.	Posterior cruciate ligament rupture
C.	Medial collateral ligament tear
D.	Lateral collateral ligament tear
E.	Torn meniscus
F.	Chondromalacia patellae
G.	Dislocated patella
H.	Fractured patella
I.	Tibial plateau fracture
A

Anterior cruciate ligament rupture

This is common in footballers as the football boot studs stick to the ground and high twisting force is applied to a flexed knee. Rapid joint swelling also supports the diagnosis.

Knee injury

Types of injury

Ruptured anterior cruciate ligament
Sport injury
Mechanism: high twisting force applied to a bent knee
Typically presents with: loud crack, pain and RAPID joint swelling (haemoarthrosis)
Poor healing
Management: intense physiotherapy or surgery
Ruptured posterior cruciate ligament
Mechanism: hyperextension injuries
Tibia lies back on the femur
Paradoxical anterior draw test
Rupture of medial collateral ligament
Mechanism: leg forced into valgus via force outside the leg
Knee unstable when put into valgus position
Menisceal tear
Rotational sporting injuries
Delayed knee swelling
Joint locking (Patient may develop skills to “unlock” the knee
Recurrent episodes of pain and effusions are common, often following minor trauma
Chondromalacia patellae
Teenage girls, following an injury to knee e.g. Dislocation patella
Typical history of pain on going downstairs or at rest
Tenderness, quadriceps wasting
Dislocation of the patella
Most commonly occurs as a traumatic primary event, either through direct trauma or through severe contraction of quadriceps with knee stretched in valgus and external rotation
Genu valgum, tibial torsion and high riding patella are risk factors
Skyline x-ray views of patella are required, although displaced patella may be clinically obvious
An osteochondral fracture is present in 5%
The condition has a 20% recurrence rate
Fractured patella
2 types:
i. Direct blow to patella causing undisplaced fragments
ii. Avulsion fracture
Tibial plateau fracture
Occur in the elderly (or following significant trauma in young)
Mechanism: knee forced into valgus or varus, but the knee fractures before the ligaments rupture
Varus injury affects medial plateau and if valgus injury, lateral plateau depressed fracture occurs
Classified using the Schatzker system (see below)

How well did you know this?
1
Not at all
2
3
4
5
Perfectly
95
Q

A 65 year old diabetic female presents with a painless ulcer at the medial malleolus, it has been present for the past 16 years. On examination she has evidence of truncal varicosities and a brownish discolouration of the skin overlying the affected area.

A.	Mixed ulcer
B.	Chronic obliterative arterial disease
C.	Superficial venous insufficiency
D.	Deep venous insufficiency
E.	Neuropathic ulcer
F.	Basal cell carcinoma
G.	Squamous cell carcinoma
A

Superficial venous insufficiency

Venous ulcers are usually associated with features of venous insufficiency. These include haemosiderin deposition and varicose veins. Neuropathic ulcers will tend to present at sites of pressure, which is not typically at the medial malleolus.

Venous leg ulcers
Most due to venous hypertension, secondary to chronic venous insufficiency (other causes include calf pump dysfunction or neuromuscular disorders)
Ulcers form due to capillary fibrin cuff or leucocyte sequestration
Features of venous insufficiency include oedema, brown pigmentation, lipodermatosclerosis, eczema
Location above the ankle, painless
Deep venous insufficiency is related to previous DVT and superficial venous insufficiency is associated with varicose veins
Doppler ultrasound looks for presence of reflux and duplex ultrasound looks at the anatomy/ flow of the vein
Management: 4 layer compression banding after exclusion of arterial disease or surgery
If fail to heal after 12 weeks or >10cm2 skin grafting may be needed

Marjolin’s ulcer

Image sourced from Wikipedia

Squamous cell carcinoma
Occurring at sites of chronic inflammation e.g; burns, osteomyelitis after 10-20 years
Mainly occur on the lower limb

Arterial ulcers
Occur on the toes and heel
Painful
There may be areas of gangrene
Cold with no palpable pulses
Low ABPI measurements

Neuropathic ulcers
Commonly over plantar surface of metatarsal head and plantar surface of hallux
The plantar neuropathic ulcer is the condition that most commonly leads to amputation in diabetic patients
Due to pressure
Management includes cushioned shoes to reduce callus formation

Pyoderma gangrenosum

Image sourced from Wikipedia
Associated with inflammatory bowel disease/RA
Can occur at stoma sites
Erythematous nodules or pustules which ulcerate

How well did you know this?
1
Not at all
2
3
4
5
Perfectly
96
Q

A 71 year old man presents with a painful lower calf ulcer, mild pitting oedema and an ABPI of 0.3.

A.	Mixed ulcer
B.	Chronic obliterative arterial disease
C.	Superficial venous insufficiency
D.	Deep venous insufficiency
E.	Neuropathic ulcer
F.	Basal cell carcinoma
G.	Squamous cell carcinoma
A

Chronic obliterative arterial disease

Painful ulcers associated with a low ABPI are usually arterial in nature. The question does not indicate that features of chronic venous insufficiency are present. Patients may have mild pitting oedema as many vascular patients will also have ischaemic heart disease and elevated right heart pressures. The absence of more compelling signs of venous insufficiency makes a mixed ulcer less likely.

Venous leg ulcers
Most due to venous hypertension, secondary to chronic venous insufficiency (other causes include calf pump dysfunction or neuromuscular disorders)
Ulcers form due to capillary fibrin cuff or leucocyte sequestration
Features of venous insufficiency include oedema, brown pigmentation, lipodermatosclerosis, eczema
Location above the ankle, painless
Deep venous insufficiency is related to previous DVT and superficial venous insufficiency is associated with varicose veins
Doppler ultrasound looks for presence of reflux and duplex ultrasound looks at the anatomy/ flow of the vein
Management: 4 layer compression banding after exclusion of arterial disease or surgery
If fail to heal after 12 weeks or >10cm2 skin grafting may be needed

Marjolin’s ulcer

Image sourced from Wikipedia

Squamous cell carcinoma
Occurring at sites of chronic inflammation e.g; burns, osteomyelitis after 10-20 years
Mainly occur on the lower limb

Arterial ulcers
Occur on the toes and heel
Painful
There may be areas of gangrene
Cold with no palpable pulses
Low ABPI measurements

Neuropathic ulcers
Commonly over plantar surface of metatarsal head and plantar surface of hallux
The plantar neuropathic ulcer is the condition that most commonly leads to amputation in diabetic patients
Due to pressure
Management includes cushioned shoes to reduce callus formation

Pyoderma gangrenosum

Image sourced from Wikipedia
Associated with inflammatory bowel disease/RA
Can occur at stoma sites
Erythematous nodules or pustules which ulcerate

How well did you know this?
1
Not at all
2
3
4
5
Perfectly
97
Q

A 79 year old retired teacher has had an ulcer for 15 years. It is at the medial malleolus and has associated lipodermatosclerosis of the lower limb. The ulcer base is heaped up and irregular.

A.	Mixed ulcer
B.	Chronic obliterative arterial disease
C.	Superficial venous insufficiency
D.	Deep venous insufficiency
E.	Neuropathic ulcer
F.	Basal cell carcinoma
G.	Squamous cell carcinoma
A

Squamous cell carcinoma

If after many years an ulcer becomes heaped up and irregular, with rolled edges then suspect a squamous cell carcinoma.

Venous leg ulcers
Most due to venous hypertension, secondary to chronic venous insufficiency (other causes include calf pump dysfunction or neuromuscular disorders)
Ulcers form due to capillary fibrin cuff or leucocyte sequestration
Features of venous insufficiency include oedema, brown pigmentation, lipodermatosclerosis, eczema
Location above the ankle, painless
Deep venous insufficiency is related to previous DVT and superficial venous insufficiency is associated with varicose veins
Doppler ultrasound looks for presence of reflux and duplex ultrasound looks at the anatomy/ flow of the vein
Management: 4 layer compression banding after exclusion of arterial disease or surgery
If fail to heal after 12 weeks or >10cm2 skin grafting may be needed

Marjolin’s ulcer

Image sourced from Wikipedia

Squamous cell carcinoma
Occurring at sites of chronic inflammation e.g; burns, osteomyelitis after 10-20 years
Mainly occur on the lower limb

Arterial ulcers
Occur on the toes and heel
Painful
There may be areas of gangrene
Cold with no palpable pulses
Low ABPI measurements

Neuropathic ulcers
Commonly over plantar surface of metatarsal head and plantar surface of hallux
The plantar neuropathic ulcer is the condition that most commonly leads to amputation in diabetic patients
Due to pressure
Management includes cushioned shoes to reduce callus formation

Pyoderma gangrenosum

Image sourced from Wikipedia
Associated with inflammatory bowel disease/RA
Can occur at stoma sites
Erythematous nodules or pustules which ulcerate

How well did you know this?
1
Not at all
2
3
4
5
Perfectly
98
Q

Which of the following structures is not transmitted by the jugular foramen?

	Hypoglossal nerve
	Accessory nerve
	Internal jugular vein
	Inferior petrosal sinus
	Vagus nerve
A

Hypoglossal nerve

The jugular foramen may be divided into three compartments:
Anterior compartment transmits the inferior petrosal sinus
Middle compartment transmits cranial nerves IX, X and XI
Posterior compartment transmits the sigmoid sinus

Foramen ovale	Sphenoid bone	Otic ganglion
V3 (Mandibular nerve:3rd branch of 
trigeminal)
Accessory meningeal artery
Lesser petrosal nerve
Emissary veins

Foramen spinosum Sphenoid bone Middle meningeal artery
Meningeal branch of the Mandibular nerve
Foramen rotundum Sphenoid bone Maxillary nerve (V2)

Foramen lacerum/ carotid canal Sphenoid bone Base of the medial pterygoid plate.
Internal carotid artery*
Nerve and artery of the pterygoid canal

Jugular foramen Temporal bone Anterior: inferior petrosal sinus
Intermediate: glossopharyngeal, vagus, and accessory nerves.
Posterior: sigmoid sinus (becoming the internal jugular vein) and some meningeal branches from the occipital and ascending pharyngeal arteries.

Foramen magnum Occipital bone Anterior and posterior spinal arteries
Vertebral arteries
Medulla oblongata
Stylomastoid foramen Temporal bone Stylomastoid artery
Facial nerve

Superior orbital fissure Sphenoid bone Oculomotor nerve (III)
Recurrent meningeal artery
Trochlear nerve (IV)
Lacrimal, frontal and nasociliary branches of ophthalmic nerve (V1)
Abducent nerve (VI)
Superior ophthalmic vein

How well did you know this?
1
Not at all
2
3
4
5
Perfectly
99
Q

A 45 year old motor cyclist sustains a tibial fracture and is noted to have anaesthesia of the web space between his first and second toes. Which of the nerves listed below is most likely to be compromised?

	Superficial peroneal nerve
	Deep peroneal nerve
	Sural nerve
	Long saphenous nerve
	Tibial nerve
A

The deep peroneal nerve lies in the anterior muscular compartment of the lower leg and can be compromised by compartment syndrome affecting this area. It provides cutaneous sensation to the first web space. The superficial peroneal nerve provides more lateral cutaneous innervation.

Deep peroneal nerve

Origin From the common peroneal nerve, at the lateral aspect of the fibula, deep to peroneus longus
Nerve root values L4, L5, S1, S2
Course and relation
Pierces the anterior intermuscular septum to enter the anterior compartment of the lower leg
Passes anteriorly down to the ankle joint, midway between the two malleoli
Terminates In the dorsum of the foot
Muscles innervated
Tibialis anterior
Extensor hallucis longus
Extensor digitorum longus
Peroneus tertius
Extensor digitorum brevis
Cutaneous innervation Web space of the first and second toes
Actions
Dorsiflexion of ankle joint
Extension of all toes (extensor hallucis longus and extensor digitorum longus)
Inversion of the foot

After its bifurcation past the ankle joint, the lateral branch of the deep peroneal nerve innervates the extensor digitorum brevis and the extensor hallucis brevis
The medial branch supplies the web space between the first and second digits.

How well did you know this?
1
Not at all
2
3
4
5
Perfectly
100
Q

Damage to the accessory nerve during a lymph node excision biopsy.

A.	Teres major
B.	Brachialis
C.	Serratus anterior
D.	Trapezius
E.	Flexor digitorum profundus
F.	Biceps
G.	Supinator
H.	Adductor pollicis
I.	Abductor pollicis brevis
J.	Abductor digiti minimi
A

Trapezius
Nerve lesions during surgery

A variety of different procedures carry the risk of iatrogenic nerve injury. These are important not only from the patients perspective but also from a medicolegal standpoint.

The following operations and their associated nerve lesions are listed here:
Posterior triangle lymph node biopsy and accessory nerve lesion.
Lloyd Davies stirrups and common peroneal nerve.
Thyroidectomy and laryngeal nerve.
Anterior resection of rectum and hypogastric autonomic nerves.
Axillary node clearance; long thoracic nerve, thoracodorsal nerve and intercostobrachial nerve.
Inguinal hernia surgery and ilioinguinal nerve.
Varicose vein surgery- sural and saphenous nerves.
Posterior approach to the hip and sciatic nerve.
Carotid endarterectomy and hypoglossal nerve.

There are many more, with sound anatomical understanding of the commonly performed procedures the incidence of nerve lesions can be minimised. They commonly occur when surgeons operate in an unfamiliar tissue plane or by blind placement of haemostats (not recommended).

How well did you know this?
1
Not at all
2
3
4
5
Perfectly
101
Q

Damage to the median nerve during a carpal tunnel release.

A.	Teres major
B.	Brachialis
C.	Serratus anterior
D.	Trapezius
E.	Flexor digitorum profundus
F.	Biceps
G.	Supinator
H.	Adductor pollicis
I.	Abductor pollicis brevis
J.	Abductor digiti minimi
A

Abductor pollicis brevis

Adductor pollicis and abductor digiti minimi are innervated by the ulnar nerve

Nerve lesions during surgery

A variety of different procedures carry the risk of iatrogenic nerve injury. These are important not only from the patients perspective but also from a medicolegal standpoint.

The following operations and their associated nerve lesions are listed here:
Posterior triangle lymph node biopsy and accessory nerve lesion.
Lloyd Davies stirrups and common peroneal nerve.
Thyroidectomy and laryngeal nerve.
Anterior resection of rectum and hypogastric autonomic nerves.
Axillary node clearance; long thoracic nerve, thoracodorsal nerve and intercostobrachial nerve.
Inguinal hernia surgery and ilioinguinal nerve.
Varicose vein surgery- sural and saphenous nerves.
Posterior approach to the hip and sciatic nerve.
Carotid endarterectomy and hypoglossal nerve.

There are many more, with sound anatomical understanding of the commonly performed procedures the incidence of nerve lesions can be minimised. They commonly occur when surgeons operate in an unfamiliar tissue plane or by blind placement of haemostats (not recommended).

How well did you know this?
1
Not at all
2
3
4
5
Perfectly
102
Q

Injury to the radial nerve in a humeral shaft fracture.

A.	Teres major
B.	Brachialis
C.	Serratus anterior
D.	Trapezius
E.	Flexor digitorum profundus
F.	Biceps
G.	Supinator
H.	Adductor pollicis
I.	Abductor pollicis brevis
J.	Abductor digiti minimi
A

Supinator

Nerve lesions during surgery

A variety of different procedures carry the risk of iatrogenic nerve injury. These are important not only from the patients perspective but also from a medicolegal standpoint.

The following operations and their associated nerve lesions are listed here:
Posterior triangle lymph node biopsy and accessory nerve lesion.
Lloyd Davies stirrups and common peroneal nerve.
Thyroidectomy and laryngeal nerve.
Anterior resection of rectum and hypogastric autonomic nerves.
Axillary node clearance; long thoracic nerve, thoracodorsal nerve and intercostobrachial nerve.
Inguinal hernia surgery and ilioinguinal nerve.
Varicose vein surgery- sural and saphenous nerves.
Posterior approach to the hip and sciatic nerve.
Carotid endarterectomy and hypoglossal nerve.

There are many more, with sound anatomical understanding of the commonly performed procedures the incidence of nerve lesions can be minimised. They commonly occur when surgeons operate in an unfamiliar tissue plane or by blind placement of haemostats (not recommended).

How well did you know this?
1
Not at all
2
3
4
5
Perfectly
103
Q

A patient undergoes a femoral hernia repair and at operation the surgeon decides to enter the abdominal cavity to resect small bowel. She makes a transverse incision two thirds of the way between umbilicus and the symphysis pubis. Which of the structures listed below is least likely to be divided?

	Rectus abdominis
	External oblique aponeurosis
	Peritoneum
	Fascia transversalis
	Posterior lamina of the rectus sheath
A

Posterior lamina of the rectus sheath

An incision at this level lies below the arcuate line and the posterior wall of the rectus sheath is deficient at this level.

Rectus abdominis muscle

The rectus sheath is formed by the aponeuroses of the lateral abdominal wall muscles. The rectus sheath has a composition that varies according to anatomical level.

  1. Above the costal margin the anterior sheath is composed of external oblique aponeurosis, the costal cartilages are posterior to it.
  2. From the costal margin to the arcuate line, the anterior rectus sheath is composed of external oblique aponeurosis and the anterior part of the internal oblique aponeurosis. The posterior part of the internal oblique aponeurosis and transversus abdominis form the posterior rectus sheath.
  3. Below the arcuate line the aponeuroses of all the abdominal muscles lie in anterior aspect of the rectus sheath. Posteriorly lies the transversalis fascia and peritoneum.

The arcuate line is the point at which the inferior epigastric vessels enter the rectus sheath.

How well did you know this?
1
Not at all
2
3
4
5
Perfectly
104
Q

A 44 year old lady has undergone a mastectomy and axillary node clearance. Post operatively, she notices a patch of anaesthesia of her axillary skin when she applies an underarm deodorant.

A.	Medial pectoral nerve
B.	Thoracodorsal nerve
C.	Lateral pectoral nerve
D.	Intercostobrachial nerve
E.	Medial cord of the brachial plexus
F.	Long thoracic nerve
G.	Axillary nerve
H.	Accessory nerve
A

Intercostobrachial nerve

The intercostobrachial nerves traverse the axilla and innervate the overlying skin. These can be injured or divided during axillary surgery and the result is anaesthesia of the overlying skin.

Axilla

Boundaries of the axilla
Medially Chest wall and Serratus anterior
Laterally Humeral head
Floor Subscapularis
Anterior aspect Lateral border of Pectoralis major
Fascia Clavipectoral fascia

Content:
Long thoracic nerve (of Bell) Derived from C5-C7 and passes behind the brachial plexus to enter the axilla. It lies on the medial chest wall and supplies serratus anterior. Its location puts it at risk during axillary surgery and damage will lead to winging of the scapula.
Thoracodorsal nerve and thoracodorsal trunk Innervate and vascularise latissimus dorsi.
Axillary vein Lies at the apex of the axilla, it is the continuation of the basilic vein. Becomes the subclavian vein at the outer border of the first rib.
Intercostobrachial nerves Traverse the axillary lymph nodes and are often divided during axillary surgery. They provide cutaneous sensation to the axillary skin.
Lymph nodes The axilla is the main site of lymphatic drainage for the breast.

How well did you know this?
1
Not at all
2
3
4
5
Perfectly
105
Q

A 44 year old lady has undergone a mastectomy and axillary node clearance to treat breast cancer. Post operatively, it is noted that she has winging of the scapula.

A.	Medial pectoral nerve
B.	Thoracodorsal nerve
C.	Lateral pectoral nerve
D.	Intercostobrachial nerve
E.	Medial cord of the brachial plexus
F.	Long thoracic nerve
G.	Axillary nerve
H.	Accessory nerve
A

Long thoracic nerve

Injury to the long thoracic nerve (which innervates the serratus anterior) can occur as it lies at the medial aspect of the axilla, winging of the scapula will then result.

Axilla

Boundaries of the axilla
Medially Chest wall and Serratus anterior
Laterally Humeral head
Floor Subscapularis
Anterior aspect Lateral border of Pectoralis major
Fascia Clavipectoral fascia

Content:
Long thoracic nerve (of Bell) Derived from C5-C7 and passes behind the brachial plexus to enter the axilla. It lies on the medial chest wall and supplies serratus anterior. Its location puts it at risk during axillary surgery and damage will lead to winging of the scapula.
Thoracodorsal nerve and thoracodorsal trunk Innervate and vascularise latissimus dorsi.
Axillary vein Lies at the apex of the axilla, it is the continuation of the basilic vein. Becomes the subclavian vein at the outer border of the first rib.
Intercostobrachial nerves Traverse the axillary lymph nodes and are often divided during axillary surgery. They provide cutaneous sensation to the axillary skin.
Lymph nodes The axilla is the main site of lymphatic drainage for the breast.

How well did you know this?
1
Not at all
2
3
4
5
Perfectly
106
Q

A 44 year old lady who works as an interior decorator has undergone a mastectomy and axillary node clearance to treat breast cancer. Post operatively, she comments that her arm easily becomes fatigued when she is painting walls.

A.	Medial pectoral nerve
B.	Thoracodorsal nerve
C.	Lateral pectoral nerve
D.	Intercostobrachial nerve
E.	Medial cord of the brachial plexus
F.	Long thoracic nerve
G.	Axillary nerve
H.	Accessory nerve
A

Thoracodorsal nerve

The most likely explanation for this is that the thoracodorsal nerve has been injured. This will result in atrophy of latissimus dorsi and this will become evident with repetitive arm movements where the arm is elevated and moving up and down (such as in painting). Injury to the pectoral nerves may produce a similar picture but this pattern of injury is very rare and the pectoral nerves are seldom injured in breast surgery.

Axilla

Boundaries of the axilla
Medially Chest wall and Serratus anterior
Laterally Humeral head
Floor Subscapularis
Anterior aspect Lateral border of Pectoralis major
Fascia Clavipectoral fascia

Content:
Long thoracic nerve (of Bell) Derived from C5-C7 and passes behind the brachial plexus to enter the axilla. It lies on the medial chest wall and supplies serratus anterior. Its location puts it at risk during axillary surgery and damage will lead to winging of the scapula.
Thoracodorsal nerve and thoracodorsal trunk Innervate and vascularise latissimus dorsi.
Axillary vein Lies at the apex of the axilla, it is the continuation of the basilic vein. Becomes the subclavian vein at the outer border of the first rib.
Intercostobrachial nerves Traverse the axillary lymph nodes and are often divided during axillary surgery. They provide cutaneous sensation to the axillary skin.
Lymph nodes The axilla is the main site of lymphatic drainage for the breast.

How well did you know this?
1
Not at all
2
3
4
5
Perfectly
107
Q

A 35 year old farm labourer injures the posterior aspect of his hand with a mechanical scythe. He severs some of his extensor tendons in this injury. How many tunnels lie in the extensor retinaculum that transmit the tendons of the extensor muscles?

	One
	Three
	Four
	Five
	Six
A

There are six tunnels, each lined by its own synovial sheath.

Extensor retinaculum

The extensor rentinaculum is a thickening of the deep fascia that stretches across the back of the wrist and holds the long extensor tendons in position.
Its attachments are:
The pisiform and triquetral medially
The end of the radius laterally

Structures related to the extensor retinaculum
Structures superficial to the retinaculum
Basilic vein
Dorsal cutaneous branch of the ulnar nerve
Cephalic vein
Superficial branch of the radial nerve
Structures passing deep to the extensor retinaculum
Extensor carpi ulnaris tendon
Extensor digiti minimi tendon
Extensor digitorum and extensor indicis tendon
Extensor pollicis longus tendon
Extensor carpi radialis longus tendon
Extensor carpi radialis brevis tendon
Abductor pollicis longus and extensor pollicis brevis tendons

Beneath the extensor retinaculum fibrous septa form six compartments that contain the extensor muscle tendons. Each compartment has its own synovial sheath.

The radial artery
The radial artery passes between the lateral collateral ligament of the wrist joint and the tendons of the abductor pollicis longus and extensor pollicis brevis.

How well did you know this?
1
Not at all
2
3
4
5
Perfectly
108
Q

Which of the muscles listed below is not innervated by the median nerve?

	Flexor pollicis brevis
	Lateral two lumbricals
	Pronator teres
	Opponens pollicis
	Adductor pollicis
A

Adductor pollicis is innervated by the ulnar nerve.
Medial two lumbricals innervated by the ulnar nerve.

Median nerve

The median nerve is formed by the union of a lateral and medial root respectively from the lateral (C5,6,7) and medial (C8 and T1) cords of the brachial plexus; the medial root passes anterior to the third part of the axillary artery. The nerve descends lateral to the brachial artery, crosses to its medial side (usually passing anterior to the artery). It passes deep to the bicipital aponeurosis and the median cubital vein at the elbow.
It passes between the two heads of the pronator teres muscle, and runs on the deep surface of flexor digitorum superficialis (within its fascial sheath).
Near the wrist it becomes superficial between the tendons of flexor digitorum superficialis and flexor carpi radialis, deep to palmaris longus tendon. It passes deep to the flexor retinaculum to enter the palm, but lies anterior to the long flexor tendons within the carpal tunnel.

Branches
Region	Branch
Upper arm	No branches, although the nerve commonly communicates with the musculocutaneous nerve
Forearm	Pronator teres
Flexor carpi radialis
Palmaris longus
Flexor digitorum superficialis
Flexor pollicis longus
Flexor digitorum profundus (only the radial half)
Distal forearm	Palmar cutaneous branch
Hand (Motor)	Motor supply (LOAF)
Lateral 2 lumbricals
Opponens pollicis
Abductor pollicis brevis
Flexor pollicis brevis
Hand (Sensory)	
Over thumb and lateral 2 ½ fingers
On the palmar aspect this projects proximally, on the dorsal aspect only the distal regions are innervated with the radial nerve providing the more proximal cutaneous innervation.

Patterns of damage
Damage at wrist
e.g. carpal tunnel syndrome
paralysis and wasting of thenar eminence muscles and opponens pollicis (ape hand deformity)
sensory loss to palmar aspect of lateral (radial) 2 ½ fingers

Damage at elbow, as above plus:
unable to pronate forearm
weak wrist flexion
ulnar deviation of wrist

Anterior interosseous nerve (branch of median nerve)
leaves just below the elbow
results in loss of pronation of forearm and weakness of long flexors of thumb and index finger

How well did you know this?
1
Not at all
2
3
4
5
Perfectly
109
Q

A 44 year old lady is undergoing an abdominal hysterectomy and the ureter is identified during the ligation of the uterine artery. At which site does it insert into the bladder?

	Posterior
	Apex
	Anterior
	Base
	Superior aspect of the lateral side
A

The ureters enter the bladder at the upper lateral aspect of the base of the bladder. They are about 5cm apart from each other in the empty bladder. Internally this aspect is contained within the bladder trigone.

Ureter

25-35 cm long
Muscular tube lined by transitional epithelium
Surrounded by thick muscular coat. Becomes 3 muscular layers as it crosses the bony pelvis
Retroperitoneal structure overlying transverse processes L2-L5
Lies anterior to bifurcation of iliac vessels
Blood supply is segmental; renal artery, aortic branches, gonadal branches, common iliac and internal iliac
Lies beneath the uterine artery

How well did you know this?
1
Not at all
2
3
4
5
Perfectly
110
Q

An injured axillary artery is ligated between the thyrocervical trunk of the subclavian and subscapular artery. Subsequent collateral circulation is likely to result in reversal of blood flow in which of the vessels listed below?

	Circumflex scapular artery
	Transverse cervical artery
	Posterior intercostal arteries
	Suprascapular artery
	Profunda brachii artery
A

The circumflex scapular artery is a branch of the subscapular artery and normally supplies the muscle on the dorsal aspect of the scapula. In this instance, flow is reversed in the circumflex scapular and subscapular arteries forming a collateral circulation around the scapula.

It’s an easy question really, we just made the wording difficult (on purpose). It is asking about the branches of the axillary artery and knowledge of the fact that there is an extensive collateral network around the shoulder joint. As a result, the occlusion of the proximal aspect of the circumflex humeral inflow (from the axillary artery) ceases and there is then retrograde flow through it from collaterals.

Axillary artery

The axillary artery extends from the outer border of the first rib to the lower border of teres major, where it becomes the brachial artery. The vessel is subdivided into three zones; the first part lies above pectoralis minor, the second part is behind the muscle and the third part lies inferior to it.

First part
Together with the axillary vein, the artery is enclosed within the cords of the brachial plexus. Both vessels are contained within the axillary sheath, a prolongation of the prevertebral fascia. Posteriomedial to the sheath lies the first intercostal space, the superior aspect of the serratus anterior and the long thoracic nerve. Within the sheath, the medial cord of the brachial plexus lies behind the artery. Anteriorly lies the clavipectoral fascia. Superolaterally, lie the lateral and posterior cords of the brachial plexus. Inferomedially lies the axillary vein.

Second part
Posterior to the second part lies the posterior cord of the brachial plexus and the subscapularis muscle. Anteriorly, lie pectoralis minor and major. The lateral cord of the brachial plexus lies laterally. Medially, lies the medial cord of the brachial plexus, here it separates the artery from the vein.

Third part
Posterior to the artery lie suscapularis, latissimus dorsi and teres major. Interspersed between the vessel and subscapularis are the axillary and radial nerves. Anterior to the vessel is the medial root of the median nerve. Laterally, the lies the median and musculocutaneous nerves and coracobrachialis. The axillary vein is related medially.

Branches of the axillary artery
Highest thoracic artery
Thoraco-acromial artery
Lateral thoracic artery
Subscapular artery
Posterior circumflex humeral artery
Anterior circumflex humeral artery
How well did you know this?
1
Not at all
2
3
4
5
Perfectly
111
Q

A 20 year old lady presents with pain on the medial aspect of her thigh. Investigations show a large ovarian cyst. Compression of which of the nerves listed below is the most likely underlying cause?

	Sciatic
	Genitofemoral
	Obturator
	Ilioinguinal
	Femoral cutaneous
A

Obturator nerve

The cutaneous branch of the obturator nerve is frequently absent. However, the obturator nerve is a recognised contributor to innervation of the medial thigh and large pelvic tumours may compress this nerve with resultant pain radiating distally.

The obturator nerve arises from L2, L3 and L4 by branches from the ventral divisions of each of these nerve roots. L3 forms the main contribution and the second lumbar branch is occasionally absent. These branches unite in the substance of psoas major, descending vertically in its posterior part to emerge from its medial border at the lateral margin of the sacrum. It then crosses the sacroiliac joint to enter the lesser pelvis, it descends on obturator internus to enter the obturator groove. In the lesser pelvis the nerve lies lateral to the internal iliac vessels and ureter, and is joined by the obturator vessels lateral to the ovary or ductus deferens.

Supplies
Medial compartment of thigh
Muscles supplied: external obturator, adductor longus, adductor brevis, adductor magnus (not the lower part-sciatic nerve), gracilis
The cutaneous branch is often absent. When present, it passes between gracilis and adductor longus near the middle part of the thigh, and supplies the skin and fascia of the distal two thirds of the medial aspect.

Obturator canal
Connects the pelvis and thigh: contains the obturator artery, vein, nerve which divides into anterior and posterior branches.

How well did you know this?
1
Not at all
2
3
4
5
Perfectly
112
Q

A 21 year old man undergoes surgical removal of an impacted 3rd molar. Post operatively, he is noted to have anaesthesia on the anterolateral aspect of the tongue. What is the most likely explanation?

	Injury to the hypoglossal nerve
	Injury to the inferior alveolar nerve
	Injury to the lingual nerve
	Injury to the mandibular branch of the facial nerve
	Injury to the glossopharyngeal nerve
A

The lingual nerve is closely related to the third molar and up to 10% of patients undergoing surgical extraction of these teeth may subsequently develop a lingual neuropraxia. The result is anaesthesia of the ipsilateral anterior aspect of the tongue. The inferior alveolar nerve innervates the teeth themselves.

Lingual nerve

Sensory nerve to the mucosa of the presulcal part of the tongue, floor of mouth and mandibular lingual gingivae
Arises from posterior trunk of the mandibular nerve (branch of trigeminal)
Course runs past tensor veli palatini and lateral pterygoid (where it is joined by the chorda tympani branch of the facial nerve). Emerging from the cover of the lateral pterygoid it proceeds antero inferiorly lying on the surface of the medial pterygoid and lies close to the medial aspect of the mandibular ramus. At the junction of the vertical and horizontal rami of the mandible it is anterior to the inferior alveolar nerve. It then passes below the mandibular attachment of the superior pharyngeal constrictor. Eventually, it lies on the periosteum of the root of the third molar tooth. It then passes medial to the mandibular origin of mylohyoid and then passes forwards on the inferior surface of this muscle

How well did you know this?
1
Not at all
2
3
4
5
Perfectly
113
Q

What is the most important structure involved in supporting the uterus?

	Round ligament
	Broad ligament
	Uterosacral ligaments
	Cardinal ligaments
	Central perineal tendon
A

The central perineal tendon provides the main structural support to the uterus. Damage to this structure is commonly associated with the development of pelvic organ prolapse, even when other structures are intact.
Uterus

The non pregnant uterus resides entirely within the pelvis. The peritoneum invests the uterus and the structure is contained within the peritoneal cavity. The blood supply to the uterine body is via the uterine artery (branch of the internal iliac). The uterine artery passes from the inferior aspect of the uterus (lateral to the cervix) and runs alongside the uterus. It frequently anastomoses with the ovarian artery superiorly. Inferolaterally the ureter is a close relation and ureteric injuries are a recognised complication when pathology brings these structures into close proximity.

The supports of the uterus include the central perineal tendon (the most important). The lateral cervical, round and uterosacral ligaments are condensations of the endopelvic fascia and provide additional structural support.

How well did you know this?
1
Not at all
2
3
4
5
Perfectly
114
Q

A 34 year old lady suffers from hyperparathyroidism. The right inferior parathyroid is identified as having an adenoma and is scheduled for resection. From which of the following embryological structures is it derived?

	Second pharyngeal pouch
	Third pharyngeal pouch
	Fourth pharyngeal pouch
	First pharyngeal pouch
	None of the above
A

The inferior parathyroid is a derivative of the third pharyngeal pouch. The superior parathyroid originates from the fourth pharyngeal pouch.

Parathyroid glands- anatomy

Four parathyroid glands
Located posterior to the thyroid gland
They lie within the pretracheal fascia

Embryology
The parathyroids develop from the extremities of the third and fourth pharyngeal pouches. The parathyroids derived from the fourth pharyngeal pouch are located more superiorly and are associated with the thyroid gland. Those derived from the third pharyngeal pouch lie more inferiorly and may become associated with the thymus.

Blood supply
The blood supply to the parathyroid glands is derived from the inferior and superior thyroid arteries[1]. There is a rich anastomosis between the two vessels. Venous drainage is into the thyroid veins.

Relations
Laterally	Common carotid
Medially	Recurrent laryngeal nerve, trachea
Anterior	Thyroid
Posterior	Pretracheal fascia
How well did you know this?
1
Not at all
2
3
4
5
Perfectly
115
Q

A 23 year old man falls and slips at a nightclub. A shard of glass penetrates the skin at the level of the medial epicondyle, which of the following sequelae is least likely to occur?

Atrophy of the first dorsal interosseous muscle
Difficulty in abduction of the the 2nd, 3rd, 4th and 5th fingers
Claw like appearance of the hand
Loss of sensation on the anterior aspect of the 5th finger
Partial denervation of flexor digitorum profundus
A

Claw like appearance of the hand

Injury to the ulnar nerve in the mid to distal forearm will typically produce a claw hand. This consists of flexion of the 4th and 5th interphalangeal joints and extension of the metacarpophalangeal joints. The effects are potentiated when flexor digitorum profundus is not affected, and the clawing is more pronounced.More proximally sited ulnar nerve lesions produce a milder clinical picture owing to the simultaneous paralysis of flexor digitorum profundus (ulnar half).

This is the ‘ulnar paradox’, due to the more proximal level of transection the hand will typically not have a claw like appearance that may be seen following a more distal injury. The first dorsal interosseous muscle will be affected as it is supplied by the ulnar nerve.

Effects of injury
Damage at the wrist
Wasting and paralysis of intrinsic hand muscles (claw hand)
Wasting and paralysis of hypothenar muscles
Loss of sensation medial 1 and half fingers
Damage at the elbow
Radial deviation of the wrist
Clawing less in 4th and 5th digits
Ulnar nerve

Origin
C8, T1

Supplies (no muscles in the upper arm)
Flexor carpi ulnaris
Flexor digitorum profundus
Flexor digiti minimi
Abductor digiti minimi
Opponens digiti minimi
Adductor pollicis
Interossei muscle
Third and fourth lumbricals
Palmaris brevis

Path
Posteromedial aspect of upper arm to flexor compartment of forearm, then along the ulnar. Passes beneath the flexor carpi ulnaris muscle, then superficially through the flexor retinaculum into the palm of the hand.

How well did you know this?
1
Not at all
2
3
4
5
Perfectly
116
Q

A 78 year old man is due to undergo an endarterectomy of the internal carotid artery. Which of the following nervous structures are most at risk during the dissection?

	Recurrent laryngeal nerve
	Sympathetic chain
	Hypoglossal nerve
	Phrenic nerve
	Lingual nerve
A

Nerves at risk during a carotid endarterectomy:
Hypoglossal nerve
Greater auricular nerve
Superior laryngeal nerve

During a carotid endarterectomy the sternocleidomastoid muscle is dissected, with ligation of the common facial vein and then the internal jugular is dissected exposing the common and the internal carotid arteries. The nerves at risk during the operation include:
Hypoglossal nerve
Greater auricular nerve
Superior laryngeal nerve
The sympathetic chain lies posteriorly and is less prone to injury in this procedure.

The internal carotid artery is formed from the common carotid opposite the upper border of the thyroid cartilage. It extends superiorly to enter the skull via the carotid canal. From the carotid canal it then passes through the cavernous sinus, above which it divides into the anterior and middle cerebral arteries.

Relations in the carotid canal
Internal carotid plexus
Cochlea and middle ear cavity
Trigeminal ganglion (superiorly)
Leaves canal lies above the foramen lacerum

Path and relations in the cranial cavity
The artery bends sharply forwards in the cavernous sinus, the aducens nerve lies close to its inferolateral aspect. The oculomotor, trochlear, opthalmic and, usually, the maxillary nerves lie in the lateral wall of the sinus. Near the superior orbital fissure it turns posteriorly and passes postero-medially to pierce the roof of the cavernous sinus inferior to the optic nerve. It then passes between the optic and oculomotor nerves to terminate below the anterior perforated substance by dividing into the anterior and middle cerebral arteries.

Branches
Anterior and middle cerebral artery
Ophthalmic artery
Posterior communicating artery
Anterior choroid artery
Meningeal arteries
Hypophyseal arteries
How well did you know this?
1
Not at all
2
3
4
5
Perfectly
117
Q

Which of the structures listed below articulates with the head of the radius superiorly?

	Capitulum
	Trochlea
	Lateral epicondyle
	Ulna
	Medial epicondyle
A

The head of the radius articulates with the capitulum of the humerus.

Radius

The radius is one of the two long forearm bones that extends from the lateral side of the elbow to the thumb side of the wrist. It has two expanded ends, of which the distal end is the larger. Key points relating to its topography and relations are outlined below;

Upper end
Articular cartilage- covers medial > lateral side
Articulates with radial notch of the ulna by the annular ligament
Muscle attachment- biceps brachii at the tuberosity

Shaft
Muscle attachment
Upper third of the body	Supinator
Flexor digitorum superficialis
Flexor pollicis longus
Middle third of the body	Pronator teres
Lower quarter of the body	Pronator quadratus
Tendon of supinator longus
Lower end
Quadrilateral
Anterior surface- capsule of wrist joint
Medial surface- head of ulna
Lateral surface- ends in the styloid process
Posterior surface: 3 grooves containing:
1. Tendons of extensor carpi radialis longus and brevis 
2. Tendon of extensor pollicis longus 
3. Tendon of extensor indicis
How well did you know this?
1
Not at all
2
3
4
5
Perfectly
118
Q

Which of the following fascial structures encases the apex of the lungs?

	Waldeyers fascia
	Sibsons fascia
	Pretracheal fascia
	Clavipectoral fascia
	None of the above
A

Sibson’s fascia overlies the apices of both lungs

The suprapleural fascia (Sibson’s fascia) runs from C7 to the first rib and overlies the apex of both lungs.It lies between the parietal pleura and the thoracic cage.

Lung anatomy

The right lung is composed of 3 lobes divided by the oblique and transverse fissures. The left lung has two lobes divided by the oblique fissure.The apex of both lungs is approximately 4cm superior to the sterno-costal joint of the first rib. Immediately below this is a sulcus created by the subclavian artery.

Peripheral contact points of the lung
Base: diaphragm
Costal surface: corresponds to the cavity of the chest
Mediastinal surface: Contacts the mediastinal pleura. Has the cardiac impression. Above and behind this concavity is a triangular depression named the hilum, where the structures which form the root of the lung enter and leave the viscus. These structures are invested by pleura, which, below the hilum and behind the pericardial impression, forms the pulmonary ligament

Right lung
Above the hilum is the azygos vein; Superior to this is the groove for the superior vena cava and right innominate vein; behind this, and nearer the apex, is a furrow for the innominate artery. Behind the hilum and the attachment of the pulmonary ligament is a vertical groove for the oesophagus; In front and to the right of the lower part of the oesophageal groove is a deep concavity for the extrapericardiac portion of the inferior vena cava.

The root of the right lung lies behind the superior vena cava and the right atrium, and below the azygos vein.

The right main bronchus is shorter, wider and more vertical than the left main bronchus and therefore the route taken by most foreign bodies.

Image sourced from Wikipedia

Left lung
Above the hilum is the furrow produced by the aortic arch, and then superiorly the groove accommodating the left subclavian artery; Behind the hilum and pulmonary ligament is a vertical groove produced by the descending aorta, and in front of this, near the base of the lung, is the lower part of the oesophagus.

The root of the left lung passes under the aortic arch and in front of the descending aorta.

Image sourced from Wikipedia

Inferior borders of both lungs
6th rib in mid clavicular line
8th rib in mid axillary line
10th rib posteriorly
The pleura runs two ribs lower than the corresponding lung level.
How well did you know this?
1
Not at all
2
3
4
5
Perfectly
119
Q

As regards the internal jugular vein, which of the following statements is untrue?

It lies within the carotid sheath
It is the continuation of the sigmoid sinus
The terminal part of the thoracic duct crosses anterior to it to insert into the right subclavian vein
The hypoglossal nerve is closely related to it as it passes near the atlas
The vagus nerve is closely related to it within the carotid sheath
A

The terminal part of the thoracic duct crosses anterior to it to insert into the right subclavian vein

Internal jugular vein

Each jugular vein begins in the jugular foramen, where they are the continuation of the sigmoid sinus. They terminate at the medial end of the clavicle where they unite with the subclavian vein.

The vein lies within the carotid sheath throughout its course. Below the skull the internal carotid artery and last four cranial nerves are anteromedial to the vein. Thereafter it is in contact medially with the internal (then common) carotid artery. The vagus lies posteromedially.

At its superior aspect, the vein is overlapped by sternocleidomastoid and covered by it at the inferior aspect of the vein.

Below the transverse process of the atlas it is crossed on its lateral side by the accessory nerve. At its mid point it is crossed by the inferior root of the ansa cervicalis.
Posterior to the vein are the transverse processes of the cervical vertebrae, the phenic nerve as it descends on the scalenus anterior, and the first part of the subclavian artery.

On the left side its also related to the thoracic duct.

How well did you know this?
1
Not at all
2
3
4
5
Perfectly
120
Q

At the level of the wrist joint, which of the statements below best describes the relationship of the ulnar artery to the ulnar nerve?

	It lies on its radial side
	It lies deep to it
	It lies superficial to it
	It lies on its ulnar side
	None of the above
A

It lies on its radial side

In the middle of the forearm, the artery is overlapped by the flexor carpi ulnaris and on the flexor retinaculum it is covered by a superficial layer from that structure. In its distal two-thirds, flexor digitorum superficialis lies on its radial side, and the ulnar nerve is situated on its ulnar side.

Ulnar artery

Path
Starts: middle of antecubital fossa
Passes obliquely downward, reaching the ulnar side of the forearm at a point about midway between the elbow and the wrist. It follows the ulnar border to the wrist, crossing over the flexor retinaculum. It then divides into the superficial and deep volar arches.

Relations
Deep to- Pronator teres, Flexor carpi radialis, Palmaris longus
Lies on- Brachialis and Flexor digitorum profundus
Superficial to the flexor retinaculum at the wrist

The median nerve is in relation with the medial side of the artery for about 2.5 cm. And then crosses the vessel, being separated from it by the ulnar head of the Pronator teres

The ulnar nerve lies medially to the lower two-thirds of the artery

Branch
Anterior interosseous artery

How well did you know this?
1
Not at all
2
3
4
5
Perfectly
121
Q

Which of the following anatomical structures lies within the spiral groove of the humerus?

	Median nerve
	Radial nerve
	Tendon of triceps
	Musculocutaneous nerve
	Axillary nerve
A

The radial nerve lies in this groove and may be compromised by fractures involving the shaft.

Humerus

The humerus extends from the scapula to the elbow joint. It has a body and two ends. It is almost completely covered with muscle but can usually be palpated throughout its length. The smooth rounded surface of the head articulates with the shallow glenoid cavity. The head is connected to the body of the humerus by the anatomical neck. The surgical neck is the region below the head and tubercles and where they join the shaft and is the commonest site of fracture. The capsule of the shoulder joint is attached to the anatomical neck superiorly but extends down to 1.5cm on the surgical neck.

The greater tubercle is the prominence on the lateral side of the upper end of the bone. It merges with the body below and can be felt through the deltoid inferior to the acromion. The tendons of the supraspinatus and infraspinatus are inserted into impressions on its superior aspect. The lesser tubercle is a distinct prominence on the front of the upper end of the bone. It can be palpated through the deltoid just lateral to the tip of the coracoid process.

The intertubercular groove passes on the body between the greater and lesser tubercles, continuing down from the anterior borders of the tubercles to form the edges of the groove. The tendon of biceps within its synovial sheath passes through this groove, held within it by a transverse ligament.

The posterior surface of the body is marked by a spiral groove for the radial nerve which runs obliquely across the upper half of the body to reach the lateral border below the deltoid tuberosity. Within this groove lie the radial nerve and brachial vessels and both may be affected by fractures involving the shaft of the humerus.

The lower end of the humerus is wide and flattened anteroposteriorly, and inclined anteriorly. The middle third of the distal edge forms the trochlea. Superior to this are indentations for the coronoid fossa anteriorly and olecranon fossa posteriorly. Lateral to the trochlea is a rounded capitulum which articulates with the radius.

The medial epicondyle is very prominent with a smooth posterior surface which contains a sulcus for the ulnar nerve and collateral vessels. It’s distal margin gives attachment for the ulnar collateral ligament and, in front of this, the anterior surface has an impression for the common flexor tendon.

How well did you know this?
1
Not at all
2
3
4
5
Perfectly
122
Q

A 24 year old man falls and sustains a fracture through his scaphoid bone. From which of the following areas does the scaphoid derive the majority of its blood supply?

	From its proximal medial border
	From its proximal lateral border
	From its proximal posterior surface
	From the proximal end
	From the distal end
A

The blood supply to the scaphoid enters from a small non articular surface near its distal end. Transverse fractures through the scaphoid therefore carry a risk of non union.

Scaphoid bone

The scaphoid has a concave articular surface for the head of the capitate and at the edge of this is a crescentic surface for the corresponding area on the lunate.
Proximally, it has a wide convex articular surface with the radius. It has a distally sited tubercle that can be palpated. The remaining articular surface is to the lateral side of the tubercle. It faces laterally and is associated with the trapezium and trapezoid bones.

The narrow strip between the radial and trapezial surfaces and the tubercle gives rise to the radial collateral carpal ligament. The tubercle receives part of the flexor retinaculum. This area is the only part of the scaphoid that is available for the entry of blood vessels. It is commonly fractured and avascular necrosis may result.

How well did you know this?
1
Not at all
2
3
4
5
Perfectly
123
Q

Which of the following laryngeal tumours will not typically metastasise to the cervical lymph nodes?

	Glottic
	Supraglottic
	Subglottic
	Transglottic
	Aryepiglottic fold
A

Glottic
The vocal cords have no lymphatic drainage and therefore this region serves as a lymphatic watershed. The supraglottic part drains to the upper deep cervical nodes through vessels piercing the thyrohyoid membrane. The sub glottic part drains to the pre laryngeal, pre tracheal and inferior deep cervical nodes. The aryepiglottic and vestibular folds have a rich lymphatic drainage and will metastasise early.

Larynx

The larynx lies in the anterior part of the neck at the levels of C3 to C6 vertebral bodies. The laryngeal skeleton consists of a number of cartilagenous segments. Three of these are paired; arytenoid, corniculate and cuneiform. Three are single; thyroid, cricoid and epiglottic. The cricoid cartilage forms a complete ring (the only one to do so).
The laryngeal cavity extends from the laryngeal inlet to the level of the inferior border of the cricoid cartilage.

Divisions of the laryngeal cavity
Laryngeal vestibule Superior to the vestibular folds
Laryngeal ventricle Lies between vestibular folds and superior to the vocal cords
Infraglottic cavity Extends from vocal cords to inferior border of the cricoid cartilage

The vocal folds (true vocal cords) control sound production. The apex of each fold projects medially into the laryngeal cavity. Each vocal fold includes:
Vocal ligament
Vocalis muscle (most medial part of thyroarytenoid muscle)
The glottis is composed of the vocal folds, processes and rima glottidis. The rima glottidis is the narrowest potential site within the larynx, as the vocal cords may be completely opposed, forming a complete barrier.
Blood supply
Arterial supply is via the laryngeal arteries, branches of the superior and inferior thyroid arteries. The superior laryngeal artery is closely related to the internal laryngeal nerve. The inferior laryngeal artery is related to the inferior laryngeal nerve. Venous drainage is via superior and inferior laryngeal veins, the former draining into the superior thyroid vein and the latter draining into the middle thyroid vein, or thyroid venous plexus.

Lymphatic drainage
The vocal cords have no lymphatic drainage and this site acts as a lymphatic watershed.
Supraglottic part Upper deep cervical nodes
Subglottic part Prelaryngeal and pretracheal nodes and inferior deep cervical nodes
The aryepiglottic fold and vestibular folds have a dense plexus of lymphatics associated with them and malignancies at these sites have a greater propensity for nodal metastasis.

How well did you know this?
1
Not at all
2
3
4
5
Perfectly
124
Q

A patient presents with superior vena caval obstruction. How many collateral circulations exist as alternative pathways of venous return?

	None
	One
	Two
	Three
	Four
A

There are 4 collateral venous systems:
Azygos venous system
Internal mammary venous pathway
Long thoracic venous system with connections to the femoral and vertebral veins (2 pathways)

Despite this, venous hypertension still occurs.
Superior vena cava

Drainage
Head and neck
Upper limbs
Thorax
Part of abdominal walls

Formation
Subclavian and internal jugular veins unite to form the right and left brachiocephalic veins
These unite to form the SVC
Azygos vein joins the SVC before it enters the right atrium

How well did you know this?
1
Not at all
2
3
4
5
Perfectly
125
Q

Which of the nerves listed below is directly responsible for the innervation of the lateral aspect of flexor digitorum profundus?

	Ulnar nerve
	Anterior interosseous nerve
	Radial nerve
	Median nerve
	Posterior interosseous nerve
A

The anterior interosseous nerve is a branch of the median nerve and is responsible for innervation of the lateral aspect of the flexor digitorum profundus.

Flexor digitorum profundus originates in the upper 3/4 of the anterior and medial surfaces of the ulna, interosseous membrane and deep fascia of the forearm. The muscle fans out into four tendons (one to each of the second to fifth fingers) to the palmar base of the distal phalanx.

Along with the flexor digitorum superficialis, it has long tendons that run down the arm and through the carpal tunnel and attach to the palmar side of the phalanges of the fingers.

Flexor digitorum profundus lies deep to the superficialis, but it attaches more distally. Therefore, profundus’s tendons go through the tendons of superficialis, and end up attaching to the distal phalanx. For this reason profundus is also called the perforating muscle.[1]

The lumbricals of the hand arise from the radial side of its tendons.[1]

Nerve supply[edit]
Flexor digitorum profundus is a composite muscle innervated by the anterior interosseous nerve and ulnar nerves.

The medial aspect of the muscle (which flexes the 4th and 5th digit) is supplied by the ulnar nerve (C8, T1)[2]
The lateral aspect (which flexes the 2nd and 3rd digit) is innervated by the median nerve[2] specifically the anterior interosseous branch (C8, T1).
It is one of two flexor muscles that is not exclusively supplied by the median nerve (the other is flexor carpi ulnaris).

126
Q

A 43 year old lady is undergoing a total thyroidectomy for an extremely large goitre. The surgeons decide that access may be improved by division of the infra hyoid strap muscles. At which of the following sites should they be divided?

	In their upper half
	In their lower half
	In the middle
	At their origin from the hyoid
	At the point of their insertion
A

Should the strap muscles require division during surgery they should be divided in their upper half. This is because their nerve supply from the ansa cervicalis enters in their lower half.
Anterior triangle of the neck

Boundaries
Anterior border of the Sternocleidomastoid
Lower border of mandible
Anterior midline

Sub triangles (divided by Digastric above and Omohyoid)
Muscular triangle: Neck strap muscles
Carotid triangle: Carotid sheath
Submandibular Triangle (digastric)

Nerve supply to digastric muscle
Anterior: Mylohyoid nerve
Posterior: Facial nerve

127
Q

A 67 year old man undergoes a carotid endarterectomy and seems to recover well following surgery. When he is reviewed on the ward post operatively he complains that his voice is hoarse. What is the most likely cause?

	Damage to the accessory nerve
	Damage to the cervical plexus
	Damage to the glossopharyngeal nerve
	Damage to the hypoglossal nerve
	Damage to the vagus
A

Damage to the vagus
Many of these nerves are at risk of injury during carotid surgery. However, only damage to the vagus would account for a hoarse voice.
Vagus nerve

The vagus nerve has mixed functions and supplies the structures from the fourth and sixth pharyngeal arches. It also supplies the fore and midgut sections of the embryonic gut tube. It carries afferent fibres from these areas (viz; pharynx, larynx, oesophagus, stomach, lungs, heart and great vessels). The efferent fibres of the vagus are of two main types. The first are preganglionic parasympathetic fibres distributed to the parasympathetic ganglia that innervate smooth muscle of the innervated organs (such as gut). The second type of efferent fibres have direct skeletal muscle innervation, these are largely to the muscles of the larynx and pharynx.

Origin and course
The vagus arises from the lateral surface of the medulla oblongata by a series of rootlets. It is related to the glossopharyngeal nerve cranially and the accessory nerve caudally. It exits through the jugular foramen and is contained within its own dural sheath alongside the accessory nerve. In the neck it descends vertically in the carotid sheath where it is closely related to the internal and common carotid arteries. It leaves the neck and enters the mediastinum. On the right it passes anterior to the first part of the subclavian artery, on the left it lies in the interval between the common carotid and subclavian arteries.
In the mediastinum both nerves pass postero-inferiorly and reach the posterior surface of the corresponding lung root. These then branch into both lungs. At the inferior end of the mediastinum these plexuses reunite to form the formal vagal trunks that pass through the oesophageal hiatus and into the abdomen. The anterior and posterior vagal trunks are formal nerve fibres these then splay out once again sending fibres over the stomach and posteriorly to the coeliac plexus. Branches pass to the liver, spleen and kidney.

128
Q

A 35 year old man falls and sustains a fracture to the medial third of his clavicle. Which vessel is at greatest risk of injury?

	Subclavian vein
	Subclavian artery
	External carotid artery
	Internal carotid artery
	Vertebral artery
A

The subclavian vein lies behind subclavius and the medial part of the clavicle. It rests on the first rib, below and in front of the third part of the subclavian artery, and then on scalenus anterior which separates it from the second part of the artery (posteriorly).

Clavicle

The clavicle extends from the sternum to the acromion and helps prevent the shoulder falling forwards and downwards.
The inferior surface is irregular and strongly marked by ligaments at each end. Laterally, lies the trapezoid line and this runs anterolaterally. Posteriorly, lies the conoid tubercle. These give attachment to the conoid and trapezoid parts of the coracoclavicular ligament. The medial part of the inferior surface has an irregular surface which marks the surface attachment of the costoclavicular ligament. The intermediate portion is marked by a groove for the subclavius muscle.
Medially, the superior part of the bone has a raised surface which gives attachment to the clavicular head of sternocleidomastoid. Sternohyoid gains attachment to the posterior surface.
Laterally there is an oval articular facet for the acromion and a disk lies between the clavicle and acromion. The capsule of the joint is attached to the ridge on the margin of the facet.

129
Q

Which of the nerves listed below provides sensory innervation to the skin overlying the lateral aspect of the nose?

	Infratrochlear nerve
	Zygomatic nerve
	Nasopalatine nerve
	Lateral nasal branches of the ethmoidal nerve
	Frontal nerve
A

The lateral aspect of the external nose is innervated by lateral nasal branches of the anterior ethmoidal nerve. The ethmoidal nerve is a branch of the nasociliary nerve which is one of the divisions of the trigeminal.

Path
Originates at the pons
Sensory root forms the large, crescentic trigeminal ganglion within Meckel’s cave, and contains the cell bodies of incoming sensory nerve fibres. Here the 3 branches exit.
The motor root cell bodies are in the pons and the motor fibres are distributed via the mandibular nerve. The motor root is not part of the trigeminal ganglion.

Branches of the trigeminal nerve
Ophthalmic nerve Sensory only
Maxillary nerve Sensory only
Mandibular nerve Sensory and motor

Sensory
Ophthalmic Exits skull via the superior orbital fissure
Sensation of: scalp and forehead, the upper eyelid, the conjunctiva and cornea of the eye, the nose (including the tip of the nose, except alae nasi), the nasal mucosa, the frontal sinuses, and parts of the meninges (the dura and blood vessels).
Maxillary nerve Exit skull via the foramen rotundum
Sensation: lower eyelid and cheek, the nares and upper lip, the upper teeth and gums, the nasal mucosa, the palate and roof of the pharynx, the maxillary, ethmoid and sphenoid sinuses, and parts of the meninges.
Mandibular nerve Exit skull via the foramen ovale
Sensation: lower lip, the lower teeth and gums, the chin and jaw (except the angle of the jaw), parts of the external ear, and parts of the meninges.

Motor
Distributed via the mandibular nerve.
The following muscles of mastication are innervated:
Masseter
Temporalis
Medial pterygoid
Lateral pterygoid
Other muscles innervated include:
Tensor veli palatini
Mylohyoid
Anterior belly of digastric
Tensor tympani
130
Q

A 33 year old man is stabbed in the right chest and undergoes a thoracotomy. The right lung is mobilised and the pleural reflection at the lung hilum is opened. Which of the structures listed below does not lie within this region?

	Pulmonary artery
	Azygos vein
	Pulmonary vein
	Bronchus
	None of the above
A

The pleural reflections encase the hilum of the lung and continue inferiorly as the pulmonary ligament. It encases the pulmonary vessels and bronchus. The azygos vein is not contained within it.
The right lung is composed of 3 lobes divided by the oblique and transverse fissures. The left lung has two lobes divided by the oblique fissure.The apex of both lungs is approximately 4cm superior to the sterno-costal joint of the first rib. Immediately below this is a sulcus created by the subclavian artery.

Peripheral contact points of the lung
Base: diaphragm
Costal surface: corresponds to the cavity of the chest
Mediastinal surface: Contacts the mediastinal pleura. Has the cardiac impression. Above and behind this concavity is a triangular depression named the hilum, where the structures which form the root of the lung enter and leave the viscus. These structures are invested by pleura, which, below the hilum and behind the pericardial impression, forms the pulmonary ligament

Right lung
Above the hilum is the azygos vein; Superior to this is the groove for the superior vena cava and right innominate vein; behind this, and nearer the apex, is a furrow for the innominate artery. Behind the hilum and the attachment of the pulmonary ligament is a vertical groove for the oesophagus; In front and to the right of the lower part of the oesophageal groove is a deep concavity for the extrapericardiac portion of the inferior vena cava.

The root of the right lung lies behind the superior vena cava and the right atrium, and below the azygos vein.

The right main bronchus is shorter, wider and more vertical than the left main bronchus and therefore the route taken by most foreign bodies.

Left lung
Above the hilum is the furrow produced by the aortic arch, and then superiorly the groove accommodating the left subclavian artery; Behind the hilum and pulmonary ligament is a vertical groove produced by the descending aorta, and in front of this, near the base of the lung, is the lower part of the oesophagus.

The root of the left lung passes under the aortic arch and in front of the descending aorta.
Inferior borders of both lungs
6th rib in mid clavicular line
8th rib in mid axillary line
10th rib posteriorly
The pleura runs two ribs lower than the corresponding lung level.

131
Q

A 53 year old man is undergoing a radical gastrectomy for carcinoma of the stomach. Which of the following structures will need to be divided to gain access to the coeliac axis?

	Lesser omentum
	Greater omentum
	Falciform ligament
	Median arcuate ligament
	Gastrosplenic ligament
A

The lesser omentum will need to be divided. During a radical gastrectomy this forms one of the nodal stations that will need to be taken.
Coeliac axis

The coeliac axis has three main branches.
Left gastric
Hepatic: branches-Right Gastric, Gastroduodenal, Superior Pancreaticoduodenal, Cystic (occasionally).
Splenic: branches- Pancreatic, Short Gastric, Left Gastroepiploic

132
Q

A 23 year old lady with troublesome axillary hyperhidrosis is undergoing a thorascopic sympathectomy to treat the condition. Which of the following structures will need to be divided to access the sympathetic trunk?

	Intercostal vein
	Intercostal artery
	Parietal pleura
	Visceral pleura
	None of the above
A

The sympathetic chain lies posterior to the parietal pleura. During a thorascopic sympathetomy this structure will need to be divided. The intercostal vessels lie posteriorly. They may be damaged with troublesome bleeding but otherwise are best left alone as deliberate division will not improve surgical access.
Sympathetic nervous system- anatomy

The cell bodies of the pre-ganglionic efferent neurones lie in the lateral horn of the grey matter of the spinal cord in the thoraco-lumbar regions.
The pre-ganglionic efferents leave the spinal cord at levels T1-L2. These pass to the sympathetic chain.
Lateral branches of the sympathetic chain connect it to every spinal nerve. These post ganglionic nerves will pass to structures that receive sympathetic innervation at the periphery.

Sympathetic chains
These lie on the vertebral column and run from the base of the skull to the coccyx.
Cervical region Lie anterior to the transverse processes of the cervical vertebrae and posterior to the carotid sheath.
Thoracic region Lie anterior to the neck of the upper ribs and and lateral sides of the lower thoracic vertebrae.They are covered by the parietal pleura
Lumbar region Enter by passing posterior to the medial arcuate ligament. Lie anteriorly to the vertebrae and medial to psoas major.

Sympathetic ganglia
Superior cervical ganglion lies anterior to C2 and C3.
Middle cervical ganglion (if present) C6
Stellate ganglion- anterior to transverse process of C7, lies posterior to the subclavian artery, vertebral artery and cervical pleura.
Thoracic ganglia are segmentally arranged.
There are usually 4 lumbar ganglia.

Clinical importance
Interruption of the head and neck supply of the sympathetic nerves will result in an ipsilateral Horners syndrome.
For treatment of hyperhidrosis the sympathetic denervation can be achieved by removing the second and third thoracic ganglia with their rami. Removal of T1 will cause a Horners syndrome and is therefore not performed.
In patients with vascular disease of the lower limbs a lumbar sympathetomy may be performed, either radiologically or (more rarely now) surgically. The ganglia of L2 and below are disrupted. If L1 is removed then ejaculation may be compromised (and little additional benefit conferred as the preganglionic fibres do not arise below L2.

133
Q

A 5 year old boy presents with recurrent headaches. As part of his assessment he undergoes an MRI scan of his brain. This demonstrates enlargement of the lateral and third ventricles. Where is the most likely site of obstruction?

	Foramen of Luschka
	Foramen of Magendie
	Foramen of Munro
	Aqueduct of Sylvius
	None of the above
A

The CSF flows from the 3rd to the 4th ventricle via the Aqueduct of Sylvius.

Cerebrospinal fluid

The CSF fills the space between the arachnoid mater and pia mater (covering surface of the brain). The total volume of CSF in the brain is approximately 150ml. Approximately 500 ml is produced by the ependymal cells in the choroid plexus (70%), or blood vessels (30%). It is reabsorbed via the arachnoid granulations which project into the venous sinuses.

Circulation

  1. Lateral ventricles (via foramen of Munro)
  2. 3rd ventricle
  3. Cerebral aqueduct (aqueduct of Sylvius)
  4. 4th ventricle (via foramina of Magendie and Luschka)
  5. Subarachnoid space
  6. Reabsorbed into the venous system via arachnoid granulations into superior sagittal sinus
Composition
Glucose: 50-80mg/dl
Protein: 15-40 mg/dl
Red blood cells: Nil
White blood cells: 0-3 cells/ mm3
134
Q

Which of the cranial nerves listed below is least likely to carry parasympathetic fibres?

	III
	VII
	IX
	X
	II
A

Cranial nerves carrying parasympathetic fibres
X IX VII III (1973)

The parasympathetic functions served by the cranial nerves include:
III (oculomotor) Pupillary constriction and accommodation
VII (facial) Lacrimal gland, submandibular and sublingual glands
IX (glossopharyngeal) Parotid
X (vagus) Heart and abdominal viscera
The optic nerve carries no parasympathetic fibres.

The cranial preganglionic parasympathetic nerves arise from specific nuclei in the CNS. These synapse at one of four parasympathetic ganglia; otic, pterygopalatine, ciliary and submandibular. From these ganglia the parasympathetic nerves complete their journey to their target tissues via CN V (trigeminal) branches (ophthalmic nerve CNV branch 1, Maxillary nerve CN V branch2, mandibular nerve CN V branch 3)

Cranial nerve lesions
Olfactory nerve May be injured in basal skull fractures or involved in frontal lobe tumour extension. Loss of olfactory nerve function in relation to major CNS pathology is seldom an isolated event and thus it is poor localiser of CNS pathology.
Optic nerve Problems with visual acuity may result from intra ocular disorders. Problems with the blood supply such as amaurosis fugax may produce temporary visual distortion. More important surgically is the pupillary response to light. The pupillary size may be altered in a number of disorders. Nerves involved in the resizing of the pupil connect to the pretectal nucleus of the high midbrain, bypassing the lateral geniculate nucleus and the primary visual cortex. From the pretectal nucleus neurones pass to the Edinger - Westphal nucleus, motor axons from here pass along with the oculomotor nerve. They synapse with ciliary ganglion neurones; the parasympathetic axons from this then innervate the iris and produce miosis. The miotic pupil is seen in disorders such as Horner’s syndrome or opiate overdose.
Mydriasis is the dilatation of the pupil in response to disease, trauma, drugs (or the dark!). It is pathological when light fails to induce miosis. The radial muscle is innervated by the sympathetic nervous system. Because the parasympathetic fibres travel with the oculomotor nerve they will be damaged by lesions affecting this nerve (e.g. cranial trauma).
The response to light shone in one eye is usually a constriction of both pupils. This indicates intact direct and consensual light reflexes. When the optic nerve has an afferent defect the light shining on the affected eye will produce a diminished pupillary response in both eyes. Whereas light shone on the unaffected eye will produce a normal pupillary response in both eyes. This is referred to as the Marcus Gunn pupil and is seen in conditions such as optic neuritis. In a total CN II lesion shining the light in the affected eye will produce no response.
Oculomotor nerve The pupillary effects are described above. In addition it supplies all ocular muscles apart from lateral rectus and superior oblique. Thus the affected eye will be deviated inferolaterally. Levator palpebrae superioris may also be impaired resulting in impaired ability to open the eye.
Trochlear nerve The eye will not be able to look down.
Trigeminal nerve Largest cranial nerve. Exits the brainstem at the pons. Branches are ophthalmic, maxillary and mandibular. Only the mandibular branch has both sensory and motor fibres. Branches converge to form the trigeminal ganglion (located in Meckels cave). It supplies the muscles of mastication and also tensor veli palatine, mylohyoid, anterior belly of digastric and tensor tympani. The detailed descriptions of the various sensory functions are described in other areas of the website. The corneal reflex is important and is elicited by applying a small tip of cotton wool to the cornea, a reflex blink should occur if it is intact. It is mediated by: the naso ciliary branch of the ophthalmic branch of the trigeminal (sensory component) and the facial nerve producing the motor response. Lesions of the afferent arc will produce bilateral absent blink and lesions of the efferent arc will result in a unilateral absent blink.
Abducens nerve The affected eye will have a deficit of abduction. This cranial nerve exits the brainstem between the pons and medulla. It thus has a relatively long intra cranial course which renders it susceptible to damage in raised intra cranial pressure.
Facial nerve Emerges from brainstem between pons and medulla. It controls muscles of facial expression and taste from the anterior 2/3 of the tongue. The nerve passes into the petrous temporal bone and into the internal auditory meatus. It then passes through the facial canal and exits at the stylomastoid foramen. It passes through the parotid gland and divides at this point. It does not innervate the parotid gland. Its divisions are considered in other parts of the website. Its motor fibres innervate orbicularis oculi to produce the efferent arm of the corneal reflex. In surgical practice it may be injured during parotid gland surgery or invaded by malignancies of the gland and a lower motor neurone on the ipsilateral side will result.
Vestibulo-cochlear nerve Exits from the pons and then passes through the internal auditory meatus. It is implicated in sensorineural hearing loss. Individuals with sensorineural hearing loss will localise the sound in webers test to the normal ear. Rinnes test will be reduced on the affected side but should still work. These two tests will distinguish sensorineural hearing loss from conductive deafness. In the latter condition webers test will localise to the affected ear and Rinnes test will be impaired on the affected side. Surgical lesions affecting this nerve include CNS tumours and basal skull fractures. It may also be damaged by the administration of ototoxic drugs (of which gentamicin is the most commonly used in surgical practice).
Glossopharyngeal nerve Exits the pons just above the vagus. Receives sensory fibres from posterior 1/3 tongue, tonsils, pharynx and middle ear (otalgia may occur following tonsillectomy). It receives visceral afferents from the carotid bodies. It supplies parasympathetic fibres to the parotid gland via the otic ganglion and motor function to stylopharyngeaus muscle. The sensory function of the nerve is tested using the gag reflex.
Vagus nerve Leaves the medulla between the olivary nucleus and the inferior cerebellar peduncle. Passes through the jugular foramen and into the carotid sheath. Details of the functions of the vagus nerve are covered in the website under relevant organ sub headings.
Accessory nerve Exists from the caudal aspect of the brainstem (multiple branches) supplies trapezius and sternocleidomastoid muscles. The distal portion of this nerve is most prone to injury during surgical procedures.
Hypoglossal nerve Emerges from the medulla at the preolivary sulcus, passes through the hypoglossal canal. It lies on the carotid sheath and passes deep to the posterior belly of digastric to supply muscles of the tongue (except palatoglossus). Its location near the carotid sheath makes it vulnerable during carotid endarterectomy surgery and damage will produce ipsilateral defect in muscle function.

135
Q

Which of the following structures suspends the spinal cord in the dural sheath?

	Filum terminale
	Conus medullaris
	Ligamentum flavum
	Denticulate ligaments
	Anterior longitudinal ligament
A

Denticulate ligaments
The spinal cord is approximately 45cm in men and 43cm in women. The denticulate ligament is a continuation of the pia mater (innermost covering of the spinal cord) which has intermittent lateral projections attaching the spinal cord to the dura mater.
Spinal cord

Located in a canal within the vertebral column that affords it structural support.
Rostrally it continues to the medulla oblongata of the brain and caudally it tapers at a level corresponding to the L1-2 interspace (in the adult), a central structure, the filum terminale anchors the cord to the first coccygeal vertebra.
The spinal cord is characterised by cervico-lumbar enlargements and these, broadly speaking, are the sites which correspond to the brachial and lumbar plexuses respectively.

There are some key points to note when considering the surgical anatomy of the spinal cord:

  • During foetal growth the spinal cord becomes shorter than the spinal canal, hence the adult site of cord termination at the L1-2 level.
  • Due to growth of the vertebral column the spine segmental levels may not always correspond to bony landmarks as they do in the cervical spine.
  • The spinal cord is incompletely divided into two symmetrical halves by a dorsal median sulcus and ventral median fissure. Grey matter surrounds a central canal that is continuous rostrally with the ventricular system of the CNS.
  • The grey matter is sub divided cytoarchitecturally into Rexeds laminae.
  • Afferent fibres entering through the dorsal roots usually terminate near their point of entry but may travel for varying distances in Lissauers tract. In this way they may establish synaptic connections over several levels
  • At the tip of the dorsal horn are afferents associated with nociceptive stimuli. The ventral horn contains neurones that innervate skeletal muscle.

The key point to remember when revising CNS anatomy is to keep a clinical perspective in mind. So it is worth classifying the ways in which the spinal cord may become injured. These include:

Trauma either direct or as a result of disc protrusion
Neoplasia either by direct invasion (rare) or as a result of pathological vertebral fracture
Inflammatory diseases such as Rheumatoid disease, or OA (formation of osteophytes compressing nerve roots etc.
Vascular either as a result of stroke (rare in cord) or as complication of aortic dissection
Infection historically diseases such as TB, epidural abscesses.

The anatomy of the cord will, to an extent dictate the clinical presentation. Some points/ conditions to remember:

Brown- Sequard syndrome-Hemisection of the cord producing ipsilateral loss of proprioception and upper motor neurone signs, plus contralateral loss of pain and temperature sensation. The explanation of this is that the fibres decussate at different levels.
Lesions below L1 will tend to present with lower motor neurone signs

136
Q

Which of the nerves listed below is responsible for providing innervation to the lower molar teeth?

	Greater palatine nerve
	Nasopalatine nerve
	Inferior alveolar nerve
	Zygomatic nerve
	Mandibular nerve
A

The branches of the lower molar and premolar teeth are supplied by branches of the inferior alveolar nerve. Those of the canine and incisors by the incisive branch of the same nerve. The gingiva and supporting structures are innervated by the lingual nerve.
Trigeminal nerve

The trigeminal nerve is the main sensory nerve of the head. In addition to its major sensory role, it also innervates the muscles of mastication.

Distribution of the trigeminal nerve
Sensory	
Scalp
Face
Oral cavity (and teeth)
Nose and sinuses
Dura mater
Motor	
Muscles of mastication
Mylohyoid
Anterior belly of digastric
Tensor tympani
Tensor palati
Autonomic connections (ganglia)	
Ciliary
Sphenopalatine
Otic
Submandibular

Path
Originates at the pons
Sensory root forms the large, crescentic trigeminal ganglion within Meckel’s cave, and contains the cell bodies of incoming sensory nerve fibres. Here the 3 branches exit.
The motor root cell bodies are in the pons and the motor fibres are distributed via the mandibular nerve. The motor root is not part of the trigeminal ganglion.

Branches of the trigeminal nerve
Ophthalmic nerve Sensory only
Maxillary nerve Sensory only
Mandibular nerve Sensory and motor

Sensory
Ophthalmic Exits skull via the superior orbital fissure
Sensation of: scalp and forehead, the upper eyelid, the conjunctiva and cornea of the eye, the nose (including the tip of the nose, except alae nasi), the nasal mucosa, the frontal sinuses, and parts of the meninges (the dura and blood vessels).
Maxillary nerve Exit skull via the foramen rotundum
Sensation: lower eyelid and cheek, the nares and upper lip, the upper teeth and gums, the nasal mucosa, the palate and roof of the pharynx, the maxillary, ethmoid and sphenoid sinuses, and parts of the meninges.
Mandibular nerve Exit skull via the foramen ovale
Sensation: lower lip, the lower teeth and gums, the chin and jaw (except the angle of the jaw), parts of the external ear, and parts of the meninges.

Motor
Distributed via the mandibular nerve.
The following muscles of mastication are innervated:
Masseter
Temporalis
Medial pterygoid
Lateral pterygoid
Other muscles innervated include:
Tensor veli palatini
Mylohyoid
Anterior belly of digastric
Tensor tympani
137
Q

During a radical neck dissection, division of which of the following fascial layers will expose the ansa cervicalis?

	Pretracheal fascia
	Carotid sheath
	Prevertebral fascia
	Investing layer of fascia
	Sibsons fascia
A

The ansa cervicalis lies anterior to the carotid sheath. It may be exposed by division of the pretracheal fascia at the posterolateral aspect of the thyroid gland. The pre vertebral fascia lies more posteriorly and division of the investing layer of fascia will not expose this nerve.
The ansa cervicalis lies anterior to the carotid sheath. The nerve supply to the inferior strap muscles enters at their inferior aspect. Therefore when dividing these muscles to expose a large goitre, the muscles should be divided in their upper half.

138
Q

Which of the following structures lies posterior to the femoral nerve in the femoral triangle?

	Adductor longus
	Pectineus
	Psoas major
	Iliacus
	None of the above
A

The iliacus lies posterior to the femoral nerve in the femoral triangle. The femoral sheath lies anterior to the iliacus and pectineus muscles.
Femoral nerve

Root values	L2, 3, 4
Innervates	
Pectineus
Sartorius
Quadriceps femoris
Vastus lateralis/medialis/intermedius
Branches	
Medial cutaneous nerve of thigh
Saphenous nerve
Intermediate cutaneous nerve of thigh

Path
Penetrates psoas major and exits the pelvis by passing under the inguinal ligament to enter the femoral triangle, lateral to the femoral artery and vein.

Mnemonic for femoral nerve supply

(don’t) M I S V Q Scan for PE
M edial cutaneous nerve of the thigh
I ntermediate cutaneous nerve of the thigh
S aphenous nerve

V astus
Q uadriceps femoris
S artorius

PE ectineus

139
Q

You are assisting in an open right adrenalectomy for a large adrenal adenoma. The consultant is distracted and you helpfully pull the adrenal into the wound to improve the view. Unfortunately this is followed by brisk bleeding. The vessel responsible for this is most likely to be:

	Portal vein
	Phrenic vein
	Right renal vein
	Superior mesenteric vein
	Inferior vena cava
A

Inferior vena cava
It drains directly via a very short vessel. If the sutures are not carefully tied then it may be avulsed off the IVC. An injury best managed using a Satinsky clamp and a 6/0 prolene suture.
Adrenal gland anatomy

Anatomy

Location Superomedially to the upper pole of each kidney
Relationships of the right adrenal Diaphragm-Posteriorly, Kidney-Inferiorly, Vena Cava-Medially, Hepato-renal pouch and bare area of the liver-Anteriorly
Relationships of the left adrenal Crus of the diaphragm-Postero- medially, Pancreas and splenic vessels-Inferiorly, Lesser sac and stomach-Anteriorly

Arterial supply Superior adrenal arteries- from inferior phrenic artery, Middle adrenal arteries - from aorta, Inferior adrenal arteries -from renal arteries
Venous drainage of the right adrenal Via one central vein directly into the IVC
Venous drainage of the left adrenal Via one central vein into the left renal vein

140
Q

An enthusiastic surgical registrar undertakes his first solo splenectomy. The operation is far more difficult than anticipated and the registrar leaves a tube drain to the splenic bed at the end of the procedure. Over the following 24 hours approximately 500ml of clear fluid has entered the drain. Biochemical testing of the fluid is most likely to reveal:

	Elevated creatinine
	Elevated triglycerides
	Elevated glucagon
	Elevated amylase
	None of the above
A

During splenectomy the tail of the pancreas may be damaged. The pancreatic duct will then drain into the splenic bed, amylase is the most likely biochemical finding. Glucagon is not secreted into the pancreatic duct.
Splenic anatomy

The spleen is the largest lymphoid organ in the body. It is an intraperitoneal organ, the peritoneal attachments condense at the hilum where the vessels enter the spleen. Its blood supply is from the splenic artery (derived from the coeliac axis) and the splenic vein (which is joined by the IMV and unites with the SMV).

Embryology: derived from mesenchymal tissue
Shape: clenched fist
Position: below 9th-12th ribs
Weight: 75-150g

Relations
Superiorly- diaphragm
Anteriorly- gastric impression
Posteriorly- kidney
Inferiorly- colon
Hilum: tail of pancreas and splenic vessels
Forms apex of lesser sac (containing short gastric vessels)

141
Q

A 53 year old lady is recovering following a difficult mastectomy and axillary nodal clearance for carcinoma of the breast. She complains of shoulder pain and on examination has obvious winging of the scapula. Loss of innervation to which of the following is the most likely underlying cause?

	Latissimus dorsi
	Serratus anterior
	Pectoralis minor
	Pectoralis major
	Rhomboids
A

Winging of the scapula is most commonly the result of long thoracic nerve injury or dysfunction. Iatrogenic damage during the course of the difficult axillary dissection is the most likely cause in this scenario. Damage to the rhomboids may produce winging of the scapula but would be rare in the scenario given.
Long thoracic nerve

Derived from ventral rami of C5, C6, and C7 (close to their emergence from intervertebral foramina)
It runs downward and passes either anterior or posterior to the middle scalene muscle
It reaches upper tip of serratus anterior muscle and descends on outer surface of this muscle, giving branches into it
Winging of Scapula occurs in long thoracic nerve injury (most common) or from spinal accessory nerve injury (which denervates the trapezius) or a dorsal scapular nerve injury

142
Q

A 20 year old man is hit with a hammer on the right side of the head. He dies on arrival in the emergency department. Which of these features is most likely to be found at post mortem?

	Hydrocephalus
	Supra tentorial herniation
	Laceration of the middle meningeal artery
	Sub dural haematoma
	Posterior fossa haematoma
A

Laceration of the middle meningeal artery

Head injury

Patients who suffer head injuries should be managed according to ATLS principles and extra cranial injuries should be managed alongside cranial trauma. Inadequate cardiac output will compromise CNS perfusion irrespective of the nature of the cranial injury.

Types of traumatic brain injury
Extradural haematoma Bleeding into the space between the dura mater and the skull. Often results from acceleration-deceleration trauma or a blow to the side of the head. The majority of extradural haematomas occur in the temporal region where skull fractures cause a rupture of the middle meningeal artery.

Features
Raised intracranial pressure
Some patients may exhibit a lucid interval
Subdural haematoma Bleeding into the outermost meningeal layer. Most commonly occur around the frontal and parietal lobes. May be either acute or chronic.

Risk factors include old age and alcoholism.

Slower onset of symptoms than a extradural haematoma.
Subarachnoid haemorrhage Usually occurs spontaneously in the context of a ruptured cerebral aneurysm, but may be seen in association with other injuries when a patient has sustained a traumatic brain injury.

Pathophysiology
Primary brain injury may be focal (contusion/ haematoma) or diffuse (diffuse axonal injury)
Diffuse axonal injury occurs as a result of mechanical shearing following deceleration, causing disruption and tearing of axons
Intra-cranial haematomas can be extradural, subdural or intracerebral, while contusions may occur adjacent to (coup) or contralateral (contre-coup) to the side of impact
Secondary brain injury occurs when cerebral oedema, ischaemia, infection, tonsillar or tentorial herniation exacerbates the original injury. The normal cerebral auto regulatory processes are disrupted following trauma rendering the brain more susceptible to blood flow changes and hypoxia
The Cushings reflex (hypertension and bradycardia) often occurs late and is usually a pre terminal event

Management
Where there is life threatening rising ICP such as in extra dural haematoma and whilst theatre is prepared or transfer arranged use of IV mannitol/ frusemide may be required.
Diffuse cerebral oedema may require decompressive craniotomy
Exploratory Burr Holes have little management in modern practice except where scanning may be unavailable and to thus facilitate creation of formal craniotomy flap
Depressed skull fractures that are open require formal surgical reduction and debridement, closed injuries may be managed non operatively if there is minimal displacement.
ICP monitoring is appropriate in those who have GCS 3-8 and normal CT scan.
ICP monitoring is mandatory in those who have GCS 3-8 and abnormal CT scan.
Hyponatraemia is most likely to be due to syndrome of inappropriate ADH secretion.
Minimum of cerebral perfusion pressure of 70mmHg in adults.
Minimum cerebral perfusion pressure of between 40 and 70 mmHg in children.

143
Q

A 72 year old man develops a hydrocele which is being surgically managed. As part of the procedure the surgeons divide the tunica vaginalis. From which of the following is this structure derived?

	Peritoneum
	External oblique aponeurosis
	Internal oblique aponeurosis
	Transversalis fascia
	Rectus sheath
A

The tunica vaginalis is derived from peritoneum, it secretes the fluid that fills the hydrocele cavity.
Spermatic cord
Formed by the vas deferens and is covered by the following structures:
Layer Origin
Internal spermatic fascia Transversalis fascia
Cremasteric fascia From the fascial coverings of internal oblique
External spermatic fascia External oblique aponeurosis

Scrotum
Composed of skin and closely attached dartos fascia.
Arterial supply from the anterior and posterior scrotal arteries
Lymphatic drainage to the inguinal lymph nodes
Parietal layer of the tunica vaginalis is the innermost layer

Testes
The testes are surrounded by the tunica vaginalis (closed peritoneal sac). The parietal layer of the tunica vaginalis adjacent to the internal spermatic fascia.
The testicular arteries arise from the aorta immediately inferiorly to the renal arteries.
The pampiniform plexus drains into the testicular veins, the left drains into the left renal vein and the right into the inferior vena cava.
Lymphatic drainage is to the para-aortic nodes.

144
Q

A 63 year old man is undergoing a coronary artery bypass procedure. During the median sternotomy which structure would routinely require division?

	Parietal pleura
	Interclavicular ligament
	Internal mammary artery
	Brachiocephalic vein
	Left vagus nerve
A

The interclavicular ligament lies at the upper end of a median sternotomy and is routinely divided to provide access. The pleural reflections are often encountered and should not be intentionally divided, if they are, then a chest drain will need to be inserted on the affected side as collections may then accumulate in the pleural cavity. Other structures encountered include the pectoralis major muscles, again if the incision is truly midline then these should not require formal division. The close relationship of the brachiocephalic vein should be borne in mind and it should be avoided, iatrogenic injury to this structure will result in considerable haemorrhage.

Sternotomy

A median sternotomy is the commonest incision utilised to access the heart and great vessels.
A midline incision is made from the interclavicular fossa to the xiphoid process and the fat and subcutaneous tissues are divided to the level of the sternum. The periosteum may be gently mobilised off the midline, although vigorous periosteal stripping is best avoided. A bone saw is used to divide the bone itself. Posteriorly the reflections of the parietal pleura should be identified and avoided (unless surgery to the lung is planned). The fibrous pericardium is incised and the heart brought into view. Bleeding from the bony edges of the cut sternum is stopped using roller ball diathermy or bone wax. The left brachiocephalic vein is an important posterior relation at the superior aspect of the sternotomy incision and should be avoided. More inferior the thymic remnants may be identified. At the inferior aspect of the incision the abdominal cavity may be entered (though this is seldom troublesome).

145
Q

During an Ivor Lewis Oesophagectomy for carcinoma of the lower third of the oesophagus which structure is divided to allow mobilisation of the oesophagus?

	Vagus nerve
	Azygos vein
	Right inferior lobar bronchus
	Phrenic nerve
	Pericardiophrenic artery
A

The azygos vein is routinely divided during an oesophagectomy to allow mobilisation. It arches anteriorly to insert into the SVC on the right hand side.

Treatment of oesophageal cancer

In general resections are not offered to those patients with distant metastasis, and usually not to those with N2 disease.
Local nodal involvement is not in itself a contra indication to resection.
Surgical resection is the mainstay of treatment.
Neoadjuvent chemotherapy is given in most cases prior to surgery.
In situ disease may be managed by endoscopic mucosal resection, although this is still debated.
In patients with lower third lesions an Ivor - Lewis type procedure is most commonly performed. Very distal tumours may be suitable to a transhiatal procedure. Which is an attractive option as the penetration of two visceral cavities required for an Ivor- Lewis type procedure increases the morbidity considerably.
More proximal lesions will require a total oesphagectomy (Mckeown type) with anastomosis to the cervical oesophagus.
Patients with unresectable disease may derive benefit from local ablative procedures, palliative chemotherapy or stent insertion.

Operative details of Ivor- Lewis procedure
Combined laparotomy and right thoracotomy

Indication
Lower and middle third oesophageal tumours

Preparation
Staging with a combination of CT chest abdomen and pelvis- if no metastatic disease detected then patients will undergo a staging laparoscopy to detect peritoneal disease.
If both these modalities are negative then patients will finally undergo a PET CT scan to detect occult metastatic disease. Only in those whom no evidence of advanced disease is detected will proceed to resection.
Patients receive a GA, double lumen endotracheal tube to allow for lung deflation, CVP and arterial monitoring.

Procedure
A rooftop incision is made to access the stomach and duodenum.

Laparotomy To mobilize the stomach
The greater omentum is incised away from its attachment to the right gastroepiploic vessels along the greater curvature of the stomach.
Then the short gastric vessels are ligated and detached from the greater curvature from the spleen.
The lesser omentum is incised, preserving the right gastric artery.
The retroperitoneal attachments of the duodenum in its second and third portions are incised, allowing the pylorus to reach the oesophageal hiatus. Some surgeons perform a pyloroplasty at this point to facilitate gastric emptying.
The left gastric vessels are then ligated, avoiding any injury to the common hepatic or splenic arteries. Care must be taken to avoid inadvertently devascularising the liver owing to variations in anatomy.

Right Thoracotomy Oesophageal resection and oesophagogastric anastomosis
Through 5th intercostal space
Dissection performed 10cm above the tumour
This may involve transection of the azygos vein.
The oesophagus is then removed with the stomach creating a gastric tube.
An anastomosis is created.

The chest is closed with underwater seal drainage and tube drains to the abdominal cavity.

Post operatively
Patients will typically recover in ITU initially.
A nasogastric tube will have been inserted intraoperatively and must remain in place during the early phases of recovery.
Post operatively these patients are at relatively high risk of developing complications:

  • Atelectasis- due to the effects of thoracotomy and lung collapse
  • Anastomotic leakage. The risk is relatively high owing to the presence of a relatively devascularised stomach. Often the only blood supply is from the gastroepiploic artery as all others will have been divided. If a leak does occur then many will attempt to manage conservatively with prolonged nasogastric tube drainage and TPN. The reality is that up to 50% of patients developing an anastomotic leak will not survive to discharge.
  • Delayed gastric emptying (may be avoided by performing a pyloroplasty).
146
Q

Which of the following statements relating to quadratus lumborum is false?

	Causes flexion of the thoracic spine
	Causes the rib cage to be pulled down
	Innervated by anterior primary rami of T12 and L1-3
	Attached to the iliac crest
	Inserts into the 12th rib
A

Causes flexion of the thoracic spine

Quadratus lumborum
Origin: Medial aspect of iliac crest and iliolumbar ligament
Insertion: 12th rib
Action: Pulls the rib cage inferiorly. Lateral flexion.
Nerve supply: Anterior primary rami of T12 and L1-3

The rectus abdominis causes flexion of the thoracic spine and therefore the statement suggesting that quaratus lumborum does so is incorrect.

Abdominal wall

The 2 main muscles of the abdominal wall are the rectus abdominis (anterior) and the quadratus lumborum (posterior).
The remaining abdominal wall consists of 3 muscular layers. Each muscle passes from the lateral aspect of the quadratus lumborum posteriorly to the lateral margin of the rectus sheath anteriorly. Each layer is muscular posterolaterally and aponeurotic anteriorly.

Muscles of abdominal wall
External oblique
Lies most superficially
Originates from 5th to 12th ribs
Inserts into the anterior half of the outer aspect of the iliac crest, linea alba and pubic tubercle
More medially and superiorly to the arcuate line, the aponeurotic layer overlaps the rectus abdominis muscle
The lower border forms the inguinal ligament
The triangular expansion of the medial end of the inguinal ligament is the lacunar ligament.
Internal oblique
Arises from the thoracolumbar fascia, the anterior 2/3 of the iliac crest and the lateral 2/3 of the inguinal ligament
The muscle sweeps upwards to insert into the cartilages of the lower 3 ribs
The lower fibres form an aponeurosis that runs from the tenth costal cartilage to the body of the pubis
At its lowermost aspect it joins the fibres of the aponeurosis of transversus abdominis to form the conjoint tendon.
Transversus abdominis
Innermost muscle
Arises from the inner aspect of the costal cartilages of the lower 6 ribs , from the anterior 2/3 of the iliac crest and lateral 1/3 of the inguinal ligament
Its fibres run horizontally around the abdominal wall ending in an aponeurosis. The upper part runs posterior to the rectus abdominis. Lower down the fibres run anteriorly only.
The rectus abdominis lies medially; running from the pubic crest and symphysis to insert into the xiphoid process and 5th, 6th and 7th costal cartilages. The muscles lies in a aponeurosis as described above.
Nerve supply: anterior primary rami of T7-12

Surgical notes
During abdominal surgery it is usually necessary to divide either the muscles or their aponeuroses. During a midline laparotomy it is desirable to divide the aponeurosis. This will leave the rectus sheath intact above the arcuate line and the muscles intact below it. Straying off the midline will often lead to damage to the rectus muscles, particularly below the arcuate line where they may often be in close proximity to each other.

147
Q

A 38 year old lady is due to undergo a parathyroidectomy for hyperparathyroidism. At operation the inferior parathyroid gland is identified as being enlarged. A vessel is located adjacent to the gland laterally. This vessel is most likely to be the:

	External carotid artery
	Common carotid artery
	Internal carotid artery
	External jugular vein
	None of the above
A

The common carotid artery is a lateral relation of the inferior parathyroid.
Parathyroid glands- anatomy

Four parathyroid glands
Located posterior to the thyroid gland
They lie within the pretracheal fascia

Embryology
The parathyroids develop from the extremities of the third and fourth pharyngeal pouches. The parathyroids derived from the fourth pharyngeal pouch are located more superiorly and are associated with the thyroid gland. Those derived from the third pharyngeal pouch lie more inferiorly and may become associated with the thymus.

Blood supply
The blood supply to the parathyroid glands is derived from the inferior and superior thyroid arteries[1]. There is a rich anastomosis between the two vessels. Venous drainage is into the thyroid veins.

Relations
Laterally	Common carotid
Medially	Recurrent laryngeal nerve, trachea
Anterior	Thyroid
Posterior	Pretracheal fascia
148
Q

Which of the positions listed below best describes the location of the coeliac autonomic plexus?

	Anterolateral to the aorta
	Posterolateral to the aorta
	Anterolateral to the sympathetic chain
	Anteromedial to the sympathetic chain
	Posterior to L1
A

Anterolateral to the aorta

Coeliac plexus

The coeliac plexus is the largest of the autonomic plexuses. It is located on a level of the last thoracic and first lumbar vertebrae. It surrounds the coeliac axis and the SMA. It lies posterior to the stomach and the lesser sac. It lies anterior to the crura of the diaphragm and the aorta. The plexus and ganglia are joined by the greater and lesser splanchnic nerves on both sides and branches from both the vagus and phrenic nerves.

149
Q

Two teenagers are playing with an airgun when one accidentally shoots his friend in the abdomen. He is brought to the emergency department. On examination there is a bullet entry point immediately to the right of the rectus sheath at the level of the 1st lumbar vertebra. Which of the following structures is most likely to be injured by the bullet?

	Head of pancreas
	Right ureter
	Right adrenal gland
	Fundus of the gallbladder
	Gastric antrum
A

The fundus of the gallbladder lies at this level and is the most superficially located structure.

Transpyloric plane
Level of the body of L1

Pylorus stomach
Left kidney hilum (L1- left one!)
Fundus of the gallbladder
Neck of pancreas
Duodenojejunal flexure
Superior mesenteric artery
Portal vein
Left and right colic flexure
Root of the transverse mesocolon
2nd part of the duodenum
Upper part of conus medullaris
Spleen

Can be identified by asking the supine patient to sit up without using their arms. The plane is located where the lateral border of the rectus muscle crosses the costal margin.

Anatomical planes
Subcostal plane Lowest margin of 10th costal cartilage
Intercristal plane Level of body L4 (highest point of iliac crest)
Intertubercular plane Level of body L5

Common level landmarks
Inferior mesenteric artery	L3
Bifurcation of aorta into common iliac arteries	L4
Formation of IVC	L5 (union of common iliac veins)
Diaphragm apertures	
Vena cava T8
Oesophagus T10
Aortic hiatus T12
150
Q

A 73 year old lady suffers a fracture at the surgical neck of the humerus. The decision is made to operate. There are difficulties in reducing the fracture and a vessel lying posterior to the surgical neck is injured. Which of the following is this vessel most likely to be?

	Axillary artery
	Brachial artery
	Thoracoacromial artery
	Transverse scapular artery
	Posterior circumflex humeral artery
A

The circumflex humeral arteries lie at the surgical neck and is this scenario the posterior circumflex is likely to be injured. The thoracoacromial and transverse scapular arteries lie more superomedially. The posterior circumflex humeral artery is a branch of the axillary artery.

Shoulder joint

Shallow synovial ball and socket type of joint.
It is an inherently unstable joint, but is capable to a wide range of movement.
Stability is provided by muscles of the rotator cuff that pass from the scapula to insert in the greater tuberosity (all except sub scapularis-lesser tuberosity).

Glenoid labrum
Fibrocartilaginous rim attached to the free edge of the glenoid cavity
Tendon of the long head of biceps arises from within the joint from the supraglenoid tubercle, and is fused at this point to the labrum.
The long head of triceps attaches to the infraglenoid tubercle

Fibrous capsule
Attaches to the scapula external to the glenoid labrum and to the labrum itself (postero-superiorly)
Attaches to the humerus at the level of the anatomical neck superiorly and the surgical neck inferiorly
Anteriorly the capsule is in contact with the tendon of subscapularis, superiorly with the supraspinatus tendon, and posteriorly with the tendons of infraspinatus and teres minor. All these blend with the capsule towards their insertion.
Two defects in the fibrous capsule; superiorly for the tendon of biceps. Anteriorly there is a defect beneath the subscapularis tendon.
The inferior extension of the capsule is closely related to the axillary nerve at the surgical neck and this nerve is at risk in anteroinferior dislocations. It also means that proximally sited osteomyelitis may progress to septic arthritis.

Important anatomical relations
Anteriorly	Brachial plexus
Axillary artery and vein
Posterior	Suprascapular nerve
Suprascapular vessels
Inferior	Axillary nerve
Circumflex humeral vessels
151
Q

Which of the structures listed below lies posterior to the carotid sheath at the level of the 6th cervical vertebra?

	Hypoglossal nerve
	Vagus nerve
	Cervical sympathetic chain
	Ansa cervicalis
	Glossopharyngeal nerve
A

The carotid sheath is crossed anteriorly by the hypoglossal nerves and the ansa cervicalis. The vagus lies within it. The cervical sympathetic chain lies posteriorly between the sheath and the prevertebral fascia.
Common carotid artery

The right common carotid artery arises at the bifurcation of the brachiocephalic trunk, the left common carotid arises from the arch of the aorta. Both terminate at the level of the upper border of the thyroid cartilage (the lower border of the third cervical vertebra) by dividing into the internal and external carotid arteries.

Left common carotid artery
This vessel arises immediately to the left and slightly behind the origin of the brachiocephalic trunk. Its thoracic portion is 2.5- 3.5 cm in length and runs superolaterally to the sternoclavicular joint.

In the thorax
The vessel is in contact, from below upwards, with the trachea, left recurrent laryngeal nerve, left margin of the oesophagus. Anteriorly the left brachiocephalic vein runs across the artery, and the cardiac branches from the left vagus descend in front of it. These structures together with the thymus and the anterior margins of the left lung and pleura separate the artery from the manubrium.

In the neck
The artery runs superiorly deep to sternocleidomastoid and then enters the anterior triangle. At this point it lies within the carotid sheath with the vagus nerve and the internal jugular vein. Posteriorly the sympathetic trunk lies between the vessel and the prevertebral fascia. At the level of C7 the vertebral artery and thoracic duct lie behind it. The anterior tubercle of C6 transverse process is prominent and the artery can be compressed against this structure (it corresponds to the level of the cricoid).
Anteriorly at C6 the omohyoid muscle passes superficial to the artery.
Within the carotid sheath the jugular vein lies lateral to the artery.

Right common carotid artery
The right common carotid arises from the brachiocephalic artery. The right common carotid artery corresponds with the cervical portion of the left common carotid, except that there is no thoracic duct on the right. The oesophagus is less closely related to the right carotid than the left.

Summary points about the carotid anatomy

Path
Passes behind the sternoclavicular joint (12% patients above this level) to the upper border of the thyroid cartilage, to divide into the external (ECA) and internal carotid arteries (ICA).

Relations
Level of 6th cervical vertebra crossed by omohyoid
Then passes deep to the thyrohyoid, sternohyoid, sternomastoid muscles.
Passes anterior to the carotid tubercle (transverse process 6th cervical vertebra)-NB compression here stops haemorrhage.
The inferior thyroid artery passes posterior to the common carotid artery.
Then : Left common carotid artery crosses the thoracic duct, Right common carotid artery crossed by recurrent laryngeal nerve

152
Q

A 73 year old man presents with symptoms of mesenteric ischaemia. As part of his diagnostic work up a diagnostic angiogram is performed .The radiologist is attempting to cannulate the coeliac axis from the aorta. At which of the following vertebral levels does this is usually originate?

	T10
	L2
	L3
	T8
	T12
A

The coeliac axis branches off the aorta at T12.

Coeliac axis

The coeliac axis has three main branches.
Left gastric
Hepatic: branches-Right Gastric, Gastroduodenal, Superior Pancreaticoduodenal, Cystic (occasionally).
Splenic: branches- Pancreatic, Short Gastric, Left Gastroepiploic

Anteriorly Lesser omentum
Right Right coeliac ganglion and caudate process of liver
Left Left coeliac ganglion and gastric cardia
Inferiorly Upper border of pancreas and renal vein

153
Q

A 45 year old man presents with a lipoma located posterior to the posterior border of the sternocleidomastoid muscle, approximately 4cm superior to the middle third of the clavicle. During surgical excision of the lesion troublesome bleeding is encountered. Which of the following is the most likely source?

	Internal jugular vein
	External jugular vein
	Common carotid artery
	Vertebral artery
	Second part of the subclavian artery
A

The external jugular vein runs obliquely in the superficial fascia of the posterior triangle. It drains into the subclavian vein. During surgical exploration of this area the external jugular vein may be injured and troublesome bleeding may result. The internal jugular vein and carotid arteries are located in the anterior triangle. The third, and not the second, part of the subclavian artery is also a content of the posterior triangle

Posterior triangle of the neck

Boundaries
Apex Sternocleidomastoid and the Trapezius muscles at the Occipital bone
Anterior Posterior border of the Sternocleidomastoid
Posterior Anterior border of the Trapezius
Base Middle third of the clavicle

Contents
Nerves	
Accessory nerve
Phrenic nerve
Three trunks of the brachial plexus
Branches of the cervical plexus: Supraclavicular nerve, transverse cervical nerve, great auricular nerve, lesser occipital nerve
Vessels	
External jugular vein
Subclavian artery
Muscles	
Inferior belly of omohyoid
Scalene
Lymph nodes	
Supraclavicular
Occipital
154
Q

A 62 year old male complains of back pain. He has had a recent fall. Walking causes pain of the left lower leg. On examination he is noted to have reduced sensation over the knee.

A.	C2
B.	C3
C.	C4
D.	C5
E.	C6
F.	L1
G.	L2
H.	L3
I.	L4
J.	L5
A

L3

Sensation over the knee is equivalent to the L3 dermatome. The four nerves involved include the infrapatellar branch of the saphenous nerve, the lateral cutaneous nerve of the thigh, anterior cutaneous nerve of the thigh (both lateral and medial branches).

Spinal disorders

Dorsal column lesion	
Loss vibration and proprioception
Tabes dorsalis, SACD
Spinothalamic tract lesion	
Loss of pain, sensation and temperature
Central cord lesion	
Flaccid paralysis of the upper limbs
Osteomyelitis	
Normally progressive
Staph aureus in IVDU, normally cervical region affected
Fungal infections in immunocompromised
Thoracic region affected in TB
Infarction spinal cord	
Dorsal column signs (loss of proprioception and fine discrimination)
Cord compression	
UMN signs
Malignancy
Haematoma
Fracture
Brown-sequard syndrome	
Hemisection of the spinal cord
Ipsilateral paralysis
Ipsilateral loss of proprioception and fine discrimination
Contralateral loss of pain and temperature
155
Q

A 42 year old woman is found to have a burst fracture of the C5 vertebral body. After a few months where would the level of injury be?

A.	C2
B.	C3
C.	C4
D.	C5
E.	C6
F.	L1
G.	L2
H.	L3
I.	L4
J.	L5
A

C6

A C5 burst fracture usually injures the C6 spinal cord situated at the C5 vertebrae and also the C4 spinal roots that exit the spinal column between the C4 and C5 vertebra. Such an injury should cause a loss of sensations in C4 dermatome and weak deltoids. Due to oedema , the biceps (C5) may be initially weak but should recover. The wrist extensors (C6), however, should remain weak and sensation at and below C6 should be severely compromised. A neurosurgeon would conclude that there is a burst fracture at C5 from the x-rays, an initial sensory level at C4 (the first abnormal sensory dermatome) and the partial loss of deltoids and biceps would imply a motor level at C4 (the highest abnormal muscle level). Over time, as the patient recovers the C4 roots and the C5 spinal cord, both the sensory level and motor level should end up at C6. Such recovery is often attributed to ‘root’ recovery.

Spinal disorders

Dorsal column lesion	
Loss vibration and proprioception
Tabes dorsalis, SACD
Spinothalamic tract lesion	
Loss of pain, sensation and temperature
Central cord lesion	
Flaccid paralysis of the upper limbs
Osteomyelitis	
Normally progressive
Staph aureus in IVDU, normally cervical region affected
Fungal infections in immunocompromised
Thoracic region affected in TB
Infarction spinal cord	
Dorsal column signs (loss of proprioception and fine discrimination)
Cord compression	
UMN signs
Malignancy
Haematoma
Fracture
Brown-sequard syndrome	
Hemisection of the spinal cord
Ipsilateral paralysis
Ipsilateral loss of proprioception and fine discrimination
Contralateral loss of pain and temperature
156
Q

A 56 year old man suddenly develops severe back pain. His pain has a radicular pattern. On examination he is unable to extend his great toe.

A.	C2
B.	C3
C.	C4
D.	C5
E.	C6
F.	L1
G.	L2
H.	L3
I.	L4
J.	L5
A

L5

Extensor hallucis longus is derived from L5 and loss of EHL function is a useful test to determine whether this level is involved.
Spinal disorders

Dorsal column lesion	
Loss vibration and proprioception
Tabes dorsalis, SACD
Spinothalamic tract lesion	
Loss of pain, sensation and temperature
Central cord lesion	
Flaccid paralysis of the upper limbs
Osteomyelitis	
Normally progressive
Staph aureus in IVDU, normally cervical region affected
Fungal infections in immunocompromised
Thoracic region affected in TB
Infarction spinal cord	
Dorsal column signs (loss of proprioception and fine discrimination)
Cord compression	
UMN signs
Malignancy
Haematoma
Fracture
Brown-sequard syndrome	
Hemisection of the spinal cord
Ipsilateral paralysis
Ipsilateral loss of proprioception and fine discrimination
Contralateral loss of pain and temperature

Dermatomes
C2 to C4 The C2 dermatome covers the occiput and the top part of the neck. C3 covers the lower part of the neck to the clavicle. C4 covers the area just below the clavicle.
C5 to T1 Situated in the arms. C5 covers the lateral arm at and above the elbow. C6 covers the forearm and the radial (thumb) side of the hand. C7 is the middle finger, C8 is the medial aspect of the hand, and T1 covers the medial side of the forearm.
T2 to T12 The thoracic covers the axillary and chest region. T3 to T12 covers the chest and back to the hip girdle. The nipples are situated in the middle of T4. T10 is situated at the umbilicus. T12 ends just above the hip girdle.
L1 to L5 The cutaneous dermatome representing the hip girdle and groin area is innervated by L1 spinal cord. L2 and 3 cover the front part of the thighs. L4 and L5 cover medial and lateral aspects of the lower leg.
S1 to S5 S1 covers the heel and the middle back of the leg. S2 covers the back of the thighs. S3 cover the medial side of the buttocks and S4-5 covers the perineal region. S5 is of course the lowest dermatome and represents the skin immediately at and adjacent to the anus.

157
Q

The sciatic nerve lies deep to the following structures except:

	Gluteus maximus
	The femoral cutaneous nerve
	Long head of biceps femoris
	Gluteus medius
	Branch of the inferior gluteal artery
A

The gluteus medius does not extend around to the sciatic nerve.
Sciatic nerve

The sciatic nerve is formed from the sacral plexus and is the largest nerve in the body. It is the continuation of the main part of the plexus arising from ventral rami of L4 to S3. These rami converge at the inferior border of piriformis to form the nerve itself. It passes through the inferior part of the greater sciatic foramen and emerges beneath piriformis. Medially, lie the inferior gluteal nerve and vessels and the pudendal nerve and vessels. It runs inferolaterally under the cover of gluteus maximus midway between the greater trochanter and ischial tuberosity. It receives its blood supply from the inferior gluteal artery. The nerve provides cutaneous sensation to the skin of the foot and the leg. It also innervates the posterior thigh muscles and the lower leg and foot muscles. The nerve splits into the tibial and common peroneal nerves approximately half way down the posterior thigh. The tibial nerve supplies the flexor muscles and the common peroneal nerve supplies the extensor muscles and the abductor muscles.

Summary points
Origin	Spinal nerves L4 - S3
Articular Branches	Hip joint
Muscular branches in upper leg	
Semitendinosus
Semimembranosus
Biceps femoris
Part of adductor magnus
Cutaneous sensation	
Posterior aspect of thigh (via cutaneous nerves)
Gluteal region
Entire lower leg (except the medial aspect)
Terminates	At the upper part of the popliteal fossa by dividing into the tibial and peroneal nerves

The nerve to the short head of the biceps femoris comes from the common peroneal part of the sciatic and the other muscular branches arise from the tibial portion.
The tibial nerve goes on to innervate all muscles of the foot except the extensor digitorum brevis (which is innervated by the common peroneal nerve).

158
Q

Which of the following upper limb muscles is not innervated by the radial nerve?

	Extensor carpi ulnaris
	Abductor digiti minimi
	Anconeus
	Supinator
	Brachioradialis
A

Mnemonic for radial nerve muscles: BEST

B rachioradialis
E xtensors
S upinator
T riceps

Abductor digiti minimi is innervated by the ulnar nerve.

Radial nerve

Continuation of posterior cord of the brachial plexus (root values C5 to T1)

Path
In the axilla: lies posterior to the axillary artery on subscapularis, latissimus dorsi and teres major.
Enters the arm between the brachial artery and the long head of triceps (medial to humerus).
Spirals around the posterior surface of the humerus in the groove for the radial nerve.
At the distal third of the lateral border of the humerus it then pierces the intermuscular septum and descends in front of the lateral epicondyle.
At the lateral epicondyle it lies deeply between brachialis and brachioradialis where it then divides into a superficial and deep terminal branch.
Deep branch crosses the supinator to become the posterior interosseous nerve.

Regions innervated
Motor (main nerve)	
Triceps
Anconeus
Brachioradialis
Extensor carpi radialis
Motor (posterior interosseous branch)	
Supinator
Extensor carpi ulnaris
Extensor digitorum
Extensor indicis
Extensor digiti minimi
Extensor pollicis longus and brevis
Abductor pollicis longus
Sensory	The area of skin supplying the proximal phalanges on the dorsal aspect of the hand is supplied by the radial nerve (this does not apply to the little finger and part of the ring finger)
159
Q

A 72 year old lady is suspected of having a femoral hernia. At which of the following sites is it most likely to be identifiable clinically?

	Mid inguinal point
	Above and medial to the pubic tubercle
	Below and lateral to the pubic tubercle
	Mid point of the inguinal ligament
	3 cm superomedially to the superficial inguinal ring
A

Femoral hernias exit the femoral canal below and lateral to the pubic tubercle. Femoral hernia occur mainly in women due to their difference in pelvic anatomy. They are at high risk of strangulation and therefore should be repaired.
Femoral canal

The femoral canal lies at the medial aspect of the femoral sheath. The femoral sheath is a fascial tunnel containing both the femoral artery laterally and femoral vein medially. The canal lies medial to the vein.

Borders of the femoral canal
Laterally	Femoral vein
Medially	Lacunar ligament
Anteriorly	Inguinal ligament
Posteriorly	Pectineal ligament

Contents
Lymphatic vessels
Cloquet’s lymph node

Physiological significance
Allows the femoral vein to expand to allow for increased venous return to the lower limbs.

Pathological significance
As a potential space, it is the site of femoral hernias. The relatively tight neck places these at high risk of strangulation.

160
Q

Which of the following is not considered a major branch of the descending thoracic aorta?

	Bronchial artery
	Mediastinal artery
	Inferior thyroid artery
	Posterior intercostal artery
	Oesophageal artery
A

The inferior thyroid artery is usually derived from the thyrocervical trunk, a branch of the subclavian artery.

Thoracic aorta

Origin T4
Terminates T12
Relations
Anteriorly (from top to bottom)-root of the left lung, the pericardium, the oesophagus, and the diaphragm
Posteriorly-vertebral column, azygos vein
Right- hemiazygos veins, thoracic duct
Left- left pleura and lung
Branches
Lateral segmental branches: Posterior intercostal arteries
Lateral visceral: Bronchial arteries supply bronchial walls and lung excluding the alveoli
Midline branches: Oesophageal arteries

161
Q

An 18 year old lady with troublesome hyperhidrosis of the hands and arms is due to undergo a sympathectomy to treat the condition. Which of the following should the surgeons divide to most effectively treat her condition?

	Sympathetic ganglia at T1, T2 and T3
	Sympathetic ganglia at T2 and T3
	Sympathetic ganglia at T1 and T2
	Stellate ganglion
	Superior cervical ganglion
A

To treat hyperhidrosis the sympathetic ganglia at T2 and T3 should be divided. Dividing the other structures listed would either carry a risk of Horners syndrome or be ineffective.

Sympathetic nervous system- anatomy

The cell bodies of the pre-ganglionic efferent neurones lie in the lateral horn of the grey matter of the spinal cord in the thoraco-lumbar regions.
The pre-ganglionic efferents leave the spinal cord at levels T1-L2. These pass to the sympathetic chain.
Lateral branches of the sympathetic chain connect it to every spinal nerve. These post ganglionic nerves will pass to structures that receive sympathetic innervation at the periphery.

Sympathetic chains
These lie on the vertebral column and run from the base of the skull to the coccyx.
Cervical region Lie anterior to the transverse processes of the cervical vertebrae and posterior to the carotid sheath.
Thoracic region Lie anterior to the neck of the upper ribs and and lateral sides of the lower thoracic vertebrae.They are covered by the parietal pleura
Lumbar region Enter by passing posterior to the medial arcuate ligament. Lie anteriorly to the vertebrae and medial to psoas major.

Sympathetic ganglia
Superior cervical ganglion lies anterior to C2 and C3.
Middle cervical ganglion (if present) C6
Stellate ganglion- anterior to transverse process of C7, lies posterior to the subclavian artery, vertebral artery and cervical pleura.
Thoracic ganglia are segmentally arranged.
There are usually 4 lumbar ganglia.

Clinical importance
Interruption of the head and neck supply of the sympathetic nerves will result in an ipsilateral Horners syndrome.
For treatment of hyperhidrosis the sympathetic denervation can be achieved by removing the second and third thoracic ganglia with their rami. Removal of T1 will cause a Horners syndrome and is therefore not performed.
In patients with vascular disease of the lower limbs a lumbar sympathetomy may be performed, either radiologically or (more rarely now) surgically. The ganglia of L2 and below are disrupted. If L1 is removed then ejaculation may be compromised (and little additional benefit conferred as the preganglionic fibres do not arise below L2.

162
Q

During a right hemicolectomy the caecum is mobilised. As the bowel is retracted medially a vessel is injured, posterior to the colon. Which of the following is the most likely vessel?

	Right colic artery
	Inferior vena cava
	Aorta
	External iliac artery
	Gonadal vessels
A

The gonadal vessels and ureter are important posterior relations that are at risk during a right hemicolectomy.
Caecum

Location	
Proximal right colon below the ileocaecal valve
Intraperitoneal
Posterior relations	
Psoas
Iliacus
Femoral nerve
Genitofemoral nerve
Gonadal vessels
Anterior relations	Greater omentum
Arterial supply	Ileocolic artery
Lymphatic drainage	Mesenteric nodes accompany the venous drainage

The caecum is the most distensible part of the colon and in complete large bowel obstruction with a competent ileocaecal valve the most likely site of eventual perforation.

163
Q

A 53 year old man with a carcinoma of the lower third of the oesophagus is undergoing an oesophagogastrectomy. As the surgeons mobilise the lower part of the oesophagus, where are they most likely to encounter the thoracic duct?

	Anterior to the oesophagus
	On the left side of the oesophagus
	On the right side of the oesophagus
	Immediately anterior to the azygos vein
	Posterior to the oesophagus
A

The thoracic duct lies posterior to the oesophagus and passes to the left at the level of the Angle of Louis. It enters the thorax at T12 together with the aorta.
Thoracic duct

Continuation of the cisterna chyli in the abdomen.
Enters the thorax at T12.
Lies posterior to the oesophagus for most of its intrathoracic course. Passes to the left at T5.
Lymphatics draining the left side of the head and neck join the thoracic duct prior to its insertion into the left brachiocephalic vein.
Lymphatics draining the right side of the head and neck drain via the subclavian and jugular trunks into the right lymphatic duct and thence into the mediastinal trunk and eventually the right brachiocephalic vein.
Its location in the thorax makes it prone to injury during oesophageal surgery. Some surgeons administer cream to patients prior to oesophagectomy so that it is easier to identify the cut ends of the duct.

164
Q

The common peroneal nerve, or its branches, supply the following muscles except:

	Peroneus longus
	Tibialis anterior
	Extensor hallucis longus
	Flexor digitorum brevis
	Extensor digitorum longus
A

Flexor digitorum is supplied by the tibial nerve.
Common peroneal nerve

Derived from the dorsal divisions of the sacral plexus (L4, L5, S1 and S2).

This nerve supplies the skin and fascia of the anterolateral surface of the leg and the dorsum of the foot. It also innervates the muscles of the anterior and peroneal compartments of the leg, extensor digitorum brevis as well as the knee, ankle and foot joints.

It is laterally placed within the sciatic nerve. From the bifurcation of the sciatic nerve it passes inferolaterally in the lateral and proximal part of the popliteal fossa, under the cover of biceps femoris and its tendon. To reach the posterior aspect of the fibular head. It ends by dividing into the deep and superficial peroneal nerves at the point where it winds around the lateral surface of the neck of the fibula in the body of peroneus longus, approximately 2cm distal to the apex of the head of the fibula. It is palpable posterior to the head of the fibula.

Branches
In the thigh Nerve to the short head of biceps
Articular branch (knee)
In the popliteal fossa Lateral cutaneous nerve of the calf
Neck of fibula Superficial and deep peroneal nerves

165
Q

An 83 year old lady presents with a femoral hernia and undergoes a femoral hernia repair. Which of the following forms the posterior wall of the femoral canal?

	Pectineal ligament
	Lacunar ligament
	Inguinal ligament
	Adductor longus
	Sartorius
A

Pectineal ligament

Femoral canal

The femoral canal lies at the medial aspect of the femoral sheath. The femoral sheath is a fascial tunnel containing both the femoral artery laterally and femoral vein medially. The canal lies medial to the vein.

Borders of the femoral canal
Laterally	Femoral vein
Medially	Lacunar ligament
Anteriorly	Inguinal ligament
Posteriorly	Pectineal ligament

Contents
Lymphatic vessels
Cloquet’s lymph node

Physiological significance
Allows the femoral vein to expand to allow for increased venous return to the lower limbs.

Pathological significance
As a potential space, it is the site of femoral hernias. The relatively tight neck places these at high risk of strangulation.

166
Q

A 45 year man presents with hand weakness. He is given a piece of paper to hold between his thumb and index finger. When the paper is pulled, the patient has difficulty maintaining a grip. Grip pressure is maintained by flexing the thumb at the interphalangeal joint. What is the most likely nerve lesion?

	Posterior interosseous nerve
	Deep branch of ulnar nerve
	Anterior interosseous nerve
	Superficial branch of the ulnar nerve
	Radial nerve
A

This is a description of Froment’s sign, which tests for ulnar nerve palsy. It mainly tests for the function of adductor pollicis. This is supplied by the deep branch of the ulnar nerve. Remember the anterior interosseous branch (of the median nerve), which innervates the flexor pollicis longus (hence causing flexion of the thumb IP joint), branches off more proximally to the wrist.

Ulnar nerve

Origin
C8, T1

Supplies (no muscles in the upper arm)
Flexor carpi ulnaris
Flexor digitorum profundus
Flexor digiti minimi
Abductor digiti minimi
Opponens digiti minimi
Adductor pollicis
Interossei muscle
Third and fourth lumbricals
Palmaris brevis

Path
Posteromedial aspect of upper arm to flexor compartment of forearm, then along the ulnar. Passes beneath the flexor carpi ulnaris muscle, then superficially through the flexor retinaculum into the palm of the hand.

Effects of injury
Damage at the wrist
Wasting and paralysis of intrinsic hand muscles (claw hand)
Wasting and paralysis of hypothenar muscles
Loss of sensation medial 1 and half fingers
Damage at the elbow
Radial deviation of the wrist
Clawing less in 4th and 5th digits

167
Q

Which of the following statements relating to the greater omentum is false?

It is less well developed in children under 5.
It has no relationship to the lesser sac.
It contains the gastroepiploic arteries.
Has an attachment to the transverse colon.
It may be a site of metastatic disease in ovarian cancer.
A

It is connected with the lesser sac and the transverse colon. This plane is entered when performing a colonic resection. It is a common site of metastasis in many visceral malignancies.
Omentum

The omentum is divided into two parts which invest the stomach. Giving rise to the greater and lesser omentum. The greater omentum is attached to the inferolateral border of the stomach and houses the gastro-epiploic arteries.
It is of variable size but is less well developed in children. This is important as the omentum confers protection against visceral perforation (e.g. Appendicitis).
Inferiorly between the omentum and transverse colon is one potential entry point into the lesser sac.
Several malignant processes may involve the omentum of which ovarian cancer is the most notable.

168
Q

A 48 year old man with newly diagnosed hypertension is found to have a phaeochromocytoma of the left adrenal gland and is due to undergo a laparoscopic left adrenalectomy. Which of the following structures is not directly related to the left adrenal gland?

	Crus of the diaphragm
	Lesser curvature of the stomach
	Kidney
	Pancreas
	Splenic artery
A

Lesser curvature of the stomach

The left adrenal gland is slightly larger than the right. It is crescent in shape and its concavity is adapted to the medial border of the upper part of the left kidney. The upper area is covered by peritoneum of the omental bursa which separates it from the cardia of the stomach. The lower area is in contact with the pancreas and splenic artery and is not covered by peritoneum. On the anterior surface is a hilum from which the suprarenal vein emerges. The lateral aspect rests on the kidney. The medial is small and is on the left crus of the diaphragm.

Adrenal gland anatomy

Anatomy

Location Superomedially to the upper pole of each kidney
Relationships of the right adrenal Diaphragm-Posteriorly, Kidney-Inferiorly, Vena Cava-Medially, Hepato-renal pouch and bare area of the liver-Anteriorly
Relationships of the left adrenal Crus of the diaphragm-Postero- medially, Pancreas and splenic vessels-Inferiorly, Lesser sac and stomach-Anteriorly

Arterial supply Superior adrenal arteries- from inferior phrenic artery, Middle adrenal arteries - from aorta, Inferior adrenal arteries -from renal arteries
Venous drainage of the right adrenal Via one central vein directly into the IVC
Venous drainage of the left adrenal Via one central vein into the left renal vein

169
Q

Which of the following nerves innervates the long head of the biceps femoris muscle?

	Inferior gluteal nerve
	Tibial division of sciatic nerve
	Superior gluteal nerve
	Common peroneal division of sciatic nerve
	Obturator nerve
A

The short head of biceps femoris, which may occasionally be absent, is innervated by the common peroneal component of the sciatic nerve. The long head is innervated by the tibial division of the sciatic nerve.

Biceps femoris

The biceps femoris is one of the hamstring group of muscles located in the posterior upper thigh. It has two heads.

Long head
Origin Ischial tuberosity
Insertion Fibular head
Action Knee flexion, lateral rotation tibia, extension hip
Innervation Tibial division of sciatic nerve (L5, S1, S2)
Arterial supply Profunda femoris artery, inferior gluteal artery, and the superior muscular branches of popliteal artery

Short head
Origin Lateral lip of linea aspera, lateral supracondylar ridge of femur
Insertion Fibular head
Action Knee flexion, lateral rotation tibia
Innervation Common peroneal division of sciatic nerve (L5, S1, S2)
Arterial supply Profunda femoris artery, inferior gluteal artery, and the superior muscular branches of popliteal artery

170
Q

A 72 year old male with end stage critical ischaemia is undergoing an axillo-femoral bypass. What structure is not closely related to the axillary artery?

	Posterior cord of the brachial plexus
	Scalenus anterior muscle
	Pectoralis minor muscle
	Axillary vein
	Lateral cord of the brachial plexus
A

Scalene anterior muscle

The axillary artery is the continuation of the subclavian artery. It is surrounded by the cords of the brachial plexus (from which they are named). The axillary vein runs alongside the axillary artery throughout its length.
Axilla

Boundaries of the axilla
Medially Chest wall and Serratus anterior
Laterally Humeral head
Floor Subscapularis
Anterior aspect Lateral border of Pectoralis major
Fascia Clavipectoral fascia

Content:
Long thoracic nerve (of Bell) Derived from C5-C7 and passes behind the brachial plexus to enter the axilla. It lies on the medial chest wall and supplies serratus anterior. Its location puts it at risk during axillary surgery and damage will lead to winging of the scapula.
Thoracodorsal nerve and thoracodorsal trunk Innervate and vascularise latissimus dorsi.
Axillary vein Lies at the apex of the axilla, it is the continuation of the basilic vein. Becomes the subclavian vein at the outer border of the first rib.
Intercostobrachial nerves Traverse the axillary lymph nodes and are often divided during axillary surgery. They provide cutaneous sensation to the axillary skin.
Lymph nodes The axilla is the main site of lymphatic drainage for the breast.

171
Q

A 28 year old man is shot in the right chest and develops a right haemothorax necessitating a thoracotomy. The surgeons decide to place a vascular clamp across the hilum of the right lung. Which of the following structures will lie most anteriorly at this point?

	Thoracic duct
	Phrenic nerve
	Vagus nerve
	Pulmonary artery
	Pulmonary vein
A

The phrenic nerve lies anteriorly at the root of the right lung.

Lung anatomy

The right lung is composed of 3 lobes divided by the oblique and transverse fissures. The left lung has two lobes divided by the oblique fissure.The apex of both lungs is approximately 4cm superior to the sterno-costal joint of the first rib. Immediately below this is a sulcus created by the subclavian artery.

Peripheral contact points of the lung
Base: diaphragm
Costal surface: corresponds to the cavity of the chest
Mediastinal surface: Contacts the mediastinal pleura. Has the cardiac impression. Above and behind this concavity is a triangular depression named the hilum, where the structures which form the root of the lung enter and leave the viscus. These structures are invested by pleura, which, below the hilum and behind the pericardial impression, forms the pulmonary ligament

Right lung
Above the hilum is the azygos vein; Superior to this is the groove for the superior vena cava and right innominate vein; behind this, and nearer the apex, is a furrow for the innominate artery. Behind the hilum and the attachment of the pulmonary ligament is a vertical groove for the oesophagus; In front and to the right of the lower part of the oesophageal groove is a deep concavity for the extrapericardiac portion of the inferior vena cava.

The root of the right lung lies behind the superior vena cava and the right atrium, and below the azygos vein.

The right main bronchus is shorter, wider and more vertical than the left main bronchus and therefore the route taken by most foreign bodies.

Left lung
Above the hilum is the furrow produced by the aortic arch, and then superiorly the groove accommodating the left subclavian artery; Behind the hilum and pulmonary ligament is a vertical groove produced by the descending aorta, and in front of this, near the base of the lung, is the lower part of the oesophagus.

The root of the left lung passes under the aortic arch and in front of the descending aorta.
Inferior borders of both lungs
6th rib in mid clavicular line
8th rib in mid axillary line
10th rib posteriorly
The pleura runs two ribs lower than the corresponding lung level.

172
Q

An 18 year old boy is undergoing an appendicectomy for appendicitis. At which of the following locations is the appendix most likely to be found?

	Pre ileal
	Pelvic
	Retrocaecal
	Post ileal
	None of the above
A

Most appendixes lie in the retrocaecal position. If a retrocaecal appendix is difficult to remove then mobilisation of the right colon significantly improves access.

Appendix

Location: Base of caecum.
Up to 10cm long.
Mainly lymphoid tissue (Hence mesenteric adenitis may mimic appendicitis).
Caecal taenia coli converge at base of appendix and form a longitudinal muscle cover over the appendix. This convergence should facilitate its identification at surgery if it is retrocaecal and difficult to find (which it can be when people start doing appendicectomies!)
Arterial supply: Appendicular artery (branch of the ileocolic).
It is intra peritoneal.

McBurney’s point
1/3 of the way along a line drawn from the Anterior Superior Iliac Spine to the Umbilicus

6 Positions:

Retrocaecal 74%
Pelvic 21%
Postileal
Subcaecal
Paracaecal
Preileal
173
Q

A 56 year old man is undergoing a pancreatectomy for carcinoma. During resection of the gland which of the following structures will the surgeon not encounter posterior to the pancreas itself?

	Left crus of the diaphragm
	Superior mesenteric vein
	Common bile duct
	Portal vein
	Gastroduodenal artery
A

The gastroduodenal artery divides into the gastro-epiploic and pancreaticoduodenal arteries at the superior aspect of the pancreas.

The pancreas is a retroperitoneal organ and lies posterior to the stomach. It may be accessed surgically by dividing the peritoneal reflection that connects the greater omentum to the transverse colon. The pancreatic head sits in the curvature of the duodenum. Its tail lies close to the hilum of the spleen, a site of potential injury during splenectomy.

Relations
Posterior to the pancreas
Pancreatic head	Inferior vena cava
Common bile duct 
Right and left renal veins 
Superior mesenteric vein and artery
Pancreatic neck	Superior mesenteric vein, portal vein
Pancreatic body-	Left renal vein
Crus of diaphragm
Psoas muscle
Adrenal gland
Kidney 
Aorta
Pancreatic tail	Left kidney
Anterior to the pancreas
Pancreatic head	1st part of the duodenum
Pylorus
Gastroduodenal artery
SMA and SMV(uncinate process)
Pancreatic body	Stomach
Duodenojejunal flexure
Pancreatic tail	Splenic hilum

Superior to the pancreas
Coeliac trunk and its branches common hepatic artery and splenic artery

Grooves of the head of the pancreas
2nd and 3rd part of the duodenum

Arterial supply
Head: pancreaticoduodenal artery
Rest: splenic artery

Venous drainage
Head: superior mesenteric vein
Body and tail: splenic vein

Ampulla of Vater
Merge of pancreatic duct and common bile duct
Is an important landmark, halfway along the second part of the duodenum, that marks the anatomical transition from foregut to midgut (also the site of transition between regions supplied by coeliac trunk and SMA).

174
Q

Which of the following bones is related to the cuboid’s distal articular surface?

	All metatarsals
	5th metatarsal
	Calcaneum
	Medial cuneiform
	3rd metatarsal
A

The cuboid is located at the lateral aspect of the foot between the calcaneus posteriorly and the 4th and 5th metatarsals distally.

Arches of the foot
The foot is conventionally considered to have two arches.
The longitudinal arch is higher on the medial than on the lateral side. The posterior part of the calcaneum forms a posterior pillar to support the arch. The lateral part of this structure passes via the cuboid bone and the lateral two metatarsal bones. The medial part of this structure is more important. The head of the talus marks the summit of this arch, located between the sustentaculum tali and the navicular bone. The anterior pillar of the medial arch is composed of the navicular bone, the three cuneiforms and the medial three metatarsal bones.
The transverse arch is situated on the anterior part of the tarsus and the posterior part of the metatarsus. The cuneiforms and metatarsal bases narrow inferiorly, which contributes to the shape of the arch.

Intertarsal joints
Sub talar joint Formed by the cylindrical facet on the lower surface of the body of the talus and the posterior facet on the upper surface of the calcaneus. The facet on the talus is concave anteroposteriorly, the other is convex. The synovial cavity of this joint does not communicate with any other joint.
Talocalcaneonavicular joint The anterior part of the socket is formed by the concave articular surface of the navicular bone, posteriorly by the upper surface of the sustentaculum tali. The talus sits within this socket
Calcaneocuboid joint Highest point in the lateral part of the longitudinal arch. The lower aspect of this joint is reinforced by the long plantar and plantar calcaneocuboid ligaments.
Transverse tarsal joint The talocalcaneonavicular joint and the calcaneocuboid joint extend across the tarsus in an irregular transverse plane, between the talus and calcaneus behind and the navicular and cuboid bones in front. This plane is termed the transverse tarsal joint.
Cuneonavicular joint Formed between the convex anterior surface of the navicular bone and the concave surface of the the posterior ends of the three cuneiforms.
Intercuneiform joints Between the three cuneiform bones.
Cuneocuboid joint Between the circular facets on the lateral cuneiform bone and the cuboid. This joint contributes to the tarsal part of the transverse arch.

Nerves in the foot

Lateral plantar nerve
Passes anterolaterally towards the base of the 5th metatarsal between flexor digitorum brevis and flexor accessorius. On the medial aspect of the lateral plantar artery. At the base of the 5th metatarsal it splits into superficial and deep branches.

Medial plantar nerve
Passes forwards with the medial plantar artery under the cover of the flexor retinaculum to the interval between abductor hallucis and flexor digitorum brevis on the sole of the foot.

Plantar arteries
Arise under the cover of the flexor retinaculum, midway between the tip of the medial malleolus and the most prominent part of the medial side of the heel.

Medial plantar artery. Passes forwards medial to medial plantar nerve in the space between abductor hallucis and flexor digitorum brevis.Ends by uniting with a branch of the 1st plantar metatarsal artery.
Lateral plantar artery. Runs obliquely across the sole of the foot. It lies lateral to the lateral plantar nerve. At the base of the 5th metatarsal bone it arches medially across the foot on the metatarsals

Dorsalis pedis artery
This vessel is a direct continuation of the anterior tibial artery. It commences on the front of the ankle joint and runs to the proximal end of the first metatarsal space. Here is gives off the arcuate artery and continues forwards as the first dorsal metatarsal artery. It is accompanied by two veins throughout its length. It is crossed by the extensor hallucis brevis

175
Q

From which structure is the central tendon of the diaphragm derived?

	Septum transversum
	Pleuroperitoneal folds
	Diaphragmatic crura
	Dorsal mesocardium
	Oropharyngeal membrane
A

The septum transversum is a thick ridge of mesodermal tissue in the developing embryo that separates the thoracic and abdominal cavities and forms the central tendon of the diaphragm.

Embryology
The diaphragm is formed between the 5th and 7th weeks of gestation through the progressive fusion of the septum transversum, pleuroperitoneal folds and via lateral muscular ingrowth. The muscular origins of the diaphragm are somites located in cervical segments 3 to 5, which accounts for the long path taken by the phrenic nerve. The components contribute to the following diaphragmatic segments:
Septum transversum - Central tendon
Pleuroperitoneal membranes - Parietal membranes surrounding viscera
Cervical somites C5 to C7 - Muscular component of the diaphragm

Diaphragmatic hernia
Type of hernia	Features
Morgagni	Anteriorly located
Minimal compromise on lung development
Minimal signs on antenatal ultrasound
Usually present later
Usually good prognosis
Bochdalek hernia	Posteriorly located
Larger defect
Often diagnosed antenatally
Associated with pulmonary hypoplasia
Poor prognosis

The posterior hernias of Bochdalek are the most common type and if not diagnosed antenatally will typically present soon after birth with respiratory distress. The classical finding is that of a scaphoid abdomen on clinical examination because of herniation of the abdominal contents into the chest. Bochdalek hernias are associated with a number of chromosomal abnormalities such as Trisomy 21 and 18. Infants have considerable respiratory distress due to hypoplasia of the developing lung. Historically this was considered to be due to direct compression of the lung by herniated viscera. This view over simplifies the situation and the pulmonary hypoplasia occurs concomitantly with the hernial development, rather than as a direct result of it. The pulmonary hypoplasia is associated with pulmonary hypertension and abnormalities of pulmonary vasculature. The pulmonary hypertension renders infants at risk of right to left shunting (resulting in progressive and worsening hypoxia).
Diagnostic work up of these infants includes chest x-rays/ abdominal ultrasound scans and cardiac echo.
Surgery forms the mainstay of treatment and both thoracic and abdominal approaches may be utilised. Following reduction of the hernial contents a careful search needs to be made for a hernial sac as failure to recognise and correct this will result in a high recurrence rate. Smaller defects may be primarily closed, larger defects may require a patch to close the defect. Malrotation of the viscera is a recognised association and may require surgical correct at the same procedure (favoring an abdominal approach).
The mortality rate is 50-75% and is related to the degree of lung compromise and age at presentation (considerably better in infants >24 hours old).

176
Q

Where does Stensens duct primarily open?

	Immediately lateral to the foramen caecum
	Floor of mouth
	Opposite the second molar tooth
	Opposite the fifth molar tooth
	Into the post nasal space
A

Stensens duct conveys secretions from the parotid gland and these enter the oral cavity at the level of the second molar tooth.

Anatomy of the parotid gland
Location Overlying the mandibular ramus; anterior and inferior to the ear.
Salivary duct Crosses the masseter, pierces the buccinator and drains adjacent to the 2nd upper molar tooth (Stensen’s duct).
Structures passing through the gland
Facial nerve (Mnemonic: The Zebra Buggered My Cat; Temporal Zygomatic, Buccal, Mandibular, Cervical)
External carotid artery
Retromandibular vein
Auriculotemporal nerve
Relations
Anterior: masseter, medial pterygoid, superficial temporal and maxillary artery, facial nerve, stylomandibular ligament
Posterior: posterior belly digastric muscle, sternocleidomastoid, stylohyoid, internal carotid artery, mastoid process, styloid process
Arterial supply Branches of external carotid artery
Venous drainage Retromandibular vein
Lymphatic drainage Deep cervical nodes
Nerve innervation
Parasympathetic-Secretomotor
Sympathetic-Superior cervical ganglion
Sensory- Greater auricular nerve

Parasympathetic stimulation produces a water rich, serous saliva. Sympathetic stimulation leads to the production of a low volume, enzyme-rich saliva.

177
Q

Which of the following nerves is responsible for the innervation of the posterior belly of the digastric muscle?

	Facial nerve
	Hypoglossal nerve
	Trigeminal nerve
	Ansa cervicalis
	Mylohoid nerve
A

The posterior belly of digastric is innervated by the facial nerve and the anterior belly by the mylohoid nerve.

Anterior triangle of the neck

Boundaries
Anterior border of the Sternocleidomastoid
Lower border of mandible
Anterior midline

Sub triangles (divided by Digastric above and Omohyoid)
Muscular triangle: Neck strap muscles
Carotid triangle: Carotid sheath
Submandibular Triangle (digastric)

Contents of the anterior triangle
Digastric triangle	Submandibular gland
Submandibular nodes
Facial vessels
Hypoglossal nerve
Muscular triangle	Strap muscles
External jugular vein
Carotid triangle	Carotid sheath (Common carotid, vagus and internal jugular vein)
Ansa cervicalis

Nerve supply to digastric muscle
Anterior: Mylohyoid nerve
Posterior: Facial nerve

178
Q

Which of the following carpal bones is a sesamoid bone in the tendon of flexor carpi ulnaris?

	Triquetrum
	Lunate
	Pisiform
	Scaphoid
	Capitate
A

Pisiform

This small bone has a single articular facet. It projects from the triquetral bone at the ulnar aspect of the wrist where most regard it as a sesamoid bone lying within the tendon of flexor carpi ulnaris.
Carpal bones

The wrist is comprised of 8 carpal bones, these are arranged in two rows of 4. It is convex from side to side posteriorly and concave anteriorly.
No tendons attach to: Scaphoid, lunate, triquetrum (stabilised by ligaments)

179
Q

A 70 year old man falls and fractures his scaphoid bone. The fracture is displaced and the decision is made to insert a screw to fix the fracture. Which of the following structures lies directly medial to the scaphoid?

	Lunate
	Pisiform
	Trapezoid
	Trapezium
	None of the above
A

The lunate lies medially in the anatomical plane. Fractures of the scaphoid that are associated with high velocity injuries may cause associated lunate dislocation.

The scaphoid has a concave articular surface for the head of the capitate and at the edge of this is a crescentic surface for the corresponding area on the lunate.
Proximally, it has a wide convex articular surface with the radius. It has a distally sited tubercle that can be palpated. The remaining articular surface is to the lateral side of the tubercle. It faces laterally and is associated with the trapezium and trapezoid bones.

The narrow strip between the radial and trapezial surfaces and the tubercle gives rise to the radial collateral carpal ligament. The tubercle receives part of the flexor retinaculum. This area is the only part of the scaphoid that is available for the entry of blood vessels. It is commonly fractured and avascular necrosis may result.

180
Q

A 55 year old man is admitted with a brisk haematemesis. He is taken to the endoscopy department and an upper GI endoscopy is performed by the gastroenterologist. He identifies an ulcer on the posterior duodenal wall and spends an eternity trying to control the bleeding with all the latest haemostatic techniques. He eventually asks the surgeons for help. A laparotomy and anterior duodenotomy are performed, as the surgeon opens the duodenum a vessel is spurting blood into the duodenal lumen. From which of the following does this vessel arise?

	Left gastric artery
	Common hepatic artery
	Right hepatic artery
	Superior mesenteric artery
	Splenic artery
A

The vessel will be the gastroduodenal artery, this arises from the common hepatic artery.

Gastroduodenal artery

Supplies
Pylorus, proximal part of the duodenum, and indirectly to the pancreatic head (via the anterior and posterior superior pancreaticoduodenal arteries)

Path
The gastroduodenal artery most commonly arises from the common hepatic artery of the coeliac trunk. It terminates by bifurcating into the right gastroepiploic artery and the superior pancreaticoduodenal artery

181
Q

A 73 year old lady is hit by a car. She suffers a complex fracture of the distal aspect of her humerus with associated injury to the radial nerve. Which of the following movements will be most impaired as a result?

	Elbow extension
	Elbow flexion
	Shoulder abduction
	Wrist extension
	None of the above
A

The triceps will not be affected so elbow extension will be preserved. Loss of wrist extension will be the most obvious effect.

Radial nerve

Continuation of posterior cord of the brachial plexus (root values C5 to T1)

Path
In the axilla: lies posterior to the axillary artery on subscapularis, latissimus dorsi and teres major.
Enters the arm between the brachial artery and the long head of triceps (medial to humerus).
Spirals around the posterior surface of the humerus in the groove for the radial nerve.
At the distal third of the lateral border of the humerus it then pierces the intermuscular septum and descends in front of the lateral epicondyle.
At the lateral epicondyle it lies deeply between brachialis and brachioradialis where it then divides into a superficial and deep terminal branch.
Deep branch crosses the supinator to become the posterior interosseous nerve.

182
Q

A 10 year old boy is referred to the orthopaedic clinic with symptoms of right knee pain. He has suffered pain for the past 3 months and the pain typically lasts for several hours. On examination he walks with an antalgic gait and has apparent right leg shortening. The right knee is normal but the right hip reveals pain on internal and external rotation. Imaging shows flattening of the femoral head. Which of the following is the most likely underlying diagnosis?

	Osteogenesis imperfecta
	Child abuse
	Osteosarcoma
	Osteopetrosis
	Perthes disease
A

This is a typical description of Perthes disease. Management involves keeping the femoral head in the acetabulum by braces, casts or surgery.

Perthes disease
Idiopathic avascular necrosis of the femoral epiphysis of the femoral head
Impaired blood supply to femoral head, causing bone infarction. New vessels develop and ossification occurs. The bone either heals or a subchondral fracture occurs.

Clinical features
Males 4x's greater than females
Age between 2-12 years (the younger the age of onset, the better the prognosis)
Limp
Hip pain
Bilateral in 20%

Diagnosis
Plain x-ray, Technetium bone scan or magnetic resonance imaging if normal x-ray and symptoms persist.

Catterall staging
Stage Features
Stage 1 Clinical and histological features only
Stage 2 Sclerosis with or without cystic changes and preservation of the articular surface
Stage 3 Loss of structural integrity of the femoral head
Stage 4 Loss of acetabular integrity

Management
To keep the femoral head within the acetabulum: cast, braces
If less than 6 years: observation
Older: surgical management with moderate results
Operate on severe deformities

Prognosis
Most cases will resolve with conservative management. Early diagnosis improves outcomes.

183
Q

Which statement relating to talipes equinovarus is untrue?

It has an annual incidence of around 1 in 1000 in the UK.
The muscles involved in the disorder are intrinsically abnormal.
The cuboid is classically displaced medially.
All cases should be treated with an Ilizarov frame initially unless there is minor deformity.
The talocalcaneal angle is typically less than 20 degrees in club foot.
A

False: All cases should be treated with an Ilizarov frame initially unless there is minor deformity.

In most cases of Club Foot conservative measures should be tried first. The Ponsetti method is a popular approach. Severe cases may benefit from Ilizarov frame re-aligment.

Congenital talipes equinovarus.
Features:
Equinus of the hindfoot.
Adduction and varus of the midfoot.
High arch.

Most cases in developing countries. Incidence in UK is 1 per 1000 live births. It is more common in males and is bilateral in 50% cases. There is a strong familial link(1). It may also be associated with other developmental disorders such as Down’s syndrome.

Key anatomical deformities (2):
Adducted and inverted calcaneus
Wedge shaped distal calcaneal articular surface
Severe Tibio-talar plantar flexion.
Medial Talar neck inclination
Displacement of the navicular bone (medially)
Wedge shaped head of talus
Displacement of the cuboid (medially)

Management
Conservative first, the Ponseti method is best described and gives comparable results to surgery. It consists of serial casting to mold the foot into correct shape. Following casting around 90% will require a Achilles tenotomy. This is then followed by a phase of walking braces to maintain the correction.

Surgical correction is reserved for those cases that fail to respond to conservative measures. The procedures involve multiple tenotomies and lengthening procedures. In patients who fail to respond surgically an Ilizarov frame reconstruction may be attempted and gives good results.

184
Q

Which of the following is not a content of the rectus sheath?

	Pyramidalis
	Superior epigastric artery
	Inferior epigastric vein
	Internal iliac artery
	Rectus abdominis
A

Rectus sheath doesn’t contain ext. iliac artery

The rectus sheath also contains:
superior epigastric vein
inferior epigastric artery
The 2 main muscles of the abdominal wall are the rectus abdominis (anterior) and the quadratus lumborum (posterior).
The remaining abdominal wall consists of 3 muscular layers. Each muscle passes from the lateral aspect of the quadratus lumborum posteriorly to the lateral margin of the rectus sheath anteriorly. Each layer is muscular posterolaterally and aponeurotic anteriorly.

Muscles of abdominal wall
External oblique
Lies most superficially
Originates from 5th to 12th ribs
Inserts into the anterior half of the outer aspect of the iliac crest, linea alba and pubic tubercle
More medially and superiorly to the arcuate line, the aponeurotic layer overlaps the rectus abdominis muscle
The lower border forms the inguinal ligament
The triangular expansion of the medial end of the inguinal ligament is the lacunar ligament.
Internal oblique
Arises from the thoracolumbar fascia, the anterior 2/3 of the iliac crest and the lateral 2/3 of the inguinal ligament
The muscle sweeps upwards to insert into the cartilages of the lower 3 ribs
The lower fibres form an aponeurosis that runs from the tenth costal cartilage to the body of the pubis
At its lowermost aspect it joins the fibres of the aponeurosis of transversus abdominis to form the conjoint tendon.
Transversus abdominis
Innermost muscle
Arises from the inner aspect of the costal cartilages of the lower 6 ribs , from the anterior 2/3 of the iliac crest and lateral 1/3 of the inguinal ligament
Its fibres run horizontally around the abdominal wall ending in an aponeurosis. The upper part runs posterior to the rectus abdominis. Lower down the fibres run anteriorly only.
The rectus abdominis lies medially; running from the pubic crest and symphysis to insert into the xiphoid process and 5th, 6th and 7th costal cartilages. The muscles lies in a aponeurosis as described above.
Nerve supply: anterior primary rami of T7-12

Surgical notes
During abdominal surgery it is usually necessary to divide either the muscles or their aponeuroses. During a midline laparotomy it is desirable to divide the aponeurosis. This will leave the rectus sheath intact above the arcuate line and the muscles intact below it. Straying off the midline will often lead to damage to the rectus muscles, particularly below the arcuate line where they may often be in close proximity to each other.

185
Q

Which of the following vessels does not drain directly into the inferior vena cava?

	Superior mesenteric vein
	Right common iliac
	Right hepatic vein
	Left hepatic vein
	Right testicular vein
A

The superior mesenteric vein drains into the portal vein. The right and left hepatic veins drain into it directly, this can account for major bleeding in more extensive liver shearing type injuries.

Inferior vena cava

Origin
L5

Path
Left and right common iliac veins merge to form the IVC.
Passes right of midline
Paired segmental lumbar veins drain into the IVC throughout its length
The right gonadal vein empties directly into the cava and the left gonadal vein generally empties into the left renal vein.
The next major veins are the renal veins and the hepatic veins
Pierces the central tendon of diaphragm at T8
Right atrium

Relations
Anteriorly Small bowel, first and third part of duodenum, head of pancreas, liver and bile duct, right common iliac artery, right gonadal artery
Posteriorly Right renal artery, right psoas, right sympathetic chain, coeliac ganglion

Levels
Level	Vein
T8	Hepatic vein, inferior phrenic vein, pierces diaphragm
L1	Suprarenal veins, renal vein
L2	Gonadal vein
L1-5	Lumbar veins
L5	Common iliac vein, formation of IVC
186
Q

A 17 year old male has a suspected testicular torsion and the scrotum is to be explored surgically. The surgeon incises the skin and then the dartos muscle. What is the next tissue layer that will be encountered during the dissection?

	Visceral layer of the tunica vaginalis
	Cremasteric fascia
	Parietal layer of the tunica vaginalis
	External spermatic fascia
	Internal spermatic fascia
A

The layers that will be encountered are (in order):

  1. Skin
  2. Dartos fascia and muscle
  3. External spermatic fascia
  4. Cremasteric muscle and fascia
  5. Internal spermatic fascia
  6. Parietal layer of the tunica vaginalis

The layers of the spermatic cord and scrotum are a popular topic in the MRCS exam.
A mnemonic which may help:
Some Damned Examiner Called It The Testes (skin dartos external fascia cremaster internal fascia tunica Testes)
Spermatic cord
Formed by the vas deferens and is covered by the following structures:
Layer Origin
Internal spermatic fascia Transversalis fascia
Cremasteric fascia From the fascial coverings of internal oblique
External spermatic fascia External oblique aponeurosis

Contents of the cord
Vas deferens Transmits sperm and accessory gland secretions
Testicular artery Branch of abdominal aorta supplies testis and epididymis
Artery of vas deferens Arises from inferior vesical artery
Cremasteric artery Arises from inferior epigastric artery
Pampiniform plexus Venous plexus, drains into right or left testicular vein
Sympathetic nerve fibres Lie on arteries, the parasympathetic fibres lie on the vas
Genital branch of the genitofemoral nerve Supplies cremaster
Lymphatic vessels Drain to lumbar and para-aortic nodes

Scrotum
Composed of skin and closely attached dartos fascia.
Arterial supply from the anterior and posterior scrotal arteries
Lymphatic drainage to the inguinal lymph nodes
Parietal layer of the tunica vaginalis is the innermost layer

Testes
The testes are surrounded by the tunica vaginalis (closed peritoneal sac). The parietal layer of the tunica vaginalis adjacent to the internal spermatic fascia.
The testicular arteries arise from the aorta immediately inferiorly to the renal arteries.
The pampiniform plexus drains into the testicular veins, the left drains into the left renal vein and the right into the inferior vena cava.
Lymphatic drainage is to the para-aortic nodes.

187
Q

A 66 year old male is admitted to the vascular ward for an amputation. He reports episodes of vertigo and dysarthria to the house officer. He suddenly collapses with a Glasgow Coma Score of 3. What is the most likely diagnosis?

Cerebral haemorrhage in left temporal parietal area
Opiate overdose
Cerebral haemorrhage in right temporal parietal area
Diazepam overdose
Basilar artery occlusion
A

Vertigo and dysarthria suggest a posterior circulation event. In the scenario of a patient complaining of posterior symptoms and a sudden deterioration in consciousness, the main differential diagnosis is of a basilar artery occlusion.

Primary intracerebral haemorrhage (PICH, c. 10%)
Presents with headache, vomiting, loss of consciousness
Total anterior circulation infarcts (TACI, c. 15%)
Involves middle and anterior cerebral arteries
Hemiparesis/hemisensory loss
Homonymous hemianopia
Higher cognitive dysfunction e.g. Dysphasia
Partial anterior circulation infarcts (PACI, c. 25%)
Involves smaller arteries of anterior circulation e.g. upper or lower division of middle cerebral artery
Higher cognitive dysfunction or two of the three TACI features
Lacunar infarcts (LACI, c. 25%)
Involves perforating arteries around the internal capsule, thalamus and basal ganglia
Present with either isolated hemiparesis, hemisensory loss or hemiparesis with limb ataxia
Posterior circulation infarcts (POCI, c. 25%)
Vertebrobasilar arteries
Presents with features of brainstem damage
Ataxia, disorders of gaze and vision, cranial nerve lesions
Lateral medullary syndrome (posterior inferior cerebellar artery)
Wallenberg’s syndrome
Ipsilateral: ataxia, nystagmus, dysphagia, facial numbness, cranial nerve palsy
Contralateral: limb sensory loss
Weber’s syndrome
Ipsilateral III palsy
Contralateral weakness

Anterior cerebral artery
Contralateral hemiparesis and sensory loss, lower extremity > upper
Disconnection syndrome

Middle cerebral artery
Contralateral hemiparesis and sensory loss, upper extremity > lower
Contralateral hemianopia
Aphasia (Wernicke's)
Gaze abnormalities

Posterior cerebral artery
Contralateral hemianopia with macular sparing
Disconnection syndrome

Lacunar
Present with either isolated hemiparesis, hemisensory loss or hemiparesis with limb ataxia

Lateral medulla (posterior inferior cerebellar artery)
Ipsilateral: ataxia, nystagmus, dysphagia, facial numbness, cranial nerve palsy e.g.

Horner’s
Contralateral: limb sensory loss

Pontine
VI nerve: horizontal gaze palsy
VII nerve
Contralateral hemiparesis

188
Q

A 22 year old man is brought to the emergency department. He was found lying unconscious on his right arm and it is evident that he has taken a temazepam overdose. His right arm is mottled in colour and swollen, his hand is insensate and stiff. What substance is most likely to be present in the urine in increased quantities?

	Protein
	Haemoglobin
	Myoglobin
	Erythrocytes
	Lymphocytes
A

This man is likely to have muscle death secondary to compartment syndrome. This will result in muscle breakdown and release of myoglobin. This may accumulate in the kidney and result in renal failure.

Compartment syndrome
This is a particular complication that may occur following fractures (or following ischaemia re-perfusion injury in vascular patients). It is characterised by raised pressure within a closed anatomical space.
The raised pressure within the compartment will eventually compromise tissue perfusion resulting in necrosis. The two main fractures carrying this complication include supracondylar fractures and tibial shaft injuries.

Symptoms and signs
Pain, especially on movement (even passive)
Parasthesiae
Pallor may be present
Arterial pulsation may still be felt as the necrosis occurs as a result of microvascular compromise
Paralysis of the muscle group may occur

Diagnosis
Is made by measurement of intracompartmental pressure measurements. Pressures in excess of 20mmHg are abnormal and >40mmHg is diagnostic.

Treatment
This is essentially prompt and extensive fasciotomies
In the lower limb the deep muscles may be inadequately decompressed by the inexperienced operator when smaller incisions are performed
Myoglobinuria may occur following fasciotomy and result in renal failure and for this reason these patients require aggressive IV fluids
Where muscle groups are frankly necrotic at fasciotomy they should be debrided and amputation may have to be considered
Death of muscle groups may occur within 4-6 hours

189
Q

A 25 year old man is stabbed in the groin and the area, which lies within the femoral triangle is explored. Which structure forms the lateral wall of the femoral triangle?

	Adductor longus
	Pectineus
	Adductor magnus
	Sartorius
	Conjoint tendon
A

The sartorius forms the lateral wall of the femoral triangle

Femoral triangle anatomy
Boundaries
Superiorly	Inguinal ligament
Laterally	Sartorius
Medially	Adductor longus
Floor	Iliopsoas, adductor longus and pectineus
Roof	
Fascia lata and Superficial fascia
Superficial inguinal lymph nodes (palpable below the inguinal ligament)
Long saphenous vein

Contents
Femoral vein (medial to lateral)
Femoral artery-pulse palpated at the mid inguinal point
Femoral nerve
Deep and superficial inguinal lymph nodes
Lateral cutaneous nerve
Great saphenous vein
Femoral branch of the genitofemoral nerve

190
Q

An 18 year old man develops a severe spreading sepsis of the hand. The palm is explored surgically and the flexor digiti minimi brevis muscle is mobilised to facilitate drainage of the infection. Which of the following structures is not closely related to this muscle?

	The hook of hamate
	Median nerve
	Superficial palmar arterial arch
	Digital nerves arising from the ulnar nerve
	None of the above
A

Median Nerve

The flexor digiti minimi brevis originates from the Hamate, on its under- surface lie the ulnar contribution to the superficial palmar arterial arch and digital nerves derived from the ulnar nerve. The median nerve overlies the flexor tendons.

Anatomy of the hand
Bones	
8 Carpal bones
5 Metacarpals
14 phalanges
Intrinsic Muscles	7 Interossei - Supplied by ulnar nerve
3 palmar-adduct fingers
4 dorsal- abduct fingers
Intrinsic muscles	Lumbricals
Flex MCPJ and extend the IPJ.
Origin deep flexor tendon and insertion dorsal extensor hood mechanism.
Innervation: 1st and 2nd- median nerve, 3rd and 4th- deep branch of the ulnar nerve.
Thenar eminence	
Abductor pollicis brevis
Opponens pollicis
Flexor pollicis brevis
Hypothenar eminence	
Opponens digiti minimi
Flexor digiti minimi brevis
Abductor digiti minimi

Fascia and compartments of the palm
The fascia of the palm is continuous with the antebrachial fascia and the fascia of the dorsum of the hand. The palmar fascia is thin over the thenar and hypothenar eminences. In contrast, the central palmar fascia is relatively thick. The palmar aponeurosis covers the soft tissues and overlies the flexor tendons. The apex of the palmar aponeurosis is continuous with the flexor retinaculum and the palmaris longus tendon. Distally, it forms four longitudinal digital bands that attach to the bases of the proximal phalanges, blending with the fibrous digital sheaths.
A medial fibrous septum extends deeply from the medial border of the palmar aponeurosis to the 5th metacarpal. Lying medial to this are the hypothenar muscles. In a similar fashion, a lateral fibrous septum extends deeply from the lateral border of the palmar aponeurosis to the 3rd metacarpal. The thenar compartment lies lateral to this area.
Lying between the thenar and hypothenar compartments is the central compartment. It contains the flexor tendons and their sheaths, the lumbricals, the superficial palmar arterial arch and the digital vessels and nerves.
The deepest muscular plane is the adductor compartment, which contains adductor pollicis.

Short muscles of the hand
These comprise the lumbricals and interossei. The four slender lumbrical muscles flex the fingers at the metacarpophalangeal joints and extend the interphalangeal joint. The four dorsal interossei are located between the metacarpals and the four palmar interossei lie on the palmar surface of the metacarpals in the interosseous compartment of the hand.

Long flexor tendons and sheaths in the hand
The tendons of FDS and FDP enter the common flexor sheath deep to the flexor retinaculum. The tendons enter the central compartment of the hand and fan out to their respective digital synovial sheaths. Near the base of the proximal phalanx, the tendon of FDS splits to permit the passage of FDP. The FDP tendons are attached to the margins of the anterior aspect of the base of the distal phalanx.
The fibrous digital sheaths contain the flexor tendons and their synovial sheaths. These extend from the heads of the metacarpals to the base of the distal phalanges.

191
Q

A 19 year old man undergoes an open inguinal hernia repair. The cord is mobilised and the deep inguinal ring identified. Which of the following structures forms its lateral wall?

	External oblique aponeurosis
	Transversalis fascia
	Conjoint tendon
	Inferior epigastric artery
	Inferior epigastric vein
A

The transversalis fascia forms the superolateral edge of the deep inguinal ring. The epigastric vessels form its inferomedial wall.

Inguinal canal
Location
Above the inguinal ligament
The inguinal canal is 4cm long
The superficial ring is located anterior to the pubic tubercle
The deep ring is located approximately 1.5-2cm above the half way point between the anterior superior iliac spine and the pubic tubercle

Boundaries of the inguinal canal
Floor	
External oblique aponeurosis
Inguinal ligament
Lacunar ligament
Roof	
Internal oblique
Transversus abdominis
Anterior wall	External oblique aponeurosis
Posterior wall	
Transversalis fascia
Conjoint tendon
Laterally	
Internal ring
Transversalis fascia
Fibres of internal oblique
Medially	
External ring
Conjoint tendon

Contents
Males Spermatic cord and ilioinguinal nerve As it passes through the canal the spermatic cord has 3 coverings:
External spermatic fascia from external oblique aponeurosis
Cremasteric fascia
Internal spermatic fascia
Females Round ligament of uterus and ilioinguinal nerve

192
Q

A 22 year old man develops an infection in the pulp of his little finger. What is the most proximal site to which this infection may migrate?

	The metacarpophalangeal joint
	The distal interphalangeal joint
	The proximal interphalangeal joint
	Proximal to the flexor retinaculum
	Immediately distal to the carpal tunnel
A

The 5th tendon sheath extends from the little finger to the proximal aspect of the carpal tunnel. This carries a significant risk of allowing infections to migrate proximally.

Hand
Anatomy of the hand
Bones	
8 Carpal bones
5 Metacarpals
14 phalanges
Intrinsic Muscles	7 Interossei - Supplied by ulnar nerve
3 palmar-adduct fingers
4 dorsal- abduct fingers
Intrinsic muscles	Lumbricals
Flex MCPJ and extend the IPJ.
Origin deep flexor tendon and insertion dorsal extensor hood mechanism.
Innervation: 1st and 2nd- median nerve, 3rd and 4th- deep branch of the ulnar nerve.
Thenar eminence	
Abductor pollicis brevis
Opponens pollicis
Flexor pollicis brevis
Hypothenar eminence	
Opponens digiti minimi
Flexor digiti minimi brevis
Abductor digiti minimi

Fascia and compartments of the palm
The fascia of the palm is continuous with the antebrachial fascia and the fascia of the dorsum of the hand. The palmar fascia is thin over the thenar and hypothenar eminences. In contrast, the central palmar fascia is relatively thick. The palmar aponeurosis covers the soft tissues and overlies the flexor tendons. The apex of the palmar aponeurosis is continuous with the flexor retinaculum and the palmaris longus tendon. Distally, it forms four longitudinal digital bands that attach to the bases of the proximal phalanges, blending with the fibrous digital sheaths.
A medial fibrous septum extends deeply from the medial border of the palmar aponeurosis to the 5th metacarpal. Lying medial to this are the hypothenar muscles. In a similar fashion, a lateral fibrous septum extends deeply from the lateral border of the palmar aponeurosis to the 3rd metacarpal. The thenar compartment lies lateral to this area.
Lying between the thenar and hypothenar compartments is the central compartment. It contains the flexor tendons and their sheaths, the lumbricals, the superficial palmar arterial arch and the digital vessels and nerves.
The deepest muscular plane is the adductor compartment, which contains adductor pollicis.

Short muscles of the hand
These comprise the lumbricals and interossei. The four slender lumbrical muscles flex the fingers at the metacarpophalangeal joints and extend the interphalangeal joint. The four dorsal interossei are located between the metacarpals and the four palmar interossei lie on the palmar surface of the metacarpals in the interosseous compartment of the hand.

Long flexor tendons and sheaths in the hand
The tendons of FDS and FDP enter the common flexor sheath deep to the flexor retinaculum. The tendons enter the central compartment of the hand and fan out to their respective digital synovial sheaths. Near the base of the proximal phalanx, the tendon of FDS splits to permit the passage of FDP. The FDP tendons are attached to the margins of the anterior aspect of the base of the distal phalanx.
The fibrous digital sheaths contain the flexor tendons and their synovial sheaths. These extend from the heads of the metacarpals to the base of the distal phalanges.

193
Q

Which of the following muscles is not innervated by the deep branch of the ulnar nerve?

	Adductor pollicis
	Hypothenar muscles
	All the interosseous muscles
	Opponens pollicis
	Third and fourth lumbricals
A

Opponens Pollicis

Ulnar nerve
Origin
C8, T1

Supplies (no muscles in the upper arm)
Flexor carpi ulnaris
Flexor digitorum profundus
Flexor digiti minimi
Abductor digiti minimi
Opponens digiti minimi
Adductor pollicis
Interossei muscle
Third and fourth lumbricals
Palmaris brevis

Path
Posteromedial aspect of upper arm to flexor compartment of forearm, then along the ulnar. Passes beneath the flexor carpi ulnaris muscle, then superficially through the flexor retinaculum into the palm of the hand.

Branch Supplies
Muscular branch Flexor carpi ulnaris
Medial half of the flexor digitorum profundus
Palmar cutaneous branch (Arises near the middle of the forearm) Skin on the medial part of the palm
Dorsal cutaneous branch Dorsal surface of the medial part of the hand
Superficial branch Cutaneous fibres to the anterior surfaces of the medial one and one-half digits
Deep branch Hypothenar muscles
All the interosseous muscles
Third and fourth lumbricals
Adductor pollicis
Medial head of the flexor pollicis brevis

Effects of injury
Damage at the wrist
Wasting and paralysis of intrinsic hand muscles (claw hand)
Wasting and paralysis of hypothenar muscles
Loss of sensation medial 1 and half fingers
Damage at the elbow
Radial deviation of the wrist
Clawing less in 4th and 5th digits

194
Q

During an inguinal hernia repair the surgeon identifies a small nerve whilst mobilising the cord structures at the level of the superficial inguinal ring. Which nerve is this most likely to be?

	Subcostal
	Iliohypogastric
	Ilioinguinal
	Obturator
	Pudendal
A

Ilioinguinal nerve entrapment may be a cause of neuropathic pain following inguinal hernia surgery.

The ilioinguinal nerve passes through the superfical inguinal ring and is routinely encountered when exploring the inguinal canal during hernia surgery. The iliohypogastric nerve pierces the aponeurosis of the external oblique muscle superior to the superficial inguinal ring

Ilioinguinal nerve
Arises from the first lumbar ventral ramus with the iliohypogastric nerve. It passes inferolaterally through the substance of psoas major and over the anterior surface of quadratus lumborum. It pierces the internal oblique muscle and passes deep to the aponeurosis of the external oblique muscle. It enters the inguinal canal and then passes through the superficial inguinal ring to reach the skin.

Branches
To supply those muscles of the abdominal wall through which it passes.
Skin and fascia over the pubic symphysis, superomedial part of the femoral triangle, surface of the scrotum, root and dorsum of penis or labum majus in females.

195
Q

A 19 year intravenous drug abuser is recovering following a surgical drainage of a psoas abscess. He is found collapsed in the ward toilet unresponsive and with pinpoint pupils. What is the most appropriate immediate management?

	Intravenous flumazenil
	Intravenous nalaxone
	Intravenous benxhexol
	No further management
	Intravenous glycopyrolate
A

Intravenous nalaxone is needed to treat the patient who has had an overdose of opiate. Naloxone has the quickest onset of action, however it is important to be aware of its short acting duration and the need for further administration. There is also the risk of rebound pain once naloxone is given.

Opioid misuse
Opioids are substances which bind to opioid receptors. This includes both naturally occurring opiates such as morphine and synthetic opioids such as buprenorphine and methadone.

Features of opioid misuse
Rhinorrhoea
Needle track marks
Pinpoint pupils
Drowsiness

Complications of intravenous opioid misuse
Viral infection secondary to sharing needles: HIV, hepatitis B & C
Bacterial infection secondary to injection: infective endocarditis, septic arthritis, septicaemia, necrotising fasciitis, groin abscess
Pseudoaneurysm
Venous thromboembolism
Osteomyelitis
Overdose may lead to respiratory depression and death

Emergency management of opioid overdose
IV or IM naloxone: has a rapid onset and relatively short duration of action

196
Q

Which of the following is least likely to impair bone fracture healing?

	Radiotherapy
	Osteoporosis
	Administration of non steroidal anti inflammatory drugs
	Preservation of periosteum
	Presence of osteomyelitic sequestra
A

Periosteal preservation helps fractures to heal.

Bone fracture

  • Bleeding vessels in the bone and periosteum
  • Clot and haematoma formation
  • The clot organises over a week (improved structure and collagen)
  • The periosteum contains osteoblasts which produce new bone
  • Mesenchymal cells produce cartilage (fibrocartilage and hyaline cartilage) in the soft tissue around the fracture
  • Connective tissue + hyaline cartilage = callus
  • As the new bone approaches the new cartilage, endochondral ossification occurs to bridge the gap
  • Trabecular bone forms
  • Trabecular bone is resorbed by osteoclasts and replaced with compact bone

Factors affecting fracture healing
Age
Malnutrition
Bone disorders: osteoporosis
Systemic disorders: diabetes, Marfan’s syndrome and Ehlers-Danlos syndrome cause abnormal musculoskeletal healing.
Drugs: steroids, non steroidal anti inflammatory agents.
Type of bone: Cancellous (spongy) bone fractures are usually more stable, involve greater surface areas, and have a better blood supply than cortical (compact) bone fractures.
Degree of Trauma: The more extensive the injury to bone and surrounding soft tissue, the poorer the outcome.
Vascular Injury: Especially the femoral head, talus, and scaphoid bones.
Degree of Immobilization
Intra-articular Fractures: These fractures communicate with synovial fluid, which contains collagenases that retard bone healing.
Separation of Bone Ends: Normal apposition of fracture fragments is needed for union to occur. Inadequate reduction, excessive traction, or interposition of soft tissue will prevent healing.
Infection

197
Q

A 68 year old male is admitted to the surgical ward for assessment of severe epigastric pain. His abdomen is soft and non tender. However the Nurse forces you to look at the ECG. It looks abnormal. Which of the following features is an indication for urgent coronary thrombolysis or percutaneous intervention?

	Right bundle branch block
	ST elevation of 1mm in leads V1 to V6
	Ventricular tachycardia
	Q waves in leads V1 to V6
	ST elevation of greater than 1mm in leads II, III and aVF
A

ECG changes for thrombolysis or percutaneous intervention:
ST elevation of > 2mm (2 small squares) in 2 or more consecutive anterior leads (V1-V6) OR

ST elevation of greater than 1mm (1 small square) in greater than 2 consecutive inferior leads (II, III, avF, avL) OR

New Left bundle branch block

ST elevation of 1mm in leads II, III and aVF reflects significant cardiac ischaemia due to the right coronary artery occlusion. The medical registrar should be contacted to urgently assess the patient. Note right coronary artery occlusions puts the patient at risk of cardiac arrhythmias (due to blood supply to the sino atrial node).

Thrombolysis or percutaneous intervention in myocardial infarction
Thrombolytic drugs activate plasminogen to form plasmin. This in turn degrades fibrin and help breaks up thrombi. They in primarily used in patients who present with a ST elevation myocardial infarction. Other indications include acute ischaemic stroke and pulmonary embolism, although strict inclusion criteria apply.

Examples
alteplase
tenecteplase
streptokinase

Contraindications to thrombolysis
active internal bleeding
recent haemorrhage, trauma or surgery (including dental extraction)
coagulation and bleeding disorders
intracranial neoplasm
stroke < 3 months
aortic dissection
recent head injury
pregnancy
severe hypertension

Side-effects
haemorrhage
hypotension - more common with streptokinase
allergic reactions may occur with streptokinase

198
Q

During an arch aortogram the brachiocephalic artery is entered with an angiography catheter. The radiologist continues to advance the catheter. Into which of the following vessels is it likely to enter?

	Left subclavian artery
	Left axillary artery
	Right subclavian artery
	Right axillary artery
	None of the above
A

The axillary artery is a branch of the subclavian artery and although developmental anomalies may occur they are rare. The catheter may also enter the right carotid. There is no brachiocephalic artery on the left side.

Brachiocephalic artery
The brachiocephalic artery is the largest branch of the aortic arch. From its aortic origin it ascends superiorly, it initially lies anterior to the trachea and then on its right hand side. It branches into the common carotid and right subclavian arteries at the level of the sternoclavicular joint.

Path
Origin- apex of the midline of the aortic arch
Passes superiorly and posteriorly to the right
Divides into the right subclavian and right common carotid artery

Relations
Anterior	
Sternohyoid
Sternothyroid
Thymic remnants
Left brachiocephalic vein
Right inferior thyroid veins
Posterior	
Trachea
Right pleura
Right lateral	
Right brachiocephalic vein
Superior part of SVC
Left lateral	
Thymic remnants
Origin of left common carotid
Inferior thyroid veins
Trachea (higher level)

Branches
Normally none but may have the thyroidea ima artery

199
Q

Which of the following structures lie between the lateral and medial heads of the triceps muscle?

	Radial nerve
	Median nerve
	Ulnar nerve
	Axillary nerve
	Medial cutaneous nerve of the forearm
A

The radial nerve runs in its groove on between the two heads. The ulnar nerve lies anterior to the medial head. The axillary nerve passes through the quadrangular space. This lies superior to lateral head of the triceps muscle and thus the lateral border of the quadrangular space is the humerus. Therefore the correct answer is the radial nerve.

Triceps
Origin
Long head- infraglenoid tubercle of the scapula.
Lateral head- dorsal surface of the humerus, lateral and proximal to the groove of the radial nerve
Medial head- posterior surface of the humerus on the inferomedial side of the radial groove and both of the intermuscular septae
Insertion
Olecranon process of the ulna. Here the olecranon bursa is between the triceps tendon and olecranon.
Some fibres insert to the deep fascia of the forearm, posterior capsule of the elbow (preventing the capsule from being trapped between olecranon and olecranon fossa during extension)
Innervation Radial nerve
Blood supply Profunda brachii artery
Action Elbow extension. The long head can adduct the humerus and and extend it from a flexed position
Relations The radial nerve and profunda brachii vessels lie between the lateral and medial heads

200
Q

Into which of the following structures does the superior part of the fibrous capsule of the shoulder joint insert?

	The surgical neck of the humerus
	The body of the humerus
	The bicipital groove
	Immediately distal to the greater tuberosity
	The anatomical neck of the humerus
A

The shoulder joint is a shallow joint, hence its great mobility. However, this comes at the expense of stability. The fibrous capsule attaches to the anatomical neck superiorly and the surgical neck inferiorly

Shoulder joint
Shallow synovial ball and socket type of joint.
It is an inherently unstable joint, but is capable to a wide range of movement.
Stability is provided by muscles of the rotator cuff that pass from the scapula to insert in the greater tuberosity (all except sub scapularis-lesser tuberosity).

Glenoid labrum
Fibrocartilaginous rim attached to the free edge of the glenoid cavity
Tendon of the long head of biceps arises from within the joint from the supraglenoid tubercle, and is fused at this point to the labrum.
The long head of triceps attaches to the infraglenoid tubercle

Fibrous capsule
Attaches to the scapula external to the glenoid labrum and to the labrum itself (postero-superiorly)
Attaches to the humerus at the level of the anatomical neck superiorly and the surgical neck inferiorly
Anteriorly the capsule is in contact with the tendon of subscapularis, superiorly with the supraspinatus tendon, and posteriorly with the tendons of infraspinatus and teres minor. All these blend with the capsule towards their insertion.
Two defects in the fibrous capsule; superiorly for the tendon of biceps. Anteriorly there is a defect beneath the subscapularis tendon.
The inferior extension of the capsule is closely related to the axillary nerve at the surgical neck and this nerve is at risk in anteroinferior dislocations. It also means that proximally sited osteomyelitis may progress to septic arthritis.

Movements and muscles
Flexion	Anterior part of deltoid
Pectoralis major
Biceps
Coracobrachialis
Extension	Posterior deltoid
Teres major
Latissimus dorsi
Adduction	Pectoralis major
Latissimus dorsi
Teres major
Coracobrachialis
Abduction	Mid deltoid
Supraspinatus
Medial rotation	Subscapularis
Anterior deltoid
Teres major
Latissimus dorsi
Lateral rotation	Posterior deltoid
Infraspinatus
Teres minor
Important anatomical relations
Anteriorly	Brachial plexus
Axillary artery and vein
Posterior	Suprascapular nerve
Suprascapular vessels
Inferior	Axillary nerve
Circumflex humeral vessels
201
Q

A 34 year old lady presents with symptoms of faecal incontinence. Ten years previously she gave birth to a child by normal vaginal delivery. Injury to which of the following nerves is most likely to account for this process?

	Genitofemoral
	Ilioinguinal
	Pudendal
	Hypogastric autonomic nerve
	Obturator
A

S2,3,4 keeps the poo up off the floor - POOdendal nerve

Damage to the pudendal nerve is classically associated with faecal incontinence and it is for this reason that sacral neuromodulation is a popular treatment for the condition. Injury to the hypogastric autonomic nerves is an aetiological factor in the development of constipation.

The pudendal nerve arises from nerve roots S2, S3 and S4 and exits the pelvis through the greater sciatic foramen. It re-enters the perineum through the lesser sciatic foramen. It travels inferior to give innervation to the anal sphincters and external urethral sphincter. It also provides cutaneous innervation to the region of perineum surrounding the anus and posterior vulva.

Traction and compression of the pudendal nerve by the foetus in late pregnancy may result in late onset pudendal neuropathy which may be part of the process involved in the development of faecal incontinence.

202
Q

During a difficult thyroidectomy haemorrhage is noted from the thyroidea ima vessel. From which structure does this vessel usually arise?

	External carotid artery
	Internal carotid artery
	Brachiocephalic artery
	Axillary artery
	Superior thyroid artery
A

This accessory vessel which usually lies at the inferior aspect of the gland is derived either from the brachiocephalic artery or the arch of the aorta.

Thyroid gland
Right and left lobes connected by isthmus
Surrounded by sheath from pretracheal layer of deep fascia
Apex: Lamina of thyroid cartilage
Base: 4th-5th tracheal ring
Pyramidal lobe: from isthmus
May be attached to foramen caecum at the base of the tongue

Relations
Anteromedially	
Sternothyroid
Superior belly of omohyoid
Sternohyoid
Anterior aspect of sternocleidomastoid
Posterolaterally	Carotid sheath
Medially	
Larynx
Trachea
Pharynx
Oesophagus
Cricothyroid muscle
External laryngeal nerve (near superior thyroid artery)
Recurrent laryngeal nerve (near inferior thyroid artery)
Posterior	
Parathyroid glands
Anastomosis of superior and inferior thyroid arteries
Isthmus	
Anteriorly: Sternothyroids, sternohyoids, anterior jugular veins
Posteriorly: 2nd, 3rd, 4th tracheal rings (attached via Ligament of Berry)

Blood Supply
Arterial
Superior thyroid artery (1st branch of external carotid)
Inferior thyroid artery (from thyrocervical trunk)
Thyroidea ima (in 10% of population -from brachiocephalic artery or aorta)
Venous
Superior and middle thyroid veins - into the IJV
Inferior thyroid vein - into the brachiocephalic veins

203
Q

A 49 year old man undergoes a low anterior resection for cancer. He is assessed in the outpatient clinic post operatively. His wounds are well healed. However, he complains of impotence. Which of the following best explains this problem?

	Sciatic nerve injury
	Damage to the internal iliac artery
	Damage to the nervi erigentes
	Damage to the vas
	Damage to the genitofemoral nerve
A

The penis takes autonomic nerves from the nervi erigentes that lie near the seminal vesicles. These may be compromised by direct surgical trauma (such as use of diathermy in this area) and also by radiotherapy that is used in these patients pre operatively. The result is that up to 50% of patients may develop impotence following rectal cancer surgery.

Penile erection
Physiology of erection
Autonomic
Sympathetic nerves originate from T11-L2 and parasympathetic nerves from S2-4 join to form pelvic plexus.
Parasympathetic discharge causes erection, sympathetic discharge causes ejaculation and detumescence.
Somatic nerves Supplied by dorsal penile and pudendal nerves. Efferent signals are relayed from Onufs nucleus (S2-4) to innervate ischiocavernosus and bulbocavernosus muscles.

Autonomic discharge to the penis will trigger the veno-occlusive mechanism which triggers the flow of arterial blood into the penile sinusoidal spaces. As the inflow increases the increased volume in this space will secondarily lead to compression of the subtunical venous plexus with reduced venous return. During the detumesence phase the arteriolar constriction will reduce arterial inflow and thereby allow venous return to normalise.

Priapism
Prolonged unwanted erection, in the absence of sexual desire, lasting more than 4 hours.

Classification of priapism
Low flow priapism Due to veno-occlusion (high intracavernosal pressures).
Most common type
Often painful
Often low cavernosal flow
If present for >4 hours requires emergency treatment
High flow priapism Due to unregulated arterial blood flow.
Usually presents as semi rigid painless erection
Recurrent priapism Typically seen in sickle cell disease, most commonly of high flow type.

Causes
Intracavernosal drug therapies (e.g. for erectile dysfunction>
Blood disorders such as leukaemia and sickle cell disease
Neurogenic disorders such as spinal cord transection
Trauma to penis resulting in arterio-venous malformations

Tests
Exclude sickle cell/ leukaemia
Consider blood sampling from cavernosa to determine whether high or low flow (low flow is often hypoxic)

Management
Ice packs/ cold showers
If due to low flow then blood may be aspirated from copora or try intracavernosal alpha adrenergic agonists.
Delayed therapy of low flow priapism may result in erectile dysfunction.

204
Q

An obese 14 year old boy presents with difficulty running and mild knee and hip pain. There is no antecedent history of trauma. On examination internal rotation is restricted but the knee is normal with full range of passive movement possible and no evidence of effusions. Both the C-reactive protein and white cell count are normal.

A.	Perthes disease
B.	Developmental dysplasia of the hip
C.	Osteoarthritis
D.	Slipped upper femoral epiphysis
E.	Septic arthritis
F.	Rheumatoid arthritis
G.	Intra capsular fracture of the femoral neck
H.	Extra capsular fracture of the femoral neck
A

Slipped upper femoral epiphysis is the commonest adolescent hip disorder. It occurs most commonly in obese males. It may often present as knee pain which is usually referred from the ipsilateral hip. The knee itself is normal. The hip often limits internal rotation. The diagnosis is easily missed. X-rays will show displacement of the femoral epiphysis and the degree of its displacement may be calculated using the Southwick angle. Treatment is directed at preventing further slippage which may result in avascular necrosis of the femoral head.

Typically seen in obese male adolescents. Pain is often referred to the knee. Limitation to internal rotation is usually seen. Knee pain is usually present 2 months prior to hip slipping. Bilateral in 20%. Bed rest and non-weight bearing. Aim to avoid avascular necrosis. If severe slippage or risk of it occurring then percutaneous pinning of the hip may be required. X-rays will show the femoral head displaced and falling inferolaterally (like a melting ice cream cone) The Southwick angle gives indication of disease severity

205
Q

A 6 year old boy presents with pain in the hip it is present on activity and has been worsening over the past few weeks. There is no history of trauma. He was born by normal vaginal delivery at 38 weeks gestation On examination he has an antalgic gait and limitation of active and passive movement of the hip joint in all directions. C-reactive protein is mildly elevated at 10 but the white cell count is normal.

A.	Perthes disease
B.	Developmental dysplasia of the hip
C.	Osteoarthritis
D.	Slipped upper femoral epiphysis
E.	Septic arthritis
F.	Rheumatoid arthritis
G.	Intra capsular fracture of the femoral neck
H.	Extra capsular fracture of the femoral neck
A

Perthes disease

This is a typical presentation for Perthes disease. X-ray may show flattening of the femoral head or fragmentation in more advanced cases.

Hip pain (may be referred to the knee) usually occurring between 5 and 12 years of age. Bilateral disease in 20%.	Remove pressure from joint to allow normal development. Physiotherapy. Usually self-limiting if diagnosed and treated promptly.	
X-rays will show flattened femoral head. Eventually in untreated cases the femoral head will fragment.
206
Q

A 30 year old man presents with severe pain in the left hip it has been present on and off for many years. He was born at 39 weeks gestation by emergency caesarean section after a long obstructed breech delivery. He was slow to walk and as a child was noted to have an antalgic gait. He was a frequent attender at the primary care centre and the pains dismissed as growing pains. X-rays show almost complete destruction of the femoral head and a narrow acetabulum.

A.	Perthes disease
B.	Developmental dysplasia of the hip
C.	Osteoarthritis
D.	Slipped upper femoral epiphysis
E.	Septic arthritis
F.	Rheumatoid arthritis
G.	Intra capsular fracture of the femoral neck
H.	Extra capsular fracture of the femoral neck
A

Developmental dysplasia of the hip

Developmental dysplasia of the hip. Usually diagnosed by Barlow and Ortolani tests in early childhood. Most Breech deliveries are also routinely subjected to USS of the hip joint. At this young age an arthrodesis may be preferable to hip replacement.

Usually diagnosed in infancy by screening tests. May be bilateral, when disease is unilateral there may be leg length inequality. As disease progresses child may limp and then early onset arthritis. More common in extended breech babies.	 Splints and harnesses or traction. In later years osteotomy and hip realignment procedures may be needed. In arthritis a joint replacement may be needed. However, this is best deferred if possible as it will almost certainly require revision	 Initially no obvious change on plain films and USS gives best resolution until 3 months of age. On plain films Shentons line should form a smooth arc
207
Q

A 43 year old lady develops a cerebello-pontine angle lesion. Which of the nerves listed below is likely to be affected first?

	CN X
	CN III
	CN V
	CN IX
	CN XII
A

CNV
The most likely lesion to occur in the cerebello-pontine angle is an acoustic neuroma.
The trigeminal nerve has a broad base and involvement of at least part of this nerve is the most likely initial finding. The defect may be subtle such as loss of the ipsilateral corneal reflex. Ipsilateral hearing loss will also occur. Untreated, progressive lesions, may ultimately affect cranial nerve roots in this region

Cranial nerve lesions

Olfactory nerve May be injured in basal skull fractures or involved in frontal lobe tumour extension. Loss of olfactory nerve function in relation to major CNS pathology is seldom an isolated event and thus it is poor localiser of CNS pathology.

Optic nerve Problems with visual acuity may result from intra ocular disorders. Problems with the blood supply such as amaurosis fugax may produce temporary visual distortion. More important surgically is the pupillary response to light. The pupillary size may be altered in a number of disorders. Nerves involved in the resizing of the pupil connect to the pretectal nucleus of the high midbrain, bypassing the lateral geniculate nucleus and the primary visual cortex. From the pretectal nucleus neurones pass to the Edinger - Westphal nucleus, motor axons from here pass along with the oculomotor nerve. They synapse with ciliary ganglion neurones; the parasympathetic axons from this then innervate the iris and produce miosis. The miotic pupil is seen in disorders such as Horner’s syndrome or opiate overdose.
Mydriasis is the dilatation of the pupil in response to disease, trauma, drugs (or the dark!). It is pathological when light fails to induce miosis. The radial muscle is innervated by the sympathetic nervous system. Because the parasympathetic fibres travel with the oculomotor nerve they will be damaged by lesions affecting this nerve (e.g. cranial trauma).
The response to light shone in one eye is usually a constriction of both pupils. This indicates intact direct and consensual light reflexes. When the optic nerve has an afferent defect the light shining on the affected eye will produce a diminished pupillary response in both eyes. Whereas light shone on the unaffected eye will produce a normal pupillary response in both eyes. This is referred to as the Marcus Gunn pupil and is seen in conditions such as optic neuritis. In a total CN II lesion shining the light in the affected eye will produce no response.

Oculomotor nerve The pupillary effects are described above. In addition it supplies all ocular muscles apart from lateral rectus and superior oblique. Thus the affected eye will be deviated inferolaterally. Levator palpebrae superioris may also be impaired resulting in impaired ability to open the eye.

Trochlear nerve The eye will not be able to look down.

Trigeminal nerve Largest cranial nerve. Exits the brainstem at the pons. Branches are ophthalmic, maxillary and mandibular. Only the mandibular branch has both sensory and motor fibres. Branches converge to form the trigeminal ganglion (located in Meckels cave). It supplies the muscles of mastication and also tensor veli palatine, mylohyoid, anterior belly of digastric and tensor tympani. The detailed descriptions of the various sensory functions are described in other areas of the website. The corneal reflex is important and is elicited by applying a small tip of cotton wool to the cornea, a reflex blink should occur if it is intact. It is mediated by: the naso ciliary branch of the ophthalmic branch of the trigeminal (sensory component) and the facial nerve producing the motor response. Lesions of the afferent arc will produce bilateral absent blink and lesions of the efferent arc will result in a unilateral absent blink.

Abducens nerve The affected eye will have a deficit of abduction. This cranial nerve exits the brainstem between the pons and medulla. It thus has a relatively long intra cranial course which renders it susceptible to damage in raised intra cranial pressure.

Facial nerve Emerges from brainstem between pons and medulla. It controls muscles of facial expression and taste from the anterior 2/3 of the tongue. The nerve passes into the petrous temporal bone and into the internal auditory meatus. It then passes through the facial canal and exits at the stylomastoid foramen. It passes through the parotid gland and divides at this point. It does not innervate the parotid gland. Its divisions are considered in other parts of the website. Its motor fibres innervate orbicularis oculi to produce the efferent arm of the corneal reflex. In surgical practice it may be injured during parotid gland surgery or invaded by malignancies of the gland and a lower motor neurone on the ipsilateral side will result.

Vestibulo-cochlear nerve Exits from the pons and then passes through the internal auditory meatus. It is implicated in sensorineural hearing loss. Individuals with sensorineural hearing loss will localise the sound in webers test to the normal ear. Rinnes test will be reduced on the affected side but should still work. These two tests will distinguish sensorineural hearing loss from conductive deafness. In the latter condition webers test will localise to the affected ear and Rinnes test will be impaired on the affected side. Surgical lesions affecting this nerve include CNS tumours and basal skull fractures. It may also be damaged by the administration of ototoxic drugs (of which gentamicin is the most commonly used in surgical practice).

Glossopharyngeal nerve Exits the pons just above the vagus. Receives sensory fibres from posterior 1/3 tongue, tonsils, pharynx and middle ear (otalgia may occur following tonsillectomy). It receives visceral afferents from the carotid bodies. It supplies parasympathetic fibres to the parotid gland via the otic ganglion and motor function to stylopharyngeaus muscle. The sensory function of the nerve is tested using the gag reflex.

Vagus nerve Leaves the medulla between the olivary nucleus and the inferior cerebellar peduncle. Passes through the jugular foramen and into the carotid sheath. Details of the functions of the vagus nerve are covered in the website under relevant organ sub headings.

Accessory nerve Exists from the caudal aspect of the brainstem (multiple branches) supplies trapezius and sternocleidomastoid muscles. The distal portion of this nerve is most prone to injury during surgical procedures.

Hypoglossal nerve: Emerges from the medulla at the preolivary sulcus, passes through the hypoglossal canal. It lies on the carotid sheath and passes deep to the posterior belly of digastric to supply muscles of the tongue (except palatoglossus). Its location near the carotid sheath makes it vulnerable during carotid endarterectomy surgery and damage will produce ipsilateral defect in muscle function.

208
Q

Which of the following is not a branch of the abdominal aorta?

	Inferior mesenteric artery
	Inferior phrenic artery
	Superior mesenteric artery
	Superior phrenic artery
	Renal artery
A

Mnemonic for the Descending abdominal aorta branches from diaphragm to iliacs:

‘Prostitutes Cause Sagging Swollen Red Testicles [in men] Living In Sin’:

Phrenic [inferior]
Celiac
Superior mesenteric
Suprarenal [middle]
Renal
Testicular ['in men' only]
Lumbars
Inferior mesenteric
Sacral

The superior phrenic artery branches from the aorta in the thorax.

209
Q

A toddler aged 3 years presents to the Emergency Department with swelling of his leg and is found to have a spiral fracture of the tibia. His mother reports that he had tripped and fallen the previous day but she had not noticed any sign of injury at the time. She is a single parent with little family support. The child is not on the child protection register.

A.	Non accidental injury
B.	Accidental fracture
C.	Rickets
D.	Metabolic bone disease of prematurity
E.	Hypophosphataemic rickets
F.	Osteopetrosis
G.	Osteogenesis imperfecta
H.	Hypoparathyroidism
I.	Osteoporosis
A

Non accidental injury

Delayed presentation is unusual and should raise concern. In addition spiral fractures are usually the result of rotational injury which is not compatible with the mechanism proposed by the parent.

Paediatric #s:
Complete fracture Both sides of cortex are breached
Toddlers fracture Oblique tibial fracture in infants
Plastic deformity Stress on bone resulting in deformity without cortical disruption
Greenstick fracture Unilateral cortical breach only
Buckle fracture Incomplete cortical disruption resulting in periosteal haematoma only

Growth plate fractures
In paediatric practice fractures may also involve the growth plate and these injuries are classified according to the Salter- Harris system (given below):

Type Injury pattern
I Fracture through the physis only (x-ray often normal)
II Fracture through the physis and metaphysis
III Fracture through the physis and epiphyisis to include the joint
IV Fracture involving the physis, metaphysis and epiphysis
V Crush injury involving the physis (x-ray may resemble type I, and appear normal)

As a general rule it is safer to assume that growth plate tenderness is indicative of an underlying fracture even if the x-ray appears normal. Injuries of Types III, IV and V will usually require surgery. Type V injuries are often associated with disruption to growth.

Non accidental injury
Delayed presentation
Delay in attaining milestones
Lack of concordance between proposed and actual mechanism of injury
Multiple injuries
Injuries at sites not commonly exposed to trauma
Children on the at risk register

Pathological fractures
Genetic conditions, such as osteogenesis imperfecta, may cause pathological fractures.

Osteogenesis imperfecta
Defective osteoid formation due to congenital inability to produce adequate intercellular substances like osteoid, collagen and dentine.
Failure of maturation of collagen in all the connective tissues.
Radiology may show translucent bones, multiple fractures, particularly of the long bones, wormian bones (irregular patches of ossification) and a trefoil pelvis.

Subtypes
Type I The collagen is normal quality but insufficient quantity.
Type II- Poor collagen quantity and quality.
Type III- Collagen poorly formed. Normal quantity.
Type IV- Sufficient collagen quantity but poor quality.

Osteopetrosis
Bones become harder and more dense.
Autosomal recessive condition.
It is commonest in young adults.
Radiology reveals a lack of differentiation between the cortex and the medulla described as marble bone.
210
Q

A 5 month baby boy presents with swelling of his right arm and is found to have a spiral fracture of the humerus. He had been in the care of her mother’s boyfriend who reported that he had nearly dropped him that day when reaching for his bottle and had inadvertently pulled on his arm to save him. He was immediately taken to the Emergency Department.

A.	Non accidental injury
B.	Accidental fracture
C.	Rickets
D.	Metabolic bone disease of prematurity
E.	Hypophosphataemic rickets
F.	Osteopetrosis
G.	Osteogenesis imperfecta
H.	Hypoparathyroidism
I.	Osteoporosis
A

Accidental fracture

The mechanism fits with the fracture pattern and the presentation is not delayed.

Growth plate fractures
In paediatric practice fractures may also involve the growth plate and these injuries are classified according to the Salter- Harris system (given below):

Type Injury pattern
I Fracture through the physis only (x-ray often normal)
II Fracture through the physis and metaphysis
III Fracture through the physis and epiphyisis to include the joint
IV Fracture involving the physis, metaphysis and epiphysis
V Crush injury involving the physis (x-ray may resemble type I, and appear normal)

As a general rule it is safer to assume that growth plate tenderness is indicative of an underlying fracture even if the x-ray appears normal. Injuries of Types III, IV and V will usually require surgery. Type V injuries are often associated with disruption to growth.

Non accidental injury
Delayed presentation
Delay in attaining milestones
Lack of concordance between proposed and actual mechanism of injury
Multiple injuries
Injuries at sites not commonly exposed to trauma
Children on the at risk register

Pathological fractures
Genetic conditions, such as osteogenesis imperfecta, may cause pathological fractures.

Osteogenesis imperfecta
Defective osteoid formation due to congenital inability to produce adequate intercellular substances like osteoid, collagen and dentine.
Failure of maturation of collagen in all the connective tissues.
Radiology may show translucent bones, multiple fractures, particularly of the long bones, wormian bones (irregular patches of ossification) and a trefoil pelvis.

Subtypes
Type I The collagen is normal quality but insufficient quantity.
Type II- Poor collagen quantity and quality.
Type III- Collagen poorly formed. Normal quantity.
Type IV- Sufficient collagen quantity but poor quality.

Osteopetrosis
Bones become harder and more dense.
Autosomal recessive condition.
It is commonest in young adults.
Radiology reveals a lack of differentiation between the cortex and the medulla described as marble bone.
211
Q

An infant is admitted with symptoms and signs of respiratory infection and is found to have several posterior rib fractures on chest radiograph. He was born prematurely at 37 weeks’ gestation and was observed overnight on the special care baby unit for tachypnoea which settled by the following day. On assessment it is also apparent that his head circumference has increased at an excessive rate and has crossed 3 centiles since birth

A.	Non accidental injury
B.	Accidental fracture
C.	Rickets
D.	Metabolic bone disease of prematurity
E.	Hypophosphataemic rickets
F.	Osteopetrosis
G.	Osteogenesis imperfecta
H.	Hypoparathyroidism
I.	Osteoporosis
A

Non accidental injury

Posterior rib fractures are extremely unusual in neonates. The change in head size may be accounted for by hydrocephalus which may occur as a sequelae from head in

Growth plate fractures
In paediatric practice fractures may also involve the growth plate and these injuries are classified according to the Salter- Harris system (given below):

Type Injury pattern
I Fracture through the physis only (x-ray often normal)
II Fracture through the physis and metaphysis
III Fracture through the physis and epiphyisis to include the joint
IV Fracture involving the physis, metaphysis and epiphysis
V Crush injury involving the physis (x-ray may resemble type I, and appear normal)

As a general rule it is safer to assume that growth plate tenderness is indicative of an underlying fracture even if the x-ray appears normal. Injuries of Types III, IV and V will usually require surgery. Type V injuries are often associated with disruption to growth.

Non accidental injury
Delayed presentation
Delay in attaining milestones
Lack of concordance between proposed and actual mechanism of injury
Multiple injuries
Injuries at sites not commonly exposed to trauma
Children on the at risk register

Pathological fractures
Genetic conditions, such as osteogenesis imperfecta, may cause pathological fractures.

Osteogenesis imperfecta
Defective osteoid formation due to congenital inability to produce adequate intercellular substances like osteoid, collagen and dentine.
Failure of maturation of collagen in all the connective tissues.
Radiology may show translucent bones, multiple fractures, particularly of the long bones, wormian bones (irregular patches of ossification) and a trefoil pelvis.

Subtypes
Type I The collagen is normal quality but insufficient quantity.
Type II- Poor collagen quantity and quality.
Type III- Collagen poorly formed. Normal quantity.
Type IV- Sufficient collagen quantity but poor quality.

Osteopetrosis
Bones become harder and more dense.
Autosomal recessive condition.
It is commonest in young adults.
Radiology reveals a lack of differentiation between the cortex and the medulla described as marble bone.
212
Q

A 40 year old lady presents with varicose veins, these are found to originate from the short saphenous vein. As the vein is mobilised which structure is at greatest risk of injury?

	Sciatic nerve
	Sural nerve
	Common peroneal nerve
	Tibial nerve
	Popliteal artery
A

The sural nerve is closely related and damage to this structure is a major cause of litigation. The other structures may all be injured but the risks are lower.

Boundaries of the popliteal fossa
Laterally Biceps femoris above, lateral head of gastrocnemius and plantaris below
Medially Semimembranosus and semitendinosus above, medial head of gastrocnemius below
Floor Popliteal surface of the femur, posterior ligament of knee joint and popliteus muscle
Roof Superficial and deep fascia

Contents
Popliteal artery and vein
Small saphenous vein
Common peroneal nerve
Tibial nerve
Posterior cutaneous nerve of the thigh
Genicular branch of the obturator nerve
Lymph nodes
213
Q

A 23 year old man is admitted with a suspected ureteric colic. A KUB style x-ray is obtained. In which of the following locations is the stone most likely to be visualised?

The tips of the transverse processes between L2 and L5
The tips of transverse processes between T10-L1
At the crest of the ilium
Over the S3 foramina
Over the sacrococcygeal joint
A

The ureter lies anterior to L2 to L5 and stones may be visualised at these points, they may also be identified over the sacro-iliac joints.

Ureter
25-35 cm long
Muscular tube lined by transitional epithelium
Surrounded by thick muscular coat. Becomes 3 muscular layers as it crosses the bony pelvis
Retroperitoneal structure overlying transverse processes L2-L5
Lies anterior to bifurcation of iliac vessels
Blood supply is segmental; renal artery, aortic branches, gonadal branches, common iliac and internal iliac
Lies beneath the uterine artery

214
Q

A 72 year old man with non reconstructible arterial disease is undergoing an above knee amputation. The posterior compartment muscles are divided. Which of the following muscles does not lie in the posterior compartment of the thigh?

	Biceps femoris
	Quadriceps femoris
	Semitendinosus
	Semimembranosus
	None of the above
A

The quadriceps femoris lies in the anterior compartment.

Compartments of the thigh
Formed by septae passing from the femur to the fascia lata.

Anterior compartment Femora N,l Iliacus, Tensor fasciae latae, Sartorius, Quadriceps femoris, Femoral artery

Medial compartment Obturator N
Adductor longus/magnus/brevis, Gracilis, Obturator externus, Profunda femoris artery and obturator artery

Posterior compartment (2 layers)	Sciatic	
Semimembranosus
Semitendinosus
Biceps femoris
Branches of Profunda femoris artery

Compartments of the lower leg
Separated by the interosseous membrane (anterior and posterior compartments), anterior fascial septum (separate anterior and lateral compartments) and posterior fascial septum (separate lateral and posterior compartments)

Anterior compartment	Deep peroneal nerve	
Tibialis anterior
Extensor digitorum longus
Extensor hallucis longus
Peroneus tertius
Anterior tibial artery

Posterior compartment Tibial N
Muscles: deep and superficial compartments (separated by deep transverse fascia)
Deep: Flexor hallucis longus, Flexor digitalis longus, Tibialis posterior, Popliteus
Superficial: Gastrocnemius, Soleus, Plantaris
Posterior tibial

Lateral compartment Superficial peroneal N
Peroneus longus/brevis
Peroneal artery

215
Q

A woman develops winging of the scapula following a Patey mastectomy. What is the most likely cause?

Division of pectoralis minor to access level 3 axillary nodes
Damage to the brachial plexus during axillary dissection
Damage to the long thoracic nerve during axillary dissection
Division of the thoracodorsal trunk during axillary dissection
Damage to the thoracodorsal trunk during axillary dissection
A

The serratus anterior muscle is supplied by the long thoracic nerve which runs along the surface of serratus anterior and is liable to injury during nodal dissection. Although pectoralis minor is divided during a Patey mastectomy (now seldom performed) it is rare for this alone to produce winging of the scapula

Long thoracic nerve
Derived from ventral rami of C5, C6, and C7 (close to their emergence from intervertebral foramina)
It runs downward and passes either anterior or posterior to the middle scalene muscle
It reaches upper tip of serratus anterior muscle and descends on outer surface of this muscle, giving branches into it
Winging of Scapula occurs in long thoracic nerve injury (most common) or from spinal accessory nerve injury (which denervates the trapezius) or a dorsal scapular nerve injury

216
Q

In a patient with an ectopic kidney where is the adrenal gland most likely to be located?

	In the pelvis
	On the contralateral side
	In its usual position
	Superior to the spleen
	It will be absent
A

Because the kidney is present, rather than absent, the adrenal will usual develop and in the normal location.

Adrenal gland embryology
First detected at 6 weeks’ gestation, the adrenal cortex is derived from the mesoderm of the posterior abdominal wall. Steroid secretion from the fetal cortex begins shortly thereafter. Adult-type zona glomerulosa and fasciculata are detected in fetal life but make up only a small proportion of the gland, and the zona reticularis is not present at all. The fetal cortex predominates throughout fetal life. The adrenal medulla is of ectodermal origin, arising from neural crest cells that migrate to the medial aspect of the developing cortex.

The fetal adrenal gland is relatively large. At 4 months’ gestation, it is 4 times the size of the kidney; however, at birth, it is a third of the size of the kidney. This occurs because of the rapid regression of the fetal cortex at birth. It disappears almost completely by age 1 year; by age 4-5 years, the permanent adult-type adrenal cortex has fully developed.

Anatomic anomalies of the adrenal gland may occur. Because the development of the adrenals is closely associated with that of the kidneys, agenesis of an adrenal gland is usually associated with ipsilateral agenesis of the kidney, and fused adrenal glands (whereby the 2 glands join across the midline posterior to the aorta) are also associated with a fused kidney.

Adrenal hypoplasia occurs in the following 2 forms: (1) hypoplasia or absence of the fetal cortex with a poorly formed medulla and (2) disorganized fetal cortex and medulla with no permanent cortex present. Adrenal heterotopia describes a normal adrenal gland in an abnormal location, such as within the renal or hepatic capsules. Accessory adrenal tissue (adrenal rests), which is usually comprised only of cortex but seen combined with medulla in some cases, is most commonly located in the broad ligament or spermatic cord but can be found anywhere within the abdomen. Even intracranial adrenal rests have been reported

217
Q

Which of the following structures is not closely related to the posterior tibial artery?

	Soleus posteriorly
	Tibial nerve medially
	Deep peroneal nerve laterally
	Flexor hallucis longus postero-inferiorly
	Popliteus
A

The deep peroneal nerve lies in the anterior compartment. The tibial nerve lies medially. At its termination it lies deep to the flexor retinaculum.

Posterior tibial artery
Larger terminal branch of the popliteal artery
Terminates by dividing into the medial and lateral plantar arteries
Accompanied by two veins throughout its length
Position of the artery corresponds to a line drawn from the lower angle of the popliteal fossa, at the level of the neck of the fibula, to a point midway between the medial malleolus and the most prominent part of the heel

Relations of the posterior tibial artery
Proximal to distal
Anteriorly Tibialis posterior
Flexor digitorum longus
Posterior surface of tibia and ankle joint
Posterior Tibial nerve 2.5 cm distal to its origin
Fascia overlying the deep muscular layer
Proximal part covered by gastrocnemius and soleus
Distal part covered by skin and fascia

218
Q

Which of the following is not closely related to the capitate bone?

	Lunate bone
	Scaphoid bone
	Ulnar nerve
	Hamate bone
	Trapezoid bone
A

The ulnar nerve and artery lie adjacent to the pisiform bone. The capitate bone articulates with the lunate, scaphoid, hamate and trapezoid bones, which are therefore closely related to it.

Capitate bone

This is the largest of the carpal bones. It is centrally placed with a rounded head set into the cavities of the lunate and scaphoid bones. Flatter articular surfaces are present for the hamate medially and the trapezoid laterally. Distally the bone articulates predominantly with the middle metacarpal.

219
Q

Which of the following statements relating to the tympanic membrane is false?

The umbo marks the point of attachment of the handle of the malleus to the tympanic membrane
The lateral aspect of the tympanic membrane is lined by stratified squamous epithelium
The chorda tympani nerve runs medial to the pars tensa
The medial aspect of the tympanic membrane is lined by mucous membrane
The tympanic membrane is approximately 1cm in diameter
A

The chorda tympani runs medially to the pars flaccida. The relationship is shown from the medial aspect in the dissection below.

The ear is composed of three anatomically distinct regions.

External ear
Auricle is composed of elastic cartilage covered by skin. The lobule has no cartilage and contains fat and fibrous tissue.

External auditory meatus is approximately 2.5cm long.
Lateral third of the external auditory meatus is cartilaginous and the medial two thirds is bony.

The region is innervated by the greater auricular nerve. The auriculotemporal branch of the trigeminal nerve supplies most the of external auditory meatus and the lateral surface of the auricle.

Middle ear
Space between the tympanic membrane and cochlea. The aditus leads to the mastoid air cells is the route through which middle ear infections may cause mastoiditis. Anteriorly the eustacian tube connects the middle ear to the naso pharynx.
The tympanic membrane consists of:
Outer layer of stratified squamous epithelium.
Middle layer of fibrous tissue.
Inner layer of mucous membrane continuous with the middle ear.
The tympanic membrane is approximately 1cm in diameter.
The chorda tympani nerve passes on the medial side of the pars flaccida.

The middle ear is innervated by the glossopharyngeal nerve and pain may radiate to the middle ear following tonsillectomy.

Ossicles
Malleus attaches to the tympanic membrane (the Umbo).
Malleus articulates with the incus (synovial joint).
Incus attaches to stapes (another synovial joint).

Internal ear
Cochlea, semi circular canals and vestibule

Organ of corti is the sense organ of hearing and is located on the inside of the cochlear duct on the basilar membrane.

Vestibule accommodates the utricule and the saccule. These structures contain endolymph and are surrounded by perilymph within the vestibule.

The semicircular canals lie at various angles to the petrous temporal bone. All share a common opening into the vestibule.

220
Q

An injury to the spinal accessory nerve will have the greatest affect on which of the following movements?

	Lateral rotation of the arm
	Adduction of the arm at the glenohumeral joint
	Protraction of the scapula
	Upward rotation of the scapula
	Depression of the scapula
A

The spinal accessory nerve innervates trapezius. The entire muscle will retract the scapula. However, its upper and lower fibres act together to upwardly rotate it.

Shoulder joint

Shallow synovial ball and socket type of joint.
It is an inherently unstable joint, but is capable to a wide range of movement.
Stability is provided by muscles of the rotator cuff that pass from the scapula to insert in the greater tuberosity (all except sub scapularis-lesser tuberosity).

Glenoid labrum
Fibrocartilaginous rim attached to the free edge of the glenoid cavity
Tendon of the long head of biceps arises from within the joint from the supraglenoid tubercle, and is fused at this point to the labrum.
The long head of triceps attaches to the infraglenoid tubercle

Fibrous capsule
Attaches to the scapula external to the glenoid labrum and to the labrum itself (postero-superiorly)
Attaches to the humerus at the level of the anatomical neck superiorly and the surgical neck inferiorly
Anteriorly the capsule is in contact with the tendon of subscapularis, superiorly with the supraspinatus tendon, and posteriorly with the tendons of infraspinatus and teres minor. All these blend with the capsule towards their insertion.
Two defects in the fibrous capsule; superiorly for the tendon of biceps. Anteriorly there is a defect beneath the subscapularis tendon.
The inferior extension of the capsule is closely related to the axillary nerve at the surgical neck and this nerve is at risk in anteroinferior dislocations. It also means that proximally sited osteomyelitis may progress to septic arthritis.

Movements and muscles
- Flexion	Anterior part of deltoid
Pectoralis major
Biceps
Coracobrachialis
  • Extension Posterior deltoid
    Teres major
    Latissimus dorsi
  • Adduction Pectoralis major
    Latissimus dorsi
    Teres major
    Coracobrachialis
  • Abduction Mid deltoid
    Supraspinatus
  • Medial rotation Subscapularis
    Anterior deltoid
    Teres major
    Latissimus dorsi
  • Lateral rotation Posterior deltoid
    Infraspinatus
    Teres minor
Important anatomical relations
- Anteriorly	Brachial plexus
Axillary artery and vein
- Posterior	Suprascapular nerve
Suprascapular vessels
- Inferior	Axillary nerve
Circumflex humeral vessels
221
Q

Which of the following is not contained within the middle mediastinum?

	Main bronchi
	Arch of the azygos vein
	Thoracic duct
	Pericardium
	Aortic root
A

The thoracic duct lies within the posterior and superior mediastinum.

Mediastinum

Region between the pulmonary cavities.
It is covered by the mediastinal pleura. It does not contain the lungs.
It extends from the thoracic inlet superiorly to the diaphragm inferiorly.

Mediastinal regions
Superior mediastinum (between manubriosternal angle and T4/5)
Middle mediastinum
Posterior mediastinum
Anterior mediastinum
- Superior mediastinum	
Superior vena cava
Brachiocephalic veins
Arch of aorta
Thoracic duct
Trachea
Oesophagus
Thymus
Vagus nerve
Left recurrent laryngeal nerve
Phrenic nerve
  • Anterior mediastinum
    Thymic remnants
    Lymph nodes
    Fat
- Middle mediastinum	
Pericardium
Heart
Aortic root
Arch of azygos vein
Main bronchi
- Posterior mediastinum	
Oesophagus
Thoracic aorta
Azygos vein
Thoracic duct
Vagus nerve
Sympathetic nerve trunks
Splanchnic nerves
222
Q

A 55 year old man is due to undergo a radical prostatectomy for carcinoma of the prostate gland. Which of the following vessels directly supplies the prostate?

	External iliac artery
	Common iliac artery
	Internal iliac artery
	Inferior vesical artery
	None of the above
A

The arterial supply to the prostate gland is from the inferior vesical artery, it is a branch of the prostatovesical artery. The prostatovesical artery usually arises from the internal pudendal and inferior gluteal arterial branches of the internal iliac artery.

The prostate gland is approximately the shape and size of a walnut and is located inferior to the bladder. It is separated from the rectum by Denonvilliers fascia and its blood supply is derived from the internal iliac vessels (via inferior vesical artery). The internal sphincter lies at the apex of the gland and may be damaged during prostatic surgery, affected individuals may complain of retrograde ejaculation.

Summary of prostate gland
Arterial supply	Inferior vesical artery (from internal iliac)
Venous drainage	Prostatic venous plexus (to paravertebral veins)
Lymphatic drainage	Internal iliac nodes
Innervation	Inferior hypogastric plexus
Dimensions	
Transverse diameter (4cm)
AP diameter (2cm)
Height (3cm)
Lobes	
Posterior lobe: posterior to urethra
Median lobe: posterior to urethra, in between ejaculatory ducts
Lateral lobes x 2
Isthmus
Zones	
Peripheral zone: subcapsular portion of posterior prostate. Most prostate cancers are here
Central zone
Transition zone
Stroma
Relations
- Anterior	Pubic symphysis
Prostatic venous plexus
- Posterior	Denonvilliers fascia
Rectum
Ejaculatory ducts
- Lateral	Venous plexus (lies on prostate)
Levator ani (immediately below the puboprostatic ligaments)
223
Q

Which nerve directly innervates the sinoatrial node?

	Superior cardiac nerve
	Right vagus nerve
	Left vagus nerve
	Inferior cardiac nerve
	None of the above
A

None of the above
No single one of the above nerves is responsible for direct cardiac innervation (which those who have handled the heart surgically will appreciate).
The heart receives its nerves from the superficial and deep cardiac plexuses. The cardiac plexuses send small branches to the heart along the major vessels, continuing with the right and left coronary arteries. The vagal efferent fibres emerge from the brainstem in the roots of the vagus and accessory nerves, and run to ganglia in the cardiac plexuses and within the heart itself.

The background vagal discharge serves to limit heart rate, and loss of this background vagal tone accounts for the higher resting heart rate seen following cardiac transplant.

Sinoatrial node

Located in the wall of the right atrium in the upper part of the sulcus terminalis from which it extends anteriorly over the opening of the superior vena cava.
In most cases it is supplied by the right coronary artery.
It has a complicated nerve supply from the cardiac nerve plexus that takes both sympathetic and parasympathetic fibres that run alongside the main vessels.

224
Q

In paediatric orthopaedic surgery, which of the following does not fulfill the Kocher criteria for septic arthritis?

	ESR > 40mm/h
	Positive blood culture
	Fever
	White cell count > 12, 000
	Non weight bearing on the affected side
A

Kocher criteria

  1. Non weight bearing on affected side
  2. ESR > 40 mm/hr
  3. Fever
  4. WBC count of >12,000 mm3
    - When 4/4 criteria are met, there is a 99% chance that the child has septic arthritis

The Kocher criteria do not consider blood culture results.

Septic arthritis
Staph aureus commonest organism
Urgent washout and antibiotics otherwise high risk of joint destruction

Diagnosis
Plain x-rays
Consider aspiration
Utilise the Kocher criteria (see below)

Kocher criteria:

  1. Non weight bearing on affected side
  2. ESR > 40 mm/hr
  3. Fever
  4. WBC count of >12,000 mm3
    - when 4/4 criteria are met, there is a 99% chance that the child has septic arthritis

Treatment
Surgical drainage of the affected joint is required, this should be done as soon as possible since permanent damage to the joint may occur. In some cases repeated procedures are necessary. Appropriate intravenous antibiotics should be administered.

225
Q

A 30 year old man presents with back pain and the surgeon tests the ankle reflex. Which of the following nerve roots are tested in this manoeuvre?

	S3 and S4
	L4 and L5
	L3 and L4
	S1 and S2
	S4 only
A

The ankle reflex is elicited by tapping the Achilles tendon with a tendon hammer. It tests the S1 and S2 nerve roots. It is typically delayed in L5 and S1 disk prolapses.

226
Q

Which of the following structures is not closely related to the piriformis muscle?

	Superior gluteal nerve
	Sciatic nerve
	Inferior gluteal artery
	Inferior gluteal nerve
	Medial femoral circumflex artery
A

The piriformis muscle is an important anatomical landmark in the gluteal region. The following structures are closely related:
Sciatic nerve
Inferior gluteal artery and nerve
Superior gluteal artery and nerve

The medial femoral circumflex artery runs deep to quadratus femoris.

Gluteal muscles
Gluteus maximus: inserts to gluteal tuberosity of the femur and iliotibial tract
Gluteus medius: attach to lateral greater trochanter
Gluteus minimis: attach to anterior greater trochanter
All extend and abduct the hip

Deep lateral hip rotators
Piriformis
Gemelli
Obturator internus
Quadratus femoris
Nerves
Superior gluteal nerve (L5, S1)	
Gluteus medius
Gluteus minimis
Tensor fascia lata

Inferior gluteal nerve Gluteus maximus

Damage to the superior gluteal nerve will result in the patient developing a Trendelenberg gait. Affected patients are unable to abduct the thigh at the hip joint. During the stance phase, the weakened abductor muscles allow the pelvis to tilt down on the opposite side. To compensate, the trunk lurches to the weakened side to attempt to maintain a level pelvis throughout the gait cycle. The pelvis sags on the opposite side of the lesioned superior gluteal nerve.

227
Q

Which of the following structures accompanies the posterior interventricular artery within the posterior interventricular groove?

	Great cardiac vein
	Middle cardiac vein
	Small cardiac vein
	Anterior cardiac vein
	Coronary sinus
A

Middle Cardiac Vein

Heart anatomy

The walls of each cardiac chamber comprise:
Epicardium
Myocardium
Endocardium

Cardiac muscle is attached to the cardiac fibrous skeleton.

Relations
The heart and roots of the great vessels within the pericardial sac are related to the posterior aspect of the sternum, medial ends of the 3rd to 5th ribs on the left and their associated costal cartilages. The heart and pericardial sac are situated obliquely two thirds to the left and one third to the right of the median plane.

The pulmonary valve lies at the level of the left third costal cartilage.
The mitral valve lies at the level of the fourth costal cartilage.

Coronary sinus
This lies in the posterior part of the coronary groove and receives blood from the cardiac veins. The great cardiac vein lies at its left and the middle and small cardiac veins lie on its right. The smallest cardiac vein (anterior cardiac vein) drains into the right atrium directly.

Aortic sinus
Right coronary artery arises from the right aortic sinus, the left is derived from the left aortic sinus, which lies posteriorly.

Features of the left ventricle as opposed to the right

Structure Left Ventricle
A-V Valve Mitral (double leaflet)
Walls Twice as thick as right
Trabeculae carnae Much thicker and more numerous

Right coronary artery
The RCA supplies:
Right atrium
Diaphragmatic part of the right ventricle
Usually the posterior third of the interventricular septum
The sino atrial node (60% cases)
The atrio ventricular node (80% cases)

Left coronary artery
The LCA supplies:
Left atrium
Most of left ventricle
Part of the right ventricle
Anterior two thirds of the inter ventricular septum
The sino atrial node (remaining 40% cases)

Innervation of the heart
Autonomic nerve fibres from the superficial and deep cardiac plexus. These lie anterior to the bifurcation of the trachea, posterior to the ascending aorta and superior to the bifurcation of the pulmonary trunk. The parasympathetic supply to the heart is from presynaptic fibres of the vagus nerves.

228
Q

An 18 year old male presents to casualty with a depressed skull fracture. This is managed surgically. Over the next few days he complains of double vision on walking down stairs and reading. On testing ocular convergence, the left eye faces downwards and medially, but the right side does not do so. Which of the nerves listed below is most likely to be responsible?

	Facial
	Oculomotor
	Abducens
	Trochlear
	Trigeminal nerve
A

The trochlear nerve has a relatively long intracranial course and this makes it vulnerable to injury in head trauma. Head trauma is the commonest cause of an acute fourth nerve palsy. A 4th nerve palsy is the commonest cause of a vertical diplopia. The diplopia is at its worst when the eye looks medially which it usually does as part of the accommodation reflex when walking down stairs.

Oculomotor nerve
Large nucleus at the midbrain
Fibres pass through the red nucleus and the pyramidal tract; through the cavernous sinus into the orbit
Ptosis
Eye down and out
Unable to move the eye superiorly, inferiorly, medially
Pupil fixed and dilated

Trochlear nerve	
Longest intracranial course
Only nerve to exit the dorsal aspect of brainstem
Nucleus at midbrain, passes between the posterior cerebral and superior cerebellar arteries, through the cavernous sinus into the orbit
Vertical diplopia (diplopia on descending the stairs)
Unable to look down and in

Abducens nerve Nucleus lies in the mid pons Convergence of eyes in primary position
Lateral diplopia towards side of lesion
Eye deviates medially

229
Q

A 77 year old man with symptoms of intermittent claudication is due to have his ankle brachial pressure indices measured. The dorsalis pedis artery is impalpable. Which of the following tendinous structures lies medial to it, that may facilitate its identification?

	Extensor digitorum longus tendon
	Peroneus tertius tendon
	Extensor hallucis longus tendon
	Extensor digitorum brevis tendon
	Flexor digitorum longus tendon
A

The extensor hallucis longus tendon lies medial to the dorsalis pedis artery.

Arches of the foot
The foot is conventionally considered to have two arches.
The longitudinal arch is higher on the medial than on the lateral side. The posterior part of the calcaneum forms a posterior pillar to support the arch. The lateral part of this structure passes via the cuboid bone and the lateral two metatarsal bones. The medial part of this structure is more important. The head of the talus marks the summit of this arch, located between the sustentaculum tali and the navicular bone. The anterior pillar of the medial arch is composed of the navicular bone, the three cuneiforms and the medial three metatarsal bones.
The transverse arch is situated on the anterior part of the tarsus and the posterior part of the metatarsus. The cuneiforms and metatarsal bases narrow inferiorly, which contributes to the shape of the arch.

Intertarsal joints

  • Sub talar joint Formed by the cylindrical facet on the lower surface of the body of the talus and the posterior facet on the upper surface of the calcaneus. The facet on the talus is concave anteroposteriorly, the other is convex. The synovial cavity of this joint does not communicate with any other joint.
  • Talocalcaneonavicular joint The anterior part of the socket is formed by the concave articular surface of the navicular bone, posteriorly by the upper surface of the sustentaculum tali. The talus sits within this socket
  • Calcaneocuboid joint Highest point in the lateral part of the longitudinal arch. The lower aspect of this joint is reinforced by the long plantar and plantar calcaneocuboid ligaments.
  • Transverse tarsal joint The talocalcaneonavicular joint and the calcaneocuboid joint extend across the tarsus in an irregular transverse plane, between the talus and calcaneus behind and the navicular and cuboid bones in front. This plane is termed the transverse tarsal joint.
  • Cuneonavicular joint Formed between the convex anterior surface of the navicular bone and the concave surface of the the posterior ends of the three cuneiforms.
    Intercuneiform joints Between the three cuneiform bones.
  • Cuneocuboid joint Between the circular facets on the lateral cuneiform bone and the cuboid. This joint contributes to the tarsal part of the transverse arch.

Nerves in the foot

Lateral plantar nerve
Passes anterolaterally towards the base of the 5th metatarsal between flexor digitorum brevis and flexor accessorius. On the medial aspect of the lateral plantar artery. At the base of the 5th metatarsal it splits into superficial and deep branches.

Medial plantar nerve
Passes forwards with the medial plantar artery under the cover of the flexor retinaculum to the interval between abductor hallucis and flexor digitorum brevis on the sole of the foot.

Plantar arteries
Arise under the cover of the flexor retinaculum, midway between the tip of the medial malleolus and the most prominent part of the medial side of the heel.

Medial plantar artery. Passes forwards medial to medial plantar nerve in the space between abductor hallucis and flexor digitorum brevis.Ends by uniting with a branch of the 1st plantar metatarsal artery.
Lateral plantar artery. Runs obliquely across the sole of the foot. It lies lateral to the lateral plantar nerve. At the base of the 5th metatarsal bone it arches medially across the foot on the metatarsals

Dorsalis pedis artery
This vessel is a direct continuation of the anterior tibial artery. It commences on the front of the ankle joint and runs to the proximal end of the first metatarsal space. Here is gives off the arcuate artery and continues forwards as the first dorsal metatarsal artery. It is accompanied by two veins throughout its length. It is crossed by the extensor hallucis brevis

230
Q

A 23 year old man falls over whilst intoxicated and a shard of glass transects his median nerve at the proximal border of the flexor retinaculum. His tendons escape injury. Which of the following features is least likely to be present?

Weakness of thumb abduction
Loss of sensation on the dorsal aspect of the thenar eminence
Loss of power of opponens pollicis
Adduction and lateral rotation of the thumb at rest
Loss of power of abductor pollicis brevis
A

The median nerve may be injured proximal to the flexor retinaculum. This will result in loss of abductor pollicis brevis, flexor pollicis brevis, opponens pollicis and the first and second lumbricals. When the patient is asked to close the hand slowly there is a lag of the index and middle fingers reflecting the impaired lumbrical muscle function. The sensory changes are minor and do not extend to the dorsal aspect of the thenar eminence.
Abductor pollicis longus will contribute to thumb abduction (and is innervated by the posterior interosseous nerve) and therefore abduction will be weaker than prior to the injury.

The median nerve is formed by the union of a lateral and medial root respectively from the lateral (C5,6,7) and medial (C8 and T1) cords of the brachial plexus; the medial root passes anterior to the third part of the axillary artery. The nerve descends lateral to the brachial artery, crosses to its medial side (usually passing anterior to the artery). It passes deep to the bicipital aponeurosis and the median cubital vein at the elbow.
It passes between the two heads of the pronator teres muscle, and runs on the deep surface of flexor digitorum superficialis (within its fascial sheath).
Near the wrist it becomes superficial between the tendons of flexor digitorum superficialis and flexor carpi radialis, deep to palmaris longus tendon. It passes deep to the flexor retinaculum to enter the palm, but lies anterior to the long flexor tendons within the carpal tunnel.

Branches:
- Upper arm No branches, although the nerve commonly communicates with the musculocutaneous nerve

- Forearm	Pronator teres
Flexor carpi radialis
Palmaris longus
Flexor digitorum superficialis
Flexor pollicis longus
Flexor digitorum profundus (only the radial half)
- Distal forearm	Palmar cutaneous branch
Hand (Motor)	Motor supply (LOAF)
Lateral 2 lumbricals
Opponens pollicis
Abductor pollicis brevis
Flexor pollicis brevis
  • Hand (Sensory)
    Over thumb and lateral 2 ½ fingers
    On the palmar aspect this projects proximally, on the dorsal aspect only the distal regions are innervated with the radial nerve providing the more proximal cutaneous innervation.

Patterns of damage
Damage at wrist
e.g. carpal tunnel syndrome
paralysis and wasting of thenar eminence muscles and opponens pollicis (ape hand deformity)
sensory loss to palmar aspect of lateral (radial) 2 ½ fingers

Damage at elbow, as above plus:
unable to pronate forearm
weak wrist flexion
ulnar deviation of wrist

Anterior interosseous nerve (branch of median nerve)
leaves just below the elbow
results in loss of pronation of forearm and weakness of long flexors of thumb and index finger

231
Q

The following muscles are supplied by the recurrent laryngeal nerve except:

	Transverse arytenoid
	Posterior crico-arytenoid
	Cricothyroid
	Oblique arytenoid
	Thyroarytenoid
A

The external branch of the superior laryngeal nerve innervates the cricothyroid muscle.

Recurrent laryngeal nerve
- Branch of the vagus nerve
Innervates: all intrinsic larynx muscles (excluding cricothyroid)

Path:
Right
Arises anterior to the subclavian artery and ascends obliquely next to the trachea, behind the common carotid artery
It is either anterior or posterior to the inferior thyroid artery

Left
Arises left to the arch of the aorta
Winds below the aorta
Ascends along the side of the trachea

Then both

Pass in a groove between the trachea and oesophagus
Enters the larynx behind the articulation between the thyroid cartilage and cricoid
Distributed to larynx muscles

Branches to

Cardiac plexus
Mucous membrane and muscular coat of the oesophagus and trachea

Innervates

Intrinsic larynx muscles (excluding cricothyroid)

232
Q

From which embryological structure is the ureter derived?

	Uranchus
	Cloaca
	Vitello-intestinal duct
	Mesonephric duct
	None of the above
A

The ureter develops from the mesonephric duct. The mesonephric duct is associated with the metanephric duct that develops within the metenephrogenic blastema. This forms the site of the ureteric bud which branches off the mesonephric duct.

Ureter

25-35 cm long
Muscular tube lined by transitional epithelium
Surrounded by thick muscular coat. Becomes 3 muscular layers as it crosses the bony pelvis
Retroperitoneal structure overlying transverse processes L2-L5
Lies anterior to bifurcation of iliac vessels
Blood supply is segmental; renal artery, aortic branches, gonadal branches, common iliac and internal iliac
Lies beneath the uterine artery

233
Q

A 16 year old boy is hit by a car and sustains a blow to the right side of his head. He is initially conscious but on arrival in the emergency department is comatose. On examination his right pupil is fixed and dilated. The neurosurgeons plan immediate surgery. What type of initial approach should be made?

	Left parieto-temporal craniotomy
	Right parieto-temporal craniotomy
	Posterior fossa craniotomy
	Left parieto-temporal burr holes
	None of the above
A

Right parieto-temporal craniotomy

A unilateral dilated pupil is a classic sign of transtentorial herniation. The medial aspect of the temporal lobe (uncus) herniates across the tentorium and causes pressure on the ipsilateral oculomotor nerve, interrupting parasympathetic input to the eye and resulting in a dilated pupil. In addition the brainstem is compressed. As the ipsilateral oculomotor nerve is being compressed, craniotomy (rather than Burr Holes) should be made on the ipsilateral side.

Patients who suffer head injuries should be managed according to ATLS principles and extra cranial injuries should be managed alongside cranial trauma. Inadequate cardiac output will compromise CNS perfusion irrespective of the nature of the cranial injury.

Types of traumatic brain injury
Extradural haematoma Bleeding into the space between the dura mater and the skull. Often results from acceleration-deceleration trauma or a blow to the side of the head. The majority of extradural haematomas occur in the temporal region where skull fractures cause a rupture of the middle meningeal artery.

Features
Raised intracranial pressure
Some patients may exhibit a lucid interval
Subdural haematoma Bleeding into the outermost meningeal layer. Most commonly occur around the frontal and parietal lobes. May be either acute or chronic.

Risk factors include old age and alcoholism.

Slower onset of symptoms than a extradural haematoma.
Subarachnoid haemorrhage Usually occurs spontaneously in the context of a ruptured cerebral aneurysm, but may be seen in association with other injuries when a patient has sustained a traumatic brain injury.

Pathophysiology
Primary brain injury may be focal (contusion/ haematoma) or diffuse (diffuse axonal injury)
Diffuse axonal injury occurs as a result of mechanical shearing following deceleration, causing disruption and tearing of axons
Intra-cranial haematomas can be extradural, subdural or intracerebral, while contusions may occur adjacent to (coup) or contralateral (contre-coup) to the side of impact
Secondary brain injury occurs when cerebral oedema, ischaemia, infection, tonsillar or tentorial herniation exacerbates the original injury. The normal cerebral auto regulatory processes are disrupted following trauma rendering the brain more susceptible to blood flow changes and hypoxia
The Cushings reflex (hypertension and bradycardia) often occurs late and is usually a pre terminal event

Management
Where there is life threatening rising ICP such as in extra dural haematoma and whilst theatre is prepared or transfer arranged use of IV mannitol/ frusemide may be required.
Diffuse cerebral oedema may require decompressive craniotomy
Exploratory Burr Holes have little management in modern practice except where scanning may be unavailable and to thus facilitate creation of formal craniotomy flap
Depressed skull fractures that are open require formal surgical reduction and debridement, closed injuries may be managed non operatively if there is minimal displacement.
ICP monitoring is appropriate in those who have GCS 3-8 and normal CT scan.
ICP monitoring is mandatory in those who have GCS 3-8 and abnormal CT scan.
Hyponatraemia is most likely to be due to syndrome of inappropriate ADH secretion.
Minimum of cerebral perfusion pressure of 70mmHg in adults.
Minimum cerebral perfusion pressure of between 40 and 70 mmHg in children.

234
Q

An otherwise fit 74 year old man presents with pain in the right hip following minimal trauma. On examination his leg is shortened and externally rotated. Plain films demonstrate a displaced intracapsular fracture of the femoral neck.

A.	MRI scan
B.	Hemiarthroplasty
C.	Bone scintigraphy
D.	Conservative management
E.	Total hip replacement
F.	Insertion of intra medullary nail
G.	Hip arthrodesis
H.	Internal fixation
A

Total hip replacement

In otherwise fit patients aged over 70, the best long term functional outcomes are obtained with total hip arthroplasty.

Neck of femur (NOF) fracture is a common orthopaedic presentation, with over 65000 fractures in the UK per year. Like many orthopaedic injuries, there is a bimodal age distribution. It is imperative to distinguish between the high energy injury in a young patient, and the low energy osteoporotic fracture in the elderly, as their management aims are very different:

Young patient - Usually high energy trauma (e.g road traffic accident, horse riding) and needs treating in accordance with Advanced Trauma Life Support (ATLS) principles. Will often have associated injuries. Aim is to retain the patients own anatomy, and optimise their function.

Elderly patient - Predominantly female, fall from standing height (fragility fracture). Often patients have multiple comorbidities that will ultimately dictate their prognosis. Aim of orthopaedic treatment is to immediately regain patient mobility so that morbidity (infection, thromboembolic events, pressure sores etc) and mortality associated with prolonged bed rest is avoided. Left untreated, a neck of femur fracture can be considered a terminal event. Historically, mortality associated with elderly hip fracture is 10% at one month, and 30% at one year. However, this has been improved in the UK with the introduction of multidisciplinary, orthogeriatric lead care and the National Hip Fracture Database and Best Practice Tariff.

Pertinent anatomy
Osteology - normal neck-shaft angle is 130 +/- 7 degrees, and 10 +/- 7 degrees of neck anteversion.
Vascular supply - The predominant blood supply to the femoral head and neck is from the medial and lateral femoral circumflex arteries (branches of profunda femoris). These anastomose and pierce the joint capsule at the base of the neck, mainly posteriorly. There is a small vascular contribution from the artery of the ligament teres. Understanding the blood supply is fundamental to the decision making process in treating NOF fractures.

Presentation and initial management
Typically, patients present with pain in the hip/groin, a shortened, abducted, externally rotated leg (due to the unopposed pull of the muscles that act across the hip joint) and the inability to straight-leg-raise. With undisplaced fractures, signs are more subtle.
High energy injuries should be treated in line with ATLS principles. All patients should be fluid resuscitated, have adequate pain relief (often with a fascio-iliiaca nerve block), and be optimised for surgery. In addition, elderly patients should be assessed by an orthogeriatrician.

Imaging
Anteroposterior and cross-table lateral plain radiographs are sufficient to diagnose the majority of NOF fractures. If the fracture extends below the level of the lesser trochanter, or there is any possibility of pathological fracture, full length femur views are essential to plan surgery.

Where there is a high index of suspicion of fracture, but plain radiographs are inconclusive, gold standard investigation is MRI. However, if unavailable within 24 hours, or if the patient will not tolerate MRI, CT is appropriate. The majority of fractures can be seen with modern CT techniques, and so this is becoming first line in many hospitals.

Classification
There has been a move away from named classification systems towards descriptive classification systems.
Two main types of NOF exist: Intra-capsular, and extra-capsular. Extra-capsular fractures are further divided into pertrochanteric or subtrochanteric (within 5cm distal to the lesser trochanter). All fractures are then described as undisplaced, minimally displaced, or displaced.
Femoral neck and head blood supply disruption is common with intracapsular NOF fractures, and rare with extracapsular fractures. This fundamental principle underpins the practise of arthroplasty for intracapsular fractures, and fixation for extracapsular fractures.

If you wish to use a named classification system, the most commonly used are below:
Elderly intracapsular - Garden Classification
Young intrasapsular - Pauvels Classification
Intertrochanteric - Evans
Subtrochanteric - Russell Taylor

Treatment
In general, NOF fractures are treated operatively except if the patient is deemed unlikely to survive an anaesthetic. Best Practice Tarif (BPT) dictates that surgery should happen within 36 hours, as delay of greater than 48 hours is associated with increased morbidity and mortality.

  • The priority with the young patient is to retain the femoral head if possible, even with a displaced intracapsular fracture. The risk of avascular necrosis and non-union (and therefore revision surgery) associated with internal fixation needs weighing up against the sequelae of total hip replacement in the young (wear, dislocation, revision). Discussion is necessary with the patient, on a case by case basis.

** Undisplaced fractures in the elderly can be treated with internal fixation, often with cannulated screws. This is appropriate for valgus impacted subcapital fractures which are inherently stable, to prevent secondary displacement. This does still carry the risk of AVN or non-union, and therefore a future revision. For this reason, many surgeons advocate arthroplasty as a single surgery.

*** NICE guidance - patients who fulfil these criteria should be offered total hip replacement which conveys better function and prosthetic survivorship, compared with hemiarthroplasty, but at an increased risk of dislocation.

  • Intertrochanteric fractures vary greatly in their stability. If the trochanter (and therefore lateral wall), and medial calcar is in tact, then the fracture configuration bears stability. This can be treated with a DHS, as collapse of the fracture is predictable. Where either or both structures are involved in the fracture, stability becomes compromised and many surgeons will favour using an intramedullary device. This is an ongoing debate, and difficult to test in an exam setting.
235
Q

A 72 year old retired teacher is admitted to A&E with a fall and hip pain. He is normally fit and well. He lives with his son in a detached, 2 storey house. A hip x-ray and femur views confirm a sub trochanteric fracture.

A.	MRI scan
B.	Hemiarthroplasty
C.	Bone scintigraphy
D.	Conservative management
E.	Total hip replacement
F.	Insertion of intra medullary nail
G.	Hip arthrodesis
H.	Internal fixation
A

Intramedullary devices are normally recommended for reverse oblique, transverse subtrochanteric fractures.

Neck of femur (NOF) fracture is a common orthopaedic presentation, with over 65000 fractures in the UK per year. Like many orthopaedic injuries, there is a bimodal age distribution. It is imperative to distinguish between the high energy injury in a young patient, and the low energy osteoporotic fracture in the elderly, as their management aims are very different:

Young patient - Usually high energy trauma (e.g road traffic accident, horse riding) and needs treating in accordance with Advanced Trauma Life Support (ATLS) principles. Will often have associated injuries. Aim is to retain the patients own anatomy, and optimise their function.

Elderly patient - Predominantly female, fall from standing height (fragility fracture). Often patients have multiple comorbidities that will ultimately dictate their prognosis. Aim of orthopaedic treatment is to immediately regain patient mobility so that morbidity (infection, thromboembolic events, pressure sores etc) and mortality associated with prolonged bed rest is avoided. Left untreated, a neck of femur fracture can be considered a terminal event. Historically, mortality associated with elderly hip fracture is 10% at one month, and 30% at one year. However, this has been improved in the UK with the introduction of multidisciplinary, orthogeriatric lead care and the National Hip Fracture Database and Best Practice Tariff.

Pertinent anatomy
Osteology - normal neck-shaft angle is 130 +/- 7 degrees, and 10 +/- 7 degrees of neck anteversion.
Vascular supply - The predominant blood supply to the femoral head and neck is from the medial and lateral femoral circumflex arteries (branches of profunda femoris). These anastomose and pierce the joint capsule at the base of the neck, mainly posteriorly. There is a small vascular contribution from the artery of the ligament teres. Understanding the blood supply is fundamental to the decision making process in treating NOF fractures.

Presentation and initial management
Typically, patients present with pain in the hip/groin, a shortened, abducted, externally rotated leg (due to the unopposed pull of the muscles that act across the hip joint) and the inability to straight-leg-raise. With undisplaced fractures, signs are more subtle.
High energy injuries should be treated in line with ATLS principles. All patients should be fluid resuscitated, have adequate pain relief (often with a fascio-iliiaca nerve block), and be optimised for surgery. In addition, elderly patients should be assessed by an orthogeriatrician.

Imaging
Anteroposterior and cross-table lateral plain radiographs are sufficient to diagnose the majority of NOF fractures. If the fracture extends below the level of the lesser trochanter, or there is any possibility of pathological fracture, full length femur views are essential to plan surgery.

Where there is a high index of suspicion of fracture, but plain radiographs are inconclusive, gold standard investigation is MRI. However, if unavailable within 24 hours, or if the patient will not tolerate MRI, CT is appropriate. The majority of fractures can be seen with modern CT techniques, and so this is becoming first line in many hospitals.

Classification
There has been a move away from named classification systems towards descriptive classification systems.
Two main types of NOF exist: Intra-capsular, and extra-capsular. Extra-capsular fractures are further divided into pertrochanteric or subtrochanteric (within 5cm distal to the lesser trochanter). All fractures are then described as undisplaced, minimally displaced, or displaced.
Femoral neck and head blood supply disruption is common with intracapsular NOF fractures, and rare with extracapsular fractures. This fundamental principle underpins the practise of arthroplasty for intracapsular fractures, and fixation for extracapsular fractures.

If you wish to use a named classification system, the most commonly used are below:
Elderly intracapsular - Garden Classification
Young intrasapsular - Pauvels Classification
Intertrochanteric - Evans
Subtrochanteric - Russell Taylor

Treatment
In general, NOF fractures are treated operatively except if the patient is deemed unlikely to survive an anaesthetic. Best Practice Tarif (BPT) dictates that surgery should happen within 36 hours, as delay of greater than 48 hours is associated with increased morbidity and mortality.

  • The priority with the young patient is to retain the femoral head if possible, even with a displaced intracapsular fracture. The risk of avascular necrosis and non-union (and therefore revision surgery) associated with internal fixation needs weighing up against the sequelae of total hip replacement in the young (wear, dislocation, revision). Discussion is necessary with the patient, on a case by case basis.

** Undisplaced fractures in the elderly can be treated with internal fixation, often with cannulated screws. This is appropriate for valgus impacted subcapital fractures which are inherently stable, to prevent secondary displacement. This does still carry the risk of AVN or non-union, and therefore a future revision. For this reason, many surgeons advocate arthroplasty as a single surgery.

*** NICE guidance - patients who fulfil these criteria should be offered total hip replacement which conveys better function and prosthetic survivorship, compared with hemiarthroplasty, but at an increased risk of dislocation.

  • Intertrochanteric fractures vary greatly in their stability. If the trochanter (and therefore lateral wall), and medial calcar is in tact, then the fracture configuration bears stability. This can be treated with a DHS, as collapse of the fracture is predictable. Where either or both structures are involved in the fracture, stability becomes compromised and many surgeons will favour using an intramedullary device. This is an ongoing debate, and difficult to test in an exam setting.
236
Q

A 72 year old lady stumbles and falls. On examination she is tender in the left groin and unable to weight bear. Attempts at internal rotation produce severe pain. Plain films of the hip show no obvious fracture.

A.	MRI scan
B.	Hemiarthroplasty
C.	Bone scintigraphy
D.	Conservative management
E.	Total hip replacement
F.	Insertion of intra medullary nail
G.	Hip arthrodesis
H.	Internal fixation
A

In those patients who present with a suspected hip fracture, but normal plain films, the most accurate investigation is an MRI or CT scan.

Neck of femur (NOF) fracture is a common orthopaedic presentation, with over 65000 fractures in the UK per year. Like many orthopaedic injuries, there is a bimodal age distribution. It is imperative to distinguish between the high energy injury in a young patient, and the low energy osteoporotic fracture in the elderly, as their management aims are very different:

Young patient - Usually high energy trauma (e.g road traffic accident, horse riding) and needs treating in accordance with Advanced Trauma Life Support (ATLS) principles. Will often have associated injuries. Aim is to retain the patients own anatomy, and optimise their function.

Elderly patient - Predominantly female, fall from standing height (fragility fracture). Often patients have multiple comorbidities that will ultimately dictate their prognosis. Aim of orthopaedic treatment is to immediately regain patient mobility so that morbidity (infection, thromboembolic events, pressure sores etc) and mortality associated with prolonged bed rest is avoided. Left untreated, a neck of femur fracture can be considered a terminal event. Historically, mortality associated with elderly hip fracture is 10% at one month, and 30% at one year. However, this has been improved in the UK with the introduction of multidisciplinary, orthogeriatric lead care and the National Hip Fracture Database and Best Practice Tariff.

Pertinent anatomy
Osteology - normal neck-shaft angle is 130 +/- 7 degrees, and 10 +/- 7 degrees of neck anteversion.
Vascular supply - The predominant blood supply to the femoral head and neck is from the medial and lateral femoral circumflex arteries (branches of profunda femoris). These anastomose and pierce the joint capsule at the base of the neck, mainly posteriorly. There is a small vascular contribution from the artery of the ligament teres. Understanding the blood supply is fundamental to the decision making process in treating NOF fractures.

Presentation and initial management
Typically, patients present with pain in the hip/groin, a shortened, abducted, externally rotated leg (due to the unopposed pull of the muscles that act across the hip joint) and the inability to straight-leg-raise. With undisplaced fractures, signs are more subtle.
High energy injuries should be treated in line with ATLS principles. All patients should be fluid resuscitated, have adequate pain relief (often with a fascio-iliiaca nerve block), and be optimised for surgery. In addition, elderly patients should be assessed by an orthogeriatrician.

Imaging
Anteroposterior and cross-table lateral plain radiographs are sufficient to diagnose the majority of NOF fractures. If the fracture extends below the level of the lesser trochanter, or there is any possibility of pathological fracture, full length femur views are essential to plan surgery.

Where there is a high index of suspicion of fracture, but plain radiographs are inconclusive, gold standard investigation is MRI. However, if unavailable within 24 hours, or if the patient will not tolerate MRI, CT is appropriate. The majority of fractures can be seen with modern CT techniques, and so this is becoming first line in many hospitals.

Classification
There has been a move away from named classification systems towards descriptive classification systems.
Two main types of NOF exist: Intra-capsular, and extra-capsular. Extra-capsular fractures are further divided into pertrochanteric or subtrochanteric (within 5cm distal to the lesser trochanter). All fractures are then described as undisplaced, minimally displaced, or displaced.
Femoral neck and head blood supply disruption is common with intracapsular NOF fractures, and rare with extracapsular fractures. This fundamental principle underpins the practise of arthroplasty for intracapsular fractures, and fixation for extracapsular fractures.

If you wish to use a named classification system, the most commonly used are below:
Elderly intracapsular - Garden Classification
Young intrasapsular - Pauvels Classification
Intertrochanteric - Evans
Subtrochanteric - Russell Taylor

Treatment
In general, NOF fractures are treated operatively except if the patient is deemed unlikely to survive an anaesthetic. Best Practice Tarif (BPT) dictates that surgery should happen within 36 hours, as delay of greater than 48 hours is associated with increased morbidity and mortality.

  • The priority with the young patient is to retain the femoral head if possible, even with a displaced intracapsular fracture. The risk of avascular necrosis and non-union (and therefore revision surgery) associated with internal fixation needs weighing up against the sequelae of total hip replacement in the young (wear, dislocation, revision). Discussion is necessary with the patient, on a case by case basis.

** Undisplaced fractures in the elderly can be treated with internal fixation, often with cannulated screws. This is appropriate for valgus impacted subcapital fractures which are inherently stable, to prevent secondary displacement. This does still carry the risk of AVN or non-union, and therefore a future revision. For this reason, many surgeons advocate arthroplasty as a single surgery.

*** NICE guidance - patients who fulfil these criteria should be offered total hip replacement which conveys better function and prosthetic survivorship, compared with hemiarthroplasty, but at an increased risk of dislocation.

  • Intertrochanteric fractures vary greatly in their stability. If the trochanter (and therefore lateral wall), and medial calcar is in tact, then the fracture configuration bears stability. This can be treated with a DHS, as collapse of the fracture is predictable. Where either or both structures are involved in the fracture, stability becomes compromised and many surgeons will favour using an intramedullary device. This is an ongoing debate, and difficult to test in an exam setting.
237
Q

Which of the following relationship descriptions regarding the scalene muscles is incorrect?

The brachial plexus passes anterior to the middle scalene muscle
The phrenic nerve lies anterior to the anterior scalene muscle
The subclavian artery passes posterior to the middle scalene
The subclavian vein lies anterior to the anterior scalene muscle at the level of the first rib
The anterior scalene inserts into the first rib
A

The subclavian artery passes anterior to the middle scalene.
Scalene muscles

The 3 paired muscles are:
Scalenus anterior: Elevate 1st rib and laterally flex the neck to same side
Scalenus medius: Same action as scalenus anterior
Scalenus posterior: Elevate 2nd rib and tilt neck to opposite side

Innervation Spinal nerves C4-6
Origin Transverse processes C2 to C7
Insertion First and second ribs
Important relations
The brachial plexus and subclavian artery pass between the anterior and middle scalenes through a space called the scalene hiatus/fissure.
The subclavian vein and phrenic nerve pass anteriorly to the anterior scalene as it crosses over the first rib.

Thoracic outlet syndrome
The scalenes are at risk of adhering to the fascia surrounding the brachial plexus or shortening causing compression of the brachial plexus when it passes between the clavicle and 1st rib causing thoracic outlet syndrome.

238
Q

A 56 year old man is having a long venous line inserted via the femoral vein into the right atrium for CVP measurements. The catheter is advanced through the IVC. At which of the following levels does this vessel enter the thorax?

	L2
	T10
	L1
	T8
	T6
A

The IVC passes through the diaphragm at T8.

Inferior vena cava
Origin
L5

Path
Left and right common iliac veins merge to form the IVC.
Passes right of midline
Paired segmental lumbar veins drain into the IVC throughout its length
The right gonadal vein empties directly into the cava and the left gonadal vein generally empties into the left renal vein.
The next major veins are the renal veins and the hepatic veins
Pierces the central tendon of diaphragm at T8
Right atrium

Relations
Anteriorly Small bowel, first and third part of duodenum, head of pancreas, liver and bile duct, right common iliac artery, right gonadal artery
Posteriorly Right renal artery, right psoas, right sympathetic chain, coeliac ganglion

Levels
Level	Vein
T8	Hepatic vein, inferior phrenic vein, pierces diaphragm
L1	Suprarenal veins, renal vein
L2	Gonadal vein
L1-5	Lumbar veins
L5	Common iliac vein, formation of IVC
239
Q

A 23 year old man falls and injures his hand. There are concerns that he may have a scaphoid fracture as there is tenderness in his anatomical snuffbox on clinical examination. Which of the following forms the posterior border of this structure?

	Basilic vein
	Radial artery
	Extensor pollicis brevis
	Abductor pollicis longus
	Extensor pollicis longus
A

It’s boundaries are extensor pollicis longus, medially (posterior border) and laterally (anterior border) by the tendons of abductor pollicis longus and extensor pollicis brevis.

Anatomical snuffbox

Posterior border Tendon of extensor pollicis longus
Anterior border Tendons of extensor pollicis brevis and abductor pollicis longus
Proximal border Styloid process of the radius
Distal border Apex of snuffbox triangle
Floor Trapezium and scaphoid
Content Radial artery

240
Q

Which of the following structures attaches periosteum to bone?

	Sharpeys fibres
	Peripheral lamellae
	Elastic fibres
	Fibrolamellar bundles
	Purkinje fibres
A

Periosteum is attached to bone by strong collagenous fibers called Sharpey’s fibres, which extend to the outer circumferential and interstitial lamellae. It also provides an attachment for muscles and tendons.

Periosteum is a membrane that covers the outer surface of all bones, except at the joints of long bones. Endosteum lines the inner surface of all bones.

Periosteum consists of dense irregular connective tissue. Periosteum is divided into an outer “fibrous layer” and inner “cambium layer” (or “osteogenic layer”). The fibrous layer contains fibroblasts, while the cambium layer contains progenitor cells that develop into osteoblasts. These osteoblasts are responsible for increasing the width of a long bone and the overall size of the other bone types. After a bone fracture the progenitor cells develop into osteoblasts and chondroblasts, which are essential to the healing process.

As opposed to osseous tissue, periosteum has nociceptive nerve endings, making it very sensitive to manipulation. It also provides nourishment by providing the blood supply. Periosteum is attached to bone by strong collagenous fibers called Sharpey’s fibres, which extend to the outer circumferential and interstitial lamellae. It also provides an attachment for muscles and tendons.

Periosteum that covers the outer surface of the bones of the skull is known as “pericranium” except when in reference to the layers of the scalp.

241
Q

A 62 year old man is undergoing a left hemicolectomy for carcinoma of the descending colon. The registrar commences mobilisation of the left colon by pulling downwards and medially. Blood soon appears in the left paracolic gutter. The most likely source of bleeding is the:

	Marginal artery
	Left testicular artery
	Spleen
	Left renal vein
	None of the above
A

The spleen is commonly torn by traction injuries in colonic surgery. The other structures are associated with bleeding during colonic surgery but would not manifest themselves as blood in the paracolic gutter prior to incision of the paracolonic peritoneal edge.

Left colon

Position
As the left colon passes inferiorly its posterior aspect becomes extraperitoneal, and the ureter and gonadal vessels are close posterior relations that may become involved in disease processes
At a level of L3-4 (variable) the left colon becomes the sigmoid colon and wholly intraperitoneal once again
The sigmoid colon is a highly mobile structure and may even lie on the right side of the abdomen
It passes towards the midline, the taenia blend and this marks the transition between sigmoid colon and upper rectum

Blood supply
Inferior mesenteric artery
However, the marginal artery (from the right colon) contributes, this contribution becomes clinically significant when the IMA is divided surgically (e.g. During AAA repair)

242
Q

A man is undergoing excision of a sub mandibular gland. As the gland is mobilised, a vessel is injured lying between the gland and the mandible. Which of the following is this vessel most likely to be?

	Lingual artery
	Occipital artery
	Superior thyroid artery
	Facial artery
	External jugular vein
A

The facial artery lies between the gland and mandible and is often ligated during excision of the gland. The lingual artery may be encountered but this is usually later in the operative process as Whartons duct is mobilised.

The high salivary viscosity of submandibular gland secretions favors stone formation.
Most stones are radio-opaque.
The marginal mandibular nerve is the most superficial structure.

Relations of the submandibular gland
Superficial Platysma, deep fascia and mandible
Submandibular lymph nodes
Facial vein (facial artery near mandible)
Marginal mandibular nerve
Cervical branch of the facial nerve

Deep	Facial artery (inferior to the mandible)
Mylohyoid muscle
Sub mandibular duct
Hyoglossus muscle
Lingual nerve
Submandibular ganglion
Hypoglossal nerve

Submandibular duct (Wharton’s duct)
Opens lateral to the lingual frenulum on the anterior floor of mouth.
5 cm length
Lingual nerve wraps around Wharton’s duct. As the duct passes forwards it crosses medial to the nerve to lie above it and then crosses back, lateral to it, to reach a position below the nerve.

Innervation
Sympathetic innervation- Derived from superior cervical ganglion
Parasympathetic innervation- Submandibular ganglion via lingual nerve

Arterial supply
Branch of the facial artery. The facial artery passes through the gland to groove its deep surface. It then emerges onto the face by passing between the gland and the mandible.

Venous drainage
Anterior facial vein (lies deep to the Marginal Mandibular nerve)

Lymphatic drainage
Deep cervical and jugular chains of nodes

243
Q

Supplies the motor fibres of styloglossus.

A.	Facial
B.	Trigeminal
C.	Vagus
D.	Hypoglossal
E.	Glossopharyngeal
A

Hypoglossal

The hypoglossal nerve supplies motor innervation to all extrinsic and intrinsic muscles of the tongue. The only possible exception to this is palatoglossus (which is jointly innervated by the vagus and accessory nerves.

Cranial nerve lesions
Olfactory nerve May be injured in basal skull fractures or involved in frontal lobe tumour extension. Loss of olfactory nerve function in relation to major CNS pathology is seldom an isolated event and thus it is poor localiser of CNS pathology.
Optic nerve Problems with visual acuity may result from intra ocular disorders. Problems with the blood supply such as amaurosis fugax may produce temporary visual distortion. More important surgically is the pupillary response to light. The pupillary size may be altered in a number of disorders. Nerves involved in the resizing of the pupil connect to the pretectal nucleus of the high midbrain, bypassing the lateral geniculate nucleus and the primary visual cortex. From the pretectal nucleus neurones pass to the Edinger - Westphal nucleus, motor axons from here pass along with the oculomotor nerve. They synapse with ciliary ganglion neurones; the parasympathetic axons from this then innervate the iris and produce miosis. The miotic pupil is seen in disorders such as Horner’s syndrome or opiate overdose.
Mydriasis is the dilatation of the pupil in response to disease, trauma, drugs (or the dark!). It is pathological when light fails to induce miosis. The radial muscle is innervated by the sympathetic nervous system. Because the parasympathetic fibres travel with the oculomotor nerve they will be damaged by lesions affecting this nerve (e.g. cranial trauma).
The response to light shone in one eye is usually a constriction of both pupils. This indicates intact direct and consensual light reflexes. When the optic nerve has an afferent defect the light shining on the affected eye will produce a diminished pupillary response in both eyes. Whereas light shone on the unaffected eye will produce a normal pupillary response in both eyes. This is referred to as the Marcus Gunn pupil and is seen in conditions such as optic neuritis. In a total CN II lesion shining the light in the affected eye will produce no response.
Oculomotor nerve The pupillary effects are described above. In addition it supplies all ocular muscles apart from lateral rectus and superior oblique. Thus the affected eye will be deviated inferolaterally. Levator palpebrae superioris may also be impaired resulting in impaired ability to open the eye.
Trochlear nerve The eye will not be able to look down.
Trigeminal nerve Largest cranial nerve. Exits the brainstem at the pons. Branches are ophthalmic, maxillary and mandibular. Only the mandibular branch has both sensory and motor fibres. Branches converge to form the trigeminal ganglion (located in Meckels cave). It supplies the muscles of mastication and also tensor veli palatine, mylohyoid, anterior belly of digastric and tensor tympani. The detailed descriptions of the various sensory functions are described in other areas of the website. The corneal reflex is important and is elicited by applying a small tip of cotton wool to the cornea, a reflex blink should occur if it is intact. It is mediated by: the naso ciliary branch of the ophthalmic branch of the trigeminal (sensory component) and the facial nerve producing the motor response. Lesions of the afferent arc will produce bilateral absent blink and lesions of the efferent arc will result in a unilateral absent blink.
Abducens nerve The affected eye will have a deficit of abduction. This cranial nerve exits the brainstem between the pons and medulla. It thus has a relatively long intra cranial course which renders it susceptible to damage in raised intra cranial pressure.
Facial nerve Emerges from brainstem between pons and medulla. It controls muscles of facial expression and taste from the anterior 2/3 of the tongue. The nerve passes into the petrous temporal bone and into the internal auditory meatus. It then passes through the facial canal and exits at the stylomastoid foramen. It passes through the parotid gland and divides at this point. It does not innervate the parotid gland. Its divisions are considered in other parts of the website. Its motor fibres innervate orbicularis oculi to produce the efferent arm of the corneal reflex. In surgical practice it may be injured during parotid gland surgery or invaded by malignancies of the gland and a lower motor neurone on the ipsilateral side will result.
Vestibulo-cochlear nerve Exits from the pons and then passes through the internal auditory meatus. It is implicated in sensorineural hearing loss. Individuals with sensorineural hearing loss will localise the sound in webers test to the normal ear. Rinnes test will be reduced on the affected side but should still work. These two tests will distinguish sensorineural hearing loss from conductive deafness. In the latter condition webers test will localise to the affected ear and Rinnes test will be impaired on the affected side. Surgical lesions affecting this nerve include CNS tumours and basal skull fractures. It may also be damaged by the administration of ototoxic drugs (of which gentamicin is the most commonly used in surgical practice).
Glossopharyngeal nerve Exits the pons just above the vagus. Receives sensory fibres from posterior 1/3 tongue, tonsils, pharynx and middle ear (otalgia may occur following tonsillectomy). It receives visceral afferents from the carotid bodies. It supplies parasympathetic fibres to the parotid gland via the otic ganglion and motor function to stylopharyngeaus muscle. The sensory function of the nerve is tested using the gag reflex.
Vagus nerve Leaves the medulla between the olivary nucleus and the inferior cerebellar peduncle. Passes through the jugular foramen and into the carotid sheath. Details of the functions of the vagus nerve are covered in the website under relevant organ sub headings.
Accessory nerve Exists from the caudal aspect of the brainstem (multiple branches) supplies trapezius and sternocleidomastoid muscles. The distal portion of this nerve is most prone to injury during surgical procedures.
Hypoglossal nerve Emerges from the medulla at the preolivary sulcus, passes through the hypoglossal canal. It lies on the carotid sheath and passes deep to the posterior belly of digastric to supply muscles of the tongue (except palatoglossus). Its location near the carotid sheath makes it vulnerable during carotid endarterectomy surgery and damage will produce ipsilateral defect in muscle function.

244
Q

Provides general sensation to the anterior two thirds of the tongue.

A.	Facial
B.	Trigeminal
C.	Vagus
D.	Hypoglossal
E.	Glossopharyngeal
A

Trigeminal

Taste to the anterior two thirds of the tongue is supplied by the facial nerve, the trigeminal supplies general sensation, this is mediated by the mandibular branch of the trigeminal nerve (via the lingual nerve).

Cranial nerve lesions
Olfactory nerve May be injured in basal skull fractures or involved in frontal lobe tumour extension. Loss of olfactory nerve function in relation to major CNS pathology is seldom an isolated event and thus it is poor localiser of CNS pathology.
Optic nerve Problems with visual acuity may result from intra ocular disorders. Problems with the blood supply such as amaurosis fugax may produce temporary visual distortion. More important surgically is the pupillary response to light. The pupillary size may be altered in a number of disorders. Nerves involved in the resizing of the pupil connect to the pretectal nucleus of the high midbrain, bypassing the lateral geniculate nucleus and the primary visual cortex. From the pretectal nucleus neurones pass to the Edinger - Westphal nucleus, motor axons from here pass along with the oculomotor nerve. They synapse with ciliary ganglion neurones; the parasympathetic axons from this then innervate the iris and produce miosis. The miotic pupil is seen in disorders such as Horner’s syndrome or opiate overdose.
Mydriasis is the dilatation of the pupil in response to disease, trauma, drugs (or the dark!). It is pathological when light fails to induce miosis. The radial muscle is innervated by the sympathetic nervous system. Because the parasympathetic fibres travel with the oculomotor nerve they will be damaged by lesions affecting this nerve (e.g. cranial trauma).
The response to light shone in one eye is usually a constriction of both pupils. This indicates intact direct and consensual light reflexes. When the optic nerve has an afferent defect the light shining on the affected eye will produce a diminished pupillary response in both eyes. Whereas light shone on the unaffected eye will produce a normal pupillary response in both eyes. This is referred to as the Marcus Gunn pupil and is seen in conditions such as optic neuritis. In a total CN II lesion shining the light in the affected eye will produce no response.
Oculomotor nerve The pupillary effects are described above. In addition it supplies all ocular muscles apart from lateral rectus and superior oblique. Thus the affected eye will be deviated inferolaterally. Levator palpebrae superioris may also be impaired resulting in impaired ability to open the eye.
Trochlear nerve The eye will not be able to look down.
Trigeminal nerve Largest cranial nerve. Exits the brainstem at the pons. Branches are ophthalmic, maxillary and mandibular. Only the mandibular branch has both sensory and motor fibres. Branches converge to form the trigeminal ganglion (located in Meckels cave). It supplies the muscles of mastication and also tensor veli palatine, mylohyoid, anterior belly of digastric and tensor tympani. The detailed descriptions of the various sensory functions are described in other areas of the website. The corneal reflex is important and is elicited by applying a small tip of cotton wool to the cornea, a reflex blink should occur if it is intact. It is mediated by: the naso ciliary branch of the ophthalmic branch of the trigeminal (sensory component) and the facial nerve producing the motor response. Lesions of the afferent arc will produce bilateral absent blink and lesions of the efferent arc will result in a unilateral absent blink.
Abducens nerve The affected eye will have a deficit of abduction. This cranial nerve exits the brainstem between the pons and medulla. It thus has a relatively long intra cranial course which renders it susceptible to damage in raised intra cranial pressure.
Facial nerve Emerges from brainstem between pons and medulla. It controls muscles of facial expression and taste from the anterior 2/3 of the tongue. The nerve passes into the petrous temporal bone and into the internal auditory meatus. It then passes through the facial canal and exits at the stylomastoid foramen. It passes through the parotid gland and divides at this point. It does not innervate the parotid gland. Its divisions are considered in other parts of the website. Its motor fibres innervate orbicularis oculi to produce the efferent arm of the corneal reflex. In surgical practice it may be injured during parotid gland surgery or invaded by malignancies of the gland and a lower motor neurone on the ipsilateral side will result.
Vestibulo-cochlear nerve Exits from the pons and then passes through the internal auditory meatus. It is implicated in sensorineural hearing loss. Individuals with sensorineural hearing loss will localise the sound in webers test to the normal ear. Rinnes test will be reduced on the affected side but should still work. These two tests will distinguish sensorineural hearing loss from conductive deafness. In the latter condition webers test will localise to the affected ear and Rinnes test will be impaired on the affected side. Surgical lesions affecting this nerve include CNS tumours and basal skull fractures. It may also be damaged by the administration of ototoxic drugs (of which gentamicin is the most commonly used in surgical practice).
Glossopharyngeal nerve Exits the pons just above the vagus. Receives sensory fibres from posterior 1/3 tongue, tonsils, pharynx and middle ear (otalgia may occur following tonsillectomy). It receives visceral afferents from the carotid bodies. It supplies parasympathetic fibres to the parotid gland via the otic ganglion and motor function to stylopharyngeaus muscle. The sensory function of the nerve is tested using the gag reflex.
Vagus nerve Leaves the medulla between the olivary nucleus and the inferior cerebellar peduncle. Passes through the jugular foramen and into the carotid sheath. Details of the functions of the vagus nerve are covered in the website under relevant organ sub headings.
Accessory nerve Exists from the caudal aspect of the brainstem (multiple branches) supplies trapezius and sternocleidomastoid muscles. The distal portion of this nerve is most prone to injury during surgical procedures.
Hypoglossal nerve Emerges from the medulla at the preolivary sulcus, passes through the hypoglossal canal. It lies on the carotid sheath and passes deep to the posterior belly of digastric to supply muscles of the tongue (except palatoglossus). Its location near the carotid sheath makes it vulnerable during carotid endarterectomy surgery and damage will produce ipsilateral defect in muscle function.

245
Q

Supplies general sensation to the posterior third of the tongue.

A.	Facial
B.	Trigeminal
C.	Vagus
D.	Hypoglossal
E.	Glossopharyngeal
A

Glossopharyngeal

The glossopharyngeal nerve supplies general sensation to the posterior third of the tongue and contributes to the gag reflex.

Cranial nerve lesions
Olfactory nerve May be injured in basal skull fractures or involved in frontal lobe tumour extension. Loss of olfactory nerve function in relation to major CNS pathology is seldom an isolated event and thus it is poor localiser of CNS pathology.
Optic nerve Problems with visual acuity may result from intra ocular disorders. Problems with the blood supply such as amaurosis fugax may produce temporary visual distortion. More important surgically is the pupillary response to light. The pupillary size may be altered in a number of disorders. Nerves involved in the resizing of the pupil connect to the pretectal nucleus of the high midbrain, bypassing the lateral geniculate nucleus and the primary visual cortex. From the pretectal nucleus neurones pass to the Edinger - Westphal nucleus, motor axons from here pass along with the oculomotor nerve. They synapse with ciliary ganglion neurones; the parasympathetic axons from this then innervate the iris and produce miosis. The miotic pupil is seen in disorders such as Horner’s syndrome or opiate overdose.
Mydriasis is the dilatation of the pupil in response to disease, trauma, drugs (or the dark!). It is pathological when light fails to induce miosis. The radial muscle is innervated by the sympathetic nervous system. Because the parasympathetic fibres travel with the oculomotor nerve they will be damaged by lesions affecting this nerve (e.g. cranial trauma).
The response to light shone in one eye is usually a constriction of both pupils. This indicates intact direct and consensual light reflexes. When the optic nerve has an afferent defect the light shining on the affected eye will produce a diminished pupillary response in both eyes. Whereas light shone on the unaffected eye will produce a normal pupillary response in both eyes. This is referred to as the Marcus Gunn pupil and is seen in conditions such as optic neuritis. In a total CN II lesion shining the light in the affected eye will produce no response.
Oculomotor nerve The pupillary effects are described above. In addition it supplies all ocular muscles apart from lateral rectus and superior oblique. Thus the affected eye will be deviated inferolaterally. Levator palpebrae superioris may also be impaired resulting in impaired ability to open the eye.
Trochlear nerve The eye will not be able to look down.
Trigeminal nerve Largest cranial nerve. Exits the brainstem at the pons. Branches are ophthalmic, maxillary and mandibular. Only the mandibular branch has both sensory and motor fibres. Branches converge to form the trigeminal ganglion (located in Meckels cave). It supplies the muscles of mastication and also tensor veli palatine, mylohyoid, anterior belly of digastric and tensor tympani. The detailed descriptions of the various sensory functions are described in other areas of the website. The corneal reflex is important and is elicited by applying a small tip of cotton wool to the cornea, a reflex blink should occur if it is intact. It is mediated by: the naso ciliary branch of the ophthalmic branch of the trigeminal (sensory component) and the facial nerve producing the motor response. Lesions of the afferent arc will produce bilateral absent blink and lesions of the efferent arc will result in a unilateral absent blink.
Abducens nerve The affected eye will have a deficit of abduction. This cranial nerve exits the brainstem between the pons and medulla. It thus has a relatively long intra cranial course which renders it susceptible to damage in raised intra cranial pressure.
Facial nerve Emerges from brainstem between pons and medulla. It controls muscles of facial expression and taste from the anterior 2/3 of the tongue. The nerve passes into the petrous temporal bone and into the internal auditory meatus. It then passes through the facial canal and exits at the stylomastoid foramen. It passes through the parotid gland and divides at this point. It does not innervate the parotid gland. Its divisions are considered in other parts of the website. Its motor fibres innervate orbicularis oculi to produce the efferent arm of the corneal reflex. In surgical practice it may be injured during parotid gland surgery or invaded by malignancies of the gland and a lower motor neurone on the ipsilateral side will result.
Vestibulo-cochlear nerve Exits from the pons and then passes through the internal auditory meatus. It is implicated in sensorineural hearing loss. Individuals with sensorineural hearing loss will localise the sound in webers test to the normal ear. Rinnes test will be reduced on the affected side but should still work. These two tests will distinguish sensorineural hearing loss from conductive deafness. In the latter condition webers test will localise to the affected ear and Rinnes test will be impaired on the affected side. Surgical lesions affecting this nerve include CNS tumours and basal skull fractures. It may also be damaged by the administration of ototoxic drugs (of which gentamicin is the most commonly used in surgical practice).
Glossopharyngeal nerve Exits the pons just above the vagus. Receives sensory fibres from posterior 1/3 tongue, tonsils, pharynx and middle ear (otalgia may occur following tonsillectomy). It receives visceral afferents from the carotid bodies. It supplies parasympathetic fibres to the parotid gland via the otic ganglion and motor function to stylopharyngeaus muscle. The sensory function of the nerve is tested using the gag reflex.
Vagus nerve Leaves the medulla between the olivary nucleus and the inferior cerebellar peduncle. Passes through the jugular foramen and into the carotid sheath. Details of the functions of the vagus nerve are covered in the website under relevant organ sub headings.
Accessory nerve Exists from the caudal aspect of the brainstem (multiple branches) supplies trapezius and sternocleidomastoid muscles. The distal portion of this nerve is most prone to injury during surgical procedures.
Hypoglossal nerve Emerges from the medulla at the preolivary sulcus, passes through the hypoglossal canal. It lies on the carotid sheath and passes deep to the posterior belly of digastric to supply muscles of the tongue (except palatoglossus). Its location near the carotid sheath makes it vulnerable during carotid endarterectomy surgery and damage will produce ipsilateral defect in muscle function.

246
Q

The integrity of which muscle is assessed by the Trendelenburg test?

	Sartorius
	Quadratus femoris
	Semimembranosus
	Gluteus medius
	Piriformis
A

Gluteus medius

Trendelenburg test

Injury or division of the superior gluteal nerve results in a motor deficit that consists of weakened abduction of the thigh by gluteus medius, a disabling gluteus medius limp and a compensatory list of the body to the weakened gluteal side. The compensation results in a gravitational shift so that the body is supported on the unaffected limb.

When a person is asked to stand on one leg, the gluteus medius usually contracts as soon as the contralateral leg leaves the floor, preventing the pelvis from dipping towards the unsupported side. When a person with paralysis of the superior gluteal nerve is asked to stand on one leg, the pelvis on the unsupported side descends, indicating that the gluteus medius on the affected side is weak or non functional ( a positive Trendelenburg test).

This eponymous test also refers to a vascular investigation in which tourniquets are placed around the upper thigh, these can help determine whether saphenofemoral incompetence is present.

247
Q

A 52 year old female renal patient needs a femoral catheter to allow for haemodialysis. Which of the structures listed below is least likely to be encountered during its insertion?

	Great saphenous vein
	Deep circumflex iliac artery
	Superficial circumflex iliac artery
	Femoral vein
	Femoral branch of the genitofemoral nerve
A

Femoral access catheters are typically inserted in the region of the femoral triangle. Therefore the physician may encounter the femoral, vein, nerve, branches of the femoral artery and tributaries of the femoral vein. The deep circumflex iliac artery arises above the inguinal ligament and is therefore less likely to be encountered than the superficial circumflex iliac artery which arises below the inguinal ligament.

Femoral triangle anatomy

Boundaries
Superiorly	Inguinal ligament
Laterally	Sartorius
Medially	Adductor longus
Floor	Iliopsoas, adductor longus and pectineus
Roof	
Fascia lata and Superficial fascia
Superficial inguinal lymph nodes (palpable below the inguinal ligament)
Long saphenous vein

Contents
Femoral vein (medial to lateral)
Femoral artery-pulse palpated at the mid inguinal point
Femoral nerve
Deep and superficial inguinal lymph nodes
Lateral cutaneous nerve
Great saphenous vein
Femoral branch of the genitofemoral nerve

248
Q

A 53 year old man with a chronically infected right kidney is due to undergo a nephrectomy. Which of the following structures would be encountered first during a posterior approach to the hilum of the right kidney?

	Right renal artery
	Ureter
	Right renal vein
	Inferior vena cava
	Right testicular vein
A

The ureter is the most posterior structure at the hilum of the right kidney and would therefore be encountered first during a posterior approach.

Renal arteries

The right renal artery is longer than the left renal artery
The renal vein/artery/pelvis enter the kidney at the hilum

Relations
Right Anterior- IVC, right renal vein, the head of the pancreas, and the descending part of the duodenum
Left Anterior- left renal vein, the tail of the pancreas

Branches
The renal arteries are direct branches off the aorta (upper border of L2- right side and L1 - left side)
In 30% there may be accessory arteries (mainly left side). Instead of entering the kidney at the hilum, they usually pierce the upper or lower part of the organ.
Before reaching the hilum of the kidney, each artery divides into four or five segmental branches (renal vein anterior and ureter posterior); which then divide within the sinus into lobar arteries supplying each pyramid and cortex.
Each vessel gives off some small inferior suprarenal branches to the suprarenal gland, the ureter, and the surrounding cellular tissue and muscles.

249
Q

A 28 year old man is stabbed outside a nightclub in the upper arm. The median nerve is transected. Which of the following muscles will demonstrate impaired function as a result?

	Palmaris brevis
	Second and third interossei
	Adductor pollicis
	Abductor pollicis longus
	Abductor pollicis brevis
A

The median nerve innervates all the short muscles of the thumb except the adductor and the deep head of the short flexor. Palmaris and the interossei are innervated by the ulnar nerve.

Palmaris brevis - Ulnar nerve
Palmar interossei- Ulnar nerve
Adductor pollicis - Ulnar nerve
Abductor pollicis longus - Posterior interosseous nerve
Abductor pollicis brevis - Median nerve

The median nerve is formed by the union of a lateral and medial root respectively from the lateral (C5,6,7) and medial (C8 and T1) cords of the brachial plexus; the medial root passes anterior to the third part of the axillary artery. The nerve descends lateral to the brachial artery, crosses to its medial side (usually passing anterior to the artery). It passes deep to the bicipital aponeurosis and the median cubital vein at the elbow.
It passes between the two heads of the pronator teres muscle, and runs on the deep surface of flexor digitorum superficialis (within its fascial sheath).
Near the wrist it becomes superficial between the tendons of flexor digitorum superficialis and flexor carpi radialis, deep to palmaris longus tendon. It passes deep to the flexor retinaculum to enter the palm, but lies anterior to the long flexor tendons within the carpal tunnel.

Branches
Region	Branch
Upper arm	No branches, although the nerve commonly communicates with the musculocutaneous nerve
Forearm	Pronator teres
Flexor carpi radialis
Palmaris longus
Flexor digitorum superficialis
Flexor pollicis longus
Flexor digitorum profundus (only the radial half)
Distal forearm	Palmar cutaneous branch
Hand (Motor)	Motor supply (LOAF)
Lateral 2 lumbricals
Opponens pollicis
Abductor pollicis brevis
Flexor pollicis brevis
Hand (Sensory)	
Over thumb and lateral 2 ½ fingers
On the palmar aspect this projects proximally, on the dorsal aspect only the distal regions are innervated with the radial nerve providing the more proximal cutaneous innervation.

Patterns of damage
Damage at wrist
e.g. carpal tunnel syndrome
paralysis and wasting of thenar eminence muscles and opponens pollicis (ape hand deformity)
sensory loss to palmar aspect of lateral (radial) 2 ½ fingers

Damage at elbow, as above plus:
unable to pronate forearm
weak wrist flexion
ulnar deviation of wrist

Anterior interosseous nerve (branch of median nerve)
leaves just below the elbow
results in loss of pronation of forearm and weakness of long flexors of thumb and index finger

250
Q

A 22 year old man sustains a blow to the side of his head with a baseball bat during a fight. He is initially conscious. However, he subsequently loses consciousness and then dies. Post mortem examination shows an extradural haematoma. The most likely culprit vessel is a branch of which of the following?

	Middle cerebral artery
	Internal carotid artery
	Anterior cerebral artery
	Maxillary artery
	Mandibular artery
A

The middle meningeal artery is the most likely source of the extradural haematoma in this setting. It is a branch of the maxillary artery. The middle cerebral artery does not give rise to the middle meningeal artery. Note that the question is asking for the vessel which gives rise to the middle meningeal artery (“the likely culprit vessel is a branch of which of the following”)

Middle meningeal artery is typically the third branch of the first part of the maxillary artery, one of the two terminal branches of the external carotid artery. After branching off the maxillary artery in the infratemporal fossa, it runs through the foramen spinosum to supply the dura mater (the outermost meninges) .
The middle meningeal artery is the largest of the three (paired) arteries which supply the meninges, the others being the anterior meningeal artery and the posterior meningeal artery.
The middle meningeal artery runs beneath the pterion. It is vulnerable to injury at this point, where the skull is thin. Rupture of the artery may give rise to an extra dural hematoma.
In the dry cranium, the middle meningeal, which runs within the dura mater surrounding the brain, makes a deep indention in the calvarium.
The middle meningeal artery is intimately associated with the auriculotemporal nerve which wraps around the artery making the two easily identifiable in the dissection of human cadavers and also easily damaged in surgery.

251
Q

A 72 year old man with carcinoma of the lung is undergoing a left pneumonectomy. The left main bronchus is divided. Which of the following thoracic vertebrae lies posterior to this structure?

	T3
	T7
	T6
	T10
	T1
A

The left main bronchus lies at T6. Topographical anatomy of the thorax is important as it helps surgeons to predict the likely structures to be injured in trauma scenarios (so popular with examiners)

The right lung is composed of 3 lobes divided by the oblique and transverse fissures. The left lung has two lobes divided by the oblique fissure.The apex of both lungs is approximately 4cm superior to the sterno-costal joint of the first rib. Immediately below this is a sulcus created by the subclavian artery.

Peripheral contact points of the lung
Base: diaphragm
Costal surface: corresponds to the cavity of the chest
Mediastinal surface: Contacts the mediastinal pleura. Has the cardiac impression. Above and behind this concavity is a triangular depression named the hilum, where the structures which form the root of the lung enter and leave the viscus. These structures are invested by pleura, which, below the hilum and behind the pericardial impression, forms the pulmonary ligament

Right lung
Above the hilum is the azygos vein; Superior to this is the groove for the superior vena cava and right innominate vein; behind this, and nearer the apex, is a furrow for the innominate artery. Behind the hilum and the attachment of the pulmonary ligament is a vertical groove for the oesophagus; In front and to the right of the lower part of the oesophageal groove is a deep concavity for the extrapericardiac portion of the inferior vena cava.

The root of the right lung lies behind the superior vena cava and the right atrium, and below the azygos vein.

The right main bronchus is shorter, wider and more vertical than the left main bronchus and therefore the route taken by most foreign bodies.

Left lung
Above the hilum is the furrow produced by the aortic arch, and then superiorly the groove accommodating the left subclavian artery; Behind the hilum and pulmonary ligament is a vertical groove produced by the descending aorta, and in front of this, near the base of the lung, is the lower part of the oesophagus.

The root of the left lung passes under the aortic arch and in front of the descending aorta.

Inferior borders of both lungs
6th rib in mid clavicular line
8th rib in mid axillary line
10th rib posteriorly
The pleura runs two ribs lower than the corresponding lung level.
252
Q

Which of the following regions of the male urethra is entirely surrounded by Bucks fascia?

	Preprostatic part
	Prostatic part
	Membranous part
	Spongiose part
	None of the above
A

Spongiose part

Bucks fascia is a layer of deep fascia that covers the penis it is continuous with the external spermatic fascia and the penile suspensory ligament. The membranous part of the urethra may partially pass through Bucks fascia as it passes into the penis. However, the spongiose part of the urethra is contained wholly within Bucks fascia.

Female urethra
The female urethra is shorter and more acutely angulated than the male urethra. It is an extra-peritoneal structure and embedded in the endopelvic fascia. The neck of the bladder is subjected to transmitted intra-abdominal pressure and therefore deficiency in this area may result in stress urinary incontinence. Between the layers of the urogenital diaphragm the female urethra is surrounded by the external urethral sphincter, this is innervated by the pudendal nerve. It ultimately lies anterior to the vaginal orifice.

Male urethra
In males the urethra is much longer and is divided into four parts.

Pre-prostatic urethra Extremely short and lies between the bladder and prostate gland.It has a stellate lumen and is between 1 and 1.5cm long.Innervated by sympathetic noradrenergic fibres, as this region is composed of striated muscles bundles they may contract and prevent retrograde ejaculation.
Prostatic urethra This segment is wider than the membranous urethra and contains several openings for the transmission of semen (at the midpoint of the urethral crest).
Membranous urethra Narrowest part of the urethra and surrounded by external sphincter. It traverses the perineal membrane 2.5cm postero-inferior to the symphysis pubis.
Penile urethra Travels through the corpus spongiosum on the underside of the penis. It is the longest urethral segment.It is dilated at its origin as the infrabulbar fossa and again in the gland penis as the navicular fossa. The bulbo-urethral glands open into the spongiose section of the urethra 2.5cm below the perineal membrane.

The urothelium is transitional in nature near to the bladder and becomes squamous more distally.

253
Q

Which of the following statements relating to the knee joint is false?

It is the largest synovial joint in the body
When the knee is fully extended all ligaments of the knee joint are taut
Rupture of the anterior cruciate ligament may result in haemarthrosis
The posterior aspect of the patella is extrasynovial
The joint is innervated by the femoral, sciatic and obturator nerves
A

The posterior aspect is intrasynovial and the knee itself comprises the largest synovial joint in the body. It may swell considerably following trauma such as ACL injury. Which may be extremely painful owing to rich innervation from femoral, sciatic and ( a smaller) contribution from the obturator nerve. During full extension all ligaments are taut and the knee is locked.

The knee joint is a synovial joint, the largest and most complicated. It consists of two condylar joints between the femur and tibia and a sellar joint between the patella and the femur. The tibiofemoral articular surfaces are incongruent, however, this is improved by the presence of the menisci. The degree of congruence is related to the anatomical position of the knee joint and is greatest in full extension.

Knee joint compartments
Tibiofemoral
Comprised of the patella/femur joint, lateral and medial compartments (between femur condyles and tibia)
Synovial membrane and cruciate ligaments partially separate the medial and lateral compartments
Patellofemoral
Ligamentum patellae
Actions: provides joint stability in full extension

Fibrous capsule
The capsule of the knee joint is a complex, composite structure with contributions from adjacent tendons.
Anterior fibres The capsule does not pass proximal to the patella. It blends with the tendinous expansions of vastus medialis and lateralis
Posterior fibres These fibres are vertical and run from the posterior surface of the femoral condyles to the posterior aspect of the tibial condyle
Medial fibres Attach to the femoral and tibial condyles beyond their articular margins, blending with the tibial collateral ligament
Lateral fibres Attach to the femur superior to popliteus, pass over its tendon to head of fibula and tibial condyle

Bursae
Anterior
Subcutaneous prepatellar bursa; between patella and skin
Deep infrapatellar bursa; between tibia and patellar ligament
Subcutaneous infrapatellar bursa; between distal tibial tuberosity and skin
Laterally
Bursa between lateral head of gastrocnemius and joint capsule
Bursa between fibular collateral ligament and tendon of biceps femoris
Bursa between fibular collateral ligament and tendon of popliteus
Medially
Bursa between medial head of gastrocnemius and the fibrous capsule
Bursa between tibial collateral ligament and tendons of sartorius, gracilis and semitendinosus
Bursa between the tendon of semimembranosus and medial tibial condyle and medial head of gastrocnemius
Posterior Highly variable and inconsistent

Ligaments
Medial collateral ligament Medial epicondyle femur to medial tibial condyle: valgus stability
Lateral collateral ligament Lateral epicondyle femur to fibula head: varus stability
Anterior cruciate ligament Anterior tibia to lateral intercondylar notch femur: prevents tibia sliding anteriorly
Posterior cruciate ligament Posterior tibia to medial intercondylar notch femur: prevents tibia sliding posteriorly
Patellar ligament Central band of the tendon of quadriceps femoris, extends from patella to tibial tuberosity

Menisci
Medial and lateral menisci compensate for the incongruence of the femoral and tibial condyles.
Composed of fibrous tissue.
Medial meniscus is attached to the tibial collateral ligament.
Lateral meniscus is attached to the loose fibres at the lateral edge of the joint and is separate from the fibular collateral ligament. The lateral meniscus is crossed by the popliteus tendon.

Nerve supply
The knee joint is supplied by the femoral, tibial and common peroneal divisions of the sciatic and by a branch from the obturator nerve. Hip pathology pain may be referred to the knee.

Blood supply
Genicular branches of the femoral artery, popliteal and anterior tibial arteries all supply the knee joint.

254
Q

In the distal third of the upper arm, where is the musculocutaneous nerve located?

Between the biceps brachii and brachialis muscles
Between the brachialis and brachioradialis muscles
Between the brachioradialis and triceps muscles
Between the brachialis and triceps muscles
Between the humerus and brachialis muscles
A

The musculocutaneous nerve lies between the biceps and brachialis muscles.

Musculocutaneous nerve

Branch of lateral cord of brachial plexus

Path
It penetrates the coracobrachialis muscle
Passes obliquely between the biceps brachii and the brachialis to the lateral side of the arm
Above the elbow it pierces the deep fascia lateral to the tendon of the biceps brachii
Continues into the forearm as the lateral cutaneous nerve of the forearm

Innervates
Coracobrachialis
Biceps brachii
Brachialis

255
Q

A 48 year old lady is undergoing a left sided adrenalectomy for an adrenal adenoma. The superior adrenal artery is injured and starts to bleed, from which of the following does this vessel arise?

	Left renal artery
	Inferior phrenic artery
	Aorta
	Splenic
	None of the above
A

The superior adrenal artery is a branch of the inferior phrenic artery.

Adrenal gland anatomy

Anatomy

Location Superomedially to the upper pole of each kidney
Relationships of the right adrenal Diaphragm-Posteriorly, Kidney-Inferiorly, Vena Cava-Medially, Hepato-renal pouch and bare area of the liver-Anteriorly
Relationships of the left adrenal Crus of the diaphragm-Postero- medially, Pancreas and splenic vessels-Inferiorly, Lesser sac and stomach-Anteriorly

Arterial supply Superior adrenal arteries- from inferior phrenic artery, Middle adrenal arteries - from aorta, Inferior adrenal arteries -from renal arteries
Venous drainage of the right adrenal Via one central vein directly into the IVC
Venous drainage of the left adrenal Via one central vein into the left renal vein

256
Q

Which of the following does not exit the pelvis through the greater sciatic foramen?

	Superior gluteal artery
	Internal pudendal vessels
	Sciatic nerve
	Obturator nerve
	Inferior gluteal nerve
A

The obturator nerve exits through the obturator foramen.

Greater sciatic foramen

Contents
Nerves	
Sciatic Nerve
Superior and Inferior Gluteal Nerves
Pudendal Nerve
Posterior Femoral Cutaneous Nerve
Nerve to Quadratus Femoris
Nerve to Obturator internus
Vessels	
Superior Gluteal Artery and vein
Inferior Gluteal Artery and vein
Internal Pudendal Artery and vein

Piriformis
The piriformis is a landmark for identifying structures passing out of the sciatic notch
Above piriformis: Superior gluteal vessels
Below piriformis: Inferior gluteal vessels, sciatic nerve (10% pass through it, <1% above it), posterior cutaneous nerve of the thigh

Greater sciatic foramen boundaries
Anterolaterally Greater sciatic notch of the ilium
Posteromedially Sacrotuberous ligament
Inferior Sacrospinous ligament and the ischial spine
Superior Anterior sacroiliac ligament

Structures passing between both foramina (Medial to lateral)
Pudendal nerve
Internal pudendal artery
Nerve to obturator internus

Contents of the lesser sciatic foramen
Tendon of the obturator internus
Pudendal nerve
Internal pudendal artery and vein
Nerve to the obturator internus
257
Q

Which statement is false about the foramina of the skull?

The hypoglossal canal transmits the hypoglossal nerve
The foramen spinosum is at the base of the medial pterygoid plate.
The jugular foramen transmits the accessory nerve
The foramen lacerum is located in the sphenoid bone
The stylomastoid foramen transmits the facial nerve
A

Foramen spinosum- Sphenoid bone

Foramen ovale	Sphenoid bone	Otic ganglion
V3 (Mandibular nerve:3rd branch of 
trigeminal)
Accessory meningeal artery
Lesser petrosal nerve
Emissary veins

Foramen spinosum Sphenoid bone Middle meningeal artery
Meningeal branch of the Mandibular nerve

Foramen rotundum Sphenoid bone Maxillary nerve (V2)

Foramen lacerum/ carotid canal Sphenoid bone Base of the medial pterygoid plate.
Internal carotid artery*
Nerve and artery of the pterygoid canal

Jugular foramen Temporal bone Anterior: inferior petrosal sinus
Intermediate: glossopharyngeal, vagus, and accessory nerves.
Posterior: sigmoid sinus (becoming the internal jugular vein) and some meningeal branches from the occipital and ascending pharyngeal arteries.

Foramen magnum Occipital bone Anterior and posterior spinal arteries
Vertebral arteries
Medulla oblongata

Stylomastoid foramen Temporal bone Stylomastoid artery
Facial nerve

Superior orbital fissure Sphenoid bone Oculomotor nerve (III)
Recurrent meningeal artery
Trochlear nerve (IV)
Lacrimal, frontal and nasociliary branches of ophthalmic nerve (V1)
Abducent nerve (VI)
Superior ophthalmic vein

*= In life the foramen lacerum is occluded by a cartilagenous plug. The ICA initially passes into the carotid canal which ascends superomedially to enter the cranial cavity through the foramen lacerum.

258
Q

An 80 year old lady with a caecal carcinoma is undergoing a right hemicolectomy performed through a transverse incision. The procedure is difficult and the incision is extended medially by dividing the rectus sheath. Brisk arterial haemorrhage ensues. From which of the following does the damaged vessel originate?

	Internal iliac artery
	External iliac artery
	Superior vesical artery
	Inferior vesical artery
	None of the above
A

The vessel damaged is the epigastric artery. This originates from the external iliac artery

The inferior epigastric artery arises from the external iliac artery immediately above the inguinal ligament. It then passes along the medial margin of the deep inguinal ring. From here it continues superiorly to lie behind the rectus abdominis muscle.

259
Q

A 73 year old man has a large abdominal aortic aneurysm. During a laparotomy for planned surgical repair the surgeons find the aneurysm is far more proximally located and lies near the origin of the SMA. During the dissection a vessel lying transversely across the aorta is injured. What is this vessel most likely to be?

	Left renal vein
	Right renal vein
	Inferior mesenteric artery
	Ileocolic artery
	Middle colic artery
A

The left renal vein runs across the surface of the aorta and may require deliberate ligation during juxtarenal aneurysm repair.

Abdominal aortic topography
Origin	T12
Termination	L4
Posterior relations	L1-L4 Vertebral bodies
Anterior relations	Lesser omentum
Liver
Left renal vein
Inferior mesenteric vein
Third part of duodenum
Pancreas
Parietal peritoneum
Peritoneal cavity
Right lateral relations	Right crus of the diaphragm
Cisterna chyli
Azygos vein
IVC (becomes posterior distally)
Left lateral relations	4th part of duodenum
Duodenal-jejunal flexure
Left sympathetic trunk
260
Q

Which of the following is not a branch of the posterior cord of the brachial plexus?

	Thoracodorsal nerve
	Axillary nerve
	Radial nerve
	Lower subscapular nerve
	Musculocutaneous nerve
A

Mnemonic branches off the posterior cord

S ubscapular (upper and lower)
T horacodorsal
A xillary
R adial

The musculocutaneous nerve is a branch off the lateral cord.

Brachial plexus

Origin Anterior rami of C5 to T1
Sections of the plexus
Roots, trunks, divisions, cords, branches
Mnemonic:Real Teenagers Drink Cold Beer
Roots
Located in the posterior triangle
Pass between scalenus anterior and medius
Trunks
Located posterior to middle third of clavicle
Upper and middle trunks related superiorly to the subclavian artery
Lower trunk passes over 1st rib posterior to the subclavian artery
Divisions Apex of axilla
Cords Related to axillary artery

261
Q

A 45 year old man has been admitted after being knocked off his bicycle. His ankle is grossly deformed with bilateral malleolar tenderness with severe ankle swelling and tenting of the medial soft tissues.

A. Surgical fixation
B. Below knee amputation
C. Application of below knee plaster
D. Application of ankle boot
E. Application of external fixation device
F. Application of compression dressing and physiotherapy
G. Immediate reduction and application of backslab

A

Immediate reduction and application of backslab

This is an unstable ankle injury that is likely to require surgical fixation. The immediate management of a displaced ankle fracture is to reduce the fracture to prevent soft tissues compromise and help reduce swelling. This can be performed before an x-ray is obtained if performing the x-ray will significantly delay reduction.

Ankle injuries

An ankle fracture relates to a fracture around the tibio-talar joint. It generally refers to a fracture involving the lateral, and/or medial and/or posterior malleolus. Pilon and Tillaux fractures are also considered to be ankle fractures, but are not covered here.
Ankle fractures are common. They effect men and women in equal numbers, but men have a higher rate as young adults (sports and contact injuries), and women a higher rate post-menopausal (fragility type fracture).

Osseous anatomy
The ankle (or mortise) joint consists of the distal tibia (tibial plafond and posterior malleolus), the distal fibula (lateral malleolus), and the talus. The main movement at the ankle joint is plantar and dorsiflexion.

Ligamentous anatomy
Medial side: Deltoid ligament. This is divided into superficial and deep portions. It is the primary restraint to valgus tilting of the talus.

Lateral side: Lateral ligament complex consisting from anterior to posterior of the anterior talofibular ligament (ATFL), calcaneofibular ligament (CFL), and the posterior talofibular ligament (PTFL). Together they resist valgus stress to the ankle, and are a restraint to anterior translation of the talus within the mortise joint.

Syndesmosis: The syndesmosis is a ligament complex between the distal tibia and fibula, holding the two bones together. It is fundamental to the integrity of the ankle joint, and its disruption leads to instability. It consists of (from anterior to posterior) the anterior-inferior tibiofibular ligament (AITFL), the transverse tibiofibular ligament (TTFL), the interosseous membrane, and the posterior-inferior tibiofibular ligament (PITFL).

Presentation and initial management
Patients will present following a traumatic event with a painful, swollen ankle, and reluctance/inability to weight bear. The Ottawa rules can be applied to differentiate between an ankle fracture and sprain, but can be unreliable.
In high energy injuries, management should follow ATLS principles to identify more significant injuries first. Neurovascular status of the foot should be documented, and open injuries should be excluded. If an open injury is identified, it should be managed in line with BOAST 4 principles1. If an obvious deformity exists, it should be reduced as soon as possible with appropriate analgesia or conscious sedation. Radiographs of clearly deformed or dislocated joints are not necessary, and removing the pressure on the surrounding soft tissues from the underlying bony deformity is the priority. If the fracture pattern is not clinically obvious then plain radiographs are appropriate and will guide the subsequent manipulation during plaster-of-paris below knee backslab application.

Imaging
AP, lateral and mortise views (20o internal rotation) are essential to evaluate fracture displacement and syndesmotic injury. Decreased tibiofibular overlap, medial joint clear space and lateral talar shift all indicate a syndesmotic injury. (In subtle cases of shift, imaging the uninjured ankle can be helpful as a proportion of the population have little or no tibiotalar overlap 2.)

Where there is suspicion of syndesmosis involvement in the absence of radiographic evidence, stress radiographs can be diagnostic.
Complex fracture patterns (and increasingly posterior malleolar fractures) are best defined using CT.

Classification
The most commonly used classifications are Lauge-Hansen and Danis-Weber.

Lauge-Hansen
Comprises two parts: first part is the foot position, and the second part is the force applied. Useful for understanding the forces involved and therefore predict the ligamentous or bony injury. Results in four injury patterns:
Supination - Adduction (SA) - 10-20%
Supination - External rotation (SER) - 40-75%
Pronation - Abduction (PA) - 5-20%
Pronation - External rotation (PER) - 5-20%

Not often used in clinical practice but good for understanding the principles of ankle fracture.

Danis-Weber
Commonly used. Based on the level of the fibula fracture in relation to the syndesmosis. The more proximal, the greater the risk of syndesmotic injury and therefore fracture instability.
A - fracture below the level of the syndesmosis
B - fracture at the level of the syndesmosis / level of the tibial plafond
C - fracture above the level of the syndesmosis. This includes Maisonneuve fractures (proximal fibula fracture), which can be associated with ankle instability. Beware the high fibula fracture - it may be an ankle fracture!

The Weber classification is based purely on the the lateral side. All injuries can include a medial or posterior bony or ligamentous injury which also dictates fracture stability (bimalleolar and trimalleolar fractures are more unstable).

Treatment
When deciding upon treatment for an ankle fracture, one must consider both the fracture and the patient. Diabetic patients and smokers are at greater risk of post-operative complication, especially wound problems and infection. Likewise, the long term outcome of post-traumatic arthritis from a malunited ankle fracture is extremely important for a young patient, but not as relevant in the elderly. Therefore, normal surgical decision processes apply as with all fractures.

Defining stability of an ankle fracture underpins the treatment decision.
Weber A - Unimalleolar Weber A Weber fractures by definition are stable and therefore can be mobilised fully weight bearing in an ankle boot.

Weber C - Fractures tend to include syndesmotic disruption and are usually bimalleolar (either bony or ligamentous). They are therefore unstable and usually require operative fixation. In addition to the fracture fixation, the syndesmosis usually requires reconstruction/augmentation with screws to restore the joint integrity and function.

Weber B - B fractures vary greatly. They can be part of a trimalleolar injury and therefore extremely unstable, requiring fixation. Alternatively, a uni-malleolar Weber B fracture can be a stable injury, and therefore mobilised immediately in an ankle boot. Defining the stability can be challenging, and often involves stress radiographs, or a trial of mobilisation and repeat radiographs. Defining stability is the subject of much ongoing research. However, treating undisplaced ankle fractures in a below knee plaster, non-weight bearing for six weeks is still widely practised, and a safe approach.

When operative fixation is appropriate, it is usually via open reduction and internal fixation using plates and screws. It must be carried out when soft tissue swelling has settled in order to minimise the risk of wound problems. This can often take a week to settle.

The use of fibula nails is expanding, but is not yet mainstream. Ankle fractures can also be treated with external fixation, or with a hind foot nail in patients who need fixation but where soft tissue or bone quality is poor.

Post operative management
Ankle fractures generally take 6 weeks to unite enough to prevent secondary displacement. This is therefore an appropriate time period to keep a cast on in a conservatively managed patient. Weight bearing post-operatively depends on the quality of the fixation and bone quality, and preference varies between surgeons, ranging from aggressive early mobilisation to a period of non-weight bearing. Return to activities takes approximately three months, and often requires assistance of a physiotherapist to improve range-of-movement and muscle strengthening.

262
Q

A 40 year old marine injures his ankle on an assault course. On examination he has a severely swollen ankle, as well as tenderness over the medial malleolus and proximal fibula. X-rays demonstrate a medial malleolar fracture, spiral fracture of the proximal fibula and widening of the syndesmosis.

A. Surgical fixation
B. Below knee amputation
C. Application of below knee plaster
D. Application of ankle boot
E. Application of external fixation device
F. Application of compression dressing and physiotherapy
G. Immediate reduction and application of backslab

A

Surgical fixation

This is a Maisonneuve fracture of the proximal fibula. It indicates an unstable ankle injury with likely injury to the interosseous membrane. In the setting of radiographic evidence of syndesmotic widening, this requires surgical fixation to reduce and stabilise the syndesmosis.

Ankle injuries

An ankle fracture relates to a fracture around the tibio-talar joint. It generally refers to a fracture involving the lateral, and/or medial and/or posterior malleolus. Pilon and Tillaux fractures are also considered to be ankle fractures, but are not covered here.
Ankle fractures are common. They effect men and women in equal numbers, but men have a higher rate as young adults (sports and contact injuries), and women a higher rate post-menopausal (fragility type fracture).

Osseous anatomy
The ankle (or mortise) joint consists of the distal tibia (tibial plafond and posterior malleolus), the distal fibula (lateral malleolus), and the talus. The main movement at the ankle joint is plantar and dorsiflexion.

Ligamentous anatomy
Medial side: Deltoid ligament. This is divided into superficial and deep portions. It is the primary restraint to valgus tilting of the talus.

Lateral side: Lateral ligament complex consisting from anterior to posterior of the anterior talofibular ligament (ATFL), calcaneofibular ligament (CFL), and the posterior talofibular ligament (PTFL). Together they resist valgus stress to the ankle, and are a restraint to anterior translation of the talus within the mortise joint.

Syndesmosis: The syndesmosis is a ligament complex between the distal tibia and fibula, holding the two bones together. It is fundamental to the integrity of the ankle joint, and its disruption leads to instability. It consists of (from anterior to posterior) the anterior-inferior tibiofibular ligament (AITFL), the transverse tibiofibular ligament (TTFL), the interosseous membrane, and the posterior-inferior tibiofibular ligament (PITFL).

Presentation and initial management
Patients will present following a traumatic event with a painful, swollen ankle, and reluctance/inability to weight bear. The Ottawa rules can be applied to differentiate between an ankle fracture and sprain, but can be unreliable.
In high energy injuries, management should follow ATLS principles to identify more significant injuries first. Neurovascular status of the foot should be documented, and open injuries should be excluded. If an open injury is identified, it should be managed in line with BOAST 4 principles1. If an obvious deformity exists, it should be reduced as soon as possible with appropriate analgesia or conscious sedation. Radiographs of clearly deformed or dislocated joints are not necessary, and removing the pressure on the surrounding soft tissues from the underlying bony deformity is the priority. If the fracture pattern is not clinically obvious then plain radiographs are appropriate and will guide the subsequent manipulation during plaster-of-paris below knee backslab application.

Imaging
AP, lateral and mortise views (20o internal rotation) are essential to evaluate fracture displacement and syndesmotic injury. Decreased tibiofibular overlap, medial joint clear space and lateral talar shift all indicate a syndesmotic injury. (In subtle cases of shift, imaging the uninjured ankle can be helpful as a proportion of the population have little or no tibiotalar overlap 2.)

Where there is suspicion of syndesmosis involvement in the absence of radiographic evidence, stress radiographs can be diagnostic.
Complex fracture patterns (and increasingly posterior malleolar fractures) are best defined using CT.

Classification
The most commonly used classifications are Lauge-Hansen and Danis-Weber.

Lauge-Hansen
Comprises two parts: first part is the foot position, and the second part is the force applied. Useful for understanding the forces involved and therefore predict the ligamentous or bony injury. Results in four injury patterns:
Supination - Adduction (SA) - 10-20%
Supination - External rotation (SER) - 40-75%
Pronation - Abduction (PA) - 5-20%
Pronation - External rotation (PER) - 5-20%

Not often used in clinical practice but good for understanding the principles of ankle fracture.

Danis-Weber
Commonly used. Based on the level of the fibula fracture in relation to the syndesmosis. The more proximal, the greater the risk of syndesmotic injury and therefore fracture instability.
A - fracture below the level of the syndesmosis
B - fracture at the level of the syndesmosis / level of the tibial plafond
C - fracture above the level of the syndesmosis. This includes Maisonneuve fractures (proximal fibula fracture), which can be associated with ankle instability. Beware the high fibula fracture - it may be an ankle fracture!

The Weber classification is based purely on the the lateral side. All injuries can include a medial or posterior bony or ligamentous injury which also dictates fracture stability (bimalleolar and trimalleolar fractures are more unstable).

Treatment
When deciding upon treatment for an ankle fracture, one must consider both the fracture and the patient. Diabetic patients and smokers are at greater risk of post-operative complication, especially wound problems and infection. Likewise, the long term outcome of post-traumatic arthritis from a malunited ankle fracture is extremely important for a young patient, but not as relevant in the elderly. Therefore, normal surgical decision processes apply as with all fractures.

Defining stability of an ankle fracture underpins the treatment decision.
Weber A - Unimalleolar Weber A Weber fractures by definition are stable and therefore can be mobilised fully weight bearing in an ankle boot.

Weber C - Fractures tend to include syndesmotic disruption and are usually bimalleolar (either bony or ligamentous). They are therefore unstable and usually require operative fixation. In addition to the fracture fixation, the syndesmosis usually requires reconstruction/augmentation with screws to restore the joint integrity and function.

Weber B - B fractures vary greatly. They can be part of a trimalleolar injury and therefore extremely unstable, requiring fixation. Alternatively, a uni-malleolar Weber B fracture can be a stable injury, and therefore mobilised immediately in an ankle boot. Defining the stability can be challenging, and often involves stress radiographs, or a trial of mobilisation and repeat radiographs. Defining stability is the subject of much ongoing research. However, treating undisplaced ankle fractures in a below knee plaster, non-weight bearing for six weeks is still widely practised, and a safe approach.

When operative fixation is appropriate, it is usually via open reduction and internal fixation using plates and screws. It must be carried out when soft tissue swelling has settled in order to minimise the risk of wound problems. This can often take a week to settle.

The use of fibula nails is expanding, but is not yet mainstream. Ankle fractures can also be treated with external fixation, or with a hind foot nail in patients who need fixation but where soft tissue or bone quality is poor.

Post operative management
Ankle fractures generally take 6 weeks to unite enough to prevent secondary displacement. This is therefore an appropriate time period to keep a cast on in a conservatively managed patient. Weight bearing post-operatively depends on the quality of the fixation and bone quality, and preference varies between surgeons, ranging from aggressive early mobilisation to a period of non-weight bearing. Return to activities takes approximately three months, and often requires assistance of a physiotherapist to improve range-of-movement and muscle strengthening.

263
Q

A 60 year old female injures her ankle after a falling from a horse. On examination she has severe swelling and bruising of the ankle with x-rays demonstrating a comminuted intra-articular distal tibia fracture. The ankle has been temporarily reduced and splinted in the emergency department.

A. Surgical fixation
B. Below knee amputation
C. Application of below knee plaster
D. Application of ankle boot
E. Application of external fixation device
F. Application of compression dressing and physiotherapy
G. Immediate reduction and application of backslab

A

Application of external fixation device

This is a pilon fracture, which a high energy injury of the distal tibia. The patient will ultimately require surgical fixation but early management involves applying a spanning external fixator to temporarily reduce the fracture and allow soft tissue swelling to settle. A CT scan should then be performed to aid surgical planning.

Ankle injuries

An ankle fracture relates to a fracture around the tibio-talar joint. It generally refers to a fracture involving the lateral, and/or medial and/or posterior malleolus. Pilon and Tillaux fractures are also considered to be ankle fractures, but are not covered here.
Ankle fractures are common. They effect men and women in equal numbers, but men have a higher rate as young adults (sports and contact injuries), and women a higher rate post-menopausal (fragility type fracture).

Osseous anatomy
The ankle (or mortise) joint consists of the distal tibia (tibial plafond and posterior malleolus), the distal fibula (lateral malleolus), and the talus. The main movement at the ankle joint is plantar and dorsiflexion.

Ligamentous anatomy
Medial side: Deltoid ligament. This is divided into superficial and deep portions. It is the primary restraint to valgus tilting of the talus.

Lateral side: Lateral ligament complex consisting from anterior to posterior of the anterior talofibular ligament (ATFL), calcaneofibular ligament (CFL), and the posterior talofibular ligament (PTFL). Together they resist valgus stress to the ankle, and are a restraint to anterior translation of the talus within the mortise joint.

Syndesmosis: The syndesmosis is a ligament complex between the distal tibia and fibula, holding the two bones together. It is fundamental to the integrity of the ankle joint, and its disruption leads to instability. It consists of (from anterior to posterior) the anterior-inferior tibiofibular ligament (AITFL), the transverse tibiofibular ligament (TTFL), the interosseous membrane, and the posterior-inferior tibiofibular ligament (PITFL).

Presentation and initial management
Patients will present following a traumatic event with a painful, swollen ankle, and reluctance/inability to weight bear. The Ottawa rules can be applied to differentiate between an ankle fracture and sprain, but can be unreliable.
In high energy injuries, management should follow ATLS principles to identify more significant injuries first. Neurovascular status of the foot should be documented, and open injuries should be excluded. If an open injury is identified, it should be managed in line with BOAST 4 principles1. If an obvious deformity exists, it should be reduced as soon as possible with appropriate analgesia or conscious sedation. Radiographs of clearly deformed or dislocated joints are not necessary, and removing the pressure on the surrounding soft tissues from the underlying bony deformity is the priority. If the fracture pattern is not clinically obvious then plain radiographs are appropriate and will guide the subsequent manipulation during plaster-of-paris below knee backslab application.

Imaging
AP, lateral and mortise views (20o internal rotation) are essential to evaluate fracture displacement and syndesmotic injury. Decreased tibiofibular overlap, medial joint clear space and lateral talar shift all indicate a syndesmotic injury. (In subtle cases of shift, imaging the uninjured ankle can be helpful as a proportion of the population have little or no tibiotalar overlap 2.)

Where there is suspicion of syndesmosis involvement in the absence of radiographic evidence, stress radiographs can be diagnostic.
Complex fracture patterns (and increasingly posterior malleolar fractures) are best defined using CT.

Classification
The most commonly used classifications are Lauge-Hansen and Danis-Weber.

Lauge-Hansen
Comprises two parts: first part is the foot position, and the second part is the force applied. Useful for understanding the forces involved and therefore predict the ligamentous or bony injury. Results in four injury patterns:
Supination - Adduction (SA) - 10-20%
Supination - External rotation (SER) - 40-75%
Pronation - Abduction (PA) - 5-20%
Pronation - External rotation (PER) - 5-20%

Not often used in clinical practice but good for understanding the principles of ankle fracture.

Danis-Weber
Commonly used. Based on the level of the fibula fracture in relation to the syndesmosis. The more proximal, the greater the risk of syndesmotic injury and therefore fracture instability.
A - fracture below the level of the syndesmosis
B - fracture at the level of the syndesmosis / level of the tibial plafond
C - fracture above the level of the syndesmosis. This includes Maisonneuve fractures (proximal fibula fracture), which can be associated with ankle instability. Beware the high fibula fracture - it may be an ankle fracture!

The Weber classification is based purely on the the lateral side. All injuries can include a medial or posterior bony or ligamentous injury which also dictates fracture stability (bimalleolar and trimalleolar fractures are more unstable).

Treatment
When deciding upon treatment for an ankle fracture, one must consider both the fracture and the patient. Diabetic patients and smokers are at greater risk of post-operative complication, especially wound problems and infection. Likewise, the long term outcome of post-traumatic arthritis from a malunited ankle fracture is extremely important for a young patient, but not as relevant in the elderly. Therefore, normal surgical decision processes apply as with all fractures.

Defining stability of an ankle fracture underpins the treatment decision.
Weber A - Unimalleolar Weber A Weber fractures by definition are stable and therefore can be mobilised fully weight bearing in an ankle boot.

Weber C - Fractures tend to include syndesmotic disruption and are usually bimalleolar (either bony or ligamentous). They are therefore unstable and usually require operative fixation. In addition to the fracture fixation, the syndesmosis usually requires reconstruction/augmentation with screws to restore the joint integrity and function.

Weber B - B fractures vary greatly. They can be part of a trimalleolar injury and therefore extremely unstable, requiring fixation. Alternatively, a uni-malleolar Weber B fracture can be a stable injury, and therefore mobilised immediately in an ankle boot. Defining the stability can be challenging, and often involves stress radiographs, or a trial of mobilisation and repeat radiographs. Defining stability is the subject of much ongoing research. However, treating undisplaced ankle fractures in a below knee plaster, non-weight bearing for six weeks is still widely practised, and a safe approach.

When operative fixation is appropriate, it is usually via open reduction and internal fixation using plates and screws. It must be carried out when soft tissue swelling has settled in order to minimise the risk of wound problems. This can often take a week to settle.

The use of fibula nails is expanding, but is not yet mainstream. Ankle fractures can also be treated with external fixation, or with a hind foot nail in patients who need fixation but where soft tissue or bone quality is poor.

Post operative management
Ankle fractures generally take 6 weeks to unite enough to prevent secondary displacement. This is therefore an appropriate time period to keep a cast on in a conservatively managed patient. Weight bearing post-operatively depends on the quality of the fixation and bone quality, and preference varies between surgeons, ranging from aggressive early mobilisation to a period of non-weight bearing. Return to activities takes approximately three months, and often requires assistance of a physiotherapist to improve range-of-movement and muscle strengthening.

264
Q

A 18 year old man presents with an indirect inguinal hernia and undergoes surgery. The deep inguinal ring is exposed and held with a retractor at its medial aspect. Which structure is most likely to lie under the retractor?

	Ureter
	Inferior epigastric artery
	Internal iliac vein
	Femoral artery
	Lateral border of rectus abdominis
A

Boundaries of the deep inguinal ring:
Superolaterally - transversalis fascia
Inferomedially - inferior epigastric artery

The deep inguinal ring is closely related to the inferior epigastric artery. The inferior epigastric artery forms part of the structure referred to as Hesselbach’s triangle.

Inguinal canal

Location
Above the inguinal ligament
The inguinal canal is 4cm long
The superficial ring is located anterior to the pubic tubercle
The deep ring is located approximately 1.5-2cm above the half way point between the anterior superior iliac spine and the pubic tubercle

Boundaries of the inguinal canal
Floor	
External oblique aponeurosis
Inguinal ligament
Lacunar ligament
Roof	
Internal oblique
Transversus abdominis
Anterior wall	External oblique aponeurosis
Posterior wall	
Transversalis fascia
Conjoint tendon
Laterally	
Internal ring
Transversalis fascia
Fibres of internal oblique
Medially	
External ring
Conjoint tendon

Contents
Males Spermatic cord and ilioinguinal nerve As it passes through the canal the spermatic cord has 3 coverings:
External spermatic fascia from external oblique aponeurosis
Cremasteric fascia
Internal spermatic fascia
Females Round ligament of uterus and ilioinguinal nerve

265
Q

A 73 year old man presents with a tumour at the tip of his tongue. To which of the following regions will the tumour initially metastasise?

	Sub mental nodes
	Ipsilateral deep cervical nodes
	Tonsil
	Ipsilateral superficial cervical nodes
	Contralateral deep cervical nodes
A

Submental nodes

The lymphatic drainage of the anterior two thirds of the tongue shows only minimal communication of lymphatics across the midline, so metastasis to the ipsilateral nodes is usual.
The lymphatic drainage of the posterior third of the tongue have communicating networks, as a result early bilateral nodal metastases are more common in this area.
Lymphatics from the tip of the tongue usually pass to the sub mental nodes and from there to the deep cervical nodes.
Lymphatics from the mid portion of the tongue usually drain to the submandibular nodes and then to the deep cervical nodes. Mid tongue tumours that are laterally located will usually drain to the ipsilateral deep cervical nodes, those from more central regions may have bilateral deep cervical nodal involvement.

266
Q

A 78 year old man is undergoing a femoro-popliteal bypass graft. The operation is not progressing well and the surgeon is complaining of poor access. Retraction of which of the following structures will improve access to the femoral artery in the groin?

	Quadriceps
	Adductor longus
	Adductor magnus
	Pectineus
	Sartorius
A

At the lower border of the femoral triangle the femoral artery passes under the sartorius muscle. This can be retracted to improve access.

Femoral triangle anatomy

Boundaries
Superiorly	Inguinal ligament
Laterally	Sartorius
Medially	Adductor longus
Floor	Iliopsoas, adductor longus and pectineus
Roof	
Fascia lata and Superficial fascia
Superficial inguinal lymph nodes (palpable below the inguinal ligament)
Long saphenous vein

Contents
Femoral vein (medial to lateral)
Femoral artery-pulse palpated at the mid inguinal point
Femoral nerve
Deep and superficial inguinal lymph nodes
Lateral cutaneous nerve
Great saphenous vein
Femoral branch of the genitofemoral nerve

267
Q

A builder falls off a ladder whilst laying roof tiles. He sustains a burst fracture of L3. The MRI scan shows complete nerve transection at this level, as a result of the injury. Which clinical sign will not be present initially?

	Flaccid paralysis of the legs
	Extensor plantar response
	Sensory loss in the legs
	Incontinence
	Areflexia
A

The main purpose of this question is to differentiate the features of an UMN lesion and a LMN lesion. The features of a LMN lesion include:
Flaccid paralysis of muscles supplied
Atrophy of muscles supplied.
Loss of reflexes of muscles supplied.
Muscles fasciculation
For lesions below L1 LMN signs will occur. Hence in an L3 lesion, there will be loss of the patella reflex but there will be no extensor plantar reflex.

Spinal cord

Located in a canal within the vertebral column that affords it structural support.
Rostrally it continues to the medulla oblongata of the brain and caudally it tapers at a level corresponding to the L1-2 interspace (in the adult), a central structure, the filum terminale anchors the cord to the first coccygeal vertebra.
The spinal cord is characterised by cervico-lumbar enlargements and these, broadly speaking, are the sites which correspond to the brachial and lumbar plexuses respectively.

There are some key points to note when considering the surgical anatomy of the spinal cord:

  • During foetal growth the spinal cord becomes shorter than the spinal canal, hence the adult site of cord termination at the L1-2 level.
  • Due to growth of the vertebral column the spine segmental levels may not always correspond to bony landmarks as they do in the cervical spine.
  • The spinal cord is incompletely divided into two symmetrical halves by a dorsal median sulcus and ventral median fissure. Grey matter surrounds a central canal that is continuous rostrally with the ventricular system of the CNS.
  • The grey matter is sub divided cytoarchitecturally into Rexeds laminae.
  • Afferent fibres entering through the dorsal roots usually terminate near their point of entry but may travel for varying distances in Lissauers tract. In this way they may establish synaptic connections over several levels
  • At the tip of the dorsal horn are afferents associated with nociceptive stimuli. The ventral horn contains neurones that innervate skeletal muscle.

The key point to remember when revising CNS anatomy is to keep a clinical perspective in mind. So it is worth classifying the ways in which the spinal cord may become injured. These include:

Trauma either direct or as a result of disc protrusion
Neoplasia either by direct invasion (rare) or as a result of pathological vertebral fracture
Inflammatory diseases such as Rheumatoid disease, or OA (formation of osteophytes compressing nerve roots etc.
Vascular either as a result of stroke (rare in cord) or as complication of aortic dissection
Infection historically diseases such as TB, epidural abscesses.

The anatomy of the cord will, to an extent dictate the clinical presentation. Some points/ conditions to remember:

Brown- Sequard syndrome-Hemisection of the cord producing ipsilateral loss of proprioception and upper motor neurone signs, plus contralateral loss of pain and temperature sensation. The explanation of this is that the fibres decussate at different levels.
Lesions below L1 will tend to present with lower motor neurone signs

268
Q

A 56 year old machinist has his arm entrapped in a steel grinder and is brought to the emergency department. On examination, he is unable to extend his metacarpophalangeal joints and abduct his shoulder. He has weakness of his elbow and wrist. What has been injured?

	Ulnar nerve
	Axillary nerve
	Medial cord of brachial plexus
	Lateral cord of brachial plexus
	Posterior cord of brachial plexus
A

The posterior cord gives rise to:
Radial nerve ((innervates the triceps, brachioradialis, wrist extensors, and finger extensors)
Axillary nerve (innervates deltoid and teres minor)
Upper subscapular nerve (innervates subscapularis)
Lower subscapular nerve (innervates teres major and subscapularis)
Thoracodorsal nerve (innervates latissimus dorsi)

This is a description of a posterior cord lesion. Remember that the posterior cord gives rise to the axillary and radial nerve.

Cords of the brachial plexus

The brachial plexus cords are described according to their relationship with the axillary artery. The cords pass over the 1st rib near to the dome of the lung and pass beneath the clavicle immediately posterior to the subclavian artery.

Lateral cord
Anterior divisions of the upper and middle trunks form the lateral cord
Origin of the lateral pectoral nerve (C5, C6, C7)
Medial cord
Anterior division of the lower trunk forms the medial cord
Origin of the medial pectoral nerve (C8, T1), the medial brachial cutaneous nerve (T1), and the medial antebrachial cutaneous nerve (C8, T1)
Posterior cord
Formed by the posterior divisions of the 3 trunks (C5-T1)
Origin of the upper and lower subscapular nerves (C7, C8 and C5, C6, respectively) and the thoracodorsal nerve to the latissimus dorsi (also known as the middle subscapular nerve, C6, C7, C8), axillary and radial nerve

269
Q

A 66 year old man with peripheral vascular disease is undergoing a below knee amputation. In which of the lower leg compartments does peroneus brevis lie?

	Lateral compartment
	Anterior compartment
	Superficial posterior compartment
	Deep posterior compartment
	None of the above
A

Lateral compartment

The interosseous membrane separates the anterior and posterior compartments. The deep and superficial compartments are separated by the deep transverse fascia. The peroneus brevis is part of the lateral compartment.

Compartments of the thigh

Formed by septae passing from the femur to the fascia lata:
Anterior compartment	Femoral	
Iliacus
Tensor fasciae latae
Sartorius
Quadriceps femoris
Femoral artery
Medial compartment	Obturator	
Adductor longus/magnus/brevis
Gracilis
Obturator externus
Profunda femoris artery and obturator artery
Posterior compartment (2 layers)	Sciatic	
Semimembranosus
Semitendinosus
Biceps femoris
Branches of Profunda femoris artery

Compartments of the lower leg
Separated by the interosseous membrane (anterior and posterior compartments), anterior fascial septum (separate anterior and lateral compartments) and posterior fascial septum (separate lateral and posterior compartments):

Anterior compartment	Deep peroneal nerve	
Tibialis anterior
Extensor digitorum longus
Extensor hallucis longus
Peroneus tertius
Anterior tibial artery

Posterior compartment Tibial
Muscles: deep and superficial compartments (separated by deep transverse fascia)

Deep: Flexor hallucis longus, Flexor digitalis longus, Tibialis posterior, Popliteus
Superficial: Gastrocnemius, Soleus, Plantaris

Posterior tibial
Lateral compartment Superficial peroneal
Peroneus longus/brevis
Peroneal artery

270
Q

A 70 year old man is due to undergo an arterial bypass procedure for claudication and foot ulceration. The anterior tibial artery will form the site of the distal arterial anastomosis. Which of the following structures is not closely related to it distally?

	Interosseous membrane
	Deep peroneal nerve
	Tibialis posterior
	Extensor hallucis longus
	Dorsalis pedis artery
A

Tibialis posterior

As an artery of the anterior compartment, the anterior tibial artery is closely related to tibialis anterior. The tibialis posterior is related to it at its origin.

Anterior tibial artery

Begins opposite the distal border of popliteus
Terminates in front of the ankle, continuing as the dorsalis pedis artery
As it descends it lies on the interosseous membrane, distal part of the tibia and front of the ankle joint
Passes between the tendons of extensor digitorum and extensor hallucis longus distally
It is related to the deep peroneal nerve, it lies anterior to the middle third of the vessel and lateral to it in the lower third

271
Q

Which of the muscles below does not cause lateral rotation of the hip?

	Obturator internus
	Quadratus femoris
	Gemellus inferior
	Piriformis
	Pectineus
A

Pectineus is not a lateral rotator. Pectineus adducts and medially rotates the femur.

Mnemonic lateral hip rotators: P-GO-GO-Q (top to bottom)

Piriformis
Gemellus superior
Obturator internus
Gemellus inferior
Obturator externus
Quadratus femoris

Hip joint

Head of femur articulates with acetabulum of the pelvis
Both covered by articular hyaline cartilage
The acetabulum forms at the union of the ilium, pubis, and ischium
The triradiate cartilage (Y-shaped growth plate) separates the pelvic bones
The acetabulum holds the femoral head by the acetabular labrum
Normal angle between femoral head and femoral shaft is 130o

Ligaments
Transverse ligament: joints anterior and posterior ends of the articular cartilage
Head of femur ligament (ligamentum teres): acetabular notch to the fovea. Contains arterial supply to head of femur in children.

Extracapsular ligaments
Iliofemoral ligament: inverted Y shape. Anterior iliac spine to the trochanteric line
Pubofemoral ligament: acetabulum to lesser trochanter
Ischiofemoral ligament: posterior support. Ischium to greater trochanter.

Blood supply
Medial circumflex femoral and lateral circumflex femoral arteries (Branches of profunda femoris). Also from the inferior gluteal artery. These form an anastomosis and travel to up the femoral neck to supply the head.

272
Q

Which of the following is not a content of the posterior triangle of the neck?

	Spinal accessory nerve
	Phrenic nerve
	External jugular vein
	Occipital lymph nodes
	Internal jugular vein
A

The IJV does not lie in the posterior triangle. However, the terminal branches of the external jugular vein do.

Posterior triangle of the neck

Boundaries
Apex Sternocleidomastoid and the Trapezius muscles at the Occipital bone
Anterior Posterior border of the Sternocleidomastoid
Posterior Anterior border of the Trapezius
Base Middle third of the clavicle

Contents
Nerves	
Accessory nerve
Phrenic nerve
Three trunks of the brachial plexus
Branches of the cervical plexus: Supraclavicular nerve, transverse cervical nerve, great auricular nerve, lesser occipital nerve
Vessels	
External jugular vein
Subclavian artery
Muscles	
Inferior belly of omohyoid
Scalene
Lymph nodes	
Supraclavicular
Occipital
273
Q

An obese 12 year old boy presents with knee pain. On examination he has pain on internal rotation of the hip. His knee is clinically normal.

A.	USS hip
B.	Hip x-ray
C.	Anteroposterior pelvic x-ray
D.	CT scan
E.	Discharge and reassure
F.	MRI
G.	USS knee
H.	X-ray knee
A

Hip x-ray

The main differential diagnosis in a boy over 10 years old is of slipped upper femoral epiphysis. Knee pain is a common presenting feature. An anteroposterior pelvic x-ray may miss a minor slip, therefore request a hip film.

Developmental dysplasia of the hip Usually diagnosed in infancy by screening tests. May be bilateral, when disease is unilateral there may be leg length inequality. As disease progresses child may limp and then early onset arthritis. More common in extended breech babies. Splints and harnesses or traction. In later years osteotomy and hip realignment procedures may be needed. In arthritis a joint replacement may be needed. However, this is best deferred if possible as it will almost certainly require revision Initially no obvious change on plain films and USS gives best resolution until 3 months of age. On plain films Shentons line should form a smooth arc
Perthes Disease Hip pain (may be referred to the knee) usually occurring between 5 and 12 years of age. Bilateral disease in 20%. Remove pressure from joint to allow normal development. Physiotherapy. Usually self-limiting if diagnosed and treated promptly. X-rays will show flattened femoral head. Eventually in untreated cases the femoral head will fragment.
Slipped upper femoral epiphysis Typically seen in obese male adolescents. Pain is often referred to the knee. Limitation to internal rotation is usually seen. Knee pain is usually present 2 months prior to hip slipping. Bilateral in 20%. Bed rest and non-weight bearing. Aim to avoid avascular necrosis. If severe slippage or risk of it occurring then percutaneous pinning of the hip may be required. X-rays will show the femoral head displaced and falling inferolaterally (like a melting ice cream cone) The Southwick angle gives indication of disease severity

274
Q

A baby is delivered in the breech position. Barlows and Ortolani tests are normal

A.	USS hip
B.	Hip x-ray
C.	Anteroposterior pelvic x-ray
D.	CT scan
E.	Discharge and reassure
F.	MRI
G.	USS knee
H.	X-ray knee
A

USS hip

This child is at risk of developmental dysplasia of the hip (up to 20% will have DDH), so should have the hip joints scanned to exclude this.

Developmental dysplasia of the hip Usually diagnosed in infancy by screening tests. May be bilateral, when disease is unilateral there may be leg length inequality. As disease progresses child may limp and then early onset arthritis. More common in extended breech babies. Splints and harnesses or traction. In later years osteotomy and hip realignment procedures may be needed. In arthritis a joint replacement may be needed. However, this is best deferred if possible as it will almost certainly require revision Initially no obvious change on plain films and USS gives best resolution until 3 months of age. On plain films Shentons line should form a smooth arc
Perthes Disease Hip pain (may be referred to the knee) usually occurring between 5 and 12 years of age. Bilateral disease in 20%. Remove pressure from joint to allow normal development. Physiotherapy. Usually self-limiting if diagnosed and treated promptly. X-rays will show flattened femoral head. Eventually in untreated cases the femoral head will fragment.
Slipped upper femoral epiphysis Typically seen in obese male adolescents. Pain is often referred to the knee. Limitation to internal rotation is usually seen. Knee pain is usually present 2 months prior to hip slipping. Bilateral in 20%. Bed rest and non-weight bearing. Aim to avoid avascular necrosis. If severe slippage or risk of it occurring then percutaneous pinning of the hip may be required. X-rays will show the femoral head displaced and falling inferolaterally (like a melting ice cream cone) The Southwick angle gives indication of disease severity

275
Q

A 5 year old boy presents with a painful limp. The symptoms have been present for 8 weeks. Two hip x-rays have been performed and appear normal.

A.	USS hip
B.	Hip x-ray
C.	Anteroposterior pelvic x-ray
D.	CT scan
E.	Discharge and reassure
F.	MRI
G.	USS knee
H.	X-ray knee
A

MRI

Perthes disease should be suspected in boys over 4 years old presenting with a limp. Early disease can be missed on x-ray. An MRI will often demonstrate areas of hypoperfusion and subtle changes that allow for earlier diagnosis. A bone scan is an alternative option.

Developmental dysplasia of the hip Usually diagnosed in infancy by screening tests. May be bilateral, when disease is unilateral there may be leg length inequality. As disease progresses child may limp and then early onset arthritis. More common in extended breech babies. Splints and harnesses or traction. In later years osteotomy and hip realignment procedures may be needed. In arthritis a joint replacement may be needed. However, this is best deferred if possible as it will almost certainly require revision Initially no obvious change on plain films and USS gives best resolution until 3 months of age. On plain films Shentons line should form a smooth arc
Perthes Disease Hip pain (may be referred to the knee) usually occurring between 5 and 12 years of age. Bilateral disease in 20%. Remove pressure from joint to allow normal development. Physiotherapy. Usually self-limiting if diagnosed and treated promptly. X-rays will show flattened femoral head. Eventually in untreated cases the femoral head will fragment.
Slipped upper femoral epiphysis Typically seen in obese male adolescents. Pain is often referred to the knee. Limitation to internal rotation is usually seen. Knee pain is usually present 2 months prior to hip slipping. Bilateral in 20%. Bed rest and non-weight bearing. Aim to avoid avascular necrosis. If severe slippage or risk of it occurring then percutaneous pinning of the hip may be required. X-rays will show the femoral head displaced and falling inferolaterally (like a melting ice cream cone) The Southwick angle gives indication of disease severity

276
Q

Which nerve is at risk during submandibular gland excision?

	Maxillary nerve
	Buccal nerve
	Zygomatic nerve
	Marginal mandibular nerve
	Cervical nerve
A

The marginal mandibular nerve lies deep to platysma. It supplies the depressor anguli oris and the depressor labii inferioris. If injured it may lead to facial asymmetry and dribbling.

Relations of the submandibular gland
Superficial	Platysma, deep fascia and mandible
Submandibular lymph nodes
Facial vein (facial artery near mandible)
Marginal mandibular nerve
Cervical branch of the facial nerve
Deep	Facial artery (inferior to the mandible)
Mylohyoid muscle
Sub mandibular duct
Hyoglossus muscle
Lingual nerve
Submandibular ganglion
Hypoglossal nerve

Submandibular duct (Wharton’s duct)
Opens lateral to the lingual frenulum on the anterior floor of mouth.
5 cm length
Lingual nerve wraps around Wharton’s duct. As the duct passes forwards it crosses medial to the nerve to lie above it and then crosses back, lateral to it, to reach a position below the nerve.

Innervation
Sympathetic innervation- Derived from superior cervical ganglion
Parasympathetic innervation- Submandibular ganglion via lingual nerve

Arterial supply
Branch of the facial artery. The facial artery passes through the gland to groove its deep surface. It then emerges onto the face by passing between the gland and the mandible.

Venous drainage
Anterior facial vein (lies deep to the Marginal Mandibular nerve)

Lymphatic drainage
Deep cervical and jugular chains of nodes

277
Q

In a patient with a carcinoma of the distal sigmoid colon, what is the most likely source of its blood supply?

	Ileocolic artery
	External iliac artery
	Internal iliac artery
	Superior mesenteric artery
	Inferior mesenteric artery
A

IMA

During a high anterior resection of such tumours, the inferior mesenteric artery is ligated. Note that the branches (mainly middle rectal branch) of the internal iliac artery are important in maintaining vascularity of the rectal stump and hence the integrity of the anastomoses.

Rectum

The rectum is approximately 12 cm long. It is a capacitance organ. It has both intra and extraperitoneal components. The transition between the sigmoid colon is marked by the disappearance of the tenia coli.The extra peritoneal rectum is surrounded by mesorectal fat that also contains lymph nodes. This mesorectal fatty layer is removed surgically during rectal cancer surgery (Total Mesorectal Excision). The fascial layers that surround the rectum are important clinical landmarks, anteriorly lies the fascia of Denonvilliers. Posteriorly lies Waldeyers fascia.

Extra peritoneal rectum
Posterior upper third
Posterior and lateral middle third
Whole lower third

Relations
Anteriorly (Males)	Rectovesical pouch
Bladder
Prostate
Seminal vesicles
Anteriorly (Females)	Recto-uterine pouch (Douglas)
Cervix
Vaginal wall
Posteriorly	Sacrum
Coccyx 
Middle sacral artery
Laterally	Levator ani
Coccygeus

Arterial supply
Superior rectal artery

Venous drainage
Superior rectal vein

Lymphatic drainage
Mesorectal lymph nodes (superior to dentate line)
Inguinal nodes (inferior to dentate line)

278
Q

Which of these openings transmits the facial nerve into the temporal bone?

	Internal acoustic meatus
	Foramen lacerum
	Foramen spinosum
	Stylomastoid foramen
	Jugular foramen
A

It enters the temporal bone through the internal acoustic meatus and exits through the stylomastoid foramen.

The facial nerve is the main nerve supplying the structures of the second embryonic branchial arch. It is predominantly an efferent nerve to the muscles of facial expression, digastric muscle and also to many glandular structures. It contains a few afferent fibres which originate in the cells of its genicular ganglion and are concerned with taste.

Supply - ‘face, ear, taste, tear’
Face: muscles of facial expression
Ear: nerve to stapedius
Taste: supplies anterior two-thirds of tongue
Tear: parasympathetic fibres to lacrimal glands, also salivary glands

Path
Subarachnoid path
Origin: motor- pons, sensory- nervus intermedius
Pass through the petrous temporal bone into the internal auditory meatus with the vestibulocochlear nerve. Here they combine to become the facial nerve.

Facial canal path
The canal passes superior to the vestibule of the inner ear
At the medial aspect of the middle ear, it becomes wider and contains the geniculate ganglion.
- 3 branches:
1. greater petrosal nerve
2. nerve to stapedius
3. chorda tympani

Stylomastoid foramen
Passes through the stylomastoid foramen (tympanic cavity anterior and mastoid antrum posteriorly)
Posterior auricular nerve and branch to posterior belly of digastric and stylohyoid muscle

Face
Enters parotid gland and divides into 5 branches: 
Temporal branch
Zygomatic branch
Buccal branch
Marginal mandibular branch
Cervical branch
279
Q

A motor cyclist is involved in a road traffic accident causing severe right shoulder injuries. He is found to have an adducted, medially rotated shoulder. The elbow is fully extended and the forearm pronated. Which is the most likely diagnosis?

	C8, T1 root lesion
	C5, C6 root lesion
	Radial nerve lesion
	Ulnar nerve lesion
	Axillary nerve lesion
A

Erbs Palsy C5, C6 lesion
The features include:
Waiter’s tip position
Loss of shoulder abduction (deltoid and supraspinatus paralysis)
Loss of external rotation of the shoulder (paralysis of infraspinatus)
Loss of elbow flexion (paralysis of biceps, brachialis and brachioradialis)
Loss of forearm supination (paralysis of Biceps)

The motorcyclist has had an Erb’s palsy (C5, C6 root lesion). This is commonly known to be associated with birth injury when a baby has a shoulder dystocia.

Brachial plexus

Origin Anterior rami of C5 to T1
Sections of the plexus
Roots, trunks, divisions, cords, branches
Mnemonic:Real Teenagers Drink Cold Beer
Roots
Located in the posterior triangle
Pass between scalenus anterior and medius
Trunks
Located posterior to middle third of clavicle
Upper and middle trunks related superiorly to the subclavian artery
Lower trunk passes over 1st rib posterior to the subclavian artery
Divisions Apex of axilla
Cords Related to axillary artery

280
Q

A patient is due to undergo a right hemicolectomy for a carcinoma of the caecum. Which of the following vessels will require high ligation to provide optimal oncological control?

	Middle colic artery
	Inferior mesenteric artery
	Superior mesenteric artery
	Ileo-colic artery
	None of the above
A

The ileo - colic artery supplies the caecum and would require high ligation during a right hemicolectomy. The middle colic artery should generally be preserved when resecting a caecal lesion.
This question is essentially asking you to name the vessel supplying the caecum. The SMA does not directly supply the caecum, it is the ileocolic artery which does this.

Caecum

Location	
Proximal right colon below the ileocaecal valve
Intraperitoneal
Posterior relations	
Psoas
Iliacus
Femoral nerve
Genitofemoral nerve
Gonadal vessels
Anterior relations	Greater omentum
Arterial supply	Ileocolic artery
Lymphatic drainage	Mesenteric nodes accompany the venous drainage

The caecum is the most distensible part of the colon and in complete large bowel obstruction with a competent ileocaecal valve the most likely site of eventual perforation.

281
Q

A 40-year-old man presents with pain in his lower back and ‘sciatica’ for the past three days. He describes bending down to pick up a washing machine when he felt ‘something go’. He now has severe pain radiating from his back down the right leg. On examination he describes paraesthesia over the anterior aspect of the right knee and the medial aspect of his calf. Power is intact and the right knee reflex is diminished. The femoral stretch test is positive on the right side. Which nerve or nerve root is most likely to be affected?

	Common peroneal nerve
	Lateral cutaneous nerve of the thigh
	L5
	L3
	L1
A

L3

Prolapsed disc

A prolapsed lumbar disc usually produces clear dermatomal leg pain associated with neurological deficits.

Features
Leg pain usually worse than back
Pain often worse when sitting

The table below demonstrates the expected features according to the level of compression:

L3 nerve root compression Sensory loss over anterior thigh/knee
Weak quadriceps
Reduced knee reflex
Positive femoral stretch test
L5 nerve root compression Sensory loss dorsum of foot
Weakness in foot and big toe dorsiflexion
Reflexes intact
Positive sciatic nerve stretch test
S1 nerve root compression Sensory loss posterolateral aspect of leg and lateral aspect of foot
Weakness in plantar flexion of foot
Reduced ankle reflex
Positive sciatic nerve stretch test

Management
Similar to that of other musculoskeletal lower back pain: analgesia, physiotherapy, exercises
Persistent symptoms, muscular weakness, bladder or bowel dysfunction are indications for urgent MRI scanning to delineate the disease extent to allow surgical planning
Plain spinal x-rays have no useful role in establishing the extent of disk disease

282
Q

A 72 year old man is undergoing a repair of an abdominal aortic aneurysm. The aorta is cross clamped both proximally and distally. The proximal clamp is applied immediately inferior to the renal arteries. Both common iliac arteries are clamped distally. A longitudinal aortotomy is performed. After evacuating the contents of the aneurysm sac a significant amount of ongoing bleeding is encountered. This is most likely to originate from:

	The coeliac axis
	Testicular artery
	Splenic artery
	Superior mesenteric artery
	Lumbar arteries
A

The lumbar arteries are posteriorly sited and are a common cause of back bleeding during aortic surgery. The other vessels cited all exit the aorta in the regions that have been cross clamped.

283
Q

Which of the following statements relating to sartorius is untrue?

It is supplied by the femoral nerve
It forms the lateral boundary of the femoral triangle
The middle third forms the roof of the adductor canal
It is a flexor of the hip and knee
It inserts into the medial femoral condyle
A

It inserts into the medial aspect of the upper part of the tibia.

Sartorius

Longest strap muscle in the body
Most superficial muscle in the anterior compartment of the thigh

Origin Anterior superior iliac spine
Insertion Medial surface of the of the body of the tibia (upper part). It inserts anterior to gracilis and semitendinosus
Nerve Supply Femoral nerve (L2,3)
Action
Flexor of the hip and knee, slight abducts the thigh and rotates it laterally
It assists with medial rotation of the tibia on the femur. For example it would play a pivotal role in placing the right heel onto the left knee ( and vice versa)
Important relations The middle third of this muscle, and its strong underlying fascia forms the roof of the adductor canal , in which lie the femoral vessels, the saphenous nerve and the nerve to vastus medialis.

284
Q

A 63 year old man undergoes a radical cystectomy for carcinoma of the bladder. During the procedure there is considerable venous bleeding. What is the primary site of venous drainage of the urinary bladder?

	Vesicoprostatic venous plexus
	Internal iliac vein
	External iliac vein
	Gonadal vein
	Common iliac vein
A

The urinary bladder has a rich venous plexus surrounding it, this drains subsequently into the internal iliac vein. The vesicoprostatic plexus may be a site of considerable venous bleeding during cystectomy.

Bladder

The empty bladder is contained within the pelvic cavity. It is usually a three sided pyramid. The apex of the bladder points forwards towards the symphysis pubis and the base lies immediately anterior to the rectum or vagina. Continuous with the apex is the median umbilical ligament, during development this was the site of the urachus.
The inferior aspect of the bladder is retroperitoneal and the superior aspect covered by peritoneum. As the bladder distends it will tend to separate the peritoneum from the fascia of transversalis. For this reason a bladder that is distended due to acute urinary retention may be approached with a suprapubic catheter that avoids entry into the peritoneal cavity.
The trigone is the least mobile part of the bladder and forms the site of the ureteric orifices and internal urethral orifice. In the empty bladder the ureteric orifices are approximately 2-3cm apart, this distance may increase to 5cm in the distended bladder.

Arterial supply
The superior and inferior vesical arteries provide the main blood supply to the bladder. These are branches of the internal iliac artery.

Venous drainage
In males the bladder is drained by the vesicoprostatic venous plexus. In females the bladder is drained by the vesicouterine venous plexus. In both sexes this venous plexus will ultimately drain to the internal iliac veins.

Lymphatic drainage
Lymphatic drainage is predominantly to the external iliac nodes, internal iliac and obturator nodes also form sites of bladder lymphatic drainage.

Innervation
Parasympathetic nerve fibres innervate the bladder from the pelvic splanchnic nerves. Sympathetic nerve fibres are derived from L1 and L2 via the hypogastric nerve plexuses. The parasympathetic nerve fibres will typically cause detrusor muscle contraction and result in voiding. The muscle of the trigone is innervated by the sympathetic nervous system. The external urethral sphincter is under conscious control. During bladder filling the rate of firing of nerve impulses to the detrusor muscle is low and receptive relaxation occurs. At higher volumes and increased intra vesical pressures the rate of neuronal firing will increase and eventually voiding will occur.

285
Q

A man sustains a laceration between the base of the little finger and wrist. Several weeks after the injury there is loss of thumb adduction power. Which nerve is most likely to have been injured?

	Superficial ulnar nerve
	Deep ulnar nerve
	Median nerve
	Radial nerve
	Recurrent branch of median nerve
A

Deep ulnar nerve

Branches of the ulnar nerve in the wrist and hand
At the wrist the ulnar nerve divides into superficial and deep branches. The superficial branch lies deep to the palmaris brevis. It divides into two; to produce digital nerves, which innervate the skin of the medial third of the palm and the palmar surface of one and a half fingers.
The deep branch arises from the nerve on the flexor retinaculum lateral to the pisiform bone. It passes posteriorly between the abductor and short flexor of the little finger supplying them, and supplying and piercing the opponens digiti minimi near its origin from the flexor retinaculum, turns laterally over the distal surface of the Hook of the Hamate bone. It eventually passes between the two heads of adductor pollicis with the deep palmar arch and ends in the first dorsal interosseous muscle. In the palm the deep branch also innervates the lumbricals and interosseous muscles.

286
Q

A 60 year old female is undergoing a Whipples procedure for adenocarcinoma of the pancreas. As the surgeons begin to mobilise the pancreatic head they identify a large vessel passing inferiorly over the anterior aspect of the uncinate process. What is it likely to be?

	Superior mesenteric artery
	Coeliac axis
	Inferior mesenteric artery
	Aorta
	Left gastric artery
A

The superior mesenteric artery arises from the aorta and passes anterior to the lower part of the pancreas. Invasion of this structure is a relative contra indication to resectional surgery.

The pancreas is a retroperitoneal organ and lies posterior to the stomach. It may be accessed surgically by dividing the peritoneal reflection that connects the greater omentum to the transverse colon. The pancreatic head sits in the curvature of the duodenum. Its tail lies close to the hilum of the spleen, a site of potential injury during splenectomy.

Relations
Posterior to the pancreas
Pancreatic head	Inferior vena cava
Common bile duct 
Right and left renal veins 
Superior mesenteric vein and artery
Pancreatic neck	Superior mesenteric vein, portal vein
Pancreatic body-	Left renal vein
Crus of diaphragm
Psoas muscle
Adrenal gland
Kidney 
Aorta
Pancreatic tail	Left kidney
Anterior to the pancreas
Pancreatic head	1st part of the duodenum
Pylorus
Gastroduodenal artery
SMA and SMV(uncinate process)
Pancreatic body	Stomach
Duodenojejunal flexure
Pancreatic tail	Splenic hilum

Superior to the pancreas
Coeliac trunk and its branches common hepatic artery and splenic artery

Grooves of the head of the pancreas
2nd and 3rd part of the duodenum

Arterial supply
Head: pancreaticoduodenal artery
Rest: splenic artery

Venous drainage
Head: superior mesenteric vein
Body and tail: splenic vein

Ampulla of Vater
Merge of pancreatic duct and common bile duct
Is an important landmark, halfway along the second part of the duodenum, that marks the anatomical transition from foregut to midgut (also the site of transition between regions supplied by coeliac trunk and SMA).

287
Q

A 23 year old man has a cannula inserted into his cephalic vein. Through which structure does the cephalic vein pass?

	Interosseous membrane
	Triceps
	Pectoralis major
	Clavipectoral fascia
	Tendon of biceps
A

The cephalic vein is a favored vessel for arteriovenous fistula formation and should be preserved in patients with end stage renal failure

The cephalic vein penetrates the calvipectoral fascia (but not the pectoralis major) prior to terminating in the axillary vein.

Cephalic vein

Path
Dorsal venous arch drains laterally into the cephalic vein
Crosses the anatomical snuffbox and travels laterally up the arm
At the antecubital fossa connected to the basilic vein by the median cubital vein
Pierces deep fascia of deltopectoral groove to join axillary vein

288
Q

An 18 year old man is undergoing an orchidectomy via a scrotal approach. The surgeons mobilise the spermatic cord. From which of the following is the outermost layer of this structure derived?

	Internal oblique aponeurosis
	External oblique aponeurosis
	Transversalis fascia
	Rectus sheath
	Campers fascia
A

The outermost covering of the spermatic cord is derived from the external oblique aponeurosis.This layer is added as the cord passes through the superficial inguinal ring.

Spermatic cord
Formed by the vas deferens and is covered by the following structures:
Layer Origin
Internal spermatic fascia Transversalis fascia
Cremasteric fascia From the fascial coverings of internal oblique
External spermatic fascia External oblique aponeurosis

Contents of the cord
Vas deferens Transmits sperm and accessory gland secretions
Testicular artery Branch of abdominal aorta supplies testis and epididymis
Artery of vas deferens Arises from inferior vesical artery
Cremasteric artery Arises from inferior epigastric artery
Pampiniform plexus Venous plexus, drains into right or left testicular vein
Sympathetic nerve fibres Lie on arteries, the parasympathetic fibres lie on the vas
Genital branch of the genitofemoral nerve Supplies cremaster
Lymphatic vessels Drain to lumbar and para-aortic nodes

Scrotum
Composed of skin and closely attached dartos fascia.
Arterial supply from the anterior and posterior scrotal arteries
Lymphatic drainage to the inguinal lymph nodes
Parietal layer of the tunica vaginalis is the innermost layer

Testes
The testes are surrounded by the tunica vaginalis (closed peritoneal sac). The parietal layer of the tunica vaginalis adjacent to the internal spermatic fascia.
The testicular arteries arise from the aorta immediately inferiorly to the renal arteries.
The pampiniform plexus drains into the testicular veins, the left drains into the left renal vein and the right into the inferior vena cava.
Lymphatic drainage is to the para-aortic nodes.

289
Q

Which of the following is not a carpal bone?

	Trapezium
	Triquetrum
	Trapezoid
	Trapezius
	Lunate
A

Mnemonic for the Carpal Bones

Sally Likes To Play The Tiny Chrome Harmonica
She Looks Too Pretty Try To Catch Her
Scared Lovers Try Positions That They Can’t Handle

Trapezius is a muscle of the back.
No tendons attach to: Scaphoid, lunate, triquetrum (stabilised by ligaments)
The wrist is comprised of 8 carpal bones, these are arranged in two rows of 4. It is convex from side to side posteriorly and concave anteriorly.

290
Q

A 53 year old male presents with a carcinoma of the transverse colon. Which of the following structures should be ligated close to their origin to maximise clearance of the tumour?

	Superior mesenteric artery
	Inferior mesenteric artery
	Middle colic artery
	Ileo-colic artery
	Superior rectal artery
A

The middle colic artery supplies the transverse colon and requires high ligation during cancer resections. It is a branch of the superior mesenteric artery.
Transverse colon

The right colon undergoes a sharp turn at the level of the hepatic flexure to become the transverse colon.
At this point it also becomes intraperitoneal.
It is connected to the inferior border of the pancreas by the transverse mesocolon.
The greater omentum is attached to the superior aspect of the transverse colon from which it can easily be separated. The mesentery contains the middle colic artery and vein. The greater omentum remains attached to the transverse colon up to the splenic flexure. At this point the colon undergoes another sharp turn.

Relations
Superior Liver and gall-bladder, the greater curvature of the stomach, and the lower end of the spleen
Inferior Small intestine
Anterior Greater omentum
Posterior From right to left with the descending portion of the duodenum, the head of the pancreas, convolutions of the jejunum and ileum, spleen

291
Q

Which of the following structures are not closely related to the adductor longus muscle?

	Long saphenous vein
	Tendon of iliacus
	The profunda branch of the femoral artery
	Pectineus muscle
	Femoral nerve
A

Femoral triangle:
Adductor longus medially
Inguinal ligament superiorly
Sartorius muscle laterally

Adductor longus forms the medial border of the femoral triangle. It is closely related to the long saphenous vein which overlies it and the profunda branch of the femoral artery. The femoral nerve is related to it inferiorly. However, the tendon of iliacus inserts proximally and is not in contact with adductor longs.

Adductor longus

Origin Anterior body of pubis
Insertion Middle third of linea aspera
Action Adducts and flexes the thigh, medially rotate the hip
Innervation Anterior division of obturator nerve (L2, L3, L4)

292
Q

Which of the following structures does not lie posterior to the right kidney?

	Psoas major
	12th rib
	Quadratus lumborum
	Medial arcuate ligament
	10th rib
A

The 8th and10th ribs lie more superiorly. The 12th rib is a closer relation posteriorly.
Quadratus lumborum runs between the posterior part of the iliac crest, iliolumbar ligament and the transverse processes of the lower lumbar vertebrae to the medial part of the lower border of the last rib and transverse process of the upper lumbar vertebrae. In these last two locations it is posterior to the kidney.

Renal anatomy

Each kidney is about 11cm long, 5cm wide and 3cm thick. They are located in a deep gutter alongside the projecting vertebral bodies, on the anterior surface of psoas major. In most cases the left kidney lies approximately 1.5cm higher than the right. The upper pole of both kidneys approximates with the 11th rib (beware pneumothorax during nephrectomy). On the left hand side the hilum is located at the L1 vertebral level and the right kidney at level L1-2. The lower border of the kidneys is usually alongside L3.

Fascial covering
Each kidney and suprarenal gland is enclosed within a common layer of investing fascia, derived from the transversalis fascia. It is divided into anterior and posterior layers (Gerotas fascia).

Renal structure
Kidneys are surrounded by an outer cortex and an inner medulla which usually contains between 6 and 10 pyramidal structures. The papilla marks the innermost apex of these. They terminate at the renal pelvis, into the ureter.
Lying in a hollow within the kidney is the renal sinus. This contains:
1. Branches of the renal artery
2. Tributaries of the renal vein
3. Major and minor calyces’s
4. Fat

Structures at the renal hilum
The renal vein lies most anteriorly, then renal artery (it is an end artery) and the ureter lies most posterior.

293
Q

A 56 year old man is undergoing a radical nephrectomy via a posterior approach. Which of the following structures is most likely to be encountered during the operative approach?

	8th rib
	10th rib
	6th rib
	12th rib
	9th rib
A

The 11th and 12th ribs lie posterior to the kidneys and may be encountered during a posterior approach. A pneumothorax is a recognised complication of this type of surgery.

Each kidney is about 11cm long, 5cm wide and 3cm thick. They are located in a deep gutter alongside the projecting vertebral bodies, on the anterior surface of psoas major. In most cases the left kidney lies approximately 1.5cm higher than the right. The upper pole of both kidneys approximates with the 11th rib (beware pneumothorax during nephrectomy). On the left hand side the hilum is located at the L1 vertebral level and the right kidney at level L1-2. The lower border of the kidneys is usually alongside L3.

Fascial covering
Each kidney and suprarenal gland is enclosed within a common layer of investing fascia, derived from the transversalis fascia. It is divided into anterior and posterior layers (Gerotas fascia).

Renal structure
Kidneys are surrounded by an outer cortex and an inner medulla which usually contains between 6 and 10 pyramidal structures. The papilla marks the innermost apex of these. They terminate at the renal pelvis, into the ureter.
Lying in a hollow within the kidney is the renal sinus. This contains:
1. Branches of the renal artery
2. Tributaries of the renal vein
3. Major and minor calyces’s
4. Fat

Structures at the renal hilum
The renal vein lies most anteriorly, then renal artery (it is an end artery) and the ureter lies most posterior.

294
Q

A 73 year old lady presents with a femoral hernia. Which of the following structures forms the lateral wall of the femoral canal?

	Pubic tubercle
	Femoral vein
	Femoral artery
	Conjoint tendon
	Femoral nerve
A

The canal exists to allow for the physiological expansion of the femoral vein, which lies lateral to it.

The femoral canal lies at the medial aspect of the femoral sheath. The femoral sheath is a fascial tunnel containing both the femoral artery laterally and femoral vein medially. The canal lies medial to the vein.

Borders of the femoral canal
Laterally	Femoral vein
Medially	Lacunar ligament
Anteriorly	Inguinal ligament
Posteriorly	Pectineal ligament

Contents
Lymphatic vessels
Cloquet’s lymph node

Physiological significance
Allows the femoral vein to expand to allow for increased venous return to the lower limbs.

Pathological significance
As a potential space, it is the site of femoral hernias. The relatively tight neck places these at high risk of strangulation.

295
Q

Which of these muscles is not a component of the rotator cuff?

	Subscapularis
	Teres minor
	Supraspinatus
	Infraspinatus
	Deltoid
A

Deltoid may abduct the shoulder and is not a rotator cuff muscle.

Supraspinatus muscle Suprascapular nerve
Infraspinatus muscle Suprascapular nerve
Teres minor muscle Axillary nerve
Subscapularis muscle Superior and inferior subscapular nerves

296
Q

Which of the following muscles is not within the posterior compartment of the lower leg?

	Peroneus brevis
	Flexor digitorum longus
	Soleus
	Popliteus
	Flexor hallucis longus
A

Peroneus brevis lies in the lateral compartment.

Compartments of the thigh

Formed by septae passing from the femur to the fascia lata.
Anterior compartment	Femoral	
Iliacus
Tensor fasciae latae
Sartorius
Quadriceps femoris
Femoral artery
Medial compartment	Obturator	
Adductor longus/magnus/brevis
Gracilis
Obturator externus
Profunda femoris artery and obturator artery
Posterior compartment (2 layers)	Sciatic	
Semimembranosus
Semitendinosus
Biceps femoris
Branches of Profunda femoris artery

Compartments of the lower leg
Separated by the interosseous membrane (anterior and posterior compartments), anterior fascial septum (separate anterior and lateral compartments) and posterior fascial septum (separate lateral and posterior compartments)

Anterior compartment Deep peroneal nerve
Tibialis anterior
Extensor digitorum longus
Extensor hallucis longus
Peroneus tertius
Anterior tibial artery
Posterior compartment Tibial
Muscles: deep and superficial compartments (separated by deep transverse fascia)
Deep: Flexor hallucis longus, Flexor digitalis longus, Tibialis posterior, Popliteus
Superficial: Gastrocnemius, Soleus, Plantaris
Posterior tibial
Lateral compartment Superficial peroneal
Peroneus longus/brevis
Peroneal artery

297
Q

How many unpaired branches leave the abdominal aorta to supply the abdominal viscera?

	One
	Two
	Three
	Four
	Five
A

There are three unpaired branches to the abdominal viscera. These include the coeliac axis, the SMA and IMA. Branches to the adrenals, renal arteries and gonadal vessels are paired. The fourth unpaired branch of the abdominal aorta, the median sacral artery, does not directly supply the abdominal viscera.

298
Q

A 34 year old man with a submandibular gland stone is undergoing excision of the submandibular gland. The incision is sited transversely approximately 4cm below the mandible. After incising the skin, platysma and deep fascia which of the following structures is most likely to be encountered.

	Facial artery
	Facial vein
	Lingual nerve
	Hypoglossal nerve
	Glossopharyngeal nerve
A

When approaching the submandibular gland the facial vein and submandibular lymph nodes are the most superficially encountered structures. Each sub mandibular gland has a superficial and deep part, separated by the mylohyoid muscle. The facial artery passes in a groove on the superficial aspect of the gland. It then emerges onto the surface of the face by passing between the gland and the mandible. The facial vein is encountered first in this surgical approach because the incision is made 4cm below the mandible (to avoid injury to the marginal mandibular nerve).

Relations of the submandibular gland
Superficial	Platysma, deep fascia and mandible
Submandibular lymph nodes
Facial vein (facial artery near mandible)
Marginal mandibular nerve
Cervical branch of the facial nerve
Deep	Facial artery (inferior to the mandible)
Mylohyoid muscle
Sub mandibular duct
Hyoglossus muscle
Lingual nerve
Submandibular ganglion
Hypoglossal nerve

Submandibular duct (Wharton’s duct)
Opens lateral to the lingual frenulum on the anterior floor of mouth.
5 cm length
Lingual nerve wraps around Wharton’s duct. As the duct passes forwards it crosses medial to the nerve to lie above it and then crosses back, lateral to it, to reach a position below the nerve.

Innervation
Sympathetic innervation- Derived from superior cervical ganglion
Parasympathetic innervation- Submandibular ganglion via lingual nerve

Arterial supply
Branch of the facial artery. The facial artery passes through the gland to groove its deep surface. It then emerges onto the face by passing between the gland and the mandible.

Venous drainage
Anterior facial vein (lies deep to the Marginal Mandibular nerve)

Lymphatic drainage
Deep cervical and jugular chains of nodes

299
Q

You are working as an anatomy demonstrator and the medical students decide to test your knowledge on the Circle of Willis. Which of the following comments is false?

Does not include the middle cerebral artery
Asymmetry of the circle of willis is a risk factor for the development of intracranial aneurysms
Majority of blood passing through the vessels mix together
Includes the anterior communicating artery
The circle surrounds the stalk of the pituitary gland
A

There is minimum mixing of blood passing through the vessels.

Circle of Willis

The two internal carotid arteries and two vertebral arteries form an anastomosis known as the Circle of Willis on the inferior surface of the brain. Each half of the circle is formed by:

  1. Anterior communicating artery
  2. Anterior cerebral artery
  3. Internal carotid artery
  4. Posterior communicating artery
  5. Posterior cerebral arteries and the termination of the basilar artery

The circle and its branches supply; the corpus striatum, internal capsule, diencephalon and midbrain.

Vertebral arteries
Enter the cranial cavity via foramen magnum
Lie in the subarachnoid space
Ascend on anterior surface of medulla oblongata
Unite to form the basilar artery at the base of the pons

Branches:
Posterior spinal artery
Anterior spinal artery
Posterior inferior cerebellar artery

Basilar artery
Branches:
Anterior inferior cerebellar artery
Labyrinthine artery
Pontine arteries
Superior cerebellar artery
Posterior cerebral artery
Internal carotid arteries
Branches:
Posterior communicating artery
Anterior cerebral artery
Middle cerebral artery
Anterior choroid artery
300
Q

A 64 year old man has a suspected lymphoma and lymph node biopsy from the posterolateral aspect of the right neck is planned.

A.	Recurrent laryngeal nerve
B.	Accessory nerve
C.	Hypoglossal nerve
D.	Vagus nerve
E.	Common peroneal nerve
F.	Tibial nerve
G.	Long saphenous nerve
H.	Phrenic nerve
I.	Thoracodorsal nerve
A

Accessory nerve

The accessory nerve has a superficial course and is easily injured. It lies under platysma and may be divided during the early part of the procedure.

A variety of different procedures carry the risk of iatrogenic nerve injury. These are important not only from the patients perspective but also from a medicolegal standpoint.

The following operations and their associated nerve lesions are listed here:
Posterior triangle lymph node biopsy and accessory nerve lesion.
Lloyd Davies stirrups and common peroneal nerve.
Thyroidectomy and laryngeal nerve.
Anterior resection of rectum and hypogastric autonomic nerves.
Axillary node clearance; long thoracic nerve, thoracodorsal nerve and intercostobrachial nerve.
Inguinal hernia surgery and ilioinguinal nerve.
Varicose vein surgery- sural and saphenous nerves.
Posterior approach to the hip and sciatic nerve.
Carotid endarterectomy and hypoglossal nerve.

There are many more, with sound anatomical understanding of the commonly performed procedures the incidence of nerve lesions can be minimised. They commonly occur when surgeons operate in an unfamiliar tissue plane or by blind placement of haemostats (not recommended).

301
Q

A 43 year old woman is due to undergo an axillary node clearance following a positive sentinel node biopsy.

A.	Recurrent laryngeal nerve
B.	Accessory nerve
C.	Hypoglossal nerve
D.	Vagus nerve
E.	Common peroneal nerve
F.	Tibial nerve
G.	Long saphenous nerve
H.	Phrenic nerve
I.	Thoracodorsal nerve
A

Thoracodorsal nerve

The long thoracic nerve is also at risk. The thoracodorsal nerve traverses the level 2 axillary nodes to supply latissimus dorsi and may be divided or damaged with diathermy.

A variety of different procedures carry the risk of iatrogenic nerve injury. These are important not only from the patients perspective but also from a medicolegal standpoint.

The following operations and their associated nerve lesions are listed here:
Posterior triangle lymph node biopsy and accessory nerve lesion.
Lloyd Davies stirrups and common peroneal nerve.
Thyroidectomy and laryngeal nerve.
Anterior resection of rectum and hypogastric autonomic nerves.
Axillary node clearance; long thoracic nerve, thoracodorsal nerve and intercostobrachial nerve.
Inguinal hernia surgery and ilioinguinal nerve.
Varicose vein surgery- sural and saphenous nerves.
Posterior approach to the hip and sciatic nerve.
Carotid endarterectomy and hypoglossal nerve.

There are many more, with sound anatomical understanding of the commonly performed procedures the incidence of nerve lesions can be minimised. They commonly occur when surgeons operate in an unfamiliar tissue plane or by blind placement of haemostats (not recommended).

302
Q

A 53 year old man is to undergo a thyroidectomy.

A.	Recurrent laryngeal nerve
B.	Accessory nerve
C.	Hypoglossal nerve
D.	Vagus nerve
E.	Common peroneal nerve
F.	Tibial nerve
G.	Long saphenous nerve
H.	Phrenic nerve
I.	Thoracodorsal nerve
A

Recurrent laryngeal nerve injury may complicate thyroid surgery in up to 2% of cases.

A variety of different procedures carry the risk of iatrogenic nerve injury. These are important not only from the patients perspective but also from a medicolegal standpoint.

The following operations and their associated nerve lesions are listed here:
Posterior triangle lymph node biopsy and accessory nerve lesion.
Lloyd Davies stirrups and common peroneal nerve.
Thyroidectomy and laryngeal nerve.
Anterior resection of rectum and hypogastric autonomic nerves.
Axillary node clearance; long thoracic nerve, thoracodorsal nerve and intercostobrachial nerve.
Inguinal hernia surgery and ilioinguinal nerve.
Varicose vein surgery- sural and saphenous nerves.
Posterior approach to the hip and sciatic nerve.
Carotid endarterectomy and hypoglossal nerve.

There are many more, with sound anatomical understanding of the commonly performed procedures the incidence of nerve lesions can be minimised. They commonly occur when surgeons operate in an unfamiliar tissue plane or by blind placement of haemostats (not recommended).

303
Q

A 14 year old boy jumps off a 10 foot wall and lands on both feet. An x-ray shows a bimalleolar fracture of the right ankle.

A. Smith's 
B. Bennett's 
C. Monteggia's 
D. Colle's 
E. Galeazzi 
F. Pott's 
G. Barton's
A

Pott’s

Eponymous fractures

Colles’ fracture (dinner fork deformity)
•Fall onto extended outstretched hand
•Classical Colles’ fractures have the following 3 features:

  1. Transverse fracture of the radius
  2. 1 inch proximal to the radio-carpal joint
  3. Dorsal displacement and angulation

Smith’s fracture (reverse Colles’ fracture)
•Volar angulation of distal radius fragment (Garden spade deformity)
•Caused by falling backwards onto the palm of an outstretched hand or falling with wrists flexed

Bennett’s fracture
•Intra-articular fracture of the first carpometacarpal joint
•Impact on flexed metacarpal, caused by fist fights
•X-ray: triangular fragment at ulnar base of metacarpal

Image sourced from Wikipedia
Monteggia’s fracture
•Dislocation of the proximal radioulnar joint in association with an ulna fracture
•Fall on outstretched hand with forced pronation
•Needs prompt diagnosis to avoid disability

Image sourced from Wikipedia
Galeazzi fracture
•Radial shaft fracture with associated dislocation of the distal radioulnar joint
•Direct blow

Pott’s fracture
•Bimalleolar ankle fracture
•Forced foot eversion

Barton’s fracture
•Distal radius fracture (Colles’/Smith’s) with associated radiocarpal dislocation
•Fall onto extended and pronated wrist
•Involvement of the joint is a defining feature

304
Q

A 22 year old drunk man is involved in a fight. He hurts his thumb when he punches his opponent.

A. Smith's 
B. Bennett's 
C. Monteggia's 
D. Colle's 
E. Galeazzi 
F. Pott's 
G. Barton's
A

Bennett’s

Eponymous fractures

Colles’ fracture (dinner fork deformity)
•Fall onto extended outstretched hand
•Classical Colles’ fractures have the following 3 features:

  1. Transverse fracture of the radius
  2. 1 inch proximal to the radio-carpal joint
  3. Dorsal displacement and angulation

Smith’s fracture (reverse Colles’ fracture)
•Volar angulation of distal radius fragment (Garden spade deformity)
•Caused by falling backwards onto the palm of an outstretched hand or falling with wrists flexed

Bennett’s fracture
•Intra-articular fracture of the first carpometacarpal joint
•Impact on flexed metacarpal, caused by fist fights
•X-ray: triangular fragment at ulnar base of metacarpal

Image sourced from Wikipedia
Monteggia’s fracture
•Dislocation of the proximal radioulnar joint in association with an ulna fracture
•Fall on outstretched hand with forced pronation
•Needs prompt diagnosis to avoid disability

Image sourced from Wikipedia
Galeazzi fracture
•Radial shaft fracture with associated dislocation of the distal radioulnar joint
•Direct blow

Pott’s fracture
•Bimalleolar ankle fracture
•Forced foot eversion

Barton’s fracture
•Distal radius fracture (Colles’/Smith’s) with associated radiocarpal dislocation
•Fall onto extended and pronated wrist
•Involvement of the joint is a defining feature

305
Q

A 63 year nurse falls on an extended and pronated wrist. An x-ray shows a distal radial fracture with radiocarpal dislocation

A. Smith's 
B. Bennett's 
C. Monteggia's 
D. Colle's 
E. Galeazzi 
F. Pott's 
G. Barton's
A

Barton’s

Eponymous fractures

Colles’ fracture (dinner fork deformity)
•Fall onto extended outstretched hand
•Classical Colles’ fractures have the following 3 features:

  1. Transverse fracture of the radius
  2. 1 inch proximal to the radio-carpal joint
  3. Dorsal displacement and angulation

Smith’s fracture (reverse Colles’ fracture)
•Volar angulation of distal radius fragment (Garden spade deformity)
•Caused by falling backwards onto the palm of an outstretched hand or falling with wrists flexed

Bennett’s fracture
•Intra-articular fracture of the first carpometacarpal joint
•Impact on flexed metacarpal, caused by fist fights
•X-ray: triangular fragment at ulnar base of metacarpal

Image sourced from Wikipedia
Monteggia’s fracture
•Dislocation of the proximal radioulnar joint in association with an ulna fracture
•Fall on outstretched hand with forced pronation
•Needs prompt diagnosis to avoid disability

Image sourced from Wikipedia
Galeazzi fracture
•Radial shaft fracture with associated dislocation of the distal radioulnar joint
•Direct blow

Pott’s fracture
•Bimalleolar ankle fracture
•Forced foot eversion

Barton’s fracture
•Distal radius fracture (Colles’/Smith’s) with associated radiocarpal dislocation
•Fall onto extended and pronated wrist
•Involvement of the joint is a defining feature

306
Q

Which of the following most commonly arises from the brachiocephalic artery?

 Vertebral artery 
 Subscapular artery 
 Thyroidea ima artery 
 Left Subclavian artery 
 None of the above
A

Thyroidea Ima

The brachiocephalic artery is the largest branch of the aortic arch. From its aortic origin it ascends superiorly, it initially lies anterior to the trachea and then on its right hand side. It branches into the common carotid and right subclavian arteries at the level of the sternoclavicular joint.

Path
Origin- apex of the midline of the aortic arch
Passes superiorly and posteriorly to the right
Divides into the right subclavian and right common carotid artery

Relations

Anterior •Sternohyoid
•Sternothyroid
•Thymic remnants
•Left brachiocephalic vein
•Right inferior thyroid veins

Posterior •Trachea
•Right pleura

Right lateral •Right brachiocephalic vein
•Superior part of SVC

Left lateral •Thymic remnants
•Origin of left common carotid
•Inferior thyroid veins
•Trachea (higher level)

Branches
Normally none but may have the thyroidea ima artery

307
Q

A 28 year old man is undergoing an appendicectomy. The external oblique aponeurosis is incised and the underlying muscle split in the line of its fibres. At the medial edge of the wound is a tough fibrous structure. Entry to this structure will most likely encounter which of the following?

 Internal oblique 
 Rectus abdominis 
 Transversus abdominis 
 Linea alba 
 Peritoneum
A

This structure will be the rectus sheath and when entered the rectus abdominis muscle will be encountered.

Abdominal incisions

Midline incision •Commonest approach to the abdomen
•Structures divided: linea alba, transversalis fascia, extraperitoneal fat, peritoneum (avoid falciform ligament above the umbilicus)
•Bladder can be accessed via an extraperitoneal approach through the space of Retzius

Paramedian incision •Parallel to the midline (about 3-4cm)
•Structures divided/retracted: anterior rectus sheath, rectus (retracted), posterior rectus sheath, transversalis fascia, extraperitoneal fat, peritoneum
•Incision is closed in layers

Battle •Similar location to paramedian but rectus displaced medially (and thus denervated)
•Now seldom used

Kocher’s Incision under right subcostal margin e.g. Cholecystectomy (open)

Lanz Incision in right iliac fossa e.g. Appendicectomy

Gridiron Oblique incision centered over McBurneys point- usually appendicectomy (less cosmetically acceptable than Lanz

Gable Rooftop incision

Pfannenstiel’s Transverse supra pubic, primarily used to access pelvic organs

McEvedy’s Groin incision e.g. Emergency repair strangulated femoral hernia

Rutherford Morrison Extraperitoneal approach to left or right lower quadrants. Gives excellent access to iliac vessels and is the approach of choice for first time renal transplantation.

308
Q

A 35 year old man presents to the surgical clinic with a suspected direct inguinal hernia. These will pass through Hesselbach’s triangle. Which of the following forms the medial edge of this structure?

 External oblique aponeurosis 
 Inferior epigastric artery 
 Rectus abdominis muscle 
 Inferior epigastric vein 
 Obturator nerve
A

Direct inguinal hernias pass through Hesselbachs triangle (although this is of minimal clinical significance!). Its medial boundary is the rectus muscle.

Hesselbach’s triangle

Direct hernias pass through Hesselbachs triangle.

Superolaterally Epigastric vessels
Medially Lateral edge of rectus muscle
Inferiorly Inguinal ligament

309
Q

Which of the following muscles is not innervated by the ansa cervicalis?

 Sternohyoid 
 Mylohyoid 
 Omohyoid 
 Sternothyroid 
 None of the above
A

Mylohyoid is innervated by the mylohyoid branch of the inferior alveolar nerve. (trigeminal)

Ansa cervicalis

Superior root Branch of C1 anterolateral to carotid sheath
Inferior root Derived from C2 and C3 roots, passes posterolateral to the internal jugular vein (may lie either deep or superficial to it)
Innervation Sternohyoid
Sternothyroid
Omohyoid

The ansa cervicalis lies anterior to the carotid sheath. The nerve supply to the inferior strap muscles enters at their inferior aspect. Therefore when dividing these muscles to expose a large goitre, the muscles should be divided in their upper half.

310
Q

A 58 year old lady presents with a mass in the upper outer quadrant of the right breast. Which of the following statements relating to the breast is untrue?

The internal mammary artery provides the majority of its arterial supply
Nipple retraction may occur as a result of tumour infiltration of the clavipectoral fascia
The internal mammary artery is a branch of the subclavian artery
Up to 70% of lymphatic drainage is to the ipsilateral axillary nodes
None of the above

A

Both skin dimpling and nipple retraction are features of breast malignancy. However, they usually occur as a result of tumour infiltration of the breast ligaments and ducts respectively. The clavipectoral fascia encases the axillary contents. The lymphatic drainage of the breast is to the axilla and also to the internal mammary chain. The breast is well vascularised and the internal mammary artery is a branch of the subclavian artery.

Breast

The breast itself lies on a layer of pectoral fascia and the following muscles:

  1. Pectoralis major
  2. Serratus anterior
  3. External oblique

Breast anatomy

Nerve supply Branches of intercostal nerves from T4-T6.
Arterial supply •Internal mammary (thoracic) artery
•External mammary artery (laterally)
•Anterior intercostal arteries
•Thoraco-acromial artery

Venous drainage Superficial venous plexus to subclavian, axillary and intercostal veins.
Lymphatic drainage •70% Axillary nodes
•Internal mammary chain
•Other lymphatic sites such as deep cervical and supraclavicular fossa (later in disease)